Tải bản đầy đủ (.pdf) (184 trang)

Tổng hợp đề và lời giải đề chọn đội tuyển TST Việt Nam

Bạn đang xem bản rút gọn của tài liệu. Xem và tải ngay bản đầy đủ của tài liệu tại đây (10.28 MB, 184 trang )

<span class='text_page_counter'>(1)</span><div class='page_container' data-page=1></div>
<span class='text_page_counter'>(2)</span><div class='page_container' data-page=2>

1

<b>LỜI GIẢI ĐỀ THI CHỌN ĐỘI TUYỂN QUỐC GIA </b>



<b>DỰ THI IMO 2011 </b>



<b>*********** </b>


<b>Bài 1. </b>


<i><b>Trên mặt phẳng tọa độ có một con cào cào ở điểm </b></i>(1;1)<i><b>. Nó có thể nhảy từ điểm A sang điểm </b></i>
<i><b>B khi tam giác OAB có diện tích bằng </b></i>1


2<i><b> và tọa độ của A, B nguyên dương. </b></i>


<i><b>1. Tìm các điểm </b></i>( , )<i><b>m n sao cho con cào cào có th</b><b>ể nhảy đến đó sau hữu hạn bước. </b></i>
<i><b>2. Chứng minh rằng con cào cào có thể nhảy đến </b></i>( , )<i><b>m n k</b><b>ể trên sau ít hơn </b>m</i><i>n</i> <i><b> bước.</b></i>
<b>Lời giải. </b>


Trước hết, ta cần chứng minh nhận xét sau:


Với ( , )<i>m n</i> là cặp số nguyên tố cùng nhau thì tồn tại a và b nguyên tố cùng nhau sao cho
1


<i>mb</i><i>na</i>  và <i>a</i> <i>b</i> <i>m</i> <i>n</i> 1.
Thật vậy, không mất tính tổng quát, giả sử <i>m</i><i>n</i>.


Xét các số có dạng <i>mb</i>1,1 <i>b</i> <i>n</i>. Dễ thấy ta có n số như vậy và nếu <i>b</i><i>b</i>' thì


1 ' 1(mod )


</div>
<span class='text_page_counter'>(3)</span><div class='page_container' data-page=3>

2



Đặt <i>a</i> <i>mb</i> 1 <i>a</i> <i>mn</i> <i>m</i>


<i>n</i> <i>n</i>




    và <i>mb</i><i>an</i>1.


Ta có <i>n b</i>( <i>a</i>)  (<i>m</i><i>n b</i>)  1 (<i>m</i><i>n n</i>) (<i>m</i><i>n b</i>)(   <i>n</i>) 1 (<i>m</i><i>n n</i>)   <i>b</i> <i>a</i> <i>m</i><i>n</i> .
Do đó <i>b</i> <i>a</i> <i>m</i> <i>n</i> 1. Nhận xét được chứng minh.


Trở lại bài toán, ta thấy rằng, con cào cào đang ở đỉnh <i>A a b</i>( , ) và muốn nhảy sang đỉnh <i>B c d</i>( , )


thì phải có 1 1 1 1


2 2 2


<i>OAB</i>


<i>S</i>   <i>ad</i><i>bc</i>   <i>ad</i><i>bc</i>  .


Từ đó suy ra xuất phát từ điểm ( ; )1 1 , con cào cào chỉ có thể nhảy đến các điểm ( ; )<i>m n</i> mà
( , )<i>m n</i> 1. Hơn nữa, với ( , )<i>m n</i> 1 thì theo nhận xét ở trên thì con cào cào có thể nhảy đến điểm
có tọa độ tương ứng là ( ; )<i>m n</i> . Vậy các điểm cần tìm là ( ; )<i>m n</i> với <i>m n</i>, nguyên tố cùng nhau.
2. Xét điểm nguyên dương ( ; )<i>m n</i> mà ( , )<i>m n</i> 1,<i>m n</i> 1 thì con cào cào có thể nhảy đến điểm
này chỉ sau một bước nhảy.


Ta lại xét điểm ( ; )<i>m n</i> mà ( , )<i>m n</i> 1, <i>m n</i> 1 thì theo nhận xét trên, tồn tại các số <i>m n</i>', ' sao
cho <i>mm</i>'<i>nn</i>' 1 và <i>m</i>'<i>n</i>'  <i>m</i><i>n</i> 1.



Lặp lại quá trình này, ta thấy rằng con cào cào có thể nhảy đến điểm ( ; )<i>m n</i> sau không quá
<i>m n</i> bước.


</div>
<span class='text_page_counter'>(4)</span><div class='page_container' data-page=4>

3
<b>Bài 2. </b>


<i><b>Cho đường (O) và điểm A nằm ngồi đường trịn. Kẻ hai tiếp tuyến </b><b>AB AC t</b></i>, <i><b>ới (O) với A, B là </b></i>
<i><b>các tiếp điểm. Gọi P là điểm thuộc tia đối của tia BA, Q là điểm thuộc tia đối của tia CA sao </b></i>
<i><b>cho PQ tiếp xúc với (O). Qua P kẻ đường thẳng song song với AC và cắt đường thẳng BC tại </b></i>
<i><b>E. Qua Q kẻ đường thẳng song song với AB và cắt BC tại F. </b></i>


<i><b>1. Chứng minh rằng các đường thẳng </b>QE PF</i>, <i><b>luôn đi qua điểm cố định lần lượt là M, N. </b></i>
<i><b>2. Chứng minh rằng tích </b>PM QN</i> <i><b>không đổi. </b></i>


<b>Lời giải. </b>


Trước hết, ta chứng minh nhận xét sau: <i>Cho tam giác ABC ngoại tiếp (I) có tiếp điểm của (I) lên </i>
<i>AB, AC lần lượt là E, F. Đường thẳng qua B, song song với AC cắt EF tại K; CK cắt AB tại G. </i>
<i>Khi đó, tam giác AGI vng tại I.</i>


Thật vậy:


Đặt <i>AB</i><i>c BC</i>, <i>a CA</i>, <i>b</i> và chu vi tam giác ABC là 2<i>p</i>.


Do BK // AC nên tam giác BKF cân tại B, suy ra: <i>AE</i><i>AF</i> <i>p</i> <i>a</i>.
Theo định lí Thales thì: <i>BG</i> <i>BK</i> <i>p</i> <i>b</i> <i>AB</i> <i>p</i> <i>AG</i> <i>bc</i>


<i>AG</i> <i>AC</i> <i>b</i> <i>AG</i> <i>b</i> <i>p</i>





      .


Mà <i>AF</i> <i>p</i> <i>a</i> nên <i>AF</i> <i>p p</i>( <i>a</i>)


<i>AG</i> <i>bc</i>




 .


Ta cũng có: , .sin sin2 1 cos ( )


2 2 2


sin
2


<i>AF</i> <i>A</i> <i>AH</i> <i>A</i> <i>A</i> <i>p p</i> <i>a</i>


<i>AI</i> <i>AH</i> <i>AF</i>


<i>A</i> <i><sub>AI</sub></i> <i><sub>bc</sub></i>


 


      ..


Từ đó suy ra: <i>AF</i> <i>AH</i> <i>p p</i>( <i>a</i>)



<i>AG</i> <i>AI</i> <i>bc</i>




</div>
<span class='text_page_counter'>(5)</span><div class='page_container' data-page=5>

4
Bổ đề được chứng minh. Trở lại bài toán đã cho.


1/ Gọi M, N lần lượt là giao điểm
của QE với AB và PF với AC.
Theo bổ đề trên, ta thấy rằng tam
giác OMA và ONA lần lượt
vuông tại O nên các điểm M, N
cố định.


2/ Trước hết, ta đặt


, ,


<i>AB</i><i>AC</i><i>a BP</i><i>x CQ</i><i>y</i>.
Chu vi của tam giác APQ là


2( )


<i>p</i> <i>a</i> <i>x</i> <i>y</i> .


Theo bổ đề trên, ta tính được:
2 .


( ) ( )



<i>AP PQ</i>


<i>PM</i> <i>AP</i>


<i>AP</i> <i>AQ</i> <i>PQ</i>


<i>AP AP</i> <i>AQ</i> <i>PQ</i> <i>a</i> <i>x x</i>


<i>AP</i> <i>AQ</i> <i>PQ</i> <i>a</i> <i>x</i> <i>y</i>


  


 


  <sub></sub> 


   


Tương tự <i>QN</i> (<i>a</i> <i>y y</i>)


<i>a</i> <i>x</i> <i>y</i>





  .


Cần chứng minh rằng tích ( )( <sub>2</sub> )


( )



<i>xy a</i> <i>x a</i> <i>y</i>
<i>PM QN</i>


<i>a</i> <i>x</i> <i>y</i>


 


 


  không đổi. Thật vậy:
Nếu gọi R là bán kính của (O) thì diện tích của tam giác APQ cùng bằng:






sin . . ( )( )


2 2 <sub>sin</sub>


<i>pR</i> <i>BAC AP AQ</i> <i>a</i> <i>x a</i> <i>y</i> <i>R</i>


<i>a</i> <i>x</i> <i>y</i> <i><sub>BAC</sub></i>


 


  


  .



Suy ra tỉ số (<i>a</i> <i>x a</i>)( <i>y</i>) <i>k</i>


<i>a</i> <i>x</i> <i>y</i>


  <sub></sub>


  không đổi, với sin


<i>R</i>
<i>k</i>


<i>BAC</i>


 . Từ đó, ta có


(<i>a</i> <i>x a</i>)( <i>y</i>) <i>k a</i>( <i>x</i> <i>y</i>) <i>a a</i>( <i>x</i> <i>y</i>) <i>xy</i> <i>k a</i>( <i>x</i> <i>y</i>) <i>a</i> <i>xy</i> <i>k</i>


<i>a</i> <i>x</i> <i>y</i>


              


  , tức là tỉ
số <i>xy</i>


<i>a</i> <i>x</i> <i>y</i> cũng không đổi.


Vậy ( )( <sub>2</sub> )


( )



<i>xy a</i> <i>x a</i> <i>y</i>
<i>PM QN</i>


<i>a</i> <i>x</i> <i>y</i>


 


 


</div>
<span class='text_page_counter'>(6)</span><div class='page_container' data-page=6>

5
* Cách khác:


1. Ta sẽ chứng minh giao điểm
M của EQ và AB, giao điểm N
của FP và AC là các điểm cố
định.


Giả sử (O) tiếp xúc với BC tại
D. Gọi K, L lần lượt là giao
điểm của QO và BC, PK và
OD theo thứ tự đó.


Dễ thấy rằng <i>PKO</i>900,suy
ra: các cặp tam giác LKO, LDP
và LOP, QOA đồng dạng với
nhau. Suy ra,


<i>OK</i> <i>LO</i> <i>OQ</i> <i>OK</i> <i>PB</i>



<i>PD</i>  <i>LP</i>  <i>AQ</i> <i>OQ</i>  <i>AQ</i>
Mặt khác dễ thấy là <i>PE</i><i>PB</i>


nên <i>OK</i> <i>PE</i> <i>EM</i>


<i>OQ</i>  <i>AQ</i>  <i>MQ</i>.
Áp dụng định lí thales đảo ta được: MO song song với BC là đoạn cố định mà M thuộc AB cố
định nên M cũng cố định. Tương tự, N là điểm cố định. Ta có đpcm.


2.Từ câu 1), ta có <i>M O N</i>, , thẳng hàng. Từ đó dễ có <i>PMO</i><i>ONQ g g</i>( . ).
Suy ra <i>PM QN</i> <i>OM ON</i> <i>OM</i>2là hằng số.


Ta cũng chứng minh được rằng M, N là tiếp điểm của đường tròn A-Mixitilinear trên các đoạn
AP, AQ của tam giác APQ.


<i><b>L</b></i>


<i><b>N</b></i>
<i><b>K</b></i>


<i><b>M</b></i>


<i><b>F</b></i>


<i><b>E</b></i>


<i><b>Q</b></i>
<i><b>P</b></i>


<i><b>C</b></i>


<i><b>B</b></i>


<i><b>O</b></i>
<i><b>A</b></i>


</div>
<span class='text_page_counter'>(7)</span><div class='page_container' data-page=7>

6
<b>Bài 3</b>.


<i><b>Cho cho n nguyên dương thỏa </b>n</i>3<i><b>và n số thực </b>x x x</i><sub>1</sub>, <sub>2</sub>, <sub>3</sub>,...,<i><b>x th</b><sub>n</sub></i> <i><b>ỏa mãn đồng thời </b></i>
<i><b>(i) </b>x</i><sub>1</sub>   <i>x</i><sub>2</sub> <i>x</i><sub>3</sub> ... <i>x<sub>n</sub></i>0<i><b>. </b></i>


<i><b>(ii) </b>x</i><sub>1</sub>2<i>x</i><sub>2</sub>2<i>x</i><sub>2</sub>3 ... <i>x<sub>n</sub></i>2<i>n n</i>( 1)<i><b>. </b></i>
<i><b>(iii) </b>x</i><sub>1</sub><i>x</i><sub>2</sub><i>x</i><sub>3</sub> ... <i>x<sub>n</sub><b>. </b></i>


<i><b>Tìm giá trị lớn nhất và nhỏ nhất của</b></i> <i>f</i>  <i>x</i><sub>1</sub> <i>x</i><sub>2</sub><i><b>. </b></i>
<b>Lời giải. </b>


<i>*Tìm giá trị lớn nhất. </i>


Theo BĐT Bunhiacopski cho <i>n</i>2 số thực <i>x x</i><sub>3</sub>, <sub>4</sub>,...,<i>x<sub>n</sub></i>, ta có:


2


2 2 2 2 2 2 2 2 3 4


3 3 4 3


( ... )


(1 1 ...1 )( ... ) ( ... ) ...



2
<i>n</i>


<i>n</i> <i>n</i> <i>n</i>


<i>x</i> <i>x</i> <i>x</i>


<i>x</i> <i>x</i> <i>x</i> <i>x</i> <i>x</i> <i>x</i> <i>x</i>


<i>n</i>


  


           


 . Suy ra


2 2 2


2 2 3 2 2 2 3 4 1 2 1 2


1 2 2 1 2


2


1 2 1 2


( ... ) ( ) ( )



( 1) ...


2 2 2


2 ( 1)( 2)


( ) 2( 1)( 2) 2( 1)( 2)


( 2 2)


<i>n</i>
<i>n</i>


<i>x</i> <i>x</i> <i>x</i> <i>x</i> <i>x</i> <i>x</i> <i>x</i>


<i>n n</i> <i>x</i> <i>x</i> <i>x</i> <i>x</i> <i>x</i> <i>x</i>


<i>n</i> <i>n</i>


<i>n n</i> <i>n</i>


<i>x</i> <i>x</i> <i>n</i> <i>n</i> <i>x</i> <i>x</i> <i>n</i> <i>n</i>


<i>n</i>
    
          
 
 
          
 



Do đó, giá trị lớn nhất của <i>f</i>  <i>x</i><sub>1</sub> <i>x</i><sub>2</sub> là 2(<i>n</i>1)(<i>n</i>2), đạt được khi


1 2, 3 4 ... <i>n</i>


<i>x</i> <i>x x</i> <i>x</i>  <i>x</i> hay 1 2 3 4 5


( 1)( 2) 2( 1)


, ...


2 <i>n</i> 2


<i>n</i> <i>n</i> <i>n</i>


<i>x</i> <i>x</i> <i>x</i> <i>x</i> <i>x</i> <i>x</i>


<i>n</i>


  


       


 .
<i>*Tìm giá trị nhỏ nhất.</i>


Ta xét các trường hợp:


- Nếu <i>n</i>3 thì <i>x</i><sub>1</sub>2<i>x</i><sub>2</sub> <i>x</i><sub>1</sub> <i>x</i><sub>2</sub><i>x</i><sub>3</sub>0 và 2<i>x</i><sub>1</sub><i>x</i><sub>2</sub> <i>x</i><sub>1</sub> <i>x</i><sub>2</sub><i>x</i><sub>3</sub>0.



Thay <i>x</i><sub>3</sub> (<i>x</i><sub>1</sub><i>x</i><sub>2</sub>) vào đẳng thức (ii), ta được 2 2 2 2 2


1 2 ( 1 2) 6 1 2 1 2 3
<i>x</i> <i>x</i>  <i>x</i> <i>x</i>  <i>x</i> <i>x</i> <i>x x</i>  .
Từ đánh giá (2<i>x</i><sub>1</sub><i>x</i><sub>2</sub>)(<i>x</i><sub>1</sub>2<i>x</i><sub>2</sub>) 0 <i>x</i><sub>1</sub>2<i>x</i><sub>2</sub>2<i>x x</i><sub>1</sub> <sub>2</sub>(2<i>x</i><sub>1</sub><i>x</i><sub>2</sub>)(<i>x</i><sub>1</sub>2<i>x</i><sub>2</sub>) 3 (<i>x</i><sub>1</sub><i>x</i><sub>2</sub>)21.
Dễ thấy nếu <i>x</i><sub>1</sub><i>x</i><sub>2</sub> 1 <i>x</i><sub>3</sub>   1 0 <i>x</i><sub>1</sub> <i>x</i><sub>2</sub>, mâu thuẫn nên <i>x</i><sub>1</sub><i>x</i><sub>2</sub>1.


Đẳng thức xảy ra khi <i>x</i><sub>1</sub>2,<i>x</i><sub>2</sub><i>x</i><sub>3</sub> 1


</div>
<span class='text_page_counter'>(8)</span><div class='page_container' data-page=8>

7
Đánh giá này đúng vì


2 2 2 2 2


( ) 2 2 ( ) 2 0 ( )( ) 0


<i>b</i> <i>c</i> <i>a</i>   <i>b</i> <i>c</i> <i>a</i>  <i>a</i>  <i>a b</i> <i>c</i> <i>bc</i>  <i>a</i><i>b a</i> <i>c</i>
Áp dụng đánh giá này với <i>x</i><sub>2</sub><i>x</i><sub>3</sub><i>x</i><sub>4</sub>, ta được: 2 2 2 2


3 4 2 ( 3 4 2)
<i>x</i> <i>x</i> <i>x</i>  <i>x</i>  <i>x</i> <i>x</i> .
Dễ thấy <i>x</i><sub>2</sub>  <i>x</i><sub>3</sub> <i>x</i><sub>4</sub> <i>x x</i><sub>2</sub>, <sub>2</sub><i>x</i><sub>5</sub> nên áp dụng tiếp đánh giá (*), ta có:


2 2 2 2 2 2 2 2


3 4 5 2 ( 3 4 2) 5 2 2 ( 3 4 2 2)
<i>x</i> <i>x</i> <i>x</i> <i>x</i>  <i>x</i>  <i>x</i> <i>x</i> <i>x</i>  <i>x</i>  <i>x</i>  <i>x</i> <i>x</i> .
Tiếp tục áp dụng đánh giá (*) với các số <i>a a</i><sub>6</sub>, <sub>7</sub>,...,<i>a<sub>n</sub></i>, ta được:


2

2


2 2 3 2 2 2


1 2 2 ... <i>n</i> ( 3) 2 3 4 ... <i>n</i> ( 3) 2 ( 3) 2 1 ( 2) 2
<i>x</i> <i>x</i> <i>x</i>  <i>x</i>  <i>n</i> <i>x</i>  <i>x</i>     <i>x</i> <i>x</i> <i>n</i> <i>x</i>  <i>n</i> <i>x</i>  <i>x</i>  <i>n</i> <i>x</i>
Suy ra <i>x</i><sub>1</sub>2 (<i>n</i> 2)<i>x</i><sub>2</sub>2

<i>x</i><sub>1</sub> (<i>n</i> 2)<i>x</i><sub>2</sub>

2<i>n n</i>( 1).


Ta sẽ chứng minh rằng


2 2


2 2 2 2 2 1 2


1 2 1 2 1 2 1 2


( 1)( )


( 2) ( 2) 2 ( 3 2) 2( 2)


4


<i>n n</i> <i>x</i> <i>x</i>


<i>x</i>  <i>n</i> <i>x</i>  <i>x</i>  <i>n</i> <i>x</i>  <i>x</i>  <i>n</i>  <i>n</i> <i>x</i>  <i>n</i> <i>x x</i>    .


Bất đẳng thức này tương đương với: (<i>x</i><sub>1</sub><i>x</i><sub>2</sub>) (<sub></sub><sub></sub> <i>n</i>2 <i>n</i> 8)<i>x</i><sub>1</sub> (<i>n</i> 1)(3<i>n</i>8)<i>x</i><sub>2</sub><sub></sub><sub></sub>0, mà bất đẳng
thức này đúng do <i>x</i><sub>1</sub><i>x</i><sub>2</sub> và


2 2



1 2 1 1 1


(<i>n</i>  <i>n</i> 8)<i>x</i>  (<i>n</i> 1)(3<i>n</i>8)<i>x</i> (<i>n</i>  <i>n</i> 8)<i>x</i> (3<i>n</i>8)<i>x</i> <i>n n</i>( 4)<i>x</i> 0.
Từ các điều này, ta có:


2


2
1 2


1 2


( 1)( )


( 1) ( ) 4


4


<i>n n</i> <i>x</i> <i>x</i>


<i>n n</i>     <i>x</i> <i>x</i>  , dễ thấy <i>x</i><sub>1</sub><i>x</i><sub>2</sub>2.
Trong trường hợp này, giá trị nhỏ nhất của biểu thức đã cho là 2, đạt được khi


1 2 3 ... <i>n</i> 1 1, <i>n</i> 1
<i>x</i> <i>x</i> <i>x</i>   <i>x</i><sub></sub>  <i>x</i>  <i>n</i>.


Vậy giá trị lớn nhất của <i>f</i>  <i>x</i><sub>1</sub> <i>x</i><sub>2</sub> là 2(<i>n</i>1)(<i>n</i>2), đạt được khi


1 2, 3 4 ... <i>n</i>



<i>x</i> <i>x x</i> <i>x</i>  <i>x</i> hay <sub>1</sub> <sub>2</sub> ( 1)( 2), <sub>3</sub> <sub>4</sub> <sub>5</sub> ... 2( 1)


2 <i>n</i> 2


<i>n</i> <i>n</i> <i>n</i>


<i>x</i> <i>x</i> <i>x</i> <i>x</i> <i>x</i> <i>x</i>


<i>n</i>


  


       


 .
Giá trị nhỏ nhất của <i>f</i>  <i>x</i><sub>1</sub> <i>x</i><sub>2</sub> là 1 với <i>n</i>3, đạt được khi <i>x</i><sub>1</sub>2,<i>x</i><sub>2</sub> <i>x</i><sub>3</sub> 1 và là 2 với


4


<i>n</i> , đạt được khi <sub>1</sub> <sub>2</sub> ( 1)( 2), <sub>3</sub> <sub>4</sub> <sub>5</sub> ... 2( 1)


2 <i>n</i> 2


<i>n</i> <i>n</i> <i>n</i>


<i>x</i> <i>x</i> <i>x</i> <i>x</i> <i>x</i> <i>x</i>


<i>n</i>


  



       


</div>
<span class='text_page_counter'>(9)</span><div class='page_container' data-page=9>

8
<b>Bài 4. </b>


<i><b>Cho dãy số</b></i>(<i>a<sub>n</sub></i>)<i><b>thỏa mãn </b>a</i><sub>0</sub>1,<i>a</i><sub>1</sub>3<i><b>và </b></i>


2
1
2 1
<i>n</i>
<i>n</i>
<i>n</i>
<i>a</i>
<i>a</i>
<i>a</i>


 
 
  <sub></sub> <sub></sub><i><b>. </b></i>
<i><b>Chứng minh rằng với mọi số tự nhiên n thì </b>a a<sub>n</sub></i> <i><sub>n</sub></i><sub></sub><sub>2</sub><i>a<sub>n</sub></i>2<sub></sub><sub>1</sub>2<i>n<b>. </b></i>
<b>Lời giải. </b>


Dễ thấy mọi số hạng của dãy đều là số nguyên dương.


Trước hết, bằng quy nạp, ta sẽ chứng minh rằng đã cho cũng thỏa mãn hệ thức truy hồi
,



<i>n</i> <i>n</i> <i>n</i>


<i>a</i> <sub></sub><sub>2</sub> 4<i>a</i> <sub></sub><sub>1</sub>2<i>a</i>  <i>n</i> .


Thật vậy, bằng tính tốn trực tiếp, ta có <i>a</i><sub>2</sub> 10,<i>a</i><sub>3</sub> 34,<i>a</i><sub>4</sub> 116 thỏa mãn cả hai hệ thức truy
hồi, tức là khẳng định đúng với <i>n</i>0 1 2, , .


Giả sử khẳng định đúng đến <i>n</i><i>k k</i>, 2, tức là <i>a<sub>k</sub></i><sub></sub><sub>2</sub> 4<i>a<sub>k</sub></i><sub></sub><sub>1</sub>2<i>a<sub>k</sub></i>.
Ta sẽ chứng minh rằng nó cũng đúng với <i>n</i><i>k</i>1.


Ta có <i>k</i> ( <i>k</i> <i>k</i>) <i>k</i>


<i>k</i> <i>k</i> <i>k</i>


<i>k</i> <i>k</i> <i>k</i>


<i>a</i> <i>a</i> <i>a</i> <i>a</i>


<i>a</i> <i>a</i> <i>a</i>


<i>a</i> <i>a</i> <i>a</i>


 
 
  
      
 <sub></sub> <sub></sub> <sub></sub> <sub></sub>   <sub></sub> <sub></sub>
     


2 2 2



2 1


3 1


1 1 1


4 2 4


1 1 1 16 16 .


Ta lại cần chứng minh <i>k</i> , <b> </b>

<sub> </sub>

*
<i>k</i>


<i>k</i>
<i>a</i>


<i>a</i> <i>k</i>


<i>a</i><sub></sub> 


 
  
 
 
2
1
1
4



4 1 2 .


Thật vậy,


(*) <i>k</i> <i>k</i>


<i>k</i> <i>k</i> <i>k</i> <i>k</i> <i>k</i> <i>k</i>


<i>k</i> <i>k</i>


<i>a</i> <i>a</i>


<i>a</i> <i>a</i> <i>a</i> <i>a</i> <i>a</i> <i>a</i>


<i>a</i> <sub></sub>  <i>a</i> <sub></sub>   


        


2 2


2 2


1 1 1 1


1 1


4 4 1


1 4 1



4


Hơn nữa, theo giả thiết thì <i>k</i> <i>k</i> <i>k</i>


<i>k</i> <i>k</i> <i>k</i> <i>k</i> <i>k</i> <i>k</i> <i>k</i>


<i>k</i> <i>k</i> <i>k</i>


<i>a</i> <i>a</i> <i>a</i>


<i>a</i> <i>a</i> <i>a</i> <i>a</i> <i>a</i> <i>a</i> <i>a</i>


<i>a</i> <i>a</i> <i>a</i>


    


  


 


 <sub></sub> <sub></sub>       
 


2 2 2


2 2


1 1 1 1 1


1 1 1



1 1 .


Theo giả thiết quy nạp, ta có


( ) ( )


<i>k</i> <i>k</i> <i>k</i> <i>k</i> <i>k</i> <i>k</i> <i>k</i> <i>k</i> <i>k</i> <i>k</i> <i>k</i> <i>k</i>


<i>a</i> 1  <i>a</i>  <i>a</i> 1  <i>a</i> 1 <i>a</i> 2  <i>a</i> 1 <i>a</i> 1 <i>a</i> 1<i>a</i> 2  <i>a</i> 1<i>a</i> 1 <i>a</i> 1
1


4 2 4 4 2 2 6 8 4


4 .
Do đó, ta có được <i>a<sub>k</sub></i>2 <i>a<sub>k</sub></i><sub></sub><sub>1</sub><i>a<sub>k</sub></i><sub></sub><sub>1</sub> <i>a<sub>k</sub></i>21<i>a<sub>k</sub></i><sub></sub><sub>1</sub>


4 hay (*) đúng. Từ ,


<i>k</i>


<i>k</i>
<i>k</i>


<i>a</i>


<i>a</i> <i>k</i>


<i>a</i> <sub></sub> 



 
  
 
 
2
1
1
4


</div>
<span class='text_page_counter'>(10)</span><div class='page_container' data-page=10>

9


( ) ( )


<i>k</i> <i>k</i> <i>k</i> <i>k</i> <i>k</i> <i>k</i> <i>k</i> <i>k</i> <i>k</i> <i>k</i>


<i>a</i> <sub></sub><sub>3</sub>  1 16<i>a</i> <sub></sub><sub>1</sub>16<i>a</i> 4<i>a</i> <sub></sub><sub>1</sub> 1 4 4<i>a</i> <sub></sub><sub>1</sub>2<i>a</i> 2 4<i>a</i> 2<i>a</i> <sub></sub><sub>1</sub> 4<i>a</i> <sub></sub><sub>2</sub> 2<i>a</i> <sub></sub><sub>1</sub>.


Suy ra khẳng định đúng với <i>n</i><i>k</i>1 và theo giả thiết quy nạp thì khẳng định được chứng minh.
Ta sẽ chứng minh dãy số thỏa mãn <i>a</i><sub>0</sub> 1,<i>a</i><sub>1</sub> 3,<i>a<sub>n</sub></i> 4<i>a<sub>n</sub></i><sub></sub><sub>1</sub>2<i>a<sub>n</sub></i><sub></sub><sub>2</sub>,<i>n</i>2 cũng thỏa mãn đẳng


thức <i>n</i>


<i>n</i> <i>n</i> <i>n</i>


<i>a</i> <sub></sub><sub>2</sub><i>a</i> <i>a</i>2<sub></sub><sub>1</sub> 2 với mọi n (**)


</div>
<span class='text_page_counter'>(11)</span><div class='page_container' data-page=11>

10
<b>Bài 5. </b>


<i><b>Tìm tất cả các số nguyên dương n sao cho biểu thức sau </b></i>


2


2<i>n</i> (2<i>n</i> 1) 8.3<i>n</i> 1


<i>A</i>    


<i><b>là một số chính phương. </b></i>
<b>Lời giải. </b>


Giả sử tồn tại số <i>n</i> nguyên dương thỏa mãn đề bài.
Khi đó, tồn tại <i>m</i> sao cho:


2 2 2 2 1 1 2


2<i>n</i> (2<i>n</i> 1) 8 3<i>n</i> 1 2 <i>n</i> 2 2<i>n</i> 1 8 3<i>n</i> (2<i>n</i> 1) 8.3<i>n</i>


<i>m</i>                  


hay ( )( )


<i>n</i> <i>n</i>
<i>n</i>
<i>m</i> <i>m</i>
 
   
 
1 1


2 1 2 1



2 3


4 .


Đặt ( ) , ( )


<i>n</i> <i>n</i>
<i>t</i> <i>t</i>
<i>a</i> <i>b</i>
 
   
 
1 1


2 1 2 1


2 2 thì ,


<i>n</i> <i>n</i>


<i>a b</i>  <i>ab</i>


 2 11  2 3 .


Dễ thấy t là số lẻ nên <i>a b</i>, là các số nguyên dương. Do đó, <i>a</i>3<i>u</i>,<i>b</i> 2 3<i>v</i> hoặc ngược lại.
Trong cả hai trường hợp, ta đều có 3<i>u</i>  2 3<i>v</i> 2<i>n</i>11 và <i>u</i> <i>v</i> <i>n</i>.


Ta xét các trường hợp sau:


- Nếu <i>n</i>1 thì 3<i>u</i>  2 3<i>v</i> 3,<i>u</i> <i>v</i> 1, hệ này khơng có nghiệm nguyên dương.


- Nếu <i>n</i>2 thì <i>u</i> <i>v</i> ,


<i>u</i> <i>v</i>


    


3 2 3 7 2, hệ này cũng khơng có nghiệm ngun dương.
- Nếu <i>n</i>3 thì 3<i>u</i> 2 3<i>v</i>15,<i>u</i> <i>v</i> 3, hệ này có nghiệm là <i>u</i>2,<i>v</i>1.


- Nếu <i>n</i>4 thì 3<i>u</i>  2 3<i>v</i> 31,<i>u</i> <i>v</i> 4, hệ này khơng có nghiệm ngun dương dương.
- Nếu <i>n</i>5 thì 3<i>u</i>  2 3<i>v</i> 63,<i>u</i> <i>v</i> 5, hệ này có nghiệm là <i>u</i>2,<i>v</i>3.


- Nếu <i>n</i>5, ta sẽ chứng minh rằng hệ tương ứng trong trường hợp này sẽ vơ nghiệm.
Ta có


( )


( ) ( )


<i>n</i>


<i>u</i> <i>n</i> <i>u</i> <i>n</i> <i>n</i> <i>n</i>


<i>u</i> <i>n v</i> <i>v</i>




   


          



2 1


1 3 2 1 2 1 2


3 2 9 8 1


3 3 3 .


Tương tự, ta cũng có <i>u</i><i>n</i>21


3 . Đặt min{ , }


<i>n</i>


<i>w</i> <i>u v</i>  2 1 <i>w</i>2


3 . Suy ra


(mod )
<i>n</i>


 
1


2 1 0 9 . Bằng cách thử trực tiếp, ta có được <i>n</i> 1 0(mod )6 hay *
,


</div>
<span class='text_page_counter'>(12)</span><div class='page_container' data-page=12>

11
Từ đó, ta được 3<i>u</i>  2 3<i>v</i> (2<i>k</i>1 2)( <i>k</i> 1 4)( 2<i>k</i>4<i>k</i> 1).



Dễ dàng thấy rằng 42<i>k</i> 4<i>k</i>    1 1 1 1 0(mod )3 nhưng 42<i>k</i> 4<i>k</i> 1 2<i>k</i>4<i>k</i>  1 0(mod )9
nên 3<i>w1</i> là ước của (2<i>k</i>1 2)( <i>k</i>1). Suy ra 3<i>w</i>12<i>k</i> 1 3<i>k</i> <i>n</i>2 1 <i>w</i> 1 <i>n</i>1<i>n</i>11


3 6


Dễ thấy không tồn tại số nguyên dương <i>n</i> nào mà <i>n</i> 1 0(mod )6 và 5<i>n</i>11 nên trong
trường hợp này, hệ khơng có nghiệm.


</div>
<span class='text_page_counter'>(13)</span><div class='page_container' data-page=13>

12
<b>Bài 6. </b>


<i><b>Có n học sinh ngồi quanh một bàn tròn, trong tay mỗi học sinh có một số kẹo sao cho tổng số </b></i>
<i><b>kẹo của tất cả các học sinh này là một bội số của n. Ta thực hiện một quy tắc chuyển kẹo như </b></i>
<i><b>sau: nếu có một học sinh có số kẹo lớn hơn số kẹo của người bạn bên tay phải mình thì ta sẽ </b></i>
<i><b>lấy đi của người đó chuyển sang cho người bạn bên tay phải. </b></i>


<i><b>Chứng minh rằng với quy tắc trên, sau một số hữu hạn các bước với cách chuyển thích hợp, </b></i>
<i><b>số kẹo của mỗi học sinh đều bằng nhau.</b></i>


<b>Lời giải. </b>


Xuất phát từ một học sinh nào đó, lần lượt theo chiều ngược chiều kim đồng hồ, ta đánh số các
em bởi 1 2 3, , ,...,<i>n</i>. Khi đó, với mỗi <i>i</i>1,<i>n</i>, ngồi ngay bên phải em i là em <i>i</i>1.


(với quy ước em <i>n</i>1 là em 1).


Với *


<i>t</i> , ta định nghĩa thời điểm t là thời điểm nằm giữa lần chuyển kẹo thứ t và thứ <i>t</i>1.


Với mỗi <i>i</i>1,<i>n</i>, kí hiệu x là số kẹo của em i tại mỗi thời điểm và gọi <i>x ti</i>( ) là số kẹo của
em i tại thời điểm t. Ta gọi mỗi bộ ( ,<i>x x x</i><sub>1</sub> <sub>2</sub>, <sub>3</sub>,...,<i>x<sub>n</sub></i>) là một trạng thái.


Xét đại lượng
<i>n</i>


<i>i</i>
<i>i</i>


<i>F</i> <i>x</i>



<sub></sub>

2


1


và xét một thời điểm t tùy ý. Giả sử, ở lần chuyển kẹo thứ <i>t</i>1, người
thực hiện việc chuyển kẹo là em i. Khi đó:




( ) ( ), ( ) ( ), ;


( ) ( ) , ( ) ( )


<i>i</i> <i>i</i> <i>j</i> <i>j</i>


<i>i</i> <i>i</i> <i>i</i> <i>i</i>


<i>x t</i> <i>x</i> <i>t x t</i> <i>x t</i> <i>j</i> <i>i i</i>



<i>x t</i> <i>x t</i> <i>x</i> <i>t</i> <i>x</i> <i>t</i>




 


     


     


1


1 1


1 1


1 1 1 1


Do đó <i>F t</i>( 1)<i>F t</i>( )

<i>x t<sub>i</sub></i>( )1

2

<i>x<sub>i</sub></i><sub></sub><sub>1</sub>( )<i>t</i> 1

2

<i>x t<sub>i</sub></i>( )

2

<i>x<sub>i</sub></i><sub></sub><sub>1</sub>( )<i>t</i>

2 2

<i>x<sub>i</sub></i><sub></sub><sub>1</sub>( )<i>t</i>  1 <i>x t<sub>i</sub></i>( )

0
Và <i>F t</i>( 1)<i>F t</i>( )0<i>x t<sub>i</sub></i>( )<i>x<sub>i</sub></i><sub></sub><sub>1</sub>( )<i>t</i> 1<b> </b>(*). (<i>F t</i>( ) kí hiệu giá trị của F tại thời điểm t).
Như vậy, giá trị của F không tăng trong quá trình chuyển kẹo và F chỉ nhận cùng một giá trị ở 2
thời điểm liên tiếp là <i>t t</i>, 1 khi và chỉ khi ở lần chuyển kẹo thứ t + 1, em chuyển kẹo có nhiều
hơn em nhận kẹo đúng 1 chiếc kẹo. Hơn nữa, do mỗi trạng thái cho ta một nghiệm tự nhiên của
phương trình


... <i><sub>n</sub></i>
<i>x</i><sub>1</sub><i>x</i><sub>2</sub> <i>x</i><sub>3</sub> <i>x</i> <i>n</i>


</div>
<span class='text_page_counter'>(14)</span><div class='page_container' data-page=14>

13


Xảy ra một trong hai trường hợp sau:


<b>* Trường hợp 1:</b> F nhận mỗi giá trị chỉ tại một số hữu hạn thời điểm liên tiếp .


Trong trường hợp này, sau một số hữu hạn lần chuyển kẹo, F sẽ nhận giá trị nhỏ nhất có thể và
kể từ thời điểm (đầu tiên) F nhận giá trị nhỏ nhất đó, việc chuyển kẹo chỉ có thể thực hiện thêm
một số hữu hạn lần. Khi việc chuyển kẹo không thể thực hiện được, ta phải có


... <i>n</i>


<i>x</i><sub>1</sub><i>x</i><sub>2</sub> <i>x</i><sub>3</sub>  <i>x</i> <i>x</i><sub>1</sub> hay <i>x</i>1 <i>x</i>2 <i>x</i>3 ...<i>xn</i>.
Khẳng định của bài toán được chứng minh.


<b>* Trường hợp 2 :</b> Tồn tại một giá trị mà F nhận giá trị đó tại vô hạn thời điểm liên tiếp .
Giả sử <i>F t</i>( )<i>F t</i>( 1), <i>t</i> <i>t</i><sub>0</sub>. Khi đó, theo (*), kể từ thời điểm <i>t</i><sub>0</sub>, mỗi lần chuyển kẹo chỉđược
thực hiện bởi em i mà tại thời điểm đó <i>x<sub>i</sub></i><i>x<sub>i</sub></i><sub></sub><sub>1</sub> 1 (**)


Do kể từ thời điểm <i>t</i><sub>0</sub> ta sẽ nhận được vô số trạng thái mà chỉ có hữu hạn trạng thái đơi một
khác nhau (chứng minh trên) nên phải tồn tại ít nhất một trạng thái xuất hiện tối thiểu 2 lần.
Xét một trạng thái trong số các trạng thái như vậy, gọi là A.


Giả sử A xuất hiện tại thời điểm <i>t</i><sub>1</sub><i>t</i><sub>0</sub>; gọi k là số nguyên dương nhỏ nhất sao cho tại thời điểm
<i>t</i><sub>1</sub><i>k</i>, A lại xuất hiện một lần nữa.


Gọi (C) là quá trình chuyển kẹo kể từ thời điểm <i>t</i><sub>1</sub> đến thời điểm <i>t</i><sub>1</sub><i>k</i>.


Do (C) khởi đầu và kết thúc bởi cùng một trạng thái nên trong q trình đó nếu em i đã nhận kẹo
ở một lần chuyển kẹo nào đó thì sau đó i phải thực hiện việc chuyển kẹo cho bạn ngồi ngay bên
phải mình. Từ đó, do các em ngồi quanh bàn trịn, suy ra trong (C), mỗi em đều phải thực hiện
việc chuyển kẹo ít nhất một lần.



Hơn nữa, do tổng số kẹo của các em là một bội của n nên với ( ,<i>x x</i>1 2,...,<i>xn</i>) là một trạng thái tùy
ý, ta phải có hoặc <i>x</i><sub>1</sub> <i>x</i><sub>2</sub> <i>x</i><sub>3</sub> ...<i>x<sub>n</sub></i>, hoặc tồn tại <i>i</i> <i>j</i>

1 2 3, , ,...,<i>n</i>

sao cho <i>x<sub>i</sub></i><i>x<sub>j</sub></i> 2.
Suy ra, mỗi trạng thái trong (C) đều có tính chất: tồn tại <i>i</i> <i>j</i>

<sub></sub>

1 2 3, , ,...,<i>n</i>

<sub></sub>

sao cho <i>x<sub>i</sub></i><i>x<sub>j</sub></i> 2.
Gọi <i>m</i> là số ngun dương nhỏ nhất sao cho trong (C) có ít nhất một trạng thái mà ở trạng thái đó
tồn tại <i>i</i>

<sub></sub>

1 2 3, , ,...,<i>n</i>

<sub></sub>

sao cho <i>x<sub>i</sub></i><i>x<sub>i m</sub></i><sub></sub> 2. Gọi B là trạng thái có tính chất như vậy và ở gần A
nhất (có thể <i>B</i><i>A</i>). Giả sử B xuất hiện (trong (C)) tại thời điểm <i>t</i><sub>2</sub> <i>t</i><sub>1</sub> .


</div>
<span class='text_page_counter'>(15)</span><div class='page_container' data-page=15>

14
- Khả năng 1: <i>m</i>1, ta lại xét tiếp các trường hợp:


+ Trường hợp 1.1: <i>B</i> <i>A</i>. Khi đó, ở lần đầu tiên (trong (C)) chuyển kẹo cho em <i>i</i>1, em i sẽ có
nhiều hơn i + 1 ít nhất 2 chiếc kẹo; mâu thuẫn với (2).


+ Trường hợp 1.2 : <i>B</i><i>A</i>. Khi đó, ở trạng thái A, ta có <i>x<sub>i</sub></i><i>x<sub>i</sub></i><sub></sub><sub>1</sub> 1. Do đó, để trở lại được
trạng thái A, sau thời điểm <i>t</i><sub>2</sub>, i phải chuyển kẹo cho em <i>i</i>1 ít nhất 1 lần và ở lần đầu tiên trong
các lần như vậy i sẽ có nhiều hơn <i>i</i>1ít nhất 2 chiếc kẹo; mâu thuẫn với (2).


- Khả năng 2: <i>m</i>1. Khi đó, từ định nghĩa của m suy ra số kẹo của các em


, ,..., ,


<i>i i</i>1 <i>i</i><i>m</i>1<i>i</i><i>m</i>(trong trạng thái B) phải thỏa mãn


, ...


<i>i</i> <i>i</i> <i>i</i> <i>i</i> <i>i m</i> <i>i m</i>


<i>x</i>  <i>x</i>11 <i>x</i>1 <i>x</i>1  <i>x</i> 1 <i>x</i> 1.



+ Trường hợp 2.1: <i>B</i> <i>A</i>. Trong trường hợp này, tại lần đầu tiên có một trong các em
, , ,...,


<i>i</i>1<i>i i</i>1 <i>i</i><i>m</i> thực hiện việc chuyển kẹo; em chuyển kẹo chỉ có thể hoặc là <i>i</i>1hoặc i
hoặc<i>i</i><i>m</i>1 hoặc <i>i</i><i>m</i>.


 Nếu <i>i</i>1 thực hiện việc chuyển kẹo thì sau bước chuyển đó, ta sẽ có trạng thái mà trong
đó <i>x<sub>i</sub></i><i>x<sub>i</sub></i><sub></sub><sub>1</sub>2.


 Nếu i thực hiện việc chuyển kẹo thì sau bước chuyển đó, ta sẽ có trạng thái mà trong
<i>i</i> <i>i m</i>


<i>x</i><sub></sub><sub>1</sub><i>x</i><sub></sub> 2.


 Nếu <i>i</i><i>m</i>1thực hiện việc chuyển kẹo thì sau bước chuyển đó, ta sẽ có trạng thái mà
<i>i</i> <i>i m</i>


<i>x</i> <i>x</i><sub> </sub><sub>1</sub> 2.


 Nếu <i>i</i><i>m</i> thực hiện việc chuyển kẹo thì sau bước chuyển đó, ta sẽ có trạng thái mà
<i>i m</i> <i>i m</i>


<i>x</i><sub> </sub><sub>1</sub><i>x</i><sub></sub> 2.


Trong cả 4 tình huống trên, ta đều nhận được điều mâu thuẫn với tính nhỏ nhất của m.
+ Trường hợp 2.2 : <i>B</i><i>A</i>. Khi đó, ở trạng thái A, các số ,<i>x x<sub>i</sub></i> <i><sub>i</sub></i>1,...,<i>x<sub>i m</sub></i> 1,<i>x<sub>i m</sub></i> sẽ không thỏa
mãn quan hệ (∗). Vì thế, để trở lại trạng thái A, sau thời điểm <i>t</i>2 phải có thời điểm mà ít nhất một
trong các sốđó thay đổi. Điều này chỉ có được khi có ít nhất một trong các em


, , ,...,



<i>i</i>1<i>i i</i>1 <i>i</i><i>m</i> thực hiện việc chuyển kẹo. Lập luận hoàn toàn tương tự như trường hợp 2.1,
ta sẽ nhận được những điều mâu thuẫn với tính nhỏ nhất của <i>m</i>.


</div>
<span class='text_page_counter'>(16)</span><div class='page_container' data-page=16>

<b>Vietnam TST 2012 – L</b>

<b>ờ</b>

<b>i gi</b>

<b>ả</b>

<b>i và bình lu</b>

<b>ậ</b>

<b>n </b>



<b>Trần Nam Dũng & K0 </b>


Kỳ thi chọn đội tuyển Việt Nam tham dự IMO 2012 đã diễn ra trong 2 ngày 16 và
17/04/2012 tại Hà Nội. Mỗi ngày thí sinh phải giải quyết 3 bài tốn trong vòng 4 giờ 30


phút. Theo đánh giá chung, đề thi năm nay thuộc loại khó. Vềphân mơn, 6 bài tốn được
phân bốnhư sau:


<b>Bài 1.</b> Hình học phẳng (Quỹtích và điểm cốđịnh)


<b>Bài 2.</b> Tổ hợp (Phủ)


<b>Bài 3.</b> Số học (Hệ thặng dư)
<b>Bài 4.</b> Số học (Dãy số)


<b>Bài 5.</b>Đại số (Bất đẳng thức)


<b>Bài 6.</b> Tổ hợp (Lý thuyết đồ thị)


Nếu đi sâu vào lời giải thì có thể thấy bài 4 là một bài toán thuần túy đại số. Bài 3 là bài
số học nhưng mang đậm chất tổ hợp. Như thế, có thể thấy đề thi năm nay quá nặng về
Đại số và Tổ hợp, phần Số học và Hình yếu, dù bài hình là một bài tốn tốt.


Vềđộ khó, chỉ có bài 4 là dễ chịu hơn cả, cịn lại 5 bài đều là những bài tốn khó, đều là


thách thức đáng kểđối với các thí sinh.


Một đặc điểm nữa trong đềthi năm nay là có nhiều bài tốn sử dụng ý tưởng các định lý
mạnh như định lý <i>Cauchy-Davenport </i>(bài 3), định lý <i>Dirac</i>, định lý <i>Tutte </i>(bài 6). Điều
này một mặt là tích cực vì hướng học sinh đến việc làm quen với những vấn đềcơ sở của
toán cao cấp, mặt khác cũng tạo những bất lợi cho các học sinh chưa có điều kiện làm
quen với những kiến thức này. Đây là điều mà những người dẫn dắt phong trào HSG của
Việt Nam phải thảo luận kỹđể có một định hướng đúng.


Dưới đây chúng tơi trình bày lời giải chi tiết các bài toán của Vietnam TST 2012 cùng
các bình luận.


Bài viết này được hoàn thành với sự tham gia trực tiếp của các bạn: <i>Võ Quốc Bá Cẩn </i>


<i>(ĐH Y Cần Thơ) </i>và Lê Phúc L<i>ữ(ĐH FPT), Lê Hồng Quý c</i>ũng như sự tham gia gián tiếp
của thầy <i>Nguyễn Chu Gia Vượng (Viện Toán học), các thành viên mathscope.org </i>như


</div>
<span class='text_page_counter'>(17)</span><div class='page_container' data-page=17>

<b>Bài 1. </b>


<i><b>Trên m</b><b>ặ</b><b>t ph</b><b>ẳng, cho đườ</b><b>ng tròn </b></i>( )<i><b>O </b><b>và hai điể</b><b>m c</b><b>ố</b></i> <i><b>đị</b><b>nh </b><b>B C </b></i>, <i><b>trên đườ</b><b>ng tròn này </b></i>
<i><b>sao cho BC </b><b>không là đườ</b><b>ng kính c</b><b>ủ</b><b>a </b></i>( )<i><b>O . G</b><b>ọ</b><b>i A là m</b><b>ột điểm di động trên đườ</b><b>ng </b></i>
<i><b>tròn </b></i>( )<i><b>O và A không trùng v</b><b>ới hai điể</b><b>m </b><b>B C . G</b></i>, <i><b>ọ</b><b>i </b><b>D K J l</b></i>, , <i><b>ần lượt là trung điể</b><b>m c</b><b>ủ</b><b>a </b></i>


, ,


<i><b>BC CA AB và </b></i> <i><b>E M N l</b></i>, , <i><b>ần lượ</b><b>t là hình chi</b><b>ế</b><b>u vng góc c</b><b>ủ</b><b>a </b></i> <i><b>A B C trên </b></i>, ,


, ,


<i><b>BC DJ DK . </b></i>



<i><b>Ch</b><b>ứ</b><b>ng minh r</b><b>ằ</b><b>ng các ti</b><b>ế</b><b>p tuy</b><b>ế</b><b>n t</b><b>ạ</b><b>i </b></i> <i><b>M N c</b></i>, <i><b>ủa đườ</b><b>ng tròn ngo</b><b>ạ</b><b>i ti</b><b>ế</b><b>p tam giác </b></i>
<i><b>EMN luôn c</b><b>ắ</b><b>t nhau t</b><b>ại điể</b><b>m T c</b><b>ố</b><b>định khi điể</b><b>m A </b><b>thay đổ</b><b>i trên </b></i>( )<i><b>O . </b></i>


<b>Lời giải. </b>


Đây là một bài toán khá thú vị với phát biểu nhẹ nhàng, cấu hình khơng q phức tạp và
gợi ra nhiều ý tưởng nhưng việc xử lí khơng dễ, quan trọng là phải đốn được điểm cố
định được nêu ra. Dưới đây chúng ta sẽ xem xét một sốhướng tiếp cận và xử lí mở rộng
của bài toán này.


<i><b>Cách 1. (s</b><b>ử</b><b> d</b><b>ụng hàng điểm điề</b><b>u hòa và t</b><b>ứ</b><b>giác điề</b><b>u hòa) </b></i>


Gọi H là trực tâm của tam giác ABC. Ta xét trường hợp H nằm trong tam giác, các trường
hợp còn lại chứng minh tương tự.


Trước hết, ta chứng minh rằng T nằm trên đường thẳng OD.


</div>
<span class='text_page_counter'>(18)</span><div class='page_container' data-page=18>

Đường thẳng qua H, song song với BC cắt đường thẳng OD tại điểm S. Do <i>HSD</i>900
nên S cũng thuộc đường trịn đường kính HD. Gọi X là hình chiếu của E lên AD thì X
cũng thuộc đường trịn này.


Ta sẽ chứng minh các tứ giác<i>DMSN XMEN</i>, là các tứgiác điều hòa.


Thật vậy, do <i>HS BC</i> và D là trung điểm của BC nên theo tính chất vềchùm điều hịa, ta
có (<i>HS HD HC HB</i>, , , ) 1 hay tứ giác <i>DMSN</i>tương ứng là tứ giác điều hịa. Theo tính
chất của tứgiác điều hịa, ta có T nằm trên đường thẳng DO.


Dễ thấy tứ giác<i>DEJK</i>là hình thang cân nên nên <i>ENK</i> <i>EMJ g g</i>( . ).



Suy ra <i>EM</i> <i>EJ</i> <i>AB</i>


<i>EN</i>  <i>EK</i>  <i>AC</i>. Hơn nữa,


sin sin sin


sin sin sin


<i>XM</i> <i>XNM</i> <i>XDM</i> <i>DAC</i> <i>AB</i>


<i>XN</i> <i>XMN</i> <i>XDN</i> <i>DAB</i> <i>AC</i>


  


   


   .


Do đó, <i>EM</i> <i>AB</i>


<i>EN</i>  <i>AC</i> hay tứ giác XMEN điều hịa. Ta có được T nằm trên EX hay T chính


là giao điểm của EX và AO.


Ta sẽ chứng minh rằng khoảng cách từ T đến D khơng đổi.
Gọi <i>B</i> là hình chiếu của B trên AC. Do <i>AHX</i> <i>ADE</i> nên


</div>
<span class='text_page_counter'>(19)</span><div class='page_container' data-page=19>

hay tứ giác <i>CDXB</i> nội tiếp. Suy ra <i>DXC</i> <i>DB C</i>  <i>DCA</i><i>DX DA</i> <i>DC</i>2.


Theo định lí Thales thì



2


<i>AE DX</i> <i>AD DX</i> <i>DC</i>


<i>DT</i>


<i>AX</i> <i>AH</i> <i>AH</i>


 


   .


Dễ thấy <i>DC AH</i>, đều không đổi nên độdài đoạn <i>DT</i> không đổi hay T là điểm cốđịnh.


Ta có đpcm.


<i><b>Cách 2. </b><b>(dùng phương tích, trục đẳng phương)</b></i>


Gọi <i>R S</i>, lần lượt là trung điểm của <i>DB DC</i>, thì R, S lần lượt là tâm đường tròn ngoại


tiếp các tam giác <i>BMD CND</i>, . Ta có TM <i>TN</i>, 1 1 1


2 4 2


<i>MR</i> <i>DB</i> <i>BC</i> <i>DC</i><i>NS</i> và
bằng biến đổi góc, ta thu được <i>TMR</i> <i>TNS</i> hay <i>TMR</i> <i>TNS c g c</i>( . . ).


Suy ra <i>TR</i><i>TS</i> hay T nằm trên đường trung trực của BC.



Gọi X là tâm đường trịn ngoại tiếp tam giác HBC thì X cốđịnh. Ta sẽ chứng minh T nằm
trên trục đẳng phương của đường tròn (S) và (X). Gọi U là trung điểm của OD. Ta thấy




2 2 2 2


/( ) /( )


2 2 2 2 2 2 2 2 2 2 2 2


2 <sub>2</sub> <sub>2</sub> <sub>2</sub> <sub>2</sub>


2


<i>T</i> <i>X</i> <i>T</i> <i>S</i> <i>TX</i> <i>XC</i> <i>TS</i> <i>SC</i>


<i>TX</i> <i>TS</i> <i>XD</i> <i>CD</i> <i>SC</i> <i>TD</i> <i>SD</i> <i>XD</i> <i>CD</i> <i>SC</i> <i>TC</i> <i>XD</i>


<i>TD</i> <i>XD</i> <i>TC</i> <i>XD</i> <i>CD</i> <i>TD XD</i> <i>DS</i> <i>DU DT</i>


<i></i> <i></i>    


           


</div>
<span class='text_page_counter'>(20)</span><div class='page_container' data-page=20>

Điều này tương đương với tam giác <i>TSU</i> vuông tại S. Hơn nữa, ta thấy


0 0 0


0



90 90 180


180


<i>TSU</i> <i>STU</i> <i>SUT</i> <i>RTS</i> <i>BXC</i>


<i>MTN</i> <i>MIN</i>


         


   


Đẳng thức cuối đúng nên suy ra <i>T n</i>ằm trên trục đẳng phương của (S) và (X). Do hai


đường tròn này cốđịnh nên trục đẳng phương của chúng cũng cốđịnh. T là giao điểm của
hai đường thẳng cốđịnh nên T là điểm cốđịnh. Ta có đpcm.


<b>Bình luận. </b>


So sánh với các bài tốn hình ở vị trí bài 1 nhiều năm trở lại đây thì bài này khó hơn hẳn.


Hướng giải theo con đường hình học thuần túy bắt buộc phải kẻ thêm khá nhiều đường
phụvà điều này sẽ khiến nhiều bạn phải bỏ cuộc. Có một cách giải quyết trong trường
hợp này là dùng phương pháp tọa độ do giả thiết cũng tương đối thuận lợi. Đôi khi cách


tiếp cận bằng đại số cũng đem lại hiệu quả cao. Chúng ta sẽ cùng tìm hiểu một cách làm
bằng biến đổi vector như sau:


Ta thấy các điểm M, N chính là trung điểm của các đường cao tương ứng của tam giác


<i>ABC. </i>Các điểm <i>M N E H D</i>, , , , cùng thuộc đường trịn đường kính HD. Gọi R là điểm đối
xứng với O qua đường thẳng BC và S là giao điểm của đường tròn ngoại tiếp tam giác
<i>BCR v</i>ới đường thẳng OD. Gọi F là chân đường cao kẻ từ C đến AB và T là trung điểm
của DS. Dễ thấy T là điểm cốđịnh. Ta tính được


cos 2 cos 2 cos cos


</div>
<span class='text_page_counter'>(21)</span><div class='page_container' data-page=21>



0 0 0 0 0


90 90 90 90 90


<i>FCS</i> <i>FCR</i> <i>A</i> <i>B</i> <i>A</i> <i>B</i>


            và




0 0 0


90 90 90


<i>DCS</i> <i>RCD</i> <i>A</i> <i>A</i>


        nên


2cos
<i>BC</i>
<i>CS</i>



<i>A</i>


 .


Do <i>T I N</i>, , lần lượt là trung điểm của các đoạn <i>DS HD CF</i>, , nên ta có:


2<i>NT</i> <i>CS</i><i>FD</i>, 2<i>NI</i> <i>CD</i><i>FH</i>
     


.


Suy ra: 4NT NI 

<i>CS</i><i>FD CD</i>



<i>FH</i>

<i>CS CD</i> <i>CS FH</i> <i>FD CD</i> <i>FD FH</i>
            


.


Ta tính được


2


2 2 2


sin
4


<i>BC</i>


<i>CS CD</i> <i>CD</i>  <i>R</i>  <i>A</i>
 





2 2 2


cos cos 2 sin cos 2


<i>FD CD</i> <i>DF DC</i>  <i>CDF</i> <i>CD</i> <i>B</i><i>R</i> <i>A</i> <i>B</i>
 


.


2


cos 2 cos cos sin 2sin cos sin cos


2
<i>BC</i>


<i>FD FH</i> <i>FD FH</i>  <i>DFH</i>   <i>R</i> <i>A</i> <i>B</i> <i>B</i><i>R</i>  <i>A</i> <i>B</i> <i>B</i> <i>A</i>


 


.


2


cos 2 cos cos sin( )


2 cos


2 sin cos sin( )


<i>BC</i>


<i>CS FH</i> <i>CS FH</i> <i>FCS</i> <i>R</i> <i>A</i> <i>B</i> <i>A</i> <i>B</i>


<i>A</i>


<i>R</i> <i>A</i> <i>B</i> <i>A</i> <i>B</i>


        
  
 
Do đó





2 2 2


2 2 2


4 sin sin cos 2 2sin cos sin cos 2 sin cos sin( )


2 sin cos sin cos sin cos sin( ) 0


<i>NT NI</i> <i>R</i> <i>A</i> <i>A</i> <i>B</i> <i>A</i> <i>B</i> <i>B</i> <i>A</i> <i>A</i> <i>B</i> <i>A</i> <i>B</i>


<i>R</i> <i>A</i> <i>B</i> <i>A</i> <i>B</i> <i>B</i> <i>A</i> <i>A</i> <i>B</i>



    


    





Từđó suy ra <i>NT</i> <i>NI</i> hay TN là tiếp tuyến của đường tròn ngoại tiếp tam giác <i>MNE</i>.
Chứng minh tương tự, ta có TM là tiếp tuyến của đường tròn ngoại tiếp tam giác này.


Do đó, hai tiếp tuyến kẻ từ M và N của đường tròn ngoại tiếp tam giác MNE cắt nhau tại
<i>T </i>là điểm cốđịnh, ta có đpcm.


Bài tốn này có nội dung tương tự với mở rộng của bài 2, IMO 2009:


</div>
<span class='text_page_counter'>(22)</span><div class='page_container' data-page=22>

Một kết quả quen thuộc khác cũng có được từ bài tốn này là: tiếp tuyến tại H của đường
tròn ngoại tiếp tam giác EMN cắt AB, AC tại hai điểm đối xứng nhau qua H.


Cách giải thứ 2 ở trên khá thuần túy và đẹp mắt, có thể thay việc chứng minh tam giác
bằng nhau ở trên bằng phép quay. Trong trường hợp tam giác tù (tại B hoặc C), hình vẽ


và vịtrí các điểm cũng có nhiều thay đổi, chúng ta có thể sử dụng góc định hướng để có
một lời giải tốt hơn!


<i>Lời giải và bình luận của bài 1 được thực hiện bởi Lê Phúc Lữ, dựa trên cách giải của </i>


<i>Hoàng Đỗ Kiên, Phan Đức Minh, Lê Thanh Tú và bản thân người bình luận. </i>


<b>Bài 2.</b>


<i><b>Trên m</b><b>ột cánh đồ</b><b>ng hình ch</b><b>ữ</b><b> nh</b><b>ật kích thướ</b><b>c m n</b></i> <i><b> ơ vuông g</b><b>ồ</b><b>m m hàng và n c</b><b>ộ</b><b>t, </b></i>


<i><b>người ta đặ</b><b>t m</b><b>ộ</b><b>t s</b><b>ố</b></i> <i><b>máy bơm nướ</b><b>c vào các ô vuông. Bi</b><b>ế</b><b>t r</b><b>ằ</b><b>ng m</b><b>ỗi máy bơm nướ</b><b>c có </b></i>
<i><b>th</b><b>ể</b><b> t</b><b>ưới nướ</b><b>c khơng nh</b><b>ữ</b><b>ng cho ơ vng ch</b><b>ứ</b><b>a nó và các ơ vng có chung c</b><b>ạ</b><b>nh v</b><b>ớ</b><b>i ơ </b></i>
<i><b>đó mà c</b><b>ịn có th</b><b>ể</b><b>tướ</b><b>i cho các ơ vng cùng c</b><b>ộ</b><b>t v</b><b>ới nó và cách nó đúng mộ</b><b>t ơ vng. </b></i>
<i><b>Tìm s</b><b>ố</b><b> nh</b><b>ỏ</b><b> nh</b><b>ất các máy bơm nướ</b><b>c c</b><b>ần đặt để</b><b>các máy bơm đó có thể</b><b>tướ</b><b>i h</b><b>ế</b><b>t c</b><b>ả</b><b> cánh </b></i>
<i><b>đồ</b><b>ng trong hai </b><b>trườ</b><b>ng h</b><b>ợ</b><b>p: </b></i>


<i><b> 1) </b>m</i>4<i><b>. </b></i>
<i><b> 2) </b>m</i>3<i><b>. </b></i>


<b>Lời giải. </b>


1) Với <i>m</i>4, ta sẽ chứng minh rằng số máy bơm nước nhỏ nhất thỏa mãn yêu cầu đề


bài là n.


Điều kiện đủ là hiển nhiên với cách đặt ở mỗi cột 1 máy bơm ở hàng thứ hai như sau:




X X X … X X X





Chú ý là một máy bơm chỉ tưới được tối đa 6 ô nên điều kiện cần rõ ràng đúng với <i>n</i>1
và <i>n</i>2.


Ta chứng minh điều kiện cần bằng phản chứng. Giả sử tồn tại n sao cho cánh đồng kích


</div>
<span class='text_page_counter'>(23)</span><div class='page_container' data-page=23>

Xét cánh đồng kích thước 4<i>n</i><sub>0</sub>. Theo định nghĩa của <i>n</i><sub>0</sub>, tồn tại một cách xếp <i>k</i><i>n</i><sub>0</sub>



máy bơm để tưới hết cánh đồng. Vì số máy bơm nhỏ hơn số cột nên phải tồn tại ít nhất
một cột không chứa máy bơm (ta gọi là cột trống).


<b>Bước 1.</b> Ta thấy cột trống không thể là cột ở biên vì nếu cột trống là cột biên, chẳng hạn
là cột thứ nhất thì để tưới được các ô ở cột trống, cột thứ hai phải chứa 4 máy bơm. Khi
đó, bằng cách thêm một máy bơm vào cột 3 hàng 2 (nếu ô này chưa có máy bơm), ta thấy


0 2


<i>n</i>  cột còn lại (bỏđi cột 1 và 2) sẽđược tưới bởi <i>k</i>    4 1 <i>k</i> 3 <i>n</i><sub>0</sub>2 máy bơm,
mâu thuẫn với cách chọn <i>n</i><sub>0</sub>.


<b>Bước 2.</b> Vì cột trống khơng nằm ở biên, ta xét cột trống đầu tiên từ bên trái sang. Ta giả


sử cột này là cột <i>j</i>. Để tưới được các ô ở cột trống này, tổng cộng ở hai cột hai bên cột
trống này phải có ít nhất 4 máy bơm (*).


Xét các trường hợp sau:


i) Cột <i>j</i>1 chứa ít nhất 2 máy bơm. Khi đó do các cột từ1 đến <i>j</i>2 đều không
trống nên j cột đầu chứa ít nhất j máy bơm. Suy ra <i>n</i><sub>0</sub> <i>j</i> cột sau chứa nhiều
nhất <i>k</i> <i>j</i> máy bơm. Vì được ngăn cách bởi 1 cột trống nên rõ ràng các máy


bơm này bơm được cho tất cả các ơ của cách đồng kích thước 4

<i>n</i><sub>0</sub> <i>j</i>

. Vì
0


<i>k</i> <i>j</i> <i>n</i>  <i>j</i> nên điều này mâu thuẫn với cách chọn <i>n</i><sub>0</sub>.


ii) Cột <i>j</i>1 chỉ chứa 1 máy bơm, khi đó, do (*), cột <i>j</i>1 phải chứa ít nhất 3



máy bơm. Khi đó, do <i>j</i>1 cột đầu chứa ít nhất <i>j</i>    1 0 3 <i>j</i> 2 máy bơm


nên <i>n</i><sub>0</sub>  <i>k</i> <i>j</i> 2, tức là bên cạnh cột <i>j</i>1 cịn ít nhất 2 cột nữa. Bây giờ,
bằng cách thêm vào cột 2 hàng <i>j</i>2 một máy bơm nếu cần, ta thấy cánh đồng
gồm <i>n</i><sub>0</sub> 

<i>j</i> 1

cột còn lại sau khi bỏ đi <i>j</i>1 cột đầu có thể được tưới bởi


2

1 1


<i>k</i> <i>j</i>    <i>k</i> <i>j</i> máy bơm. Vì <i>k</i>  <i>j</i> 1 <i>n</i><sub>0</sub> 

<i>j</i> 1

nên ta nhận


được mâu thuẫn với cách chọn <i>n</i><sub>0</sub>.


Như vậy điều kiện cần được chứng minh. Ta có kết luận: Với cánh đồng 4<i>n</i>, cần ít
nhất n máy bơm để tưới nước thỏa mãn yêu cầu bài toán.


2) Ta sẽ chứng minh rằng số máy bơm ít nhất để tưới được cánh đồng 3<i>n</i> là
1


4
<i>n</i>
<i>n</i>  


 


 . Trước hết ta chứng minh điều kiện đủ. Với <i>n</i>5 điều kiện đủ là hiển nhiên,


</div>
<span class='text_page_counter'>(24)</span><div class='page_container' data-page=24>

X X
X
X



Từđây dễ dàng chỉ ra cách đặt máy bơm cho <i>n b</i>ất kỳ. Chẳng hạn với <i>n</i>20, ta đặt 16


máy bơm như sau.


X X X X X X


X X X X X


X X X X X


Bây giờ ta chứng minh điều kiện cần. Chú ý là một máy bơm nước chỉ có thể tưới được
tối đa 5 ơ nên với <i>n</i>1, 2, ta thấy rằng cần phải có ít nhất <i>n </i>máy bơm nước mới có thể
tưới được tất cả các ô của cánh đồng 3<i>n</i>.


Tương tự phần 1), ta sẽ chứng minh bằng phương pháp phản chứng.


Đặt ( ) 1 , 1, 2,3,...
4


<i>n</i>


<i>f n</i>  <i>n</i>    <i>n</i>


 


  .


Giả sử tồn tại số nguyên dương <i>n </i>sao cho cánh đồng kích thước 3<i>n</i> có thể được tưới
bởi <i>k</i> <i>f n</i>

 

máy bơm nước. Gọi <i>n</i><sub>0</sub> là số nhỏ nhất như vậy. Theo chú ý ở trên <i>n</i><sub>0</sub>3.

Do <i>f n</i>

 

<sub>0</sub> <i>n</i><sub>0</sub> nên từđây ta suy ra <i>k</i><i>n</i><sub>0</sub>. Như vậy phải có ít nhất 1 cột trống.


Lý luận tương tựnhư ở phần 1, ta thấy cột trống không thểở biên. Xét cột trống đầu tiên
từ bên trái sang. Giả sửđó là cột j. Khi đó, để tưới các ơ của cột <i>j, hai c</i>ột kề bên cột <i>j </i>
phải chứa ít nhất 3 máy bơm nước.


Xét các trường hợp sau:


i) Cột <i>j</i>1 chứa ít nhất 2 máy bơm nước. Khi đó j cột đầu chứa ít nhất j máy bơm
nước (do các cột từ 1 đến <i>j</i>2 chứa ít nhất 1, cột <i>j</i>1 chứa ít nhất 2). Suy ra <i>n</i><sub>0</sub> <i>j</i>
cột cịn lại chứa không quá <i>k</i> <i>j</i> máy bơm nước và các máy bơm này tưới hết các ô của


cánh đồng kích thước 3

<i>n</i><sub>0</sub> <i>j</i>

.


Ta có 0 0


0 0 0 0


1 1


( ) ( )


4 4


<i>n</i> <i>n</i> <i>j</i>


<i>k</i> <i>j</i> <i>f n</i>  <i>j</i> <i>n</i>  <i>j</i>   <i>n</i>  <i>j</i>     <i>f n</i>  <i>j</i>


   



    nên từđây


</div>
<span class='text_page_counter'>(25)</span><div class='page_container' data-page=25>

ii) Cột <i>j</i>1 chỉ chứa 1 máy bơm nước. Khi đó cột <i>j</i>1 phải chứa ít nhất 2 máy


bơm nước. Như thế <i>j</i>1 cột đầu chứa ít nhất <i>j</i>1 máy bơm nước. Suy ra <i>n</i><sub>0</sub> 

<i>j</i> 1


cột tiếp theo chứa nhiều nhất <i>k</i> 

<i>j</i> 1

máy bơm nước. Tiếp tục xét hai trường hợp:


<b>Trường hợp 1</b>. Cột <i>j</i>2 là cột trống. Khi đó <i>n</i><sub>0</sub> 

<i>j</i> 2

cột cịn lại sau khi bỏ <i>j</i>2
cột đầu được tưới đủ bởi nhiều nhất <i>k</i> 

<i>j</i> 1

máy bơm nước. Ta có


0 0


0 0 0


0


0 0


1 4 1


( 1) ( ) ( 1) ( 1) ( 2)


4 4


( 2) 1


( 2) ( ( 2))


4



<i>n</i> <i>n</i>


<i>k</i> <i>j</i> <i>f n</i> <i>j</i> <i>n</i> <i>j</i> <i>n</i> <i>j</i>


<i>n</i> <i>j</i>


<i>n</i> <i>j</i> <i>f n</i> <i>j</i>


      
   
            
   
   
    
 
      
 
 


(do <i>j</i>2 nên <i>j</i> 2 4).


Điều này mâu thuẫn với cách chọn n<i>0</i>.


<b>Trường hợp 2</b>. Cột <i>j</i>2 có ít nhất 1 máy bơm. Khi đó các máy bơm từ cột <i>j</i>2 đến
cột n0 tưới đủ các ô ở các cột này (<i>n</i><sub>0</sub> 

<i>j</i> 1

cột). Theo tính tốn ở trên, sốmáy bơm ở


các cột này không quá <i>k</i> 

<i>j</i> 1

. Ta lại có đánh giá





0 0


0 0 0


0


1 ( 1) 1


( 1) ( ) ( 1) ( 1) ( 1)


4 4


( 1)


<i>n</i> <i>n</i> <i>j</i>


<i>k</i> <i>j</i> <i>f n</i> <i>j</i> <i>n</i> <i>j</i> <i>n</i> <i>j</i>


<i>f n</i> <i>j</i>


       
   
            
   
   
  


mâu thuẫn với cách chọn <i>n</i><sub>0</sub>.


Bài tốn được giải quyết hồn tồn. Vậy sốmáy bơm nhỏ nhất để có thểtưới tất cả các ơ


của cánh đồng 3<i>n</i> là 1


4
<i>n</i>
<i>n</i>  


 


 .


<b>Bình luận. </b>


 Đây là một bài tốn hay và thú vị theo nghĩa để giải nó khơng cần những kiến thức
cao siêu nhưng địi hỏi những suy luận rất tinh tế. Những bài toán như vậy mang


đậm chất IMO.


</div>
<span class='text_page_counter'>(26)</span><div class='page_container' data-page=26>

Nhiều thí sinh TST và cả một số bạn ở ngoài đã có dựđốn sai rằng sốmáy bơm


cần thiết vẫn là n, từđó đưa ra những lời giải sai.


 Phương pháp chứng minh được trình bày trong cả hai lời giải được gọi là <i>phương </i>


<i>pháp phản ví dụ nhỏ nhất, </i>nằm trong chủ đề Phương pháp chứng minh phản
chứng hoặc chủđề Nguyên lý cực hạn.


 Một cách khác để trình bày lời giải bài toán là dùng phép quy nạp toán học.
 Bài này có nét giống với bài 3 trong VietnamTST 2010 nhưng có phần dễhơn.


<b>Bài 3. </b>



<i><b>Cho s</b><b>ố</b><b> nguyên t</b><b>ố</b></i> <i>p</i>17<i><b>. Ch</b><b>ứ</b><b>ng minh r</b><b>ằ</b><b>ng </b>t</i>3<i><b> là s</b><b>ố</b></i> <i><b>nguyên dương lớ</b><b>n nh</b><b>ấ</b><b>t th</b><b>ỏ</b><b>a </b></i>
<i><b>mãn </b><b>điề</b><b>u ki</b><b>ệ</b><b>n: V</b><b>ớ</b><b>i các s</b><b>ố</b><b> nguyên b</b><b>ấ</b><b>t kì </b><b>a b c d sao cho sao cho </b></i>, , , <i><b>abc không chia h</b><b>ế</b><b>t </b></i>
<i><b>cho p và </b></i> <i>a</i> <i>b</i> <i>c<b> chia h</b><b>ế</b><b>t cho p thì t</b><b>ồ</b><b>n t</b><b>ạ</b><b>i các s</b><b>ố</b><b> nguyên </b></i> <i><b>x y z thu</b></i>, , <i><b>ộ</b><b>c t</b><b>ậ</b><b>p </b></i>


0,1,..., <i>p</i> 1
<i>t</i>


   


 


 <sub>  </sub> 


 


  <sub> </sub> 


 


 <i><b> sao cho </b>ax</i><i>by</i><i>cz<b> chia h</b><b>ế</b><b>t cho p. </b></i>
<b>Lời giải. </b>


Ta sẽ xửlí bài toán này theo các bước sau:


1. Trước hết ta chứng minh với <i>t</i>3 thì ln tồn tại , ,<i>x y z</i> thỏa mãn bài toán.


Đặt: 1, { | 0 , , }


3


<i>p</i>


<i>L</i>   <i>S</i> <i>ax</i><i>by</i><i>cz</i> <i>x y z</i><i>L</i>


 


  . Yêu cầu bài toán tương đương với


việc chứng minh S chứa một hệ thặng dư đầy đủ mod p.


Kí hiệu <i>a</i><i>b</i> là <i>a</i><i>b</i> chia hết cho p, <i>a</i><i>b</i> nếu a không đồng dư với b mod p, <i>a</i>1 là
số nghịch đảo của <i>a</i> theo mod p, <i>S</i> là số phần tử khác nhau của S theo mod p.


2. Bởi vì <i>a</i>  <i>b</i> <i>c</i> 0 nên <i>S</i>{<i>ax</i><i>by</i> (<i>a</i> <i>b z</i>) | 0<i>x y z</i>, , <i>L</i>}. Việc nhân <i>a b</i>, với
cùng <i>b</i>10 không làm thay đổi số phần tử của tập S theo mod <i>p</i>. Do đó ta có thể xem


1


<i>b</i> nên ta có <i>S</i>{<i>ax</i>  <i>y</i> (<i>a</i> 1) | 0<i>z</i> <i>x y z</i>, , <i>L</i>}.


</div>
<span class='text_page_counter'>(27)</span><div class='page_container' data-page=27>

{ ( 1) | 0 , , } {( 1) ( ) | 0 , , }


<i>S</i> <i>ax</i>  <i>y</i> <i>a</i> <i>z</i> <i>x y z</i><i>L</i>  <i>S</i> <i>p</i> <i>a</i> <i>z</i> <i>y</i> <i>p</i><i>a x</i> <i>x y z</i><i>L</i>


Do , ,<i>x y z</i> có thểđổi chỗ cho nhau nên ta có vai trị của <i>a</i> và <i>p</i> <i>a</i> 1là như nhau nhau
nên ta có thể giả sử là 1.


2
<i>p</i>



<i>a</i>  Với 1
2
<i>p</i>


<i>a</i>  thì <i>k</i>min

<i>L</i>, 2<i>L</i><i>a</i>

0.


4. Với mỗi 0 <i>l</i> <i>L</i>, đặt <i>Xl</i> 

<i>a z</i>

   <i>l</i>

<i>y</i>

<i>a</i> 1

<i>z</i>| 0 <i>y</i> <i>L</i>, 0  <i>z</i> <i>L</i> <i>l</i>







1 | 0 , 0



<i>l</i>


<i>Y</i>  <i>ax</i>  <i>y</i> <i>a</i> <i>x</i><i>l</i>   <i>x</i> <i>L</i> <i>l</i>  <i>y</i> <i>L</i> thì


0 0


<i>L</i> <i>L</i>


<i>l</i> <i>l</i>


<i>l</i> <i>l</i>


<i>S</i> <i>X</i> <i>Y</i>


 


  <sub></sub> <sub></sub>



 


<sub></sub> <sub></sub><sub></sub> <sub></sub> <sub></sub><sub></sub>


 

. Ta có










– 1 | 0 , 0 –


– | 0 , 0 –


– , – 1 , ,


<i>l</i>


<i>X</i> <i>a z</i> <i>l</i> <i>y</i> <i>a</i> <i>z</i> <i>y</i> <i>L</i> <i>z</i> <i>L</i> <i>l</i>


<i>y</i> <i>z</i> <i>al</i> <i>y</i> <i>L</i> <i>z</i> <i>L</i> <i>l</i>


<i>l</i> <i>L</i> <i>al l</i> <i>L</i> <i>al</i> <i>L</i> <i>al</i>


       



     


     


là tập gồm 2L – l số nguyên liên tiếp. Và tương tự,











– 1 | 0 , 0


– – 1 | 0 , 0


– – 1 , – – 1 1, , – 1


<i>l</i>


<i>Y</i> <i>ax</i> <i>y</i> <i>a</i> <i>x</i> <i>l</i> <i>x</i> <i>L</i> <i>l</i> <i>y</i> <i>L</i>


<i>y</i> <i>x</i> <i>a</i> <i>l</i> <i>x</i> <i>L</i> <i>l</i> <i>y</i> <i>L</i>


<i>l</i> <i>L</i> <i>a</i> <i>l l</i> <i>L</i> <i>a</i> <i>l</i> <i>L</i> <i>a</i> <i>l</i>


        


      



     


là tập gồm 2L –l số nguyên liên tiếp. Dễ thấy với <i>l</i> <i>k</i> min

<i>L</i>, 2<i>L</i><i>a</i>

thì




1 1 1


<i>L</i><i>al</i>     <i>L</i> <i>l</i> <i>a l</i> và <i>L</i> 

<i>a</i> 1



<i>l</i>     1

1 <i>L</i> 1 (<i>a</i>1)<i>l</i>


nên
0
<i>L</i>
<i>l</i>
<i>l</i>
<i>X</i>


chứa tất cả các số nguyên từ <i>L</i> đến <i>L</i><i>ak</i> và
0
<i>L</i>
<i>l</i>
<i>l</i>
<i>Y</i>


chứa tất cả các số từ


1




<i>L</i> <i>k</i> <i>a</i> <i>k</i>


    đến <i>L</i>.


Do đó S chứa tất cả các số từ  <i>L</i> <i>k</i>

<i>a</i>1

<i>k</i> đến <i>L</i><i>ak</i>. Dễ thấy vì <i>a k</i>, 1 nên


( 1)

1 2 2 1 2 2 2 2 1 2 2 2 1


<i>L</i><i>ak</i>   <i>L</i> <i>k</i> <i>a</i> <i>k</i>   <i>L</i> <i>ak</i>  <i>L</i> <i>a</i> <i>k</i>   <i>L</i> <i>a</i> <i>k</i>


Mặt khác do <i>k</i>min

<i>L</i>, 2<i>L</i><i>a</i>

nên:


Nếu <i>k</i><i>L</i> thì 2<i>L</i>2<i>a</i>2<i>k</i> 1 2<i>L</i>2<i>a</i>2<i>L</i> 1 4<i>L</i> 1 <i>p</i>. (Đây là chỗ sử dụng


điều kiện <i>p</i>17).


</div>
<span class='text_page_counter'>(28)</span><div class='page_container' data-page=28>

Vậy S chứa khơng ít hơn p số nguyên liên tiếp nên ta có S chứa một hệ thặng dư đầy đủ


theo mod p.


5. Với <i>t</i>4 thì có thể lấy <i>a</i> <i>b</i> 1,<i>c</i> 2 và 1
2
<i>p</i>


<i>d</i>   thì do


1 1 1


2 1 2 2 1



4 2 2 4 2


<i>p</i> <i>p</i> <i>p</i> <i>p</i> <i>p</i>


<i>x</i> <i>y</i> <i>z</i>


  <sub></sub>     <sub></sub> 


<sub> </sub> <sub> </sub>


 <sub></sub><sub></sub>  <sub></sub><sub></sub>      <sub></sub><sub></sub>  <sub></sub><sub></sub>


   


<sub> </sub>  <sub> </sub> 


Nên 3 2 1 5


2 2 2


<i>p</i>


<i>x</i> <i>y</i> <i>z</i>  <i>p</i>


      không thể đồng dư 0 mod <i>p, t</i>ức là không tồn tại


, ,


<i>x y z</i> thỏa mãn.



6. Kết hợp các lý luận trên ta có đpcm.


<b>Bình luận. </b>


 Bài tốn này xứng đáng là bài tốn số 3 của kì thi. Việc chứng minh cho trường


hợp <i>t</i>4 khá dễ dàng; tuy nhiên, với trường hợp <i>t</i>3 thì vấn đề phức tạp hơn


nhiều. Với cách giải trên thì ta có thể mở rộng bài tốn ra như sau:
<b>Bài tốn 3.1. </b>


<i>Cho số ngun tố p. Tìm số ngun dương L nhỏ nhất sao cho với mọi bộ ba số nguyên </i>


<i>không đồng dư với nhau đôi một theo mod p và a</i> <i>b</i> <i>c chia hết cho p thì với mọi d </i>
<i>đều tồn tại </i>0<i>x y z</i>, , <i>L sao cho ax</i><i>by</i><i>cz chia hết cho p. </i>


 Quay trở lại với lời giải bài toán ban đầu.


Mấu chốt của vấn đề là tận dụng tính chất: khi nhân cả ba số <i>a b c</i>, , với cùng một số


0


<i>m</i> theo mod p thì vai trị của <i>a b c</i>, , và <i>ma mb mc</i>, , là tương đương nhau ở trong bài


toán là tương đương nhau ở trong bài tốn. Lại có <i>a</i>  <i>b</i> <i>c</i> 0(mod )<i>p</i> nên ta có thể


thay c bởi  <i>a</i> <i>b</i> và do đó giảm độ phức tạp đi. Một suy nghĩ tự nhiên là sẽ tìm <i>m sao </i>
cho <i>ma</i> hoặc <i>mb</i> bằng 1. Điều này đơn giản.



Sau các bước trên thì ta sẽ thu được <i>S</i> 

<i>ax</i> <i>y</i> (<i>a</i>1) | 0<i>z</i> <i>x y z</i>, , <i>L</i>

với


1.
3
<i>p</i>
<i>L</i>  


 


  Khi đến đây thì khi ta thử với <i>a</i>1 thì


2 | 0 , ,

 

2 , 2 1,..., 2 1, 2



</div>
<span class='text_page_counter'>(29)</span><div class='page_container' data-page=29>

gồm các số nguyên liên tiếp.


Lại thử với <i>a</i>2 thì <i>S</i>

2<i>x</i> <i>y</i> 3 | 0<i>z</i> <i>x y z</i>, , <i>L</i>

 

 3 , 3<i>L</i>  <i>L</i>1,...,3<i>L</i>1,3<i>L</i>


cũng gồm các số nguyên liên tiếp.


Như vậy qua hai trường hợp <i>a</i>1 và <i>a</i>2 thì ta sẽ có ý tưởng là chứngminh S chứa


một tập con có dạng

<i>M</i>, <i>M</i> 1,...,<i>M</i> 1,<i>M</i>

.


Chú ý là tập

<i>y</i><i>z</i>| 0<i>x y z</i>, , <i>L</i>

 

   <i>L</i>, <i>L</i> 1,...,<i>L</i>1,<i>L</i>

nên ta sẽ nghĩ đến việc
đặt <i>x</i> <i>z</i> 1 và đặt




– 1 | 0 , 0 –

– | 0 , 0 –



<i>l</i>



<i>X</i>  <i>a z</i> <i>l</i> <i>y</i> <i>a</i> <i>z</i>  <i>y</i> <i>L</i>  <i>z</i> <i>L l</i>  <i>y</i> <i>z</i><i>al</i>  <i>y</i> <i>L</i>  <i>z</i> <i>L l</i>
là tập gồm các số nguyên liên tiếp. Tương tự ta có <i>Y</i><sub>1</sub> nếu đặt <i>z</i> <i>x</i> 1.


Đểcó S chứa các số nguyên liên tiếp thì ta cần có <i>X</i><sub>1</sub> và <i>X<sub>l</sub></i><sub></sub><sub>1</sub> phải giao nhau, ta thu được
điều kiện cần và đủ là <i>a</i> <i>l</i> 2<i>L</i>. Vậy ta cần phải có <i>a</i>2<i>L</i>. Kiểm tra với <i>Y</i>1 ta thu


đượcđiều kiệntương tự.


Vấn đề còn lại là phải có <i>a</i>2<i>L</i>. Quay trở lại ta chú ý ngay đến vai trò của <i>a và </i>
(<i>a</i> 1) <i>p</i> (<i>a</i> 1)


     là như nhau và một trong hai phảikhông vượt quá 1 2 .


2
<i>p</i>


<i>L</i>






 Các cách giải khác :


Hai cách giải dưới đây đều sử dụng đến định lý thuộc lĩnh vực lý thuyết số cộng tính và
tổ hợp sau đây


<b>Định lý Cauchy-Davenport </b>



<i>Cho hai tập các số nguyên A, B và số nguyên tố p. Khơng có hai phần tử nào của A đồng</i>
<i>dư với nhau theo modun p. Tương tự cho B. Khi đó tập</i> <i>C</i>

<i>a</i><i>b a</i>| <i>A b</i>, <i>B</i>

<i> sẽ </i>


<i>chứa ít nhất </i>min

<i>A</i>  <i>B</i> 1,<i>p</i>

<i> phần tử đôi một không đồng dư với nhau mod p. </i>


Đặt <i>A</i>

0,1, 2,...,<i>L</i>

thì ta có <i>S</i><i>aA</i><i>bA</i><i>cA</i>. Theo định lý <i>Cauchy-Davenport ta có </i>


3 3 2


3
<i>p</i>
<i>S</i>  <i>A</i>    


 


  .


Tuy nhiên, nói chung đây chưa phải là điều ta cần, ta có thểvượt qua điều này bằng một


</div>
<span class='text_page_counter'>(30)</span><div class='page_container' data-page=30>

<b>Cách 1.</b> (d<i><b>ựa</b><b> theo b</b><b>ạn</b><b> chemthan) </b></i>


Đặt <i>B</i>

<i>ax</i><i>by</i><i>cz</i>| 1<i>x y z</i>, ,  <i>L</i> 1

và <i>C</i>

<i>ax</i><i>by</i><i>cz</i>| 1 <i>x y z</i>, , 1

.
Áp dụngđịnh lý Cauchy-Davenport, ta có:




min 1,


<i>S</i>  <i>B</i>  <i>C</i>  <i>p</i> và <i>B</i> 3

<i>L</i> 1

2.



Do đó S min 3L – 5

 C –1, p

min 3L – 6

 C , p

.


Nếu ta chứng minh được 3<i>L</i> 6 <i>C</i>  <i>p</i> thì ta chứng minh được bài tốn cho <i>t</i>3.


Ta có 3 6 3 9 11


3
<i>p</i>


<i>L</i>      <i>p</i>


 


  nên <i>C</i> phải ít nhất là 11 (ta cần chứng minh).
Đặt <i>D</i>

<i>a b c</i>, , ,  <i>a</i>, <i>b</i>, <i>c</i>



, , , , ,

 

, , , 2 , 2 , 2



<i>E</i> <i>a</i><i>b b</i><i>c c</i><i>a a</i> <i>b</i> <i>c b</i> <i>c</i> <i>a c</i> <i>a</i> <i>b</i>  <i>a</i><i>b b</i><i>c c</i><i>a</i> <i>a b c</i>
thì dễ thấy <i>D</i>  <i>E</i> 6. Nếu D và E rời nhau thì ta có <i>C</i>  <i>D</i>  <i>E</i> 12 thỏa mãn.
Nếu D và E không rời nhau thì tồn tại.


- Trường hợp 1 : <i>a</i> <i>a</i> <i>b</i> (không xảy ra)


- Trường hợp 2 : <i>a</i> <i>b</i> <i>c</i> hay <i>a</i>    <i>c</i> <i>b</i> <i>b</i> <i>b</i> (không xảy ra)


- Trường hợp 3 : <i>a</i>2<i>b</i> thì có thể dễ dàng chỉ ra <i>C</i> 13.
Vậy ta có <i>C</i> 11 và ta có đpcm.


<i><b>Cách 2. (D</b><b>ựa</b><b> trên cách trên v</b><b>ới</b><b> cách </b><b>đặt</b><b> t</b><b>ập</b><b> C) </b></i>


Vì <i>a</i>  <i>b</i> <i>c</i> 0 nên với


2
<i>L</i>


<i>K</i>  thì ta có <i>S</i>  <i>X</i> , với


| , ,

...


<i>X</i>  <i>ax</i><i>by</i><i>cz</i>  <i>K</i> <i>x y z</i><i>K</i>     <i>C</i> <i>C</i> <i>C</i> <i>C</i> (K phiên bản <i>C) </i>
Áp dụng định lý <i>Cauchy-Davenport K</i>1 lần thì <i>X</i> min

<i>K C</i>  <i>K</i> 1,<i>p</i>

.


Ta cần có <i>K C</i>   <i>K</i> 1 <i>p</i>. Ta đếm <i>C</i> . Dễ thấy rằng


0, , , , , , , , ,



<i>D</i> <i>a b c</i>  <i>a</i> <i>b</i> <i>c a</i><i>b b</i><i>c c</i><i>a</i> chứa các phần tửđôi một khơng đồngdư


</div>
<span class='text_page_counter'>(31)</span><div class='page_container' data-page=31>

Ta có <i>K C</i>  <i>K</i> 1 9<i>K</i>1. Ta cần chứng minh 9<i>K</i> 1 <i>p</i> với

/ 3

1


2
<i>p</i>


<i>K</i>    <sub></sub>


 . (*)


Đếnđây thử với <i>p</i>6<i>l</i>1 hoặc <i>p</i> 6<i>l</i> 1 với <i>l</i>3 ta thấy (*) luôn đúng.


 Lời giải và bình luận trên đây được thực hiện bởi bạn Lê Hồng Quý (Traum), huy



chương đồng Olympic Toán quốc tếnăm 2006.


 Định lý <i>Cauchy – Davenport có nhi</i>ều cách chứng minh khác nhau, trong đó có


cách chứng minh dựa vào định lý không điểm tổ hợp (Combinatorial
<i>Nullstenlenzat, bài vi</i>ết <i><b>Đa thứ</b><b>c và các bài toán t</b><b>ổ</b><b> h</b><b>ợ</b><b>p </b></i>đăng trên Tạp chí Tốn
học và Tuổi trẻ (6/2009)). Một cách chứng minh khác dùng đa thức nhiều biến là :
/>


Dưới đây chúng tơi trình bày một cách chứng minh sơ cấp cho định lý này. Cảm ơn bạn
Nguyễn Ngọc Trung (chemthan), huy chương vàng Olympic Toán quốc tếnăm 2010, đã
cung cấp tư liệu tiếng Anh.


Đặt <i>A</i>

<i>a a</i><sub>1</sub>, <sub>2</sub>,...,<i>a<sub>m</sub></i>

, <i>B</i>

<i>b b</i><sub>1</sub>, <sub>2</sub>,...,<i>b<sub>n</sub></i>

. Ta chứng minh <i>k</i> <i>A</i><i>B</i>   <i>m</i> <i>n</i> 1 (theo
mod p). Ta giả sử <i>m</i><i>n</i> và sẽ chứng minh quy nạp theo m.


Với <i>m</i>1, mệnh đề đúng vì từ <i>a</i><sub>1</sub>  <i>b<sub>i</sub></i> <i>a</i><sub>1</sub> <i>b<sub>j</sub></i>(mod )<i>p</i> nếu <i>i</i> <i>j</i> nên


1 1 1


<i>a</i> <i>B</i>  <i>B</i>    <i>n</i> <i>n</i> .


Khi đó các bất đẳng thức 0 <i>F</i>   <i>m</i> <i>n</i> <i>G F</i>, <i>G</i>  <i>p</i> và giả thiết quy nạp cho ta
mệnh đềđúng đối với các tập hợp F và G.


Giả sử mệnh đề đã đúng với mọi hai tập hợp X và Y sao cho <i>X</i> <i>m X</i>, <i>Y</i> và
<i>X</i> <i>Y</i>  <i>p</i>. Giả sử <i>A</i>  <i>m</i> 1 và <i>B</i> <i>n</i>, trong đó <i>m</i><i>n</i> và <i>m</i> <i>n</i> <i>p</i>. Khi đó


<i>n</i> <i>p</i> và do đó tồn tại <i>c</i><i>B</i>.Ta chọn các phần tử <i>a a</i><sub>1</sub>, <sub>2</sub> khác nhau thuộc <i>A. Vì dãy </i>

2 1

(mod )


<i>c</i><i>t a</i> <i>a</i> <i>p</i> với <i>t</i>1, 2,3,...,<i>p</i>1 chứa tất cả các số dư, trừ <i>c nên </i>

2 1



<i>b</i> <i>c</i> <i>t a</i> <i>a</i> <i>B</i> với t nào đó. Gọi t là số nhỏ nhất có tính chất này.


Tập hợp <i>A</i> 

<i>b</i><i>a</i><sub>2</sub>

<i>A</i> chứa phần tử <i>b</i><i>a</i><sub>2</sub><i>a b</i><sub>1</sub>, <i>a</i><sub>2</sub><i>a</i><sub>2</sub><i>b</i>. Chú ý rằng






2 1 1 2 1


<i>b</i><i>a</i>    <i>a</i> <i>c</i> <i>t</i> <i>a</i> <i>a</i> <i>B</i>. Vì <i>A</i>  <i>B</i>

<i>b</i><i>a</i><sub>2</sub>

 <i>A</i> <i>B</i> nên ta chỉ cần


</div>
<span class='text_page_counter'>(32)</span><div class='page_container' data-page=32>

Đặt <i>F</i> <i>A</i><i>B</i> và <i>F</i> <i>A</i><i>B</i>. Vì <i>b</i><i>F b</i>, <i>a</i><sub>2</sub> <i>a</i><sub>1</sub> <i>F</i>và <i>b</i><i>a</i><sub>2</sub> <i>a</i><sub>1</sub> <i>A</i> nên <i>F là </i>
tập con thực sự khác rỗng của <i>A</i>. Như vậy B là tập con thực sự của G.


Từđây suy ra 0 <i>F</i>   <i>m</i> <i>n</i> <i>G</i> .


Mặt khác <i>m</i> <i>n</i> <i>A</i>  <i>B</i>  <i>A</i> <i>B</i> <i>A</i><i>B</i>  <i>F</i> <i>G</i> .


Ta cũng chú ý rằng <i>F</i> <i>G</i> <i>A</i><i>B</i> (Với <i>f</i> <i>F g</i>, <i>G</i>, ta có thể giả sử rằng <i>g</i><i>A</i> và


khi đó <i>f</i>  <i>F</i> <i>B</i> suy ra rằng <i>f</i>  <i>g</i> <i>A</i><i>B</i>). Suy ra <i>A</i>  <i>B</i>  <i>F</i> <i>G</i> .


Khi đó các bất đẳng thức 0 <i>F</i>   <i>m</i> <i>n</i> <i>G F</i>, <i>G</i>  <i>p</i> và giả thiết quy nạp cho ta
mệnh đềđúng đối với các tập hợp F và G. Do đó


1 1 1



<i>A</i><i>B</i>  <i>A</i> <i>B</i> <i>F</i><i>G</i>  <i>F</i>  <i>G</i>   <i>A</i>  <i>B</i>   <i>m</i> <i>n</i>


và phép quy nạp được hoàn tất.


 Theo lời giải đầu tiên thì bài tốn vẫn đúng cho <i>p</i>13.


 Đây là một bài tốn khó và nó gần với một bài toán tổ hợp hơn là một bài toán số


học (vềphương pháp chứng minh và lập luận)


 Các bạn có thể tham khảo một bài tốn có ý tưởng giải tương tựnhư sau:


<b>Bài toán 3.2.</b> Cho <i>p</i>3<i> là số nguyên tố và a a a</i><sub>1</sub>, <sub>2</sub>, <sub>3</sub>,...,<i>ap</i><sub>2</sub><i> là dãy các số tự nhiên sao </i>
<i>cho p không chia hết a và <sub>k</sub></i> <i>a<sub>k</sub>k</i> 1<i>. Chứng minh rằng ta có thể chọn ra một số số hạng </i>
<i>của dãy sốđể tích của chúng có sốdư là 2 khi chia cho p</i>.


<b>Hướng dẫn.</b>Dùng căn nguyên thủy đưa về bài tốn cộng tính.


<b>Bài 4. </b>


<i><b>Cho dãy s</b><b>ố</b><b>nguyên dương </b></i>

<sub> </sub>

<i>x<sub>n</sub></i> <i><b>được xác đị</b><b>nh b</b><b>ở</b><b>i </b></i> 1 2 <sub>*</sub>


2 1


1, 2011,


4022 ,


<i>n</i> <i>n</i> <i>n</i>



<i>x</i> <i>x</i>


<i>x</i> <sub></sub> <i>x</i> <sub></sub> <i>x</i> <i>n</i>


 






   




 


<i><b> </b></i>


<i><b>Ch</b><b>ứ</b><b>ng minh r</b><b>ằ</b><b>ng </b></i> 2012 1
2012


<i>x</i> 


<i><b> là s</b><b>ố</b><b>chính phương.</b></i>


</div>
<span class='text_page_counter'>(33)</span><div class='page_container' data-page=33>

Đây chính là bài tốn nhẹ nhàng và quen thuộc nhất trong đề thi lần này. Đểđơn giản hơn


trong việc dùng kí hiệu và hiểu rõ bản chất vấn đề, ta sẽ phát biểu và chứng minh bài toán
tổng quát như sau:



<i>Cho plà sốnguyên dương lẻ lớn hơn 1. </i>


<i>Xét dãy sốnguyên dương </i>

 

<i>xn</i> <i>được xác định bởi </i>


1 2


*


2 1


1, ,


2 ,


<i>n</i> <i>n</i> <i>n</i>


<i>x</i> <i>x</i> <i>p</i>


<i>x</i> <sub></sub> <i>px</i><sub></sub> <i>x</i> <i>n</i>


 



   

 
<i> </i>


<i>Chứng minh rằng </i> 1 1


1
<i>p</i>
<i>x</i>


<i>p</i>
 


 <i> là sốchính phương.</i>
Bài tốn này có một số lời giải như sau.


<i><b>Cách 1 (dùng công th</b><b>ứ</b><b>c t</b><b>ổ</b><b>ng quát c</b><b>ủ</b><b>a dãy và bi</b><b>ến đổ</b><b>i tr</b><b>ự</b><b>c ti</b><b>ế</b><b>p). </b></i>


Phương trình đặc trưng của dãy sốđã cho là 2 2


2 1 2 1 0


<i>t</i>  <i>pt</i> <i>t</i>  <i>pt</i>  có 2


1 0


<i>p</i>


   


nên phương trình có hai nghiệm là 2 2


1 1, 2 1


<i>t</i>  <i>p</i> <i>p</i>  <i>t</i>  <i>p</i> <i>p</i>  .



Công thức tổng quát của dãy đã cho là <i>x<sub>n</sub></i>  <i>At</i><sub>1</sub><i>n</i> <i>Bt n</i><sub>2</sub><i>n</i>, 1, 2, 3,...


Thay <i>n</i>1, 2 tương ứng với hai số hạng cho trước của dãy, ta được hệphương trình sau:
1 2


2 2
1 2


1
<i>At</i> <i>Bt</i>


<i>At</i> <i>Bt</i> <i>p</i>


 





 




Giải hệnày, ta thu được 2 1
,


2 2


<i>t</i> <i>t</i>



<i>A</i> <i>B</i> hay


1 1
1 2


, 1, 2,3,...
2
<i>n</i> <i>n</i>
<i>n</i>
<i>t</i> <i>t</i>
<i>x</i> <i>n</i>
 <sub></sub> 
 


Suy ra



2
/ 2 /2
1 2
1 1 1 2 2


1 2( 1) 2( 1)


<i>p</i> <i>p</i>


<i>p</i> <i>p</i>


<i>p</i> <i>t</i> <i>t</i>


<i>x</i> <i><sub>t</sub></i> <i><sub>t</sub></i>



<i>p</i> <i>p</i> <i>p</i>


    


 


   .


Chú ý rằng <i>t</i><sub>1</sub><i>t</i><sub>2</sub> 2 , <i>p t t</i><sub>1 2</sub>1 nên <i>t</i><sub>1</sub>  <i>t</i><sub>2</sub>  <i>t</i><sub>1</sub><i>t</i><sub>2</sub>2 <i>t t</i><sub>1 2</sub>  2(<i>p</i>1).


Hơn nữa, ta cũng có 1 2 , 1


<i>n</i> <i>n</i>
<i>n</i>


<i>S</i> <i>t</i> <i>t</i>   <i>n</i> vì <i>S S</i>1, 2 và <i>Sn</i>2 2<i>pSn</i>1<i>Sn</i>.


Đặt <i>t</i><sub>1</sub> <i>a</i>, <i>t</i><sub>2</sub> <i>b</i> thì <i>a b</i>  2(<i>p</i>1),<i>ab</i>1 và / 2 / 2
1 2


<i>p</i> <i>p</i> <i>p</i> <i>p</i>


<i>t</i> <i>t</i> <i>a</i> <i>b</i> .


Ta có


1 1


1 1



0 0


( ) ( 1) 2( 1) ( 1)


<i>p</i> <i>p</i>


<i>p</i> <i>p</i> <i>i</i> <i>i</i> <i>p</i> <i>i</i> <i>i</i> <i>i</i> <i>p</i> <i>i</i>


<i>i</i> <i>i</i>


<i>a</i> <i>b</i> <i>a b</i> <i>a b</i> <i>p</i> <i>a b</i>


 


   


 


</div>
<span class='text_page_counter'>(34)</span><div class='page_container' data-page=34>

3


1 1


1 2 1


1 1 <sub>2</sub> <sub>2</sub> 1


1


0 0



2


3 3


1 1 2 1 2


1 1


2 2


1 2 1 2 <sub>2</sub> <sub>2</sub> <sub>2</sub>


2 1


1 1


0 0


2 2


( 1) ( 1) ( 1) ( ) ( 1)


( 1) ( ) ( 1) ( ) ( 1) ( 1) ( 1) ( 1)


<i>p</i>


<i>p</i> <i>p</i>


<i>p</i> <i>p</i>



<i>i</i> <i>i</i> <i>p</i> <i>i</i> <i>i</i> <i>i</i> <i>p</i> <i>i</i> <i>i</i> <i>i</i> <i>p</i> <i>i</i>


<i>p</i>


<i>i</i> <i>i</i> <i><sub>i</sub></i>


<i>p</i> <i>p</i>


<i>p</i> <i>p</i> <i>i</i> <i>p</i> <i>i</i> <i>p</i>


<i>p</i> <i>p</i>


<i>i</i> <i>i</i> <i>p</i> <i>i</i> <i>i</i> <i>i</i> <i>p</i> <i>i</i> <i>i</i> <i>i</i>


<i>p</i> <i>p</i>


<i>i</i> <i><sub>i</sub></i> <i>i</i> <i><sub>i</sub></i>


<i>a b</i> <i>a b</i> <i>ab</i> <i>a b</i>


<i>ab b</i> <i>ab a</i> <i>t</i> <i>t</i>



 
 
     

  <sub></sub>
 


    
 
   
 
 
 
      
           



1
2
3 3


1 2 1 2 1 1


2 2


2 2 2 2


1 2 1 2


0 0 2


( 1) ( 1) ( 1) ( 1)


<i>p</i> <i>p</i>


<i>p</i> <i>i</i> <i>p</i> <i>i</i> <i>p</i> <i>p</i>



<i>i</i> <i>i</i>


<i>p</i> <i>i</i>


<i>i</i> <i>i</i>


<i>t</i> <i>t</i> <i>S</i> <i>N</i>



 
     
 
 
 
  <sub></sub>  <sub></sub>       
 



Do đó / 2 / 2

/ 2 / 2

2 2



1 2 2( 1) 1 2 2( 1)


<i>p</i> <i>p</i> <i>p</i> <i>p</i>


<i>t</i> <i>t</i> <i>N</i> <i>p</i>  <i>t</i> <i>t</i> <i>N</i> <i>p</i> .


Vậy



2



1 1 2( 1) 2


1 2( 1)


<i>p</i>


<i>x</i> <i>N</i> <i>p</i>


<i>N</i>


<i>p</i> <i>p</i>


  


 


  là sốchính phương. Ta có đpcm.


<i><b>Cách 2. (Xét dãy s</b><b>ố</b><b> ph</b><b>ụ</b><b> và dùng quy n</b><b>ạ</b><b>p). </b></i>
Ta thấy rằng


2
2


3


2 3 4 2


3 4



4 2 5 3 6 2 2


5 6


1 1


1,


1 1


1 4 3 1


2 1, 4 3 (2 1) ,


1 1


1


8 8 1, 16 20 5 (4 2 1) .


1


<i>x</i> <i>p</i>


<i>x</i> <i>p</i>


<i>p</i> <i>p</i>


<i>x</i> <i>p</i> <i>p</i>



<i>x</i> <i>p</i> <i>x</i> <i>p</i> <i>p</i> <i>p</i>


<i>p</i> <i>p</i>


<i>x</i>


<i>x</i> <i>p</i> <i>p</i> <i>x</i> <i>p</i> <i>p</i> <i>p</i> <i>p</i> <i>p</i>


<i>p</i>
 
   
 
  
       
 

         


Từ công thức truy hồi là <i>x<sub>n</sub></i><sub></sub><sub>2</sub> 2<i>px<sub>n</sub></i><sub></sub><sub>1</sub><i>x<sub>n</sub></i>, ta có 2<i>px<sub>n</sub></i><sub></sub><sub>1</sub> <i>x<sub>n</sub></i> <i>x<sub>n</sub></i><sub></sub><sub>2</sub>. Suy ra


2 2


2 2 2 1 4 2 1 (4 2) 2, 3


<i>n</i> <i>n</i> <i>n</i> <i>n</i> <i>n</i> <i>n</i> <i>n</i> <i>n</i> <i>n</i>


<i>x</i><sub></sub>  <i>p</i> <i>px</i> <i>x</i><sub></sub> <i>x</i>  <i>p x</i>  <i>px</i> <sub></sub> <i>x</i>  <i>p</i>  <i>x</i> <i>x</i> <sub></sub>  <i>n</i> .


Ta sẽ xây dựng công thức của dãy 2 1



, 1, 2, 3,...
1
<i>n</i>
<i>n</i>
<i>x</i>
<i>y</i> <i>n</i>
<i>p</i>

 


 và chứng minh các số hạng


của dãy này đều nguyên.


Xét dãy

 

<i>yn</i> thỏa mãn


1 2


2 1


1, 2 1


, 1


<i>n</i> <i>n</i> <i>n</i>


<i>y</i> <i>y</i> <i>p</i>


<i>y</i> <sub></sub> <i>ay</i> <sub></sub> <i>by n</i>



  





  




với <i>a b</i>, được chọn sau.
Do <i>y</i><sub>3</sub> <sub></sub>4<i>p</i> <sub></sub>2<i>p</i><sub></sub>1<sub> nên </sub> 2


(2 1) 4 2 1


</div>
<span class='text_page_counter'>(35)</span><div class='page_container' data-page=35>

Dãy sốtương ứng là 1 2


2 1


1, 2 1


2 , 1


<i>n</i> <i>n</i> <i>n</i>


<i>y</i> <i>y</i> <i>p</i>


<i>y</i> <sub></sub> <i>py</i> <sub></sub> <i>y n</i>


  




  

.


Ta sẽ chứng minh bằng quy nạp rằng 2 2 1


, 1
1
<i>n</i>
<i>n</i>
<i>x</i>
<i>y</i> <i>n</i>
<i>p</i>

  
 . (*)


Với <i>n</i>1, 2, khẳng định (*) đúng.


Giả sử ta có 2 2 2 2 2
1
1 1
,
1 1
<i>n</i> <i>n</i>
<i>n</i> <i>n</i>
<i>x</i> <i>x</i>
<i>y</i> <i>y</i>


<i>p</i> <i>p</i>


 
 
  .


Ta có 2 2


2 1 (2 1 ) 1(2 1) 1 1 ,


<i>n</i> <i>n</i> <i>n</i> <i>n</i> <i>n</i> <i>n</i> <i>n</i> <i>n</i> <i>n</i> <i>n</i> <i>n</i> <i>n</i>


<i>y</i> <sub></sub> <i>y</i> <i>y</i> <sub></sub>  <i>py</i> <sub></sub> <i>y y</i> <i>y</i> <sub></sub> <i>py</i> <i>y</i> <sub></sub>  <i>y</i> <sub></sub> <i>y</i> <sub></sub> <i>y</i> <i>n</i>.


Hơn nữa 2


3 1 2 2 2


<i>y y</i> <i>y</i>  <i>p</i> nên 2


2 1 2 2,


<i>n</i> <i>n</i> <i>n</i>


<i>y</i> <sub></sub> <i>y</i> <i>y</i> <sub></sub>  <i>p</i> <i>n</i>.


Từ cơng thức xác định dãy thì





2 2 2 2 2 2 2 2 2


2 2 1 2 2 2 4 1 2 2 2 2 1 4 1


<i>n</i> <i>n</i> <i>n</i> <i>n</i> <i>n</i> <i>n</i> <i>n</i> <i>n</i> <i>n</i> <i>n</i> <i>n</i> <i>n</i>


<i>y</i> <i>y</i> <sub></sub>  <i>py</i> <sub></sub>  <i>y</i> <i>y</i> <sub></sub>  <i>y y</i> <sub></sub>  <i>p y</i> <sub></sub>  <i>y</i> <i>y</i> <sub></sub>  <i>p</i> <i>y</i> <sub></sub>  <i>p y</i> <sub></sub> hay


2 2 2 2 2 2 2 2


2 1


2


2 2 2 2 4


1 1


(4 2) 4( 1) (4 2) 4( 1)


1 1


(4 2) 1 1


1 1


<i>n</i> <i>n</i>


<i>n</i> <i>n</i> <i>n</i>



<i>n</i> <i>n</i> <i>n</i>


<i>x</i> <i>x</i>


<i>y</i> <i>p</i> <i>y</i> <i>y</i> <i>p</i> <i>p</i> <i>p</i>


<i>p</i> <i>p</i>


<i>p</i> <i>x</i> <i>x</i> <i>x</i>


<i>p</i> <i>p</i>

 
 
 
         
 
   
 
 


Khẳng định (*) cũng đúng với <i>n</i>2. Theo nguyên lí quy nạp, (*) được chứng minh.


Do đó, ta đã chứng minh được với mọi n chẵn thì 1
1
<i>n</i>
<i>x</i>


<i>p</i>




 là sốchính phương; nói riêng, ta
cũng có 1 1


1
<i>p</i>
<i>x</i>


<i>p</i>
 


 cũng là sốchính phương. Ta có đpcm.


<b>Bình luận. </b>


Cách thứ nhất có thể kết hợp thêm quy nạp để chứng minh / 2 /2


1 2 2( 1)


<i>p</i> <i>p</i>


<i>t</i> <i>t</i> <i>N</i> <i>p</i> cho


đơn giản thay vì biến đổi trực tiếp.


Cách thứ hai của bài toán này dựa trên một định lí về dãy số gồm tồn sốchính phương
như sau:


</div>
<span class='text_page_counter'>(36)</span><div class='page_container' data-page=36>

<i>Xét dãy số mới là </i>

 

<i>vn</i> <i> với </i>




2 2 2 2


1 , 2 , <i>n</i> 2 ( 2) <i>n</i> 1 <i>n</i> 2 1 3 2 , 1
<i>v</i> <i>a v</i> <i>b v</i>   <i>c</i>  <i>v</i>  <i>v</i>  <i>u u</i> <i>u</i> <i>n</i> <i>. </i>


<i>Khi đó </i> 2


<i>n</i> <i>n</i>


<i>v</i> <i>u</i> <i> với mọi sốnguyên dương n</i>.


Ý tưởng để chứng minh định lí này hoàn toàn đã được áp dụng vào lời giải thứ hai ở trên.
Cũng chính cách này đã cho ta một kết quả mạnh hơn bài toán đã cho là khẳng định vẫn


đúng khi thay <i>x<sub>p</sub></i><sub></sub><sub>1</sub> bởi <i>x<sub>n</sub></i> với n chẵn bất kì.


Ngồi ra, ta có thể dựa trên tính chất của dãy số (<i>x<sub>n</sub></i>) là 2 2


2 1 1


<i>n</i> <i>n</i> <i>n</i>


<i>x</i> <sub></sub> <i>x</i> <i>x</i> <sub></sub> <i>p</i>  để suy ra rằng


2


2 1 1 1 1


1 1 1



<i>n</i> <i>n</i> <i>n</i>


<i>x</i> <i>x</i> <i>x</i>


<i>p</i> <i>p</i> <i>p</i>


 
        

     
  
     


và dùng quy nạp đểcó được điều phải chứng minh.


Ta cũng có một hướng tiếp cận khác nữa là:


Xét phương trình Pell có dạng <i>x</i>2(<i>p</i>21)<i>y</i>21 với <i>p</i> là sốnguyên dương lẻ.


Ta xét đồng thời hai dãy sốđược xác định như sau:


1 2 2 1


1 2 2 1


1, , 2


, 1, 2,3,...



0, 1, 2


<i>n</i> <i>n</i> <i>n</i>


<i>n</i> <i>n</i> <i>n</i>


<i>x</i> <i>x</i> <i>p x</i> <i>px</i> <i>x</i>


<i>n</i>


<i>y</i> <i>y</i> <i>y</i> <i>py</i> <i>y</i>


 
 
    
 <sub></sub>
    



Khi đó, bằng quy nạp, ta chứng minh được rằng <i>x<sub>n</sub></i>2

<i>p</i>21

<i>y<sub>n</sub></i>21 và

<i>x y<sub>n</sub></i>, <i><sub>n</sub></i>

cũng


chính là tất cả các nghiệm của phương trình Pell nêu trên.


Bằng quy nạp, ta cũng chứng minh được rằng

<i>x</i><sub>2</sub><i><sub>k</sub></i>1

 

 <i>p</i>1 ,

 

<i>x</i><sub>2</sub><i><sub>k</sub></i> 1

 

 <i>p</i>1

với mọi
0


<i>k</i> nên có thể viết đẳng thức quan hệ giữa <i>x y<sub>n</sub></i>, <i><sub>n</sub></i> thành 2 1 2 1 2


1 1
<i>k</i> <i>k</i>


<i>x</i> <i>x</i>
<i>y</i>
<i>p</i> <i>p</i>
 <sub></sub>  <sub></sub> <sub></sub> <sub></sub>
 <sub></sub><sub></sub> <sub></sub><sub></sub>
 <sub></sub>  <sub></sub>
 <sub></sub>   <sub></sub> 
    .


Ta cũng chứng minh được rằng cả hai số 2 1 2 1
,
1 1
<i>k</i> <i>k</i>
<i>x</i> <i>x</i>
<i>p</i> <i>p</i>
 


  nguyên tố cùng nhau nên mỗi


sốđều phải là số chính phương. Bài tốn được giải quyết nhanh chóng và có lẽđây chính
là cơ sởđể xây dựng bài số 4 này!


Từđây, ta cũng có thểthêm vào bài toán ban đầu một kết quả thú vị nữa là 2012 1
2010


<i>x</i> 


</div>
<span class='text_page_counter'>(37)</span><div class='page_container' data-page=37>

Ngồi ra, cơng thức xác định của dãy đã cho có liên quan đến đa thức Chebyshev loại I


như sau:



0( ) 1, ( )1 , <i>n</i> 2( ) 2 <i>n</i> 1( ) <i>n</i>( ), 0
<i>T x</i>  <i>T x</i> <i>x T</i><sub></sub> <i>x</i>  <i>xT</i><sub></sub> <i>x</i> <i>T x n</i> .


Do đó, dựa vào các tính chất đã biết của loại đa thức này, bài tốn đã cho có thể phát triển
theo nhiều hướng thú vịhơn.


<i>Lời giải và bình luận bài 4 được thực hiện bởi Lê Phúc Lữ, tham khảo các lời giải của </i>
<i>các bạn Lê Việt Hải, Võ Anh Đức và Hoàng Đỗ Kiên, Nguyễn Huy Tùng. </i>


<b>Bài 5. </b>


<i><b>Ch</b><b>ứ</b><b>ng minh r</b><b>ằ</b><b>ng </b>C</i>10 24<i><b> là h</b><b>ằ</b><b>ng s</b><b>ố</b><b> l</b><b>ớ</b><b>n nh</b><b>ấ</b><b>t sao cho n</b><b>ế</b><b>u có </b></i>17<i><b> s</b><b>ố</b><b> th</b><b>ực dương</b><b> </b></i>
1, 2,..., 17


<i>a a</i> <i><b>a th</b><b>ỏ</b><b>a mãn </b><b>điề</b><b>u ki</b><b>ệ</b><b>n </b>a</i><sub>1</sub>2<i>a</i><sub>2</sub>2 ... <i>a</i><sub>17</sub>2 24<i><b> và </b></i>
3


17 1 2


3 3


17


1 2


<i>a</i> <i>a</i> <i>a</i> <i>a</i> <i>a</i> <i>a</i> <i>C</i>


<i><b>thì v</b><b>ớ</b><b>i m</b><b>ọ</b><b>i 1</b></i>   <i>i</i> <i>j</i> <i>k</i> 17,<i><b> ta có </b>a a<sub>i</sub></i>, <i><sub>j</sub></i>,<i>a<sub>k</sub></i> <i><b>là độ</b><b> dài ba c</b><b>ạ</b><b>nh c</b><b>ủ</b><b>a m</b><b>ộ</b><b>t tam giác. </b></i>



<b>Lời giải. </b>


Đặt <i>a<sub>i</sub></i>  24<i>x<sub>i</sub></i>,  <i>i</i> 1, 2, , 17, khi đó yêu cầu bài toán tương đương với:


<i>Chứng minh rằng </i> <i>C</i>10<i> là hằng số lớn nhất sao cho nếu có </i>17<i> số thực dương </i>


1, 2, , 17


<i>x x</i>  <i>x</i> <i> thỏa mãn x</i><sub>1</sub>2<i>x</i><sub>2</sub>2  <i>x</i><sub>17</sub>2 1<i>và </i>


3 3 3


1 2 17 1 2 17


24(<i>x</i> <i>x</i>   <i>x</i> )(<i>x</i> <i>x</i>   <i>x</i> )<i>C</i>


<i>thì với mọi i j k</i>, , <i> thỏa mãn 1</i>   <i>i</i> <i>j</i> <i>k</i> 17,<i> ta có x x<sub>i</sub></i>, <i><sub>j</sub></i>, <i>x<sub>k</sub></i> <i>là độ dài ba cạnh của một </i>
<i>tam giác. </i>


Chứng minh dưới đây gồm hai phần:


<i><b>(a) Ch</b>ứng minh hằng số</i> <i>C</i>10<i> thỏa mãn u cầu đề bài. </i>Khơng mất tính tổng qt, ta
chỉ cần chứng minh <i>x x</i><sub>1</sub>, <sub>2</sub>, <i>x</i><sub>3</sub> là độ dài ba cạnh của một tam giác. Để ý rằng với mọi


0 <i>t</i> 1, ta có


3 4 2 2


24<i>t</i>  <i>t</i> (16<i>t</i> 9 )<i>t</i> <i>t</i>(1<i>t</i>)(4<i>t</i>1) 0.



Do đó, từ giả thiết ta suy ra


4 4 4 2 2 2


1 2 17 1 2 17


</div>
<span class='text_page_counter'>(38)</span><div class='page_container' data-page=38>

hay


4 4 4 2 2 2 2


1 2 17 1 2 17


16(<i>x</i> <i>x</i>   <i>x</i> ) 1 (<i>x</i> <i>x</i>   <i>x</i> ) .
Bây giờ, sử dụng bất đẳng thức Cauchy-Schwarz, ta có


14 so 1


4 4 4 4 4 4 4 4 4


1 2 17 1 2 3 4 5 17


2


4 4 4 2 2 2


1 2 3 4 5 17


16( ) 2 1 1 1 ( ) ( )


2( ) .



<i>x</i> <i>x</i> <i>x</i> <i>x</i> <i>x</i> <i>x</i> <i>x</i> <i>x</i> <i>x</i>


<i>x</i> <i>x</i> <i>x</i> <i>x</i> <i>x</i> <i>x</i>


 <sub></sub>
 <sub></sub> 
   <sub></sub><sub></sub>     <sub></sub><sub></sub>       <sub></sub><sub></sub>

 
 
<sub></sub>       <sub></sub>
 
 <sub></sub> 

Từđây kết hợp với trên, ta thu được


4 4 4 2 2 2


1 2 3 1 2 3


2(<i>x</i> <i>x</i> <i>x</i> ) <i>x</i> <i>x</i> <i>x</i> ,
hay


4 4 4 2 2 2 2


1 2 3 1 2 3


2(<i>x</i> <i>x</i> <i>x</i> )(<i>x</i> <i>x</i> <i>x</i> ) .



Đến đây, bằng cách sử dụng đồng nhất thức


2 2 2 2 4 4 4


1 2 3 1 2 3 1 2 3 1 2 3 2 3 1 3 1 2


(<i>x</i> <i>x</i> <i>x</i> ) 2(<i>x</i> <i>x</i> <i>x</i> )2(<i>x</i> <i>x</i> <i>x</i> )(<i>x</i> <i>x</i> <i>x</i> )(<i>x</i>  <i>x</i> <i>x</i>)(<i>x</i>  <i>x</i> <i>x</i> ),
ta dễdàng suy ra được <i>x x</i><sub>1</sub>, <sub>2</sub>, <i>x</i><sub>3</sub> là độ dài ba cạnh của một tam giác.


<i><b>(b) Ch</b>ứng minh C</i>10<i> là hằng số lớn nhất thỏa mãn yêu cầu đề bài. Gi</i>ả sử tồn tại hằng
số <i>C</i> 10 thỏa mãn yêu cầu đềbài. Khi đó, ta xét 17 sốdương <i>x x</i><sub>1</sub>, <sub>2</sub>,, <i>x</i><sub>17</sub> với


2
17


1 2 3 4


1 1 1


, , , 0 .


4 1


1


,
16


6 <i>x</i> 14 16



<i>x</i>  <i>x</i>  <i></i> <i>x</i> <i></i> <i>x</i>   <i> </i>  <i></i>


Lúc này, dễ thấy <i>x</i><sub>1</sub>2<i>x</i><sub>2</sub>2<i>x</i><sub>17</sub>2 1. Ngoài ra, ta cũng có


2


2


1 3 2


1 1 1 1 1


4 4 16 2 16 2 16


<i>x</i>    <i>x</i> <i></i> <sub></sub><sub></sub> <i></i><sub></sub>   <i></i> <i></i>   <i></i>  <i></i> <i>x</i>




nên <i>x x</i><sub>1</sub>, <sub>2</sub>, <i>x</i><sub>3</sub> không phải là độ dài ba cạnh của một tam giác.


Bây giờ, ta sẽ chứng minh rằng, bằng cách chọn <i></i> thích hợp, các số <i>x x</i>1, 2,, <i>x</i>17 sẽ
thỏa mãn bất đẳng thức


3 3 3


1 2 17 1 2 17


24(<i>x</i> <i>x</i>   <i>x</i> )(<i>x</i> <i>x</i>   <i>x</i> )<i>C</i>.


Rõ ràng chứng minh được điều này cũng có nghĩa là ta đã chứng minh được <i>C</i>10 là


hằng số lớn nhất thỏa mãn yêu cầu của bài toán.


</div>
<span class='text_page_counter'>(39)</span><div class='page_container' data-page=39>

3
3
3
2
3
2
1 1


16 14 1


1 1 1 1


24 14 14 .


4 16 4 16 6 14 <i>C</i>


<i> </i> <i> </i>
<i></i> <i></i> <i></i> <i></i>
 <sub></sub> <sub></sub> 
 
 <sub></sub> <sub></sub> <sub></sub> <sub></sub>  <sub></sub>

         
 <sub></sub><sub></sub> <sub></sub><sub></sub> <sub></sub><sub></sub> <sub></sub> 
   


 


 



Dễ thấy vế trái là một hàm liên tục của <i></i> trên 0, 1 .
16


 <sub></sub>


 <sub></sub>


 <sub></sub>


  Ngoài ra, khi <i></i> 0




 thì


3 3


3


1 1 1 1 1 1


24 14 14 10 .


4 16 16 4 16 16


<i>VT</i> <i>C</i>


 <sub> </sub> <sub> </sub> 
 <sub></sub> <sub></sub> <sub></sub> <sub></sub>  <sub></sub>
 <sub></sub>  <sub></sub><sub></sub> <sub></sub><sub></sub>  <sub></sub><sub></sub> <sub></sub><sub></sub> <sub></sub>    
   
 
 


Do đó, theo tính chất của hàm liên tục, ta thấy rằng tồn tại một giá trị <sub>0</sub> 0, 1
16
<i></i> <sub></sub> <sub></sub><sub></sub> sao
cho tại <i></i><i></i><sub>0</sub> thì <i>VT</i> <i>C</i>. Lúc này, ta có bộ số


2 2


0 0 0 0


1 2 17 0 0


1


( , , , ,


16


1 1 1


, ) , , ,


4 16 14 16 14



<i>x</i>


<i>x x</i> <i> </i> <i></i> <i></i> <i></i> <i></i>


 <sub></sub> <sub></sub> <sub></sub>
 <sub></sub>

 <sub></sub>    <sub></sub><sub></sub>
 <sub></sub>
  


thỏa mãn đồng thời các điều kiện <i>x</i><sub>1</sub>2<i>x</i><sub>2</sub>2  <i>x</i><sub>1</sub>2<sub>7</sub> 1 và


3 3 3


1 2 17 1 2 17


24(<i>x</i> <i>x</i>   <i>x</i> )(<i>x</i> <i>x</i>   <i>x</i> )<i>C</i>.


Nhưng trong chúng có ba số <i>x x</i><sub>1</sub>, <sub>2</sub>, <i>x</i><sub>3</sub> không lập thành độ dài ba cạnh của một tam giác.


Điều này mâu thuẫn với giả thiết <i>C</i> 10 là hằng số thỏa mãn yêu cầu đề bài. Vậy
max 10.


<i>C</i> 


<b>Bình luận. </b>


Lời giải ởtrên đã sử dụng đánh giá đểđưa về xét bất đẳng thức



4 4 4 2 2 2 2


1 2 17 1 2 17 ,


16(<i>x</i> <i>x</i>   <i>x</i> )(<i>x</i> <i>x</i>   <i>x</i> )


rồi từđó suy ra <i>x x<sub>i</sub></i>, <i><sub>j</sub></i>, <i>x<sub>k</sub></i> (1   <i>i</i> <i>j</i> <i>k</i> 17) là độ dài ba cạnh của một tam giác.


Như vậy, từ việc giải bài toán đã cho, ta thu được một bài toán khác là:


<i><b>Bài toán 5.1. Cho các s</b>ốdương x x</i><sub>1</sub>, <sub>2</sub>,, <i>x</i><sub>17</sub><i> thỏa mãn </i>


2 2 2 2 4 4 4


1 2 17 1 2 17


(<i>x</i> <i>x</i>   <i>x</i> ) 16(<i>x</i> <i>x</i>   <i>x</i> ).


<i>Chứng minh rằng, với mọi </i>1   <i>i</i> <i>j</i> <i>k</i> 16,<i> ta có x xi</i>, <i>j</i>, <i>xk</i> <i>là độ dài ba cạnh của một </i>
<i>tam giác. </i>


</div>
<span class='text_page_counter'>(40)</span><div class='page_container' data-page=40>

<i><b>Bài toán 5.2. Cho s</b>ố tự nhiên n</i>3<i> và a</i><sub>1</sub>, <i>a</i><sub>2</sub>,,<i>an là các số thực dương thỏa mãn </i>
2 2


2 2 4


2


1 2



4 4


1


) ( 1)(


(<i>a</i> <i>a</i> <i>a<sub>n</sub></i>  <i>n</i> <i>a</i> <i>a</i>   <i>a<sub>n</sub></i>).


<i>Chứng minh rằng, với mọi </i>1   <i>i</i> <i>j</i> <i>k</i> <i>n</i>,<i> ta có a a<sub>i</sub></i>, <i><sub>j</sub></i>,<i>a<sub>k</sub></i> <i>là độ dài ba cạnh của một </i>
<i>tam giác. </i>


Cách giải của bài toán tổng qt hồn tốn giống với bài tốn 5.1, đó là từ giả thiết, ta sẽ


tìm cách đánh giá đưa về ba biến rồi sau đó sử dụng khai triển quen thuộc


2 2 2 2 4 4 4


(<i>a</i> <i>b</i> <i>c</i> ) 2(<i>a</i> <i>b</i> <i>c</i> )(<i>a</i> <i>b</i> <i>c b</i>)(  <i>c</i> <i>a c</i>)(  <i>a</i> <i>b a</i>)(  <i>b</i> <i>c</i>)


đểsuy ra điều phải chứng minh.


Qua lời giải được trình bày ở trên, có thể thấy được đánh giá


3 4 2


24<i>t</i>  <i>t</i> 16<i>t</i> 9 ,<i>t</i>  <i>t</i> (0, 1)


chính là mấu chốt, là chìa khóa quan trọng để giải bài toán. Và khi giải bài tốn này,
chúng tơi có cảm giác rằng tác giả bài toán đã phát hiện ra đánh giá



3 4 2


24<i>t</i>  <i>t</i> 16<i>t</i> 9<i>t</i> ,  <i>t</i> (0, 1) trước rồi sau đó đi ngược từ bài toán 5.2 để đi đến bài


tốn đã cho. Rất có thểđây chính là cách mà tác giảđã tạo ra bài tốn.
Việc nhận ra được bài toán gốc cũng như bất đẳng thức trung gian


3 4 2


24<i>t</i>  <i>t</i> 16<i>t</i> 9 ,<i>t</i>  <i>t</i> (0, 1)


là điều khơng hề dễ dàng gì, và chính điều này đã gây khó khăn cho khơng ít thí sinh


trong kỳ thi vừa qua. Tuy nhiên, theo quan điểm cá nhân của mình, chúng tơi nghĩ rằng
việc sử dụng “mẹo” che giấu đề làm khó bài tốn như thế là khơng nên. Nên có những
bài tốn với ý tưởng tựnhiên hơn.


<i>Lời giải và bình luận bài 5 này được thực hiện bởi Võ Quốc Bá Cẩn. </i>


<b>Bài 6. </b>


<i><b>Có 42 h</b><b>ọ</b><b>c sinh tham d</b><b>ự</b><b> kì thi ch</b><b>ọn độ</b><b>i tuy</b><b>ể</b><b>n Olympic toán qu</b><b>ố</b><b>c t</b><b>ế</b><b>. Bi</b><b>ế</b><b>t r</b><b>ằ</b><b>ng m</b><b>ộ</b><b>t h</b><b>ọ</b><b>c </b></i>
<i><b>sinh b</b><b>ấ</b><b>t kì quen </b><b>đúng 20 họ</b><b>c sinh khác. </b></i>


<i><b>Ch</b><b>ứ</b><b>ng minh r</b><b>ằ</b><b>ng ta có th</b><b>ể</b><b> chia 42 h</b><b>ọ</b><b>c sinh thành 2 nhóm ho</b><b>ặ</b><b>c 21 nhóm sao cho s</b><b>ố</b></i>
<i><b>h</b><b>ọ</b><b>c sinh trong các nhóm b</b><b>ằ</b><b>ng nhau và 2 h</b><b>ọ</b><b>c sinh b</b><b>ấ</b><b>t kì nào trong cùng nhóm thì </b></i>
<i><b>quen nhau. </b></i>


<b>Lời giải. </b>



</div>
<span class='text_page_counter'>(41)</span><div class='page_container' data-page=41>

Bài tốn có thể phát biểu dưới dạng đồ thị một cách hiển nhiên: một đồ thị trên 2<i>n</i>2


đỉnh, bậc mỗi đỉnh bằng n thì hoặc là hợp rời của 2 đồ thị đầy đủ trên <i>n</i>1 đỉnh hoặc có
thể tìm được <i>n</i>1 cạnh với đầu mút đôi một rời nhau.


Ta xét bổ đề sau: <i>Một đồ thị</i> <i>liên thông trên N đỉnh, bậc nhỏ nhất bằng d (bậc mỗi đỉnh </i>


<i>khơng ít hơn d) ln có một đường đi có độdài khơng ít hơn </i>min 2 ,

<i>d N</i>1

<i>. </i>


Quay trở lại bài toán, nếu G khơng liên thơng thì hiển nhiên điều kiện về bậc cho thấy G


được tạo thành từhai đồ thịđầy đủ trên <i>n</i>1đỉnh.


Giả sử G liên thông, thế thì theo bổđề trên, G có một đường đicó độ dài 2n hoặc 2<i>n</i>1.
Nói rằng <i>G </i>có đường đi độ dài 2<i>n</i>1 nghĩa là nói <i>G </i>có đường đi Hamilton (đi qua


2<i>n</i>2 đỉnh, mỗi đỉnh 1 lần), trường hợp này hiển nhiên cho ta lời giải bài toán.


Giả sửG có đường đi độ dài 2n: <i>A B A B</i><sub>1</sub> <sub>1</sub> <sub>2</sub> <sub>2</sub>...<i>A B A<sub>n</sub></i> <i><sub>n</sub></i> <i><sub>n</sub></i><sub></sub><sub>1</sub>. Gọi đỉnh còn lại là <i>A<sub>n</sub></i><sub></sub><sub>2</sub>. Nếu <i>A<sub>n</sub></i><sub></sub><sub>2</sub>
kề <i>A<sub>i</sub></i> với <i>i </i>nào đó thì các cạnh <i>A<sub>n</sub></i><sub></sub><sub>2</sub><i>A A B<sub>i</sub></i>, <i><sub>j</sub></i> <i><sub>j</sub></i>, <i>j</i><i>i</i> và <i>B A<sub>j</sub></i> <i><sub>j</sub></i><sub></sub><sub>1</sub>, <i>j</i><i>i</i> thỏa mãn bài tốn.
Giả sử <i>An</i>2 khơng kề với bất kì <i>Ai</i>, thế thì do bậc của <i>An</i>2 bằng n nên ta suy ra <i>An</i>2 kề


<i>i</i>


<i>B</i> với mọi i.


Lập luận như trên với đường đi <i>A<sub>n</sub></i><sub></sub><sub>2</sub><i>B A</i><sub>1</sub> <sub>2</sub>...<i>A B A<sub>n</sub></i> <i><sub>n</sub></i> <i><sub>n</sub></i><sub></sub><sub>1</sub> (thay đổi vai trò của <i>A</i><sub>1</sub> với <i>A<sub>n</sub></i><sub></sub><sub>2</sub>) ta
suy ra <i>A</i><sub>1</sub> kề <i>B<sub>i</sub></i> với mọi i.


Tương tự, suy luận với đường đi <i>A B A</i><sub>1</sub> <sub>1</sub> <i><sub>n</sub></i><sub></sub><sub>2</sub><i>B A B</i><sub>2</sub> <sub>3</sub> <sub>3</sub>...<i>A B A<sub>n</sub></i> <i><sub>n</sub></i> <i><sub>n</sub></i><sub></sub><sub>1</sub> (thay đổi vai trò của <i>A</i><sub>2</sub> với


2


<i>n</i>


<i>A</i> ) ta suy ra <i>A</i>2 kề <i>Bi</i> với mọi i.


Bằng cách này, ta dễ dàng suy ra <i>A<sub>j</sub></i> kề <i>B<sub>i</sub></i> với mọi i, j. Thế nhưng khi đó mỗi đỉnh Bi có
bậc bằng <i>n</i>2, vơ lý.


Ta có điều phải chứng minh.


Để kết thúc phép chứng minh, ta sẽ chứng minh bổđề.


Gọi G là đồ thị có liên thơng có N đỉnh và bậc mỗi đỉnh không nhỏhơn <i>d. V</i>ới X là đồ thị


bất kỳ ta ký hiệu <i>V X E X</i>( ), ( ) tương ứng là tập các đỉnh, các cạnh của X.


Trước hết ta có nhận xét sau:


Nếu <i>P là m</i>ột đường đi trong <i>G. N</i>ếu tồn tại phần tử <i>x</i><i>V G</i>( ) \ ( )<i>V P</i> thì tồn tại
( ) \ ( )


</div>
<span class='text_page_counter'>(42)</span><div class='page_container' data-page=42>

Thật vậy, giả sử <i>x</i><i>V G</i>( ) \ ( )<i>V P</i> và <i>v</i><i>V P</i>( ). Do tính liên thơng, tồn tại đường đi <i>Q t</i>ừ x


đến v. Lấy đỉnh y là đỉnh kềtrước của v trong Q ta được điều phải chứng minh.


Bây giờ gọi <i>P</i><i>a b</i>... là đường đi dài nhất trong <i>G. G</i>ọi <i>k </i>là độ dài của <i>P và gi</i>ả sử





min 2 , 1


<i>k</i> <i>d N</i> .


Nếu <i>ab</i><i>E G</i>( ) thì <i>yzPabPz</i>* là đường đi trong <i>G </i>dài hơn <i>P </i>(đường đi từ <i>y </i>đến z, từ <i>z </i>
theo P đến <i>a</i>, đến b, từ b theo P đến z* là đỉnh kề với z trong P).


Nếu <i>ab</i><i>E G</i>( ), gọi <i>K</i><i>V P</i>( ) là tập các đỉnh trước của các đỉnh kề a trong <i>P (v</i>ới <i>P </i>


được sắp thứ tự từ <i>a</i> đến b). Vì tất cảcác đỉnh kề của <i>a</i> phải nằm trong P, ta có <i>K</i> <i>d</i>.
Vì <i>k</i>2<i>d</i>, ta có

<i>V P</i>( ) \<i>K</i>

\<i>b</i> <i>d</i>. Như vậy tồn tại <i>q</i><i>K</i> kề với b. Bây giờ ta xét chu
trình <i>C</i><i>aPqbPq a</i>* , khi đó <i>yzCz* (theo chi</i>ều bất kỳ của C) là đường đi trong <i>G </i>dài hơn


<i>P, mâu thu</i>ẫn.


Từ mâu thuẫn ở trên, ta suy ra <i>k</i>min 2 ,

<i>d N</i>1

. Bổđềđược chứng minh.


<b>Cách 2.</b> (C<i><b>ủ</b><b>a Traum@ d</b><b>ự</b><b>a trên ý t</b><b>ưở</b><b>ng c</b><b>ủ</b><b>a mnnn@ ) </b></i>


Nhắc lại một c<i>ặp ghép (matching) hay tập cạnh độc lập c</i>ủa một đồ thị G là một tập các
cạnh không có đỉnh chung. C<i>ặp ghép cực đại (maximal matching) c</i>ủa G là cặp ghép mà
không thể mở rộng được nữa.


1. Gọi <i>S là m</i>ột cặp ghép cực đại của <i>G. N</i>ếu <i>S</i> 21 thì ta có điều phải chứng minh.
2. Nếu <i>S</i> 20, gọi V(S) là tập các đỉnh trong S.


3. Gọi A và B là hai đỉnh bất kì khơng thuộc V(S). Vì S là một cặp ghép cực đại nên A và
<i>B không k</i>ề nhau.


4. Gọi <i>N A N B</i>( ), ( ) là tập các đỉnh kề với <i>A, B trong </i> <i>V(S) </i> tương ứng thì ta có



( ) ( ) 20


<i>N A</i>  <i>N B</i>  , do bậc mỗi đỉnh là 20.


5. Với mỗi C thuộc N(A), ta gọi đỉnh còn lại của cạnh chứa C trong S là C(S). Đặt M(A)
là tập các đỉnh C(S) trên thì ta có <i>M A</i>( )  <i>N A</i>( ) 20.


</div>
<span class='text_page_counter'>(43)</span><div class='page_container' data-page=43>

cặp ghép mới lớn hơn <i>S</i>, điều này mâu thuẫn với giả thiết lớn nhất của <i>S. </i>
Vậy <i>M A</i>( )<i>N B</i>( ) 0.


7. Bởi vì <i>M A</i>( ) 20 và <i>N B</i>( ) 20 suy ra <i>V S</i>( )  <i>M A</i>( )  <i>N B</i>( ) 40, mà ở trên ta
có 40<i>V S</i>( ) nên <i>V S</i>( ) 40 hay <i>S</i> 20.


8. Từ 6. và 7, ta thấy rằng nếu C kề với A mà C(S) khơng kề với A thì ta có C kề với B và
<i>S(C) khơng k</i>ề với B.


9. Gọi X(A) là tập các đỉnh C trong V(S) mà C và C(S) đều kề với A. Tương tựta có định
nghĩa cho <i>X(B). G</i>ọi Y là tập các đỉnh kề với cả A và B và Z là các đỉnh không kề với cả A
và B. Theo các nhận xét 6, 7, 8, ta có <i>Y</i>  <i>Z</i> và <i>X A</i>( )  <i>X B</i>( ) và <i>X A</i>( ) <i>Y</i> 20.
10. Ta có các điều sau:


a. Các đỉnh thuộc X(A) chỉ có thể kề với các đỉnh thuộc X(A) hoặc Y.


b. Các đỉnh thuộc X(B) chỉ có thể kề với các đỉnh thuộc X(B) hoặc Y.
c. Các đỉnh thuộc Z chỉ có thể kề với các đỉnh thuộc Y.


11. Các đỉnh thuộc Z chỉ kề với các đỉnh thuộc Y. Vì các đỉnh thuộc Z chỉ kề với các đỉnh
thuộc Y nên ta có 20<i>Z</i> 18<i>Y</i> , mà <i>Y</i>  <i>Z</i> nên <i>Y</i>  <i>Z</i> 0.



Vậy ta có <i>X A</i>( )  <i>X B</i>( ) 20. Mà theo 10. thì các đỉnh thuộc X(A) chỉ kề với A và các


đỉnh thuộc <i>X(A). Nên ta có 21 </i>đỉnh gồm <i>A </i>và các đỉnh trong <i>X(A) t</i>ạo thành một <i>K</i><sub>21</sub>.


Tương tự ta có một <i>K</i><sub>21</sub> nữa tương ứng với B.


12. Bài toán được chứng minh xong.


<b>Bình luận. </b>


 Đây là một bài tốn về sự tồn tại cặp ghép hoàn hảo (perfect matching) trong một


đồ thị có bậc chẵn. Vì thế xu hướng sử dụng các suy luận liên quan đến cặp ghép
là rất rõ ràng.


 Lời giải 1 được định hướng bởi điều kiện rất “Dirac” của đề bài. Chú ý là theo
phiên bản cơ sở của định lý Dirac thì đồ thị trên n đỉnh có bậc của mỗi đỉnh khơng
nhỏ hơn


2
<i>n</i>


</div>
<span class='text_page_counter'>(44)</span><div class='page_container' data-page=44>

nhiên, nếu sử dụng đúng định lý Dirac thì khơng giúp ích. Ở đây, ta cần đến một
biến thể của định lý Dirac, chính là bổđề trong lời giải. Bổđể này thực chất là một
bài tập trong các tài liệu về lý thuyết Graph. Chứng minh nêu trên của bổđềđược
tham khảo tại đây:



/exercises/Graph_Theory_sol_02.pdf



 Ngoài cách phát biểu như ở định lý Dirac nguyên thủy, ta cịn có một cách phát
biểu khác cho định lý Dirac, tổng quát hơn và rất gần với bổđề:


<b>Định lý (Dirac).</b> <i>Cho G là đồ thị 2-liên thông với bậc mỗi đỉnh khơng nhỏhơn d. </i>
<i>Khi đó G chứa chu trình độ dài ít nhất là </i>min 2 ,

<i>d V G</i>( )

<i>. </i>


Có thể tham khảo về chứng minh định lý này tại:
/>


 Lời giải 2 cũng khá tự nhiên, theo hướng chứng minh nếu không tồn tại cặp ghép
hồn chỉnh thì <i>G ph</i>ải là hợp của 2 phiên bản <i>K</i><sub>21</sub> rời nhau. Cũng như trong lời
giải 1, việc sử dụng nguyên lý cực hạn (cặp ghép cực đại) đem lại những thông tin
bổ sung quan trọng (A, B thuộc <i>V G</i>( ) \<i>S</i> không kề nhau) dẫn đến lời giải bài tốn.
 Ngồi hai cách giải trên còn một cách giải khác sử dụng đến một kết quả về cặp


ghép hoàn hảo là định lý Tutte sau:


<b>Định lý Tutte.</b> <i>Đồ thị G = (V, E) có cặp ghép hồn hảo khi và chỉ khi với mọi tập </i>
<i>con U của V, số thành phần liên thơng có sốđỉnh lẻtrong đồ thị con cảm sinh bởi </i>


\


<i>V U nhỏhơn hay bằng U . </i>


Chứng minh định lý Tutte có thể tham khảo tại:
/>


Lời giải bài 6 sử dụng định lý Tutte tham khảo tại:


/>


 Đây là một bài tốn khó đối với các thí sinh không quen thuộc với lý thuyết đồ thị,



nhưng đối với các bạn đã biết về cặp ghép hoàn hảo hoặc định lý Dirac (đặc biệt là


phương pháp suy luận đường đi dài nhất – <i>longest path argument) thì bài này </i>


khơng q khó. Đây là điểm khiến chúng tơi đánh giá tính phân loại của bài này
không cao bằng bài 2, và không những thế, tạo nhiều lợi thế cho những bạn “biết


</div>
<span class='text_page_counter'>(45)</span><div class='page_container' data-page=45>

1

<b>LỜI GIẢI VÀ BÌNH LUẬN </b>



<b>ĐỀ CHỌN ĐỘI TUYỂN QUỐC GIA </b>


<b>DỰ THI IMO NĂM 2013 </b>



Những người thực hiện: Trần Nam Dũng



Trần Quang Hùng



Võ Quốc Bá Cẩn


Lê Phúc Lữ




<i>Xin chân thành cảm ơn thầy Nguyễn Tăng Vũ, thầy Trần Quốc Luật, các bạn Nguyễn </i>
<i>Văn Quý, Võ Anh Đức, Hoàng Đỗ Kiên, Phạm Tuấn Huy, Nguyễn Huy Tùng cùng </i>
<i>nhiều thành viên của các diễn đàn đã đóng góp ý kiến để chúng tơi hồn tất tài liệu này! </i>


</div>
<span class='text_page_counter'>(46)</span><div class='page_container' data-page=46>

2

<b>Phần 1. </b>



<b>ĐỀ THI CHÍNH THỨC</b>


<i><b>Ngày thi thứ nhất. </b></i>




<b>Bài 1. </b>


Cho tứ giác <i>ABCD</i> có các cạnh đối không song song nội tiếp đường tròn ( , )<i>O R . </i>
Gọi <i>E</i> là giao điểm hai đường chéo và đường phân giác góc <i>AEB</i> cắt các đường thẳng


, , ,


<i>AB BC CD DA lần lượt tại các điểm M N P Q . </i>, , ,


1. Chứng minh rằng các đường tròn (<i>AQM</i>),(<i>BMN</i>),(<i>CNP</i>),(<i>DPQ cùng đi qua một </i>)
điểm duy nhất. Gọi điểm đó là <i>K</i>.


2. Đặt min

<i>AC BD</i>,

<i>m</i>. Chứng minh rằng


2


2 2


2


.
4


<i>R</i>
<i>OK</i>


<i>R</i> <i>m</i>







<b>Bài 2. </b>


1. Chứng minh rằng tồn tại vô số số nguyên dương <i>t</i> sao cho 2012<i>t</i>1 và 2013<i>t</i>1 đều
là các số chính phương.


2. Xét ,<i>m n là các số nguyên dương sao cho mn</i>1 và (<i>m</i>1)<i>n</i>1 đều là các số chính
phương. Chứng minh rằng <i>n chia hết cho </i>8(2<i>m</i>1).


<b>Bài 3. </b>


Với mỗi số <i>n nguyên dương, đặt S<sub>n</sub></i>

0,1, 2,..., 2<i>n</i>1

. Xét hàm số <i>f</i> : (<i>S<sub>n</sub></i>)[0;1]


thỏa mãn đồng thời các điều kiện sau:


i/ <i>f x</i>( ,0) <i>f x n</i>( , 2 1) 0 với mọi số nguyên <i>x </i>.


ii/ <i>f x</i>( 1, )<i>y</i>  <i>f x</i>( 1, )<i>y</i>  <i>f x y</i>( , 1) <i>f x y</i>( , 1) 1 với <i>x y</i>,  và 1<i>y</i>2<i>n</i>.
Gọi <i>F</i> là tập hợp tất cả các hàm số <i>f thỏa mãn. </i>


1. Chứng minh rằng <i>F là vô hạn. </i>


</div>
<span class='text_page_counter'>(47)</span><div class='page_container' data-page=47>

3


<i><b>Ngày thi thứ hai. </b></i>



<b>Bài 4. </b>



Tìm hằng số <i>k</i> nguyên dương lớn nhất thỏa mãn: Với mọi <i>a b c dương mà </i>, , <i>abc</i>1 thì
ta có bất đẳng thức sau


1 1 1


3
1 4


<i>k</i> <i>k</i>


<i>a</i><i>b</i><i>c</i><i>a b c</i>     .


<b>Bài 5. </b>


Cho tam giác <i>ABC</i> nhọn khơng cân có <i>A</i> bằng 45. Các đường cao <i>AD BE CF đồng </i>, ,
quy tại trực tâm <i>H</i>. Đường thẳng <i>EF</i> cắt đường thẳng <i>BC</i> tại <i>P</i>. Gọi <i>I</i> là trung điểm
của <i>BC</i>; đường thẳng <i>IF</i> cắt <i>PH</i> tại <i>Q </i>.


1. Chứng minh rằng <i>IQH</i>  <i>AIE</i>.


2. Gọi <i>K</i> là trực tâm của tam giác <i>AEF</i> và ( )<i>J là đường tròn ngoại tiếp tam giác KPD</i>.
Đường thẳng <i>CK</i> cắt ( )<i>J tại G</i>, đường thẳng <i>IG</i> cắt ( )<i>J tại M</i>, đường thẳng <i>JC cắt </i>
đường trịn đường kính <i>BC</i>tại <i>N</i>. Chứng minh rằng các điểm <i>G M N C cùng thuộc </i>, , ,
một đường tròn.


<b>Bài 6. </b>


</div>
<span class='text_page_counter'>(48)</span><div class='page_container' data-page=48>

4

<b>Phần 2. </b>




<b>LỜI GIẢI CHI TIẾT VÀ BÌNH LUẬN </b>



<b>Bài 1. </b>


<i><b>Cho tứ giác </b>ABCD<b> lồi có các cạnh đối không song song nội tiếp đường tròn ( , )</b><b>O R . </b></i>
<i><b>Gọi </b>E<b> là giao điểm hai đường chéo và đường phân giác góc </b>AEB<b> cắt các đường thẳng </b></i>


, , ,


<i><b>AB BC CD DA lần lượt tại các điểm </b><b>M N P Q . </b></i>, , ,


<i><b>1. Chứng minh rằng các đường tròn (</b>AQM</i>),(<i>BMN</i>),(<i>CNP</i>),(<i><b>DPQ cùng đi qua một điểm </b></i>)
<i><b>duy nhất. Gọi điểm đó là </b>K<b>. </b></i>


<i><b>2. Đặt </b></i>min

<i>AC BD</i>,

<i>m<b>. Chứng minh rằng </b></i>


2


2 2


2


.
4


<i>R</i>
<i>OK</i>


<i>R</i> <i>m</i>







<b>Lời giải. </b>


<i><b>R</b></i>



<i><b>S</b></i>


<i><b>K</b></i>



<i><b>P</b></i>


<i><b>M</b></i>



<i><b>Q</b></i>



<i><b>N</b></i>



<i><b>E</b></i>


<i><b>O</b></i>



<i><b>A</b></i>



<i><b>B</b></i>



<i><b>C</b></i>


<i><b>D</b></i>



</div>
<span class='text_page_counter'>(49)</span><div class='page_container' data-page=49>

5
giữa <i>A S và nằm giữa ,</i>, <i>C R như hình vẽ. Gọi K</i> là giao điểm của đường tròn ngoại tiếp


các tam giác <i>RAB SBC thì </i>,


180


<i>BKR</i> <i>BKS</i> <i>BAD</i> <i>BCD</i>


         hay <i>R K S thẳng hàng. </i>, ,


Suy ra <i>RK RS</i> <i>RB RC</i> <i>RA RD</i> và <i>SK SR SB SA SC SD</i>     nên các tứ giác <i>ADSK</i>


và <i>CDRK</i> cũng nội tiếp hay <i>K</i> cũng thuộc về các đường tròn (<i>RCD và (</i>) <i>SDA </i>).


Do đó, ta có


<i>AKD</i> <i>ASD</i> <i>BSC</i> <i>BKC</i>


       và <i>ADK</i> <i>ASK</i> <i>BSK</i> <i>BCK</i>


nên các tam giác <i>KAD</i> và <i>KBC</i> đồng dạng. Suy ra <i>KA</i> <i>AD</i> <i>AE</i> <i>AM</i>


<i>KB</i>  <i>BC</i>  <i>BE</i>  <i>BM</i> , theo tính


chất đường phân giác thì <i>KM</i> là phân giác của góc <i>AKB</i>.


Mặt khác, ta có <i>RNQ</i> <i>BNE</i> <i>CBD</i> <i>BEN</i> <i>CAD</i> <i>AEQ</i> <i>RQN</i> nên ta được


2


<i>ARB</i> <i>BNM</i>



   .


Từ đó suy ra 1 1


2 2


<i>BKM</i> <i>AKB</i> <i>ARB</i> <i>BNM</i>


       hay tứ giác <i>BMNK</i> nội tiếp, tức là


<i>K</i> thuộc đường tròn (<i>BMN . </i>)


Chứng minh tương tự, ta cũng có <i>K</i> thuộc các đường tròn (<i>AQM</i>),(<i>CNP</i>),(<i>DPQ . </i>)


Tiếp theo, ta sẽ chứng minh rằng <i>K</i> là điểm chung duy nhất của các đường tròn này.
Thật vậy, các đường trịn (<i>AMP</i>),(<i>BMQ có hai điểm chung là </i>) <i>K M còn các đường tròn </i>,
(<i>DNP CNP có hai điểm chung là </i>),( ) <i>K N . Do đó, nếu bốn đường trịn này có hai điểm </i>,
chung thì <i>M N trùng nhau, vơ lí. </i>,


Ta có đpcm.


2. Theo tính chất phương tích thì


2 2 2 2


,


<i>RK RS RB RC</i>   <i>RO</i> <i>R SK SR SB SA SO</i>    <i>R</i> nên


2 2 2 2



<i>RO</i> <i>SO</i> <i>RK RS SK SR RK</i>    <i>SK</i> .
Từ đó suy ra <i>OK</i><i>RS</i>.


Hơn nữa, theo định lí Brocard trong tứ giác nội tiếp <i>ABCD</i> thì <i>E</i> chính là trực tâm của
tam giác <i>ORS</i>, suy ra <i>OE</i><i>RS</i>.


</div>
<span class='text_page_counter'>(50)</span><div class='page_container' data-page=50>

6
Ta lại có <i>RKA</i> <i>SKC</i> <i>RBA</i> <i>SBC</i> 2 <i>ADC</i> <i>AOC</i> nên <i>AKC</i> <i>AOC</i>180


hay tứ giác <i>AOCK</i> nội tiếp. Suy ra


2 2 <sub>(</sub> <sub>)</sub> 2


<i>EO EK</i> <i>EA EC</i> <i>R</i> <i>OE</i> <i>EO EO EK</i> <i>R</i> hay


2
<i>R</i>
<i>OK</i>


<i>EO</i>
 .
Mặt khác, theo bất đẳng thức cơ bản về đường xiên và đường vng góc thì


1 2 2 1 2 2 1 2 2


max ( , ), ( , ) max 4 , 4 4


2 2 2



<i>EO</i> <i>d O AC d O BC</i>  <sub></sub> <i>R</i> <i>AC</i> <i>R</i> <i>BD</i> <sub></sub> <i>R</i> <i>m</i>


  .


Vậy ta được


2


2 2


2
4


<i>R</i>
<i>OK</i>


<i>R</i> <i>m</i>






. Đây chính là đpcm.


<b>Nhận xét. </b>


Lời giải của bài toán dựa trên hai định lý rất cơ bản của hình học là điểm Miquel của tứ
giác tồn phần và định lý Brocard.


Nói đến định lý Miquel thì nhiều người đã quen thuộc nó ở dạng phát biểu sau



<b>Định lý Miquel. Cho tam giác </b><i>ABC</i>và các điểm <i>D E F lần lượt thuộc các đường thẳng </i>, ,
, ,


<i>BC CA AB . Khi đó các đường tròn ngoại tiếp các tam giác (AEF</i>),(<i>BFD CDE có một </i>),( )
<i>điểm chung M gọi là điểm Miquel. </i>


<b>Hệ quả 1. Các điểm </b><i>D E F thẳng hàng khi và chỉ khi điểm Miquel </i>, , <i>M</i> thuộc đường
tròn (<i>ABC . </i>)


Hệ quả trên rất có ý nghĩa nếu ta coi một tam giác và một đường thẳng là một tứ giác
tồn phần khi đó ta có thể phát biểu lại hệ quả đó như sau


<b>Hệ quả 2. Cho tứ giác </b><i>ABCD</i> có <i>AB</i> giao <i>CD</i> tại <i>E</i>, <i>AD</i> giao <i>BC</i> tại <i>F</i>. Khi đó các
đường trịn (<i>EAD</i>),(<i>EBC</i>),(<i>FAB</i>),(<i>FCD có một điểm chung M. Điểm </i>) <i>M</i> gọi là điểm
Miquel của tứ giác <i>ABCD</i>.


<i>Điểm Miquel M đặc biệt có rất nhiều tính chất thú vị khi tứ giác ABCD</i> nội tiếp. Một
trong những tính chất quan trọng là như sau


<i><b>Hệ quả 3. Cho tứ giác ABCD nội tiếp đường tròn </b></i>( )<i>O . Giả sử AB</i> giao <i>CD</i> tại <i>E</i>, <i>AD</i>


giao <i>BC</i> tại <i>F</i>, <i>AC</i> giao <i>BD</i> tại <i>G</i>. Khi đó các đường trịn (<i>EAD</i>),(<i>EBC</i>),(<i>FAB</i>),(<i>FCD </i>)
có một điểm chung <i>M</i> và <i>O G M thẳng hàng. </i>, ,


</div>
<span class='text_page_counter'>(51)</span><div class='page_container' data-page=51>

7
<i><b>Định lý Brocard. Cho tứ giác ABCD nội tiếp đường tròn </b></i>( )<i>O . Giả sử AB</i> giao <i>CD</i> tại <i>E</i>,


<i>AD</i> giao <i>BC</i> tại <i>F</i>, <i>AC</i> giao <i>BD</i> tại <i>G. Khi đó O là trực tâm tam giác EFG</i>.



Một trong những bài tốn ứng dụng hay của định lý này chính là bài thi quốc gia Việt
Nam năm 2012 vừa qua.


Chứng minh của định lý Miquel và các hệ quả có thể coi là một trong những thể hiện
quan trọng nhất của việc phải sử dụng góc định hướng trong hình học. Ở bài này, nếu
khơng dùng độ dài đại số cũng như góc định hướng, ta phải quy ước vị trí các điểm
như đã trình bày để lời giải được chặt chẽ.


Bài thi là một trong những ý tưởng hay cho việc kết hợp định lý Miquel và định lý
Brocard đặc biệt là câu 2). Ý tưởng của câu này có lẽ bắt nguồn từ đẳng thức


2
<i>OK OE OR OS</i>   <i>R</i>


 


<i>; tuy nhiên, để giấu đi sự hiện diện điểm E thì bài thi đã đưa về </i>
một bất đẳng thức liên hệ giữa đường xiên và hình chiếu, đó là




2 2


2 min ,


max ( , ), ( , )


4


<i>AC BD</i>


<i>OE</i> <i>d O AC d O BD</i> <i>R</i>  .


<i>Thực sự ý tưởng muốn giấu đi điểm E khá hay xong việc phải dùng đến một bất đẳng </i>
thức hình học đã khiến cho bài tốn mất đi khá nhiều vẻ đẹp của nó.


Bài tốn này cũng có thể giải bằng cách sử dụng bổ đề sau liên quan đến phép biến
hình như sau:


Cho hai đoạn thẳng <i>AB CD sao cho </i>, <i>ABCD</i> khơng phải là hình thang. Khi đó, tồn tại
một phép vị tự quay tâm <i>O</i> biến <i>AB</i> thành <i>CD</i>. Nếu <i>P</i> là giao điểm của <i>AB</i> và <i>CD </i>,


<i>Q là giao điểm của AD</i> và <i>BC</i> thì các tứ giác <i>ADPK BCPK ABQK CDQK nội tiếp. </i>, , ,


Trong bài tốn đã cho, điểm <i>K</i> cũng chính là tâm của phép vị tự quay đó.


Ý tưởng sử dụng bất đẳng thức ở đây khá giống với bài hình số 5 trong đề thi TST 2006,
cũng là sự so sánh giữa đường xiên và đường vng góc. Tuy nhiên, ở đây ta khơng
cần chỉ ra với trường hợp nào thì đẳng thức xảy ra.


Nếu tổng quát lên, thay đường phân giác góc <i>E</i> thành đường thẳng bất kì qua <i>E</i>, ta có
bài tốn sau:


</div>
<span class='text_page_counter'>(52)</span><div class='page_container' data-page=52>

8
<i><b>thẳng </b><b>AB BC CD DA lần lượt tại các điểm </b></i>, , , <i><b>M N P Q . Chứng minh rằng giao điểm </b></i>, , ,
<i><b>khác </b><b>M N P Q của các cặp đường tròn (</b></i>, , , <i>AQM</i>),(<i><b>BMN ; (</b></i>) <i>BMN</i>),(<i><b>CNP ; (</b></i>) <i>CNP</i>),(<i><b>DPQ </b></i>)
<i><b>và (</b>DPQ</i>),(<i><b>AQM cùng thuộc một đường tròn qua điểm Miquel </b></i>) <i>K<b> của tứ giác </b>ABCD</i>.


<i><b>K</b></i>



<i><b>N</b></i>



<i><b>P</b></i>



<i><b>M</b></i>


<i><b>Q</b></i>



<i><b>R</b></i>



<i><b>S</b></i>


<i><b>E</b></i>



<i><b>O</b></i>


<i><b>A</b></i>



<i><b>B</b></i>



<i><b>C</b></i>



<i><b>D</b></i>


Trong trường hợp đường thẳng bất kì trên trở thành phân giác thì bốn giao điểm trên
trùng nhau và trùng với <i>K</i>.


<b>Bài 2. </b>


<i><b>1. Chứng minh rằng tồn tại vô số số nguyên dương </b>t<b> sao cho </b></i>2012<i>t</i>1<i><b> và </b></i>2013<i>t</i>1<i><b> đều </b></i>
<i><b>là các số chính phương. </b></i>


<i><b>2. Xét ,</b><b>m n là các số nguyên dương sao cho </b>mn</i>1<i><b> và </b></i>(<i>m</i>1)<i>n</i>1<i><b> đều là các số chính </b></i>
<i><b>phương. Chứng minh rằng </b><b>n chia hết cho </b></i>8(2<i>m</i>1)<i><b>. </b></i>



</div>
<span class='text_page_counter'>(53)</span><div class='page_container' data-page=53>

9
1. Đặt <i>d</i>(2012<i>t</i>1, 2013<i>t</i>1) thì dễ thấy <i>d</i>1. Do đó, 2012<i>t</i>1 và 2013<i>t</i>1 đều là các
số chính phương khi và chỉ khi (2012<i>t</i>1)(2013<i>t</i>1)<i>y</i>2 với <i>y</i> là số nguyên dương nào
đó. Ta biến đổi đẳng thức trên


2


2 2 2 2


2 2


(2012 1)(2013 1)


4 2012 2013 4 2012 2013 4025 4 2012 2013 4 2012 2013
(2 2012 2013 4025) 1 4 2012 2013


<i>t</i> <i>t</i> <i>y</i>


<i>t</i> <i>t</i> <i>y</i>


<i>t</i> <i>y</i>


  


             


        


Đặt <i>x</i> 2 2012 2013 <i>t</i>4025 thì ta có phương trình <i><sub>x</sub></i>2 <sub> </sub><sub>4 2012 2013</sub><sub></sub> <i><sub>y</sub></i>2 <sub></sub><sub>1</sub><sub>. </sub>



Dễ thấy 4 2012 2013  không phải là số chính phương nên phương trình Pell loại 1 này
có vơ số nghiệm. Nghiệm nhỏ nhất của phương trình này là ( , ) (4025,1)<i>x y</i>  nên các
nghiệm của nó được cho bởi cơng thức


0 1 2 1


0 1 2 1


1, 4025, 8050


, 0
1, 1, 8050


<i>n</i> <i>n</i> <i>n</i>


<i>n</i> <i>n</i> <i>n</i>


<i>x</i> <i>x</i> <i>x</i> <i>x</i> <i>x</i>


<i>n</i>


<i>y</i> <i>y</i> <i>y</i> <i>y</i> <i>y</i>


 
 
    


   


.


Bằng quy nạp, ta chứng minh được <i>x</i><sub>2 1</sub><i><sub>i</sub></i><sub></sub> chia 2 2012 2013  dư 4025 với mọi <i>i</i> và mỗi


giá trị nguyên dương 2 1 4025
2 2012 2013


<i>i</i>


<i>x</i> <sub></sub> 


  sẽ cho ta một giá trị <i>t</i> thỏa mãn đề bài.


Vậy tồn tại vô số giá trị nguyên dương <i>t</i>sao cho 2012<i>t</i>1 và 2013<i>t</i>1 đều là các số
chính phương. Ta có đpcm.


2. Đặt <i>d</i>(<i>mn</i>1,<i>mn n</i> 1) thì


1 1



<i>d mn n</i>  <i>mn</i> hay <i>d n , suy ra (d mn</i> 1 <i>mn</i>) hay<i>d . </i>1
Do đó <i>d</i>1 hay các số <i>mn</i>1,(<i>m</i>1)<i>n</i>1 nguyên tố cùng nhau.


Khi đó, <i>mn</i>1 và (<i>m</i>1)<i>n</i>1 đều là các số chính phương khi và chỉ khi




(<i>mn</i>1) (<i>m</i>1)<i>n</i>1 là số chính phương.
Giả sử (<i>mn</i>1) (

<i>m</i>1)<i>n</i>1

<i>y</i>2 với <i>y</i> 



 . Biến đổi biểu thức này, ta thu được




2 2


2 2 2 2


2 <sub>2</sub>


( 1) (2 1) 1


4 ( 1) 4 ( 1)(2 1) 4 ( 1) 4 ( 1)
2 ( 1) (2 1) 1 4 ( 1)


<i>m m</i> <i>n</i> <i>m</i> <i>n</i> <i>y</i>


<i>m m</i> <i>n</i> <i>m m</i> <i>m</i> <i>n</i> <i>m m</i> <i>m m</i> <i>y</i>


<i>m m</i> <i>n</i> <i>m</i> <i>m m</i> <i>y</i>


    


        


</div>
<span class='text_page_counter'>(54)</span><div class='page_container' data-page=54>

10
Đặt <i>x</i>2 (<i>m m</i>1)<i>n</i>(2<i>m</i>1) thì ta có phương trình sau


2 2



4 ( 1) 1


<i>x</i>  <i>m m</i> <i>y</i>  (*)


Đây chính là phương trình Pell loại 1 và do 4 (<i>m m</i>1) không là số chính phương với
mọi <i>m ngun dương nên (*) có vơ số nghiệm. </i>


Phương trình (*) có nghiệm nhỏ nhất là ( , ) (2<i>x y</i>  <i>m</i>1,1) nên công thức nghiệm ( , )<i>x y<sub>i</sub></i> <i><sub>i</sub></i>


của nó có thể được viết dưới dạng


0 1 2 1


0 1 2 1


1, 2 1, 2(2 1)


, 0
0, 1, 2(2 1)


<i>i</i> <i>i</i> <i>i</i>


<i>i</i> <i>i</i> <i>i</i>


<i>x</i> <i>x</i> <i>m</i> <i>x</i> <i>m</i> <i>x</i> <i>x</i>


<i>i</i>


<i>y</i> <i>y</i> <i>y</i> <i>m</i> <i>y</i> <i>y</i>



 
 
      


    

.


Bằng quy nạp, ta sẽ chứng minh rằng <i>x<sub>2i</sub></i> chia 2 (<i>m m</i>1) dư 1 và <i>x</i><sub>2 1</sub><i><sub>i</sub></i> chia 2 (<i>m m</i>1) dư


2<i>m</i>1 với mọi <i>i</i>0,1, 2,...(**)


Thật vậy,


- Với <i>i</i>0, theo công thức truy hồi của dãy ( )<i>x<sub>i</sub></i> thì ta thấy khẳng định (**) đúng.


- Giả sử (**) đúng đến <i>i</i>, tức là <i>x<sub>2i</sub></i> chia 2 (<i>m m</i>1) dư 1 và <i>x</i><sub>2 1</sub><i><sub>i</sub></i> chia 2 (<i>m m</i>1) dư 2<i>m</i>1.
Ta có


2<i>i</i> 2 2(2 1) 2<i>i</i> 1 2<i>i</i> 2(2 1)(2 1) 1 8 ( 1) 1 1 (mod 2 ( 1))


<i>x</i>   <i>m</i> <i>x</i>  <i>x</i>  <i>m</i> <i>m</i>   <i>m m</i>   <i>m m</i> và


2<i>i</i> 3 2(2 1) 2<i>i</i> 2 2<i>i</i> 1 2(2 1) (2 1) 2 1 (mod 2 ( 1))


<i>x</i>   <i>m</i> <i>x</i>  <i>x</i>   <i>m</i>  <i>m</i>  <i>m</i> <i>m m</i> .


Do đó, (**) cũng đúng với <i>i</i>1.



Theo ngun lí quy nạp, (**) đúng với mọi số tự nhiên <i>i</i>.


Tiếp theo, ta sẽ xây dựng công thức truy hồi cho dãy <i>r<sub>i</sub></i> <i>x</i><sub>2</sub><i><sub>i</sub></i><sub>1</sub> với <i>i</i>0,1, 2,...


Ta có








2 2 5 2 4 2 3 2 3 2 2 2 3


2 2


2 3 2 2 2 3 2 3 2 1


2 2


2 3 2 1 1


2(2 1) 2(2 1) 2(2 1)


4(2 1) 1 2(2 1) 4(2 1) 1 ( )
4(2 1) 2 4(2 1) 2


<i>i</i> <i>i</i> <i>i</i> <i>i</i> <i>i</i> <i>i</i> <i>i</i>


<i>i</i> <i>i</i> <i>i</i> <i>i</i> <i>i</i>



<i>i</i> <i>i</i> <i>i</i> <i>i</i>


<i>r</i> <i>x</i> <i>m</i> <i>x</i> <i>x</i> <i>m</i> <i>m</i> <i>x</i> <i>x</i> <i>x</i>


<i>m</i> <i>x</i> <i>m</i> <i>x</i> <i>m</i> <i>x</i> <i>x</i> <i>x</i>


<i>m</i> <i>x</i> <i>x</i> <i>m</i> <i>r</i> <i>r</i>


      
    
  
        
         
       


Đặt <i>r<sub>i</sub></i> 2 (<i>m m</i>1)<i>s<sub>i</sub></i> (2<i>m</i>1) thì dãy ( ),<i>s i<sub>i</sub></i> 0 nguyên dương và xác định duy nhất.


</div>
<span class='text_page_counter'>(55)</span><div class='page_container' data-page=55>

11

 




2
2 1
2 2
2 1
2
2 1


2 ( 1) (2 1) 4(2 1) 2 2 ( 1) (2 1) 2 ( 1) (2 1)


2 ( 1) 2 ( 1) 4(2 1) 2 2 ( 1) 4(2 1) (2 1) 1


4(2 1) 2 8(2 1)


<i>i</i> <i>i</i> <i>i</i>


<i>i</i> <i>i</i> <i>i</i>


<i>i</i> <i>i</i> <i>i</i>


<i>m m</i> <i>s</i> <i>m</i> <i>m</i> <i>m m</i> <i>s</i> <i>m</i> <i>m m</i> <i>s</i> <i>m</i>


<i>m m</i> <i>s</i> <i>m m</i> <i>m</i> <i>s</i> <i>m m</i> <i>s</i> <i>m</i> <i>m</i>


<i>s</i> <i>m</i> <i>s</i> <i>s</i> <i>m</i>


 
 
 
            
           
      


Ta tính được <i>r</i><sub>0</sub> <i>x</i><sub>1</sub>2<i>m</i>1 nên <i>s</i><sub>0</sub> 0 và


2



1 3 2(2 1) 2(2 1) 1 (2 1) 16 ( 1)(2 1) 2 1


<i>r</i> <i>x</i>  <i>m</i> <i>m</i>   <i>m</i>  <i>m m</i> <i>m</i>  <i>m</i> nên <i>s</i><sub>1</sub>8(2<i>m</i>1).



Ta có cơng thức truy hồi của dãy số ( )<i>s<sub>i</sub></i> là




0 1


2


2 1


0, 8(2 1),


4(2 1) 2 8(2 1), 0


<i>i</i> <i>i</i> <i>i</i>


<i>s</i> <i>s</i> <i>m</i>


<i>s</i><sub></sub> <i>m</i> <i>s</i><sub></sub> <i>s</i> <i>m</i> <i>i</i>


   


      



Từ đó suy ra dãy số ( )<i>s<sub>i</sub></i> có các số hạng chia hết cho 8(2<i>m</i>1) với mọi <i>i</i>0,1, 2,...



Hơn nữa, với cách đặt <i>x</i>2 (<i>m m</i>1)<i>n</i>(2<i>m</i>1) thì dễ dàng thấy rằng <i>n thỏa mãn đề bài </i>
khi và chỉ khi <i>n s i</i> <i><sub>i</sub></i>, 1, 2, 3,...(do <i>s</i><sub>0</sub> 0 không phải là số nguyên dương).


Vậy tất cả các giá trị <i>n đều chia hết cho </i>8(2<i>m</i>1). Đây chính là đpcm.


<b>Nhận xét. </b>


Ý thứ 1 của bài toán thực ra là một trường hợp đặc biệt và cũng là một sự dẫn dắt cho ý
thứ 2. Ta có thể giải ý 1 theo cách ít tính tốn hơn bằng nhận xét:


Nếu <i>x y</i>, 1 là nghiệm của phương trình 2013<i>x</i>2 2012<i>y</i>2 1 thì <i>t</i><i>y</i>2<i>x</i>2* và các
số 2012<i>t</i>1, 2013<i>t</i>1 đều chính phương.


Hơn nữa, mỗi nghiệm ( , )<i>x y của phương trình <sub>x</sub></i>2 <sub></sub><sub>2012 2013</sub><sub></sub> <i><sub>y</sub></i>2 <sub></sub><sub>1</sub><sub> cho ta một nghiệm </sub>
(<i>x</i>2012 ,<i>y x</i>2013 )<i>y</i> của phương trình 2013<i>x</i>22012<i>y</i>2 1 nên dễ thấy đpcm.


Cách tiếp cận bằng phương trình Pell hồn tồn tự nhiên và khi đã xây dựng thành
cơng thì ta chỉ cịn cần thao tác trên các dãy số nguyên. Trong những năm gần đây, các
bài toán về dãy số nguyên khá được ưa chuộng, đặc biệt dãy số nguyên liên quan đến
cơng thức nghiệm của phương trình nghiệm ngun. Chẳng hạn như


(Việt Nam TST 2012) Cho dãy số nguyên dương ( )<i>x<sub>n</sub></i> được xác định bởi


1 2


2 1


1, 2011,


4022 , 1, 2, 3,...



<i>n</i> <i>n</i> <i>n</i>


<i>x</i> <i>x</i>


<i>x</i>  <i>x</i>  <i>x n</i>


  




  


</div>
<span class='text_page_counter'>(56)</span><div class='page_container' data-page=56>

12
Chứng minh rằng 2012 1


2012


<i>x</i> 


là số chính phương.


Trong việc giải quyết ý thứ 2, nếu ta lập luận như sau:


Do <i>mn</i>1 và (<i>m</i>1)<i>n</i>1 đều là các số chính phương nên ta có thể đặt


2 2


1 ,( 1) 1



<i>mn</i> <i>a</i> <i>m</i> <i>n</i> <i>b</i> .


Từ đó ta có phương trình <sub>(</sub><i><sub>m</sub></i><sub></sub><sub>1)</sub><i><sub>a</sub></i>2<sub></sub><i><sub>mb</sub></i>2 <sub></sub><sub>1</sub><sub>. Đổi biến </sub><i><sub>a x my b x</sub></i><sub></sub> <sub></sub> <sub>,</sub> <sub> </sub><sub>(</sub><i><sub>m</sub></i><sub></sub><sub>1)</sub><i><sub>y</sub></i><sub> để đưa </sub>
về phương trình Pell liên kết là 2 2


( 1) 1


<i>x</i> <i>m m</i> <i>y</i>  .


Cách giải này cũng hoàn toàn phù hợp nhưng chú ý rằng phương trình Pell tổng quát
dạng <i><sub>Ax</sub></i>2<sub></sub><i><sub>By</sub></i>2 <sub></sub><i><sub>n</sub></i><sub> nói chung có nhiều hơn một nghiệm cơ sở. Ta cần chứng minh </sub>
trong trường hợp đặc biệt này (<i>A</i><i>B</i>1 và <i>n</i>1) thì nó chỉ có duy nhất một nghiệm cơ
sở. Nếu khơng thì lời giải cũng thực sự vẫn cịn thiếu sót.


Ngồi cách sử dụng phương trình Pell như trên, ta có thể lập luận như sau:


Do (<i>mm</i>1,<i>mn n</i> 1) 1 nên ta cần có (<i>mn</i>1)(<i>mn n</i> 1) là số chính phương hay tồn
tại <i>a</i> sao cho 4 (<i>m m</i>1)<i>n</i>2 4(2<i>m</i>1)<i>n</i>4<i>a</i>2, đưa về


2 2 2


(2<i>m</i>1)<i>n</i>2 <i>n</i> <i>a</i>


Ta chứng minh được rằng <i>n phải là số chẵn và đây là phương trình Pythagore nên phải </i>
tồn tại các số <i>p q k sao cho </i>, ,


2 2


2 ,



(2 1) 2 ( )
<i>n</i> <i>kpq</i>


<i>m</i> <i>n</i> <i>k p</i> <i>q</i>


 



   





trong đó <i>k n</i>( ,(2<i>m</i>1)<i>n</i>2) 2 nên <i>k</i>1 hoặc <i>k</i>2.


Nếu <i>k</i>1 thì dễ thấy <i><sub>p</sub></i>2<sub></sub><i><sub>q</sub></i>2<sub> chẵn, suy ra </sub><i><sub>p q</sub></i><sub>,</sub> <sub> cùng tính chẵn lẻ. Nếu </sub><i><sub>p q</sub></i><sub>,</sub> <sub> cùng lẻ thì </sub><i><sub>n </sub></i>
chia 4 dư 2 và (2<i>m</i>1)<i>n</i>2 chia hết cho 4 trong khi <i><sub>p</sub></i>2<sub></sub><i><sub>q</sub></i>2<sub> chia 4 dư 2, mâu thuẫn. Nếu </sub>


,


<i>p q</i> cùng chẵn thì <i>n chia hết cho 4 dẫn đến </i>(2<i>m</i>1)<i>n</i>2 chia 4 dư 2 trong khi <i><sub>p</sub></i>2<sub></sub><i><sub>q</sub></i>2
chia hết cho 4, cũng mâu thuẫn.


Nếu <i>k</i>2 thì <i>n</i>4<i>pq</i> và <sub>(2</sub><i><sub>m</sub></i><sub></sub><sub>1)</sub><i><sub>n</sub></i><sub></sub><sub>2 2(</sub><sub></sub> <i><sub>p</sub></i>2<sub></sub><i><sub>q</sub></i>2<sub>)</sub><sub>, suy ra </sub>


2 2


2(2 1) 1 0



</div>
<span class='text_page_counter'>(57)</span><div class='page_container' data-page=57>

13
Đến đây ta chứng minh được (sử dụng phương pháp Viète Jumping)<i>p q</i>, là hai số hạng
liên tiếp của dãy số cho bởi công thức


1 2


2 1


1, 2(2 1),


2(2 1) , 1


<i>i</i> <i>i</i> <i>i</i>


<i>x</i> <i>x</i> <i>m</i>


<i>x</i> <i>m</i> <i>x</i> <i>x i</i>


   




   




.


Từ đó suy ra <i>pq</i> chia hết cho 2(2<i>m</i>1) hay <i>n chia hết cho </i>8(2<i>m</i>1).



Phương pháp xây dựng nghiệm ở đây có nét giống với bài 6 đề VMO 2012.
<i><b>Xét các số tự nhiên lẻ </b><b>a b mà a là ước số của </b></i>, 2


2


<i>b</i>  <i><b> và </b>b<b> là ước số của </b></i> 2
2


<i>a</i>  <i><b>. Chứng </b></i>
<i><b>minh rằng </b><b>a b là các số hạng của dãy số tự nhiên </b></i>, ( )<i>v<sub>n</sub></i> <i><b> xác định bởi </b></i>


1 2 1


<i>v</i> <i>v</i>  <i><b> và </b>v<sub>n</sub></i>4<i>v<sub>n</sub></i><sub>1</sub><i>v<sub>n</sub></i><sub>2</sub><i><b> với mọi </b>n</i>2<i><b>. </b></i>


Cách giải này có phần ngắn gọn hơn và xử lí nhẹ nhàng hơn cách đã nêu ở lời giải ban
đầu nhưng để trình bày chặt chẽ không phải là điều đơn giản.


<b>Bài 3. </b>


<i><b>Với mỗi số </b><b>n nguyên dương, đặt </b>S<sub>n</sub></i> 

0,1, 2,..., 2<i>n</i>1

<i><b>. Xét hàm số </b></i> <i>f</i> : (<i>S<sub>n</sub></i>)[0;1]


<i><b>thỏa mãn đồng thời các điều kiện sau: </b></i>


<i><b>i/ </b></i> <i>f x</i>( ,0) <i>f x n</i>( , 2 1) 0 <i><b> với mọi số nguyên </b><b>x </b></i>.


<i><b>ii/ </b></i> <i>f x</i>( 1, )<i>y</i>  <i>f x</i>( 1, )<i>y</i>  <i>f x y</i>( , 1) <i>f x y</i>( , 1) 1 <i><b> với </b>x y</i>, <i><b> và 1</b></i><i>y</i>2<i>n<b>. </b></i>
<i><b>Gọi </b>F<b> là tập hợp tất cả các hàm số </b><b>f thỏa mãn. </b></i>


<i><b>1. Chứng minh rằng </b></i> <i><b>F là vô hạn. </b></i>



<i><b>2. Với mỗi hàm số </b></i> <i>f</i><i>F<b>, đặt </b><b>v là tập hợp ảnh của f . Chứng minh rằng </b><sub>f</sub></i> <i><b>v hữu hạn. </b><sub>f</sub></i>
<i><b>3. Tìm giá trị lớn nhất của </b></i> <i><b>v với f</b><sub>f</sub></i> <i>F<b>. </b></i>


<b>Lời giải. </b>


1. Trong đẳng thức ii/ đã cho, ta thấy rằng


</div>
<span class='text_page_counter'>(58)</span><div class='page_container' data-page=58>

14
Điều này có nghĩa là các giá trị của hàm <i>f x y với </i>( , ) <i>x y</i>, cùng tính chẵn lẻ và <i>x y</i>, khác
tính chẵn lẻ là khơng có liên hệ với nhau.


Ta sẽ tìm các xác định hàm <i>f trong cả hai trường hợp. </i>


Trong mặt phẳng tọa độ <i>Oxy ta xét một lưới nguyên nằm ngang có chiều cao là </i>, 2<i>n</i>1,
chiều rộng vơ hạn và điểm có tọa độ ( , )<i>i j sẽ được gán giá trị ( , )f i j . </i>


Điều kiện i/ được hiểu là tất cả các số thuộc hai biên (trên và dưới) của lưới đều gán số
0; còn các điểm nguyên bên trong lưới đều được gán giá trị thuộc [0; 1] .


Điều kiện ii/ chính là với mọi hình vng con nằm nghiêng (các đường chéo song song
với các trục tọa độ) có các đỉnh nguyên và cạnh là 2 đều có tổng các số gán cho các
đỉnh bằng 1.


Với mỗi điểm có tọa độ <i>A</i> nguyên thuộc lưới nguyên đang xét, ta đặt <i>f A f A</i><sub>1</sub>( ), ( )<sub>2</sub> là giá
trị gán cho các điểm có cùng tung độ với <i>A</i>, lần lượt có hồnh độ lớn hơn và nhỏ hơn
hồnh độ của <i>A</i> đúng 2 đơn vị.


Tiếp tục đặt <i>a<sub>k</sub></i>  <i>f k k</i>( , ) với <i>k</i>1, 2, 3,..., 2<i>n</i> là giá trị gán cho điểm <i>A k k<sub>k</sub></i>( , ). Theo điều
kiện xác định hàm số thì



1 2 1 1 1 1 1 2


2 3 1 2 1 1 1 2 1 3


2 1 2 1 2 2 1 2 1 1 2 1 1 2


2 1 2 1 1 2 1 2 1


( ) 0 1 ( ) 1 ,
( ) ( ) 1 ( ) ,
...


( ) ( ) 1 ( ) ,


0 ( ) ( ) 1 ( )


<i>n</i> <i>n</i> <i>n</i> <i>n</i> <i>n</i> <i>n</i>


<i>n</i> <i>n</i> <i>n</i> <i>n</i>


<i>a</i> <i>a</i> <i>f A</i> <i>f A</i> <i>a</i> <i>a</i>


<i>a</i> <i>a</i> <i>f A</i> <i>f A</i> <i>f A</i> <i>a</i> <i>a</i>


<i>a</i> <i>a</i> <i>f A</i> <i>f A</i> <i>f A</i> <i>a</i> <i>a</i>


<i>a</i> <i>f A</i> <i>f A</i> <i>f A</i> <i>a</i>


   



       
      
      
     


Tương tự, ta cũng có


2 2 2 1 2


2 2 1 2 2 2


2 2 1 1 2


2 2 2


( ) 1
( )
...


( ) 1 ,


( )


<i>n</i> <i>n</i> <i>n</i>


<i>n</i> <i>n</i> <i>n</i>


<i>n</i> <i>n</i>



<i>n</i> <i>n</i>


<i>f A</i> <i>a</i> <i>a</i>


<i>f A</i> <i>a</i> <i>a</i>


<i>f A</i> <i>a</i> <i>a</i>


<i>f A</i> <i>a</i>



 

  
 
  


Từ đây ta thấy rằng nếu dãy <i>a<sub>k</sub></i> xác định thì các giá trị <i>f A</i><sub>1</sub>( <i><sub>k</sub></i>), (<i>f A</i><sub>2</sub> <i><sub>k</sub></i>) cũng hoàn toàn
xác định. Ta sẽ chọn các giá trị <i>a<sub>k</sub></i> sao cho <i>f A</i><sub>1</sub>( <i><sub>k</sub></i>), (<i>f A</i><sub>2</sub> <i><sub>k</sub></i>) đều thuộc [0; 1] .


</div>
<span class='text_page_counter'>(59)</span><div class='page_container' data-page=59>

15
Hơn nữa, ta cũng thấy rằng


1 1 1 3 1 5 1 2 1 1 2 1 4 1 6 1 2 1 1 1 2


2 1 2 3 2 5 2 2 1 2 2 2 4 2 6 2 2 2 1 2 2


( ) ( ) ( ) ... ( ), ( ) ( ) ( ) ... ( ), ( ) ( ) 1,
( ) ( ) ( ) ... ( ), ( ) ( ) ( ) ... ( ), ( ) ( ) 1



<i>n</i> <i>n</i> <i>n</i>


<i>n</i> <i>n</i> <i>n</i>


<i>f A</i> <i>f A</i> <i>f A</i> <i>f A</i> <i>f A</i> <i>f A</i> <i>f A</i> <i>f A</i> <i>f A</i> <i>f A</i>
<i>f A</i> <i>f A</i> <i>f A</i> <i>f A</i> <i>f A</i> <i>f A</i> <i>f A</i> <i>f A</i> <i>f A</i> <i>f A</i>






         


         


Do đó, tính đơn điệu của hai dãy con có chỉ số chẵn và lẻ vẫn được bảo toàn.


Tương tự, từ dãy <i>f A</i><sub>1</sub>( <i><sub>k</sub></i>), ta xác định được <i>f f A</i><sub>1</sub>( (<sub>1</sub> <i><sub>k</sub></i>)) thỏa mãn, từ dãy <i>f A</i><sub>2</sub>( <i><sub>k</sub></i>), ta cũng
xác định được <i>f f A</i><sub>2</sub>( (<sub>2</sub> <i><sub>k</sub></i>)) và cứ thế, tức là xây dựng được tất cả giá trị cho hàm số


( , )


<i>f x y trong trường hợp x y</i> chẵn.


Tiếp tục đặt <i>b<sub>k</sub></i>  <i>f k</i>( 1, )<i>k</i> với <i>k</i>1, 2, 3,..., 2<i>n</i> là giá trị gán cho điểm <i>B k<sub>k</sub></i>( 1, )<i>k</i> thì
hoàn toàn tương tự như trên, ta xây dựng được tất cả các giá trị của <i>f x y với </i>( , ) <i>x y</i> lẻ.


Dễ thấy có vơ số cách chọn dãy ( ),( )<i>a<sub>k</sub></i> <i>b<sub>k</sub></i> thỏa mãn các điều kiện trên nên có vô số hàm
<i>f thỏa mãn đề bài, tức là </i> <i>F vơ hạn. Ta có đpcm. </i>



2. Trong đẳng thức <i>f x</i>( 1, )<i>y</i>  <i>f x</i>( 1, )<i>y</i>  <i>f x y</i>( , 1) <i>f x y</i>( , 1) 1 , ta thay <i>x y</i>, lần lượt
bởi <i>x</i>1,<i>y</i>1, ta được <i>f x y</i>( , 1) <i>f x</i>( 2,<i>y</i>1) <i>f x</i>( 1, )<i>y</i>  <i>f x</i>( 1,<i>y</i>2) 1 .


Từ đó suy ra <i>f x</i>( 1, )<i>y</i>  <i>f x y</i>( , 1) <i>f x</i>( 1,<i>y</i>2) <i>f x</i>( 2,<i>y</i>1) (*) hay
(1,1) (2,0) (3, 3) (4, 2) ... (2 1, 2 1) (2 2, 2 )
<i>f</i>  <i>f</i>  <i>f</i>  <i>f</i>   <i>f</i> <i>n</i> <i>n</i>  <i>f</i> <i>n</i> <i>n</i> ,


</div>
<span class='text_page_counter'>(60)</span><div class='page_container' data-page=60>

16
Tương tự, ta có được <i>f</i>(2, 2) <i>f</i>(2<i>n</i>2, 2<i>n</i>1), (4, 2)<i>f</i>  <i>f</i>(2<i>n</i>5, 2<i>n</i>1).


Tiếp tục áp dụng nhiều lần đẳng thức (*), suy ra


( , ) (2 1 ,2 1 ), ( 2, ) (2 3 , 2 1 )
<i>f k k</i>  <i>f</i> <i>n</i> <i>k n</i> <i>k f k</i> <i>k</i>  <i>f</i> <i>n</i> <i>k n</i> <i>k</i> .
Chứng minh tương tự, ta có


(2 , 2 1 ) (4 2 , ), (2 3 , 2 1 ) (4 4 , )
<i>f</i> <i>n k n</i>  <i>k</i>  <i>f</i> <i>n</i> <i>k k f</i> <i>n</i> <i>k n</i> <i>k</i>  <i>f</i> <i>n</i> <i>k k</i> .


Do đó, <i>f k k</i>( , ) <i>f</i>(2<i>n</i> 1 <i>k n</i>, 2  1 <i>k</i>) <i>f</i>(4<i>n</i> 2 <i>k k</i>, ) với mọi <i>k</i>1, 2, 3,..., 2<i>n</i> và như thế,
bằng quy nạp, ta có được


( , ) ((2 1) , 2 1 ) ((4 2) , )


<i>f k k</i>  <i>f</i> <i>n</i> <i>i k n</i>  <i>k</i>  <i>f</i> <i>n</i> <i>i k k</i> với <i>i</i> và <i>k</i>1, 2, 3,..., 2<i>n</i>.
Hơn nữa, sự xác định các giá trị trong đường chéo tiếp theo (nằm về phía phải) hồn
tồn giống nhau ở dải các điểm ( , )<i>k k và ((2n</i>1)<i>i k n</i> , 2  1 <i>k</i>) nên các giá trị trên đó
cũng tương ứng bằng nhau. Suy ra giá trị của <i>f có sự tuần hồn và được minh họa như </i>
hình bên dưới, tức là các giá trị của <i>f x y với </i>( , ) <i>x y</i> chẵn là sự lặp lại các giá trị được


gán cho các điểm nguyên trong tam giác  có tọa độ các đỉnh (1,1),(2<i>n</i>1, 2 ),(4<i>n</i> <i>n</i>1,1).


<i><b>1</b></i>
<i><b>2n+1</b></i>
<i><b>2n</b></i>


<i><b>O</b></i> <i><b>1</b></i> <i><b>2n+1</b></i> <i><b>2n+2</b></i> <i><b>4n+1</b></i> <i><b>4n+2</b></i> <i><b>4n+3</b></i> <i><b>4n+3</b></i> <i><b>6n+3</b></i>


Tuy nhiên, các giá trị này là hữu hạn và ta tính được tổng cộng có không quá
1 2 3 ... 2    <i>n n n</i> (2 1) giá trị.


Tương tự với giá trị của <i>f x y mà </i>( , ) <i>x y</i> lẻ, ta cũng có thêm không quá <i>n n</i>(2 1) giá trị
nữa. Kết hợp với 0, ta có <i>v<sub>f</sub></i> 2 (2<i>n n</i>1) 1 .


</div>
<span class='text_page_counter'>(61)</span><div class='page_container' data-page=61>

17
3. Ta sẽ xây dựng một hàm số <i>f có v<sub>f</sub></i> 2 (2<i>n n</i>1) 1 và chỉ ra đây chính là giá trị lớn
nhất của <i>v . <sub>f</sub></i>


Bằng quy nạp, ta chứng minh được rằng


1 ( 1)



1 ( 1)



( 2 , ) ( 1) ( 1)


4


<i>i</i> <i>k</i>


<i>k</i> <i>k i</i>



<i>k i</i> <i>k</i>


<i>f i</i> <i>k i</i> <i>a</i>  <i>a</i>




   


     


với mọi <i>i</i>1, 2,..., 2<i>n</i> và <i>k</i> mà 0  <i>k i</i> 2<i>n</i>1. (**)
Thật vậy, với <i>k</i>0 thì (**) hiển nhiên đúng.


Giả sử (**) đúng với mọi ( , )<i>i k mà k</i><i>m</i> và <i>k</i><i>m</i>1,<i>i</i><i>j</i>. Từ điều kiện ii/, ta có


( 1 2( 1), 1) ( 1 2 , 1) ( 2( 1), ) ( 2 2 , 2) 1
<i>f j</i>  <i>m</i> <i>j</i>  <i>f j</i>  <i>m j</i>  <i>f j</i> <i>m</i> <i>j</i>  <i>f j</i>  <i>m j</i>  .
Sử dụng giả thiết quy nạp, ta tính được


1



1



2
1


2 1


1 ( 1) 1 ( 1)


( 1 2( 1), 1) ( 1) ( 1)



4


<i>j</i> <i>m</i>


<i>m j</i>
<i>m</i>


<i>m j</i> <i>m</i>


<i>f j</i> <i>m</i> <i>j</i> <i>a</i> <i>a</i>


 
 

  
   
         .


Suy ra (**) cũng đúng với <i>i</i> <i>j</i> 1,<i>k</i><i>m</i>1.


Như thế, các giá trị được gán cho các điểm trong tam giác  đều có dạng <i><sub>ij</sub></i><i>a<sub>i</sub></i><i>a<sub>j</sub></i> với

 

0,1


<i>ij</i>


<i></i>  , trong đó giá trị <i><sub>ij</sub></i> cũng như dấu của <i>a a xác định duy nhất theo ,<sub>i</sub></i>, <i><sub>j</sub></i> <i>i j . </i>
Tiếp theo, ta chọn <sub>2</sub> <sub>1</sub> <sub>2</sub>1 <sub>1</sub> , <sub>2</sub> <sub>2(</sub>1<sub>1</sub> <sub>)</sub>


3 3



<i>k</i> <i>k</i> <i>k</i> <i>n</i> <i>k</i>


<i>a</i> <sub></sub>  <sub></sub> <i>a</i>  <sub> </sub> với <i>k</i>1, 2,...,<i>n</i> thì do trong hệ cơ số 3, một


số nguyên bất kì có duy nhất một cách biểu diễn dưới dạng tổng
0
3
<i>r</i>
<i>i</i>
<i>i</i>
<i>i</i>
<i></i>


với <i><sub>r</sub></i> 0 và

1,0,1



<i>i</i>


<i>e</i>   ) nên suy ra tất cả các giá trị của <i>f x y dùng để đánh số cho các điểm nằm </i>( , )
trong tam giác  là đôi một phân biệt và khác 0, tức là có đúng <i>n n</i>(2 1) giá trị như thế,
tức là ta đã xây dựng được giá trị cho hàm số <i>f x y với </i>( , ) <i>x y</i> chẵn.


Tương tự, để xây dựng cho <i>f x y với </i>( , ) <i>x y</i> lẻ, ta chọn dãy số


2 1 <sub>2</sub> <sub>1</sub> 2 <sub>2(</sub> <sub>1</sub> <sub>)</sub>


1 1


,



3 3 3 3


<i>k</i> <i><sub>k</sub></i> <i>k</i> <i><sub>n</sub></i> <i><sub>k</sub></i>


<i>b</i>   <sub></sub> <i>b</i>  <sub> </sub> với <i>k</i>1, 2,..., 2<i>n</i>.


</div>
<span class='text_page_counter'>(62)</span><div class='page_container' data-page=62>

18
Từ đó, ta xây dựng được toàn bộ giá trị cho hàm số <i>f x y và có tất cả </i>( , ) 2 (2<i>n n</i>1) 1 giá
trị đôi một khác nhau.


Vậy giá trị lớn nhất cần tìm của <i>v trên miền <sub>f</sub></i> <i>F</i> là 2 (2<i>n n</i>1) 1 .


<b>Nhận xét. </b>


Đây là một bài toán đòi hỏi những kiến thức tổng hợp và một phong cách làm việc từ
tốn, bài bản. Mơ hình lưới hoặc bảng là một mơ hình tự nhiên mà ta nghĩ đến, và hàm
<b>số có thể được xác định theo hàng, cột hoặc đường chéo. </b>


Trong các bài tốn có chứa tham số <i>n , ta nên bắt đầu từ những giá trị n nhỏ để hình </i>
dung bài toán một cách tốt nhất, cụ thể nhất, từ đó tìm cách tiếp cận tổng quát. Nếu
chọn cách này thì có lẽ câu 1) và 2), thậm chí cả phần chặn trên đúng ở câu 3) khơng
phải là q khó. Điểm khó ở câu 1) là cách xây dựng hàm phải thỏa mãn điều kiện


( , ) [0; 1]


<i>f x y</i>  . Để có được điều này, ta phải tìm được một điều kiện có tính bất biến đối
với dãy giá trị trên đường chéo (khơng chỉ đề dãy kề nó thuộc [0; 1] , mà còn để các dãy
tiếp theo cũng như thế).



Để làm câu 3), ta phải có những nhận xét tinh tế hơn, địi hỏi nhiều thời gian hơn. Có lẽ
câu này chỉ dành cho những bạn đã giải quyết tốt hai bài 1, 2 trong vòng 1,5 - 2 giờ, còn
khá nhiều thời gian để tập trung toàn lực cho bài này. Ý tưởng cơ bản ở đây là dự đoán
và chứng minh được công thức tổng quát của hàm dựa theo các phần tử thuộc một
đường chéo. Từ công thức tổng quát này ta mới tìm cách chọn các “giá trị khởi tạo” để
tất cả các giá trị nằm trong “tam giác tuần hồn” đơi một khác nhau.


Để chọn các giá trị khác nhau, ta có một số định hướng cơ bản sau:
+ Dùng bất đẳng thức


+ Dùng tính “độc lập tuyến tính” của các số vô tỷ, cụ thể là: Nếu <i>p p</i><sub>1</sub>, <sub>2</sub>,...,<i>p<sub>n</sub></i> là các số
ngun tố phân biệt thì khơng tồn tại các số hữu tỉ <i>c c</i><sub>1</sub>, ,...,<sub>2</sub> <i>c<sub>n</sub></i> không đồng thời bằng 0
sao cho <i>c</i><sub>1</sub> <i>p</i><sub>1</sub> <i>c</i><sub>2</sub> <i>p</i><sub>2</sub> ...<i>c<sub>n</sub></i> <i>p<sub>n</sub></i> 0.


Tuy nhiên, việc chứng minh bổ đề này là khơng đơn giản. Ta cũng có thể thay các số


<i>i</i>


</div>
<span class='text_page_counter'>(63)</span><div class='page_container' data-page=63>

19
Một cách tiếp cận thú vị và sơ cấp cho vấn đề này đã được trình bày ở trên là sử dụng
<i><b>tính chất của hệ đếm cơ số 3: Mọi số nguyên </b>N</i>0<i><b> bất kỳ đều có thể biểu diễn duy nhất </b></i>
<i><b>dưới dạng </b></i>


1


1 0


3<i>i</i> 3<i>i</i> ...


<i>i</i> <i>i</i>



<i>N</i><i></i> <i></i><sub></sub>   <i></i> <i><b> trong đó </b><sub>i</sub></i>  1,<i><sub>j</sub></i> 

1; 0;1 ,

 <i>j</i> 0,1,..,<i>i</i>1.
Mệnh đề này có thể chứng minh khá dễ dàng bằng quy nạp toán học.


Việc <i>f x y</i>( , ) [0; 1] khơng thành vấn đề vì ta có thể chia các giá trị khởi tạo cho một
hằng số đủ lớn để đạt được điều này.


Ngoài cách xây dựng theo đường chéo với <i>4n</i> giá trị khởi tạo như trên (<i>2n</i> cho trường
hợp <i>f x y mà </i>( , ) <i>x y</i> chẵn và <i>2n</i> cho trường hợp <i>f x y mà </i>( , ) <i>x y</i> lẻ), ta cũng có thể xây
dựng theo hai cạnh dọc hoặc theo hai cạnh ngang có độ dài <i>2n</i>.


Chẳng hạn, trong trường hợp xây dựng theo cạnh dọc, song song với trục <i>Oy , ta có thể </i>
làm như sau:


Bằng quy nạp, ta chứng minh được rằng






( , 2 ) ( 2 1,1) ( 2 3,1) ... ( 2 1,1)


( , 2 1) ( 2 ,1) ( 2 2,1) ... ( ,1) ... ( 2 2,1) ( 2 ,1)


<i>f x i</i> <i>i</i> <i>f x</i> <i>i</i> <i>f x</i> <i>i</i> <i>f x</i> <i>i</i>


<i>f x i</i> <i>f x</i> <i>i</i> <i>f x</i> <i>i</i> <i>f x</i> <i>f x</i> <i>i</i> <i>f x</i> <i>i</i> <i>i</i>


          



              


Ta cần chọn <i>f x</i>( ,1),<i>x</i> [ 2<i>n</i>1; 2 ]<i>n</i> sao cho:


i/ <i>i</i> 1 <i>f x</i>( 2<i>i</i>1,1) <i>f x</i>( 2<i>i</i>3,1) ...  <i>f x</i>( 2<i>i</i>1,1)<i>i</i>.


ii/ <i>i</i> <i>f x</i>( 2 ,1)<i>i</i>  <i>f x</i>( 2<i>i</i>2,1) ...  <i>f x</i>( ,1) ...  <i>f x</i>( 2<i>i</i>2,1) <i>f x</i>( 2 ,1)<i>i</i>  <i>i</i> 1.
iii/ Các giá trị <i>f x</i>( ,1) là phân biệt với <i>x</i> [ 2<i>n</i>1; 2 ]<i>n</i> .


iv/ <i>f x i</i>( , 2 1) <i>f x</i>( 2 ,1)<i>i</i>  <i>f x</i>( 2<i>i</i>2,1) ...  <i>f x</i>( ,1) ...  <i>f x</i>( 2<i>i</i>2,1) <i>f x</i>( 2 ,1)<i>i</i> <i>i</i>
với <i>i n</i> thì ln bằng <i>n </i>.


Từ đó đưa lên đường trịn và kết hợp với bất đẳng thức để chọn, tức là xét một số giá trị
đủ nhỏ thích hợp để cho chúng đơi một phân biệt. Cách xây dựng cho trường hợp cạnh
ngang cũng thực hiện tương tự.


</div>
<span class='text_page_counter'>(64)</span><div class='page_container' data-page=64>

20
<i><b>(Việt Nam TST 2003) Cho hàm số </b></i> <i>f</i> :  <i><b> thỏa mãn đồng thời các điều kiện sau: </b></i>
<i><b>i)</b></i> <i><sub>f</sub></i><sub>(0,0)</sub><sub></sub><sub>5</sub>2003<sub>, (0, )</sub><i><sub>f</sub></i> <i><sub>n</sub></i> <sub></sub><sub>0</sub><i><b><sub> với mọi n là số nguyên khác 0. </sub></b></i>


<i><b>ii)</b></i> ( , ) ( 1, ) 2 ( 1, ) ( 1, 1) ( 1, 1)


2 2 2


<i>f m</i> <i>n</i> <i>f m</i> <i>n</i> <i>f m</i> <i>n</i>


<i>f m n</i>  <i>f m</i> <i>n</i>  <sub></sub>  <sub> </sub>      <sub> </sub>    <sub></sub>


     



<i><b>với mọi số nguyên dương m và mọi số nguyên n. </b></i>


<i><b>Chứng minh rằng tồn tại số nguyên dương M sao cho</b></i> <i>f M n</i>( , )1<i><b> với mọi số nguyên n </b></i>
<i><b>thỏa </b></i>


2003
5 1


2


<i>n</i>  <i><b> và </b></i> <i>f M n</i>( , )0<i><b> với mọi số nguyên n thỏa </b></i>


2003
5 1


2
<i>n</i>  <i><b> . </b></i>
Tuy nhiên bài toán này lại là một câu chuyện khác !


<b>Bài 4. </b>


<i><b>Tìm hằng số </b>k<b> nguyên dương lớn nhất thỏa mãn: Với mọi </b><b>a b c dương mà </b></i>, , <i>abc</i>1<i><b> thì ta </b></i>
<i><b>có bất đẳng thức sau </b></i>


1 1 1


3
1 4


<i>k</i> <i>k</i>



<i>a</i><i>b</i><i>c</i><i>a b c</i>     <i><b> (*) </b></i>


<b>Lời giải. </b>


Giả sử <i>k</i> là số nguyên dương sao cho bất đẳng thức đã cho đúng với mọi <i>a b c mà </i>, ,


1.


<i>abc</i> Thay 2, 9


3 4


<i>b c</i>  <i>a</i> vào (*), ta được


3 4 880


2 3 14


2 2 9


2 9 4 63


1
3 3 4


<i>k</i> <i>k</i>


<i>k</i>



       


  


.


Hơn nữa, vì <i>k</i> là số nguyên dương nên từ đánh giá trên, ta có <i>k</i>13. Ta sẽ chứng minh
rằng với <i>k</i>13 thì bất đẳng thức (*) đúng.


Thật vậy, với <i>k</i>13 ta có bất đẳng thức 1 1 1 13 25.
1 4


<i>a</i><i>b</i><i>c</i><i>a b c</i>    (**)


Đặt ( , , ) 1 1 1 1
1


<i>f a b c</i>


<i>a</i> <i>b</i> <i>c</i> <i>a b c</i>


   


   . Khơng mất tính tổng quát, ta có thể giả sử




max , ,


</div>
<span class='text_page_counter'>(65)</span><div class='page_container' data-page=65>

21









2


1 1 2 1 1


( , , ) , , 13


1 <sub>2</sub> <sub>1</sub>


1 13


.
( 1) 2 1
<i>f a b c</i> <i>f a</i> <i>bc</i> <i>bc</i>


<i>b</i> <i>c</i> <i><sub>bc</sub></i> <i>a b c</i> <i><sub>a</sub></i> <i><sub>bc</sub></i>


<i>b</i> <i>c</i>


<i>bc</i> <i><sub>a b c</sub></i> <i><sub>a</sub></i> <i><sub>bc</sub></i>


   
 <sub></sub>   <sub></sub> <sub></sub>  <sub></sub>
    


   
 
 
  
 <sub>  </sub> <sub></sub> <sub></sub> 
 
 


Do <i>a</i>max

<i>a b c</i>, ,

và giả thiết <i>abc</i>1 nên ta có <i>bc</i>1, suy ra 1 1


<i>bc</i>  .


Mặt khác, sử dụng bất đẳng thức AM-GM và biến đổi cho biểu thức trong ngoặc ở đẳng
thức trên, ta có


 

3



3



13 13 13


1,
16
(<i>a b c</i> 1) <i>a</i> 2 <i>bc</i> 1 3 <i>abc</i> 1 3 <i>abc</i> 1


  


      





nên hiển nhiên <i>f a b c</i>( , , ) <i>f a</i>

, <i>bc</i>, <i>bc</i>

. Ta đưa được bài toán về chứng minh


2


1 25
, ,


4


<i>f</i> <i>x x</i>


<i>x</i>


 




 


  với <i>x</i> <i>bc</i>, 0 <i>x</i> 1.


  


Nếu <i>x</i>1 thì bất đẳng thức trên trở thành đẳng thức. Trong trường hợp 0<i>x</i>1, bằng
cách sử dụng lại biến đổi đã thực hiện trong quá trình tìm điều kiện cần cho <i>k ta thấy </i>,
bất đẳng thức tương đương với


3 2


3 2



4 3 2 2 4 3 2


( 2)(2 1) 13


4( 2)(2 1) 13 (2 1)
(2 1) 4


4(2 5 2 2) 26 13 8 20 18 9 8 0.


<i>x</i> <i>x</i> <i>x</i>


<i>x</i> <i>x</i> <i>x</i> <i>x x</i>


<i>x x</i>


<i>x</i> <i>x</i> <i>x</i> <i>x</i> <i>x</i> <i>x</i> <i>x</i> <i>x</i> <i>x</i> <i>x</i>


  
      

            

Ta có


4 3 2 4 2 3 2 2


2 2 2 2


8 20 18 9 8 (8 8 2) (20 20 5 ) (10 14 6)


2(2 1) 5 (2 1) 2(5 7 3) 0


<i>x</i> <i>x</i> <i>x</i> <i>x</i> <i>x</i> <i>x</i> <i>x</i> <i>x</i> <i>x</i> <i>x</i> <i>x</i>


<i>x</i> <i>x x</i> <i>x</i> <i>x</i>


            


       


do 2(2<i>x</i>21)2 0, 5 (2<i>x x</i>1)2 0 và 5<i>x</i>27<i>x</i> 3 0 (tam thức bậc hai có hệ số cao nhất
dương và biệt thức   11 0 ). Như vậy, bất đẳng thức cuối hiển nhiên đúng. Ta đi
đến kết luận cuối cùng <i>k</i>13 là giá trị cần tìm.


<b>Nhận xét. </b>


</div>
<span class='text_page_counter'>(66)</span><div class='page_container' data-page=66>

22
Do bất đẳng thức có dạng đối xứng nên ta nghĩ đến việc chọn một bộ số mà ở đó có hai
biến nhận giá trị bằng nhau. Điều này lý giải cho việc chọn <i>a b</i> <i>x</i> và <i>c</i> 1<sub>2</sub>


<i>x</i>


 như
trong lời giải ở trên. Một cách khác để chứng minh bất đẳng thức (**) dùng phương
pháp dồn biến như sau:


Biến đổi (**) về dạng đa thức ( )( 1) 13 25( )
4


<i>ab bc ca a b c</i>       <i>a b c</i>  .



Giả sử <i>a</i>max

<i>a b c</i>, ,

thì <i>a</i>1 và đặt <i>x b c</i>  thì bất đẳng thức trên có thể viết lại
thành ( )( 1) 13 25( )


4


<i>ax bc a x</i>     <i>a x</i> . Bất đẳng thức này tương đương với


2 2
2
2
2
2
25 25


( ) 14 0


4 4


25
25


4 25


4 <sub>14</sub> <sub>0</sub>


2 2 4


<i>a</i>



<i>ax</i> <i>bc a</i> <i>a</i> <i>x</i>


<i>bc a</i> <i>a</i>


<i>bc a</i> <i><sub>a</sub></i>


<i>a x</i>
<i>a</i> <i>a</i>
      
 
  <sub></sub> <sub> </sub>
   
  <sub></sub> <sub></sub>
       
 
 
 


Do <i>x b c</i> 2 <i>bc</i> 2
<i>a</i>
    nên


2 <sub>2</sub>


2 25 <sub>2</sub> 2 25 <sub>4</sub> 1 25


4 <sub>4</sub>


4 <sub>0</sub>



2 2 2


<i>a</i> <i>bc a</i> <i><sub>a</sub></i> <i><sub>a a</sub></i>


<i>bc a</i>


<i>a</i> <i>a</i>


<i>x</i>


<i>a</i> <i>a</i> <i>a</i>


   <sub></sub> <sub> </sub> <sub></sub>


 


   


Ta đưa về chứng minh


2
2


2


2 1 25


1 25


4 <sub>4</sub>



25


4 <sub>14</sub> <sub>0</sub>


2 2 4


<i>a</i> <i>a</i>


<i>a</i> <i>a a</i> <i><sub>a</sub></i>


<i>a</i>
<i>a</i>
<i>a</i>
<i>a</i> <i>a</i>
 
  <sub></sub> <sub> </sub>
     
  <sub></sub> <sub></sub>
   
 
 
 
 
.


Đặt <i>a</i>  <i>t</i> 0 thì ta đưa về


2
2


4 2
2 4
2
2
2
2
2 2
1 25
1 25
4
4 25


4 <sub>14</sub> <sub>0</sub>


4


2 2


<i>t</i> <i>t</i>


<i>t</i> <i>t</i> <i>t</i>


<i>t</i>
<i>t</i>
<i>t</i>
<i>t</i>
<i>t</i> <i>t</i>
 
  <sub></sub> <sub></sub> <sub></sub>
     


  <sub></sub> <sub></sub>
   
 
 
 
 


hay <sub>(</sub><i><sub>t</sub></i><sub></sub><sub>1) 8</sub>2

<i><sub>t</sub></i>4<sub></sub><sub>9</sub><i><sub>t</sub></i>3<sub></sub><sub>18</sub><i><sub>t</sub></i>2<sub></sub><sub>20</sub><i><sub>t</sub></i><sub></sub><sub>8</sub>

<sub></sub><sub>0</sub><sub>. Đặt </sub><i><sub>t</sub></i> 1


<i>u</i>


 thì ta đưa được về bất đẳng thức đã
đề cập trong cách ban đầu.


Ta cũng có thể chứng minh bất đẳng thức 1 1 1 13 25


1 4


<i>a</i><i>b</i><i>c</i><i>a b c</i>    không dùng đến


</div>
<span class='text_page_counter'>(67)</span><div class='page_container' data-page=67>

23
Do <i>abc</i>1 nên tồn tại các số thực dương <i>x y z</i>, , sao cho


2


2 2


, <i>y</i> ,


<i>x</i> <i>z</i>



<i>a</i> <i>b</i> <i>c</i>


<i>yz</i> <i>zx</i> <i>xy</i>


   , ta cần
chứng minh rằng


2 2 2 3 3 3


13 13


3
4


<i>yz</i> <i>zx</i> <i>xy</i> <i>xyz</i>


<i>x</i> <i>y</i>  <i>z</i> <i>x</i> <i>y</i> <i>z</i> <i>xyz</i>   .


Do <i>x y z</i>, , bình đẳng nên ta có thể giả sử <i>x</i><i>y</i><i>z</i>. Viết bất đẳng thức trên thành dạng
tương đương


3 3 3 3 3 3 2 2 2 3 3 3


2 2 2 3 3 3


3 13( 3 )


4( )



<i>x y</i> <i>y z</i> <i>z x</i> <i>x y z</i> <i>x</i> <i>y</i> <i>z</i> <i>xyz</i>


<i>x y z</i> <i>x</i> <i>y</i> <i>z</i> <i>xyz</i>


     




   .


Ta có các đánh giá <i><sub>x</sub></i>3<sub></sub><i><sub>y</sub></i>3<sub></sub><i><sub>z</sub></i>3<sub></sub><sub>3</sub><i><sub>xyz</sub></i><sub></sub><sub>(</sub><i><sub>x y z</sub></i><sub> </sub> <sub>) (</sub><sub></sub> <i><sub>x y</sub></i><sub></sub> <sub>)</sub>2<sub></sub><sub>(</sub><i><sub>z x z y</sub></i><sub></sub> <sub>)(</sub> <sub></sub> <sub>)</sub><sub></sub>


  và


3 3 3 3 3 3 2 2 2 2 2


2


3 ( ) ( ) ( )( )
( ) ( ) ( )( )


<i>x y</i> <i>y z</i> <i>z x</i> <i>x y z</i> <i>xy yz zx z x y</i> <i>xy z x z y</i>
<i>xy xy yz zx</i> <i>x y</i> <i>z x z y</i>


 


      <sub></sub>     <sub></sub>


 



   <sub></sub>     <sub></sub>


Ta cần chứng minh


2 2


2 3 3 3


( ) ( ) ( )( ) 13( ) ( ) ( )( )


4( )


<i>xy xy yz zx</i> <i>x y</i> <i>z x z y</i> <i>x y z</i> <i>x y</i> <i>z x z y</i>


<i>xyz</i> <i>x</i> <i>y</i> <i>z</i> <i>xyz</i>


   


  <sub></sub>     <sub></sub>   <sub></sub>     <sub></sub>




   hay


3 3 3 2


4(<i>xy yz zx x</i>  )( <i>y</i> <i>z</i> <i>xyz</i>) 13 <i>xyz x y z</i>(   ).


Do tính thuần nhất nên ta có thể giả sử <i>x y</i> 2 thì dẫn đến



3


3 3 <sub>2</sub> <sub>2</sub>


2
<i>x y</i>
<i>x</i> <i>y</i>   <sub></sub> <sub></sub> 


 


.
Suy ra


3 3 3 3 3 3


(<i>xy yz zx x</i>  )( <i>y</i> <i>z</i> <i>xyz</i>) ( <i>xy</i>2 )(2<i>z</i> <i>z</i> <i>xyz</i>)<i>xy</i>(2<i>z</i> ) 2 (2 <i>z</i> <i>z</i> <i>xyz</i>).
Ta đưa về


3 3 2 3 3 2


4 3 2 3 2


4 (2 ) 8 (2 ) 13 ( 2) 8 (2 ) (9 18 8)
8 9 18 16 8 (1 )(9 18 8) 0


<i>xy</i> <i>z</i> <i>z</i> <i>z</i> <i>xyz</i> <i>xyz z</i> <i>z</i> <i>z</i> <i>xy z</i> <i>z</i>


<i>z</i> <i>z</i> <i>z</i> <i>z</i> <i>xy</i> <i>z</i> <i>z</i>


          



         


Chú ý rằng <i>xy</i>1 và <i>z</i>1 nên (1<i>xy</i>)(9<i>z</i>318<i>z</i>28) 0 ; tiếp tục đặt <i>z</i> 1 <i>t</i> với <i>t</i>0


và thay vào 8<i>z</i>49<i>z</i>3 18<i>z</i>216<i>z</i> 8 8<i>t</i>423<i>t</i>33<i>t</i>215<i>t</i>5 với <i>t</i>0.


Dễ dàng chứng minh được biểu thức này không âm nên bất đẳng thức cần chứng minh
ở trên là đúng. Từ đó ta có đpcm.


Ta thấy rằng việc xử lí bất đẳng thức (**) không quá khó nhưng vấn đề là tại sao lại
nghĩ ra cách chọn 2, 9


3 4


<i>b c</i>  <i>a</i> để có được <i>k</i>14. Điểm tinh tế và cũng là khó nhất


</div>
<span class='text_page_counter'>(68)</span><div class='page_container' data-page=68>

24
Trong bất đẳng thức đã cho, đẳng thức xảy ra khi <i>a b c</i>  1 nên ta dự đoán là giảm số


biến bằng cách đặt <i>a</i> 1,<i>b</i> <i>x c</i>, 1


<i>x</i>


   thì thu được ngay <i>k</i> 4 <i>x</i> 1 2
<i>x</i>


 


 <sub></sub>   <sub></sub>



 


, tiếp tục cho


1


<i>x</i> thì có <i>k</i>16. Tuy nhiều điều kiện này cịn chưa chặt!


Ta tiếp tục cho <i><sub>b c</sub></i> 1<sub>,</sub><i><sub>a</sub></i> <i><sub>x</sub></i>2


<i>x</i>


   thì thay vào bất đẳng thức ban đầu, ta được


4 3 2 3 2


2 2


4(2 5 2 2) 4( 2)(2 1)


( )


2 2


<i>x</i> <i>x</i> <i>x</i> <i>x</i> <i>x</i> <i>x</i> <i>x</i>


<i>k</i> <i>f x</i>


<i>x</i> <i>x</i> <i>x</i> <i>x</i>



      


  


  với <i>x</i>0. (***)
Khi đó, ta phải có


0
min ( )


<i>x</i>


<i>k</i> <i>f x</i>




 .


Dưới đây ta sẽ phân tích một số cách để từ (***) có thể suy ra <i>k</i>14.


(1) Ta có <i>f x</i>( ) 0 tương đương với một phương trình bậc 5:


5 4 3


( ) 4 8 5 4 1 0


<i>g x</i>  <i>x</i>  <i>x</i>  <i>x</i>  <i>x</i>  .


Ta không giải được phương trình này, vì thế khơng thể tìm được chặn trên chính xác


cho <i>k</i>. Tuy nhiên, do bài toán yêu cầu tìm <i>k</i> nguyên dương lớn nhất, nên ta cũng
không cần đi tìm giá trị min, mà chỉ đánh giá nó nhằm tìm ra một chặn trên cho <i>k</i>.


Chú ý phương trình <i>g x</i>( ) 0 có thể viết dưới dạng 2


2 3


4 1
4<i>x</i> 8<i>x</i> 5


<i>x</i> <i>x</i>


    .


Trên miền (0;) thì vế trái là hàm tăng, vế phải là hàm giảm nên <i>g x</i>( ) 0 có nhiều
nhất 1 nghiệm dương.


Chú ý rằng 1 7 0, (1) 12 0
2 4


<i>g</i><sub></sub> <sub></sub>   <i>g</i>  


  nên nghiệm <i>x</i>0 của phương trình này nằm giữa


1


2 và 1. Ta khơng có ý định đi tìm <i>x</i>0 mà thơng tin này chỉ dùng để có định hướng để


chọn giá trị <i>x để thay vào bất đẳng thức </i>



3 2


2


4( 2)(2 1)
2


<i>x</i> <i>x</i> <i>x</i>


<i>k</i>
<i>x</i> <i>x</i>


  




 .


Ta thay các giá trị đặc biệt nằm trong 1;1
2
 
 
 .
 Thay <i>x</i>1, ta được <i>k</i>16.


 Thay 1


2


</div>
<span class='text_page_counter'>(69)</span><div class='page_container' data-page=69>

25


 Thay 2


3


<i>x</i> , ta được <i>k</i>13,98.


 Thay 3


4


<i>x</i> , ta được <i>k</i>14, 2.


Đến đây, ta chỉ ra được <i>k</i>14.


(2) Ở cách tiếp cận này, ta sẽ chứng minh rằng <i>k</i>14 không đúng (do các đánh giá đơn
giản dễ đưa về trường hợp <i>k</i>14 nhiều hơn).


Với <i>k</i>14, ta có bất đẳng thức 4<i>x</i>410<i>x</i>310<i>x</i>25<i>x</i> 4 0. Ta sẽ chứng minh rằng bất
đẳng thức này không đúng với mọi <i>x</i>0. Đặt <i>h x là vế trái của nó, ta có </i>( )


3 2 2


( ) 16 30 20 5, ( ) 48 60 20


<i>h x</i>  <i>x</i>  <i>x</i>  <i>x</i> <i>h x</i>  <i>x</i>  <i>x</i> .


Ta thấy <i>h x</i>( ) 0 có nghiệm duy nhất và <i>f</i>(0) 0  <i>f</i>(1) nên khảo sát hàm số <i>h x ta </i>( ),
được phương trình <i>h x</i>( ) 0 có nghiệm duy nhất <i>x</i><sub>0</sub> thỏa <i>x</i><sub>0</sub>(0;1) và <sub>0</sub>


0



min ( ) ( )


<i>x</i> <i>h x</i> <i>h x</i> .
Chú ý rằng 3 0 <sub>0</sub> 3 <sub>0</sub> 3;1


4 4 4


<i>f</i><sub></sub> <sub></sub> <i>a</i>  <i>a</i> <sub></sub> <sub></sub>


    và nên


3 2


0 0 0


16<i>x</i> 30<i>x</i> 20<i>x</i>  5 0 và ta có


4 3 2 3 2


0 0 0 0 0 0 0 0 0


3 2


0 0 0 0


7 ( ) 7(4 10 10 5 4) (16 30 20 5)
( 1)(12 52 2 28)


<i>f x</i> <i>x</i> <i>x</i> <i>x</i> <i>x</i> <i>x</i> <i>x</i> <i>x</i> <i>x</i>



<i>x</i> <i>x</i> <i>x</i> <i>x</i>


        


    


Ta có


3 2


3 2


0 0 0


3 3


12 52 2 28 12 52 30 0


4 4


<i>x</i>  <i>x</i>  <i>x</i>   <sub></sub> <sub></sub>   <sub> </sub>  


   


và <i>x</i><sub>0</sub> 1 0 nên <i>f x</i>( ) 0<sub>0</sub>  , mâu


thuẫn (chú ý rằng 3 2


12<i>x</i> 52<i>x</i> 2<i>x</i>28 đồng biến trên 3; 1


4


 


 


 


).
Do đó <i>k</i>14 khơng thỏa mãn.


(3) Do <i>f x có dạng khá cồng kềnh nên nếu để nguyên như vậy mà khảo sát thì sẽ khá </i>( )
phức tạp. Do đó, ta nghĩ đến việc biến đổi <i>f x về dạng đơn giản hơn để dễ tính đạo </i>( )
hàm và vì <i>f x có dạng phân thức nên ta nghĩ đến việc tách và chia đa thức: </i>( )


2 2 3


( ) 2 .


2 1


<i>f x</i> <i>x</i> <i>x</i>


<i>x</i> <i>x</i>


   




</div>
<span class='text_page_counter'>(70)</span><div class='page_container' data-page=70>

26



2 2


1 3


( ) 2 1 .


(2 1)


<i>f x</i> <i>x</i>


<i>x</i> <i>x</i>
 
  <sub></sub>    <sub></sub>

 


Ta cần giải phương trình <i>f x</i>( ) 0. Quan sát một chút, ta phát hiện được :


3 2 2


2 2 2 2 2


1 1 ( 1)( 1) 3 4( 1)


, 1 .


(2 1) (2 1)



<i>x</i> <i>x</i> <i>x</i> <i>x</i> <i>x</i> <i>x</i>


<i>x</i>


<i>x</i> <i>x</i> <i>x</i> <i>x</i> <i>x</i>


     


    


 


Như vậy, ta có thể phân tích được nhân tử:
2


2 2


1 4


( ) ( 1) ,


(2 1)


<i>x</i>
<i>f x</i> <i>x</i> <i>x</i>


<i>x</i> <i>x</i>
  
    <sub></sub>  <sub></sub>


 


từ đó ta đưa được việc xét <i>f x</i>( ) 0 về xét một phương trình đơn giản hơn:


3 2


2 2


1 4


0 4 4 3 1 0.
(2 1)


<i>x</i>


<i>x</i> <i>x</i> <i>x</i>


<i>x</i> <i>x</i>




      




Đến đây, ta gặp phải một khó khăn khá lớn, đó là phương trình <sub>4</sub><i><sub>x</sub></i>3<sub></sub><sub>4</sub><i><sub>x</sub></i>2<sub></sub><sub>3</sub><i><sub>x</sub></i><sub> </sub><sub>1 0</sub>
khơng có “nghiệm đẹp”. Khi khảo sát một phương trình, ta quan tâm đến hai việc:
phương trình đó có bao nhiêu nghiệm và đó là những giá trị nào. Việc thứ nhất có thể
dễ dàng thực hiện bằng cách viết lại phương trình dưới dạng:



2 1


4<i>x</i> 4<i>x</i> 3 0.


<i>x</i>


   


Vế trái của phương trình trên là một hàm liên tục và đồng biến với mọi <i>x</i>0, đồng thời
bằng kiểm tra trực tiếp ta dễ thấy phương trình có ít nhất một nghiệm thuộc (0, 1) nên
bằng cách kết hợp 2 điều này lại, ta có thể khẳng định phương trình <i>f x</i>( ) 0 có nghiệm
duy nhất thuộc (0, 1).


Việc thứ hai thực sự rất khó khăn trong trường hợp của bài tốn này. Rõ ràng với việc
chỉ bằng tính tay, ta rất khó tính được giá trị chính xác nghiệm của phương trình


3 2


</div>
<span class='text_page_counter'>(71)</span><div class='page_container' data-page=71>

27
trình 4<i>x</i>34<i>x</i>23<i>x</i> 1 0. Gọi <i>x</i><sub>0</sub>(0, 1) là nghiệm của phương trình. Ý tưởng của ta là
làm sao khử được dạng bậc ba để có được một phương trình bậc thấp có thể giải được
nhanh chóng bằng tính tay. Đầu tiên ta có để ý rằng với <i>x</i><sub>0</sub>(0,1) thì 4<i>x</i><sub>0</sub>3 4<i>x</i><sub>0</sub>2, do đó


2 3 2


0 0 0 0 0


8<i>x</i> 3<i>x</i>  1 4<i>x</i> 4<i>x</i> 3<i>x</i>  1 0.



Từ đây, ta tìm được chặn dưới cho <i>x</i><sub>0</sub> là <sub>0</sub> 3 41 1
16 2


<i>x</i>    Tiếp theo, ta sẽ tìm chặn trên
cho <i>x</i><sub>0</sub>. Bằng chú ý nhỏ rằng với <i>x</i><sub>0</sub>(0,1) thì các lũy thừa của nó khá nhỏ và “xấp xỉ”
với nhau nên để khử bậc ba, ta mạnh dạn sử dụng bất đẳng thức AM-GM như sau:


3 2 3 2 2 2 2


0 0 0 0 0 0 0 0 0 0 0 0


04<i>x</i> 4<i>x</i> 3<i>x</i>  1 (4<i>x</i> <i>x</i> ) 4 <i>x</i> 4<i>x</i>  1 4<i>x</i> 4<i>x</i> 4<i>x</i>  1 8<i>x</i> 4<i>x</i> 1.


Từ đây, ta tìm được chặn trên cho <i>x</i><sub>0</sub> là <sub>0</sub> 3 1
4


<i>x</i>   . Như vậy 3 41 <sub>0</sub> 3 1
16 <i>x</i> 4


 


  . Vì


các đánh giá khá sát nên ta mạnh dạn chọn 3 1
4


<i>x</i>  để thay vào <i>f x Để ý rằng </i>( ). <i>x là </i>
nghiệm của phương trình <sub>8</sub><i><sub>x</sub></i>2 <sub></sub><sub>4</sub><i><sub>x</sub></i><sub> </sub><sub>1 0,</sub><sub> do đó </sub> 2 4 1 1<sub>,</sub> <sub>8</sub> <sub>4,</sub> 3 <sub>4 4 .</sub>


8 2 1



<i>x</i>


<i>x</i> <i>x</i> <i>x</i>


<i>x</i> <i>x</i>




    




Suy ra


4 1 45 95 45 3 1 95 45 3 50


( ) 2 2(8 4) (4 4 ) .


8 2 8 2 4 8 8


<i>x</i>


<i>f x</i>    <i>x</i> <i>x</i>   <i>x</i>  <i>x</i>      


Với kết quả này, ta thu được 45 3 50 45 3 25


2 2 45 0,866 25 13,97.


<i>k</i>        



Tất nhiên, cũng có thể lựa chọn một giá trị nào đó đẹp hơn cho <i>x để thuận tiện hơn nữa </i>
cho việc tính tốn nhưng ở đây ta nên thận trọng vì chỉ cần chọn một số <i>x lệch hơi xa </i>
so với <i>x</i><sub>0</sub> thơi là có thể đưa đến việc <i>k</i> sẽ bị lệch đi mấy đơn vị sang 14, 15 thậm chí là


16. Tốt nhất, ta vẫn cứ nên sử dụng giá trị nào đó mà ta đã biết chắc rằng nó “sát” với
0


<i>x</i> dù lẻ một chút cũng không sao, bù lại ta sẽ có thể yên tâm hơn về kết quả.


Sau khi đã tìm được <i>k</i>13, ta có thể thử đi chứng minh bất đẳng thức đúng với <i>k</i>13


</div>
<span class='text_page_counter'>(72)</span><div class='page_container' data-page=72>

28
Cuối cùng, ta sẽ chỉ ra một cách để tìm hằng số thực <i>k</i> rất gần với hằng số tốt nhất để
bất đẳng thức (*) đúng.


Đặt <i><sub>F x</sub></i><sub>( ) 4(2</sub><i><sub>x</sub></i>4 <sub>5</sub><i><sub>x</sub></i>3 <sub>2</sub><i><sub>x</sub></i>2 <i><sub>x</sub></i> <sub>2)</sub> <i><sub>k x</sub></i><sub>(2</sub> 2 <sub>2) 0,</sub><i><sub>x</sub></i> 


         . Vì với hằng số tốt nhất thì
điểm cực tiểu <i>x</i><sub>0</sub> của đa thức gần với 2


3 nên ta giả sử rằng với <i>k</i> cần tìm thì điểm cực


tiểu <i>x</i><sub>0</sub> bằng đúng 2


3, tức là đạo hàm


3 2


( ) 32 60 16 4 (4 1)



<i>F x</i>  <i>x</i>  <i>x</i>  <i>x</i> <i>k x</i> bằng 0 tại


2
3


<i>x</i> , ta tính được 1372


99


<i>k</i> . Với giá trị <i>k</i> này thì <i>F x đạt được cực tiểu tại </i>( ) 2


3 nên


2 152


( ) 0


3 891
<i>F x</i> <i>F</i><sub></sub> <sub></sub> 


  .


Do đó, bất đẳng thức (*) đúng với 1372 13, 85859
99


<i>k</i>  .


Chú ý rằng với <sub>1</sub> 880 13,96825
63



<i>k</i><i>k</i>   và với <i>k</i><sub>2</sub> 13,85859 thì (*) cũng đúng nên hằng


số thực tốt nhất <i>k</i> thỏa mãn <i>k</i><sub>2</sub> <i>k</i><i>k</i><sub>1</sub> (ta tính được <i>k</i>13,96764). Một điều thú vị ở
đây là dù <i>k</i><sub>2</sub> dù lớn hơn 13 khá nhiều nhưng bất đẳng thức trong trường hợp tương
ứng lại dễ chứng minh hơn.


Nhận xét chung, bài tốn này có hình thức khá đơn giản nhưng lại địi hỏi nhiều xử lí
trung gian tinh tế, nhất là phần tính tốn trong điều kiện thời gian có giới hạn và khơng
có máy tính hỗ trợ. Nhiều bạn chủ quan khi gặp bài này, đánh giá <i>k</i> vội vàng để ra


8


<i>k</i> hoặc <i>k</i>16 rồi từ đó “kết luận” luôn và để mất điểm đáng tiếc. Một suy nghĩ
thường thấy là việc chứng minh bất đẳng thức một biến là chuyện đơn giản (các bài
dồn biến thường đưa về các bất đẳng thức 1 biến hiển nhiên đúng) nhưng thực ra
không phải vậy; cũng như các phương trình đại số, bất đẳng thức một biến cũng có thể
khó và thậm chí là rất khó nếu như trong q trình xử lí, chúng ta không thu được
nghiệm đặc biệt nào.


Có 2 bài tốn có cách giải khá giống với bài 4 (phần chứng minh), tuy nhiên đơn giản
hơn. Có lẽ bài tốn 4 được phát triển từ các bài toán dưới đây


</div>
<span class='text_page_counter'>(73)</span><div class='page_container' data-page=73>

29
1. Cho <i>a b c là các số thực dương sao cho </i>, , <i>abc</i>1. Chứng minh rằng


(<i>a b b c c a</i> )(  )(  ) 7 5(<i>a b c</i>  ).


2. Cho <i>a b c là các số thực dương sao cho </i>, , <i>abc</i>1. Chứng minh rằng



2 1 3


3


<i>a b c</i>   <i>ab bc ca</i>  .


<b>Bài 5. </b>


<i><b>Cho tam giác </b>ABC<b> nhọn không cân có góc </b>A<b> bằng </b></i>45<i><b>. Các đường cao </b><b>AD BE CF </b></i>, ,
<i><b>đồng quy tại trực tâm </b>H<b>. Đường thẳng </b>EF<b> cắt đường thẳng </b>BC<b> tại </b>P<b>. Gọi </b>I<b> là trung </b></i>
<i><b>điểm của </b>BC<b>; đường thẳng </b>IF<b> cắt </b>PH<b> tại </b><b>Q </b></i>.


<i><b>1. Chứng minh rằng </b></i><i>IQH</i> <i>AIE<b>. </b></i>


<i><b>2. Gọi </b>K<b> là trực tâm của tam giác </b>AEF<b> và </b></i>( )<i><b>J là đường tròn ngoại tiếp tam giác </b>KPD<b>. </b></i>
<i><b>Đường thẳng </b>CK<b> cắt </b></i>( )<i><b>J tại </b>G<b>, đường thẳng </b>IG<b> cắt </b></i>( )<i><b>J tại </b>M<b>, đường thẳng </b><b>JC cắt </b></i>
<i><b>đường tròn đường kính </b>BC<b>tại </b>N<b>. Chứng minh rằng các điểm </b><b>G M N C cùng thuộc </b></i>, , ,
<i><b>một đường tròn. </b></i>


<b>Lời giải. </b>


1. Giả sử <i>AB</i> <i>AC</i>, khi đó <i>B</i> sẽ nằm giữa <i>P C . Trường hợp </i>, <i>AB</i><i>AC</i> được chứng
minh hoàn toàn tương tự.


Trước hết, ta sẽ chứng minh rằng <i>PH</i> vng góc với <i>AI</i>.


Thật vậy, gọi <i>U V lần lượt là trung điểm của </i>, <i>AH IH thì ta có </i>, <i>UV AI</i> .


Dễ thấy ( , , , )<i>P D B C</i>  1 nên theo tính chất của hàng điểm điều hịa thì <i>PB PC</i> <i>PD PI</i> .



Ta cũng có <i>PE PF</i> <i>PB PC</i> nên <i>PE PF</i> <i>PD PI</i> hay <i>P</i> nằm trên trục đẳng phương
của đường trịn đường kính <i>AH</i> (tâm <i>U</i>) và <i>IH</i> (tâm <i>V</i>).


Hơn nữa <i>H</i> cũng nằm trên trục đẳng phương của hai đường trịn này nên <i>PH</i> <i>UV</i> .
Do đó <i>PH</i><i>AI</i>.


Vì <i>BAC</i>45 nên <i>EIF</i>90, suy ra<i>IQH</i>90  <i>AIF</i> <i>EIF</i> <i>AIF</i> <i>AIE</i>.


</div>
<span class='text_page_counter'>(74)</span><div class='page_container' data-page=74>

30

<i><b>T</b></i>



<i><b>N</b></i>



<i><b>M</b></i>


<i><b>G</b></i>



<i><b>J</b></i>

<i><b>K</b></i>



<i><b>Q</b></i>



<i><b>I</b></i>


<i><b>P</b></i>



<i><b>E</b></i>



<i><b>D</b></i>


<i><b>H</b></i>


<i><b>F</b></i>



<i><b>A</b></i>




<i><b>B</b></i>

<i><b>C</b></i>



2. Ta thấy rằng <i>EKF</i> <i>ECF</i> <i>EKF</i> <i>EAF</i>180 nên <i>K</i> thuộc đường trịn đường
kính <i>BC</i>. Do hàng điểm <i>D P B C điều hịa nên ta có </i>, , , <i><sub>ID IP</sub></i><sub></sub> <sub></sub><i><sub>IC</sub></i>2<sub>, mà </sub><i><sub>IM IG</sub></i><sub></sub> <sub></sub><i><sub>ID IP</sub></i><sub></sub>
(cùng bằng phương tích của <i>I</i> đến ( )<i>J ) nên </i> 2


<i>IM IG</i> <i>IC</i> hay
( . . )


<i>IMC</i> <i>ICG c g c</i>


  nên <i>IMC</i> <i>ICG</i> <i>ICK</i>45 . (1)


Gọi <i>T</i> là trung điểm <i>PD</i> thì theo hệ thức Maclaurin, ta có <i>CB CT</i> <i>CD CP CK CG</i>  


hay tứ giác <i>GTBK</i> nội tiếp và do <i>BKG</i>90 nên cũng có <i>GTD</i>90 hay <i>GT</i><i>PD</i>.


Tam giác <i>JPD cân tại J và có T</i> là trung điểm <i>PD</i> nên <i>JT vng góc với PD</i>.


Do đó, <i>G J T thẳng hàng và </i>, , <i>KGJ</i>45.


Mặt khác <i>CN CJ</i> <i>CB CT</i> <i>CK CG</i> nên tứ giác <i>KNJG nội tiếp và dẫn đến </i>
45


<i>JNG</i> <i>JKG</i> <i>JGK</i>


      . (2)


</div>
<span class='text_page_counter'>(75)</span><div class='page_container' data-page=75>

31


<b>Nhận xét. </b>


Ở ý thứ 1, việc chứng minh <i>PH</i> vng góc với <i>AI</i> có thể được thực hiện bằng nhiều
cách. Chẳng hạn, xét tứ giác <i>BCEF</i> nội tiếp đường tròn tâm <i>I</i>. Theo định lí Brocard thì


<i>H</i> chính là trực tâm của tam giác <i>AIP</i> nên <i>PH</i> vng góc với <i>AI</i>.


Cũng có thể dùng cách tính toán chứng minh <i>AD</i> <i>PD</i>


<i>ID</i> <i>HD</i> để chứng minh kết quả này.


Ý thứ 2 là một ứng dụng rất đẹp của hang điểm điều hịa trong đó có kết hợp cả hai hệ
thức cơ bản là hệ thức Newton và hệ thức Maclaurin. Số bài tốn về hàng điểm điều
hịa vận dụng đến cả hai hệ thức này trong một bài tốn là khơng nhiều, nên câu 2) là
một ý chứng minh rất hay và thú vị. Ngoài cách giải ở trên, ta có thể sử dụng hướng
tiếp cận khác như sau:


Gọi <i>L</i> là giao điểm thứ hai khác <i>K</i> của hai đường tròn ( ),( )<i>I</i> <i>J . Khi đó, để chứng minh </i>
, , ,


<i>G M N C cùng thuộc một đường tròn, ta đưa về chứng minh GI CJ LK đồng quy. </i>, ,
Trước hết, ta có thể đi theo hướng hồn tồn thuần túy như sau:


<i><b>L</b></i>



<i><b>N</b></i>



<i><b>M</b></i>


<i><b>G</b></i>




<i><b>J</b></i>

<i><b>K</b></i>



<i><b>I</b></i>


<i><b>P</b></i>



<i><b>E</b></i>



<i><b>D</b></i>


<i><b>H</b></i>


<i><b>F</b></i>



<i><b>A</b></i>



</div>
<span class='text_page_counter'>(76)</span><div class='page_container' data-page=76>

32
Chú ý ( , , , )<i>P D B C</i>  1 và <i>BKC</i>90 nên <i>KB KC lần lượt là phân giác trong và ngồi </i>,
của góc <i>PKD</i>. Từ đó dẫn đến <i>DKI</i> <i>DPK</i> hay <i>IK</i> tiếp xúc với đường trịn ( )<i>J , điều </i>
này có nghĩa là các đường tròn ( ),( )<i>I</i> <i>J trực giao và dẫn đến </i><i>ILJ</i> <i>GLC</i>90.


Hơn nữa, <i>GI</i> và <i>CJ lần lượt là các đường đối trung trong các tam giác GKL CKL . Gọi </i>,


<i>U</i> là trung điểm <i>KL</i> thì <i>GI CJ đối xứng với </i>, <i>GU CU qua phân giác các góc </i>,
,


<i>LGC</i> <i>LCG</i>


  của tam giác <i>LGC</i>.


Do các đường thẳng <i>LK GU CU đồng quy tại </i>, , <i>U</i> và <i>LK</i> là phân giác góc <i>GLC</i> nên các
đường thẳng <i>LK GI CJ đồng quy. </i>, ,



Từ đó lời giải bài tốn hồn tất.


Tuy nhiên, ngay từ đầu, ta có thể phát biểu lại bài toán thành:


<i><b>Cho tam giác vuông </b><b>KIJ ở </b>K<b> và dựng phía ngồi các tam giác </b><b>KJG vuông cân đỉnh J </b></i>
<i><b>và </b>KIC<b> vuông cân đỉnh </b>I<b>. Chứng minh </b><b>GI CJ đồng quy tại một điểm nằm trên đường </b></i>,
<i><b>cao đỉnh </b>K<b> của tam giác </b><b>KIJ . </b></i>


(Chú ý rằng các tam giác <i>GJK CIK vuông cân là có thể chứng minh hồn tồn dễ dàng). </i>,
Để giải bài toán này, ta chỉ cần biến đổi đại số thơng qua định lí Ceva, Thales là có thể
xử lí nhanh chóng.


Tuy rằng một điều chưa hồn hảo lắm trong bài tốn này là câu 1) và câu 2) hầu như
không liên quan gì tới nhau.


Thực chất vai trị của giả thiết góc<i>A</i> bằng 45 cũng khơng thật sự cần thiết lắm trong
bài tốn này. Các bạn có thể tìm hiểu một mở rộng đơn giản nhất như sau


<i><b>Cho tam giác </b>ABC<b> đường cao </b><b>AD BE CF đồng quy tại </b></i>, , <i>H<b>, </b>EF<b>cắt </b>BC<b> tại </b>G<b>. Gọi </b></i>( )<i><b>K </b></i>
<i><b>là đường trịn đường kính </b>BC<b>. Trung trực của </b>BC<b> cắt ( )</b><b>K tại điểm </b>L<b> sao cho </b><b>A L nằm </b></i>,
<i><b>cùng phía đối với </b>BC<b>. Gọi ( )</b><b>N là đường tròn ngoại tiếp tam giác </b>GDL<b>. Đường thẳng </b></i>


<i>CL<b> cắt </b></i>( )<i><b>N tại </b>M<b> khác </b>L<b>. Đường thẳng </b>MK<b> cắt </b></i>( )<i><b>N tại </b>P<b> khác </b>M<b>. Đường thẳng </b></i>


<i>CN<b> cắt ( )</b><b>K tại Q khác </b>C<b>. Chứng minh rằng </b><b>M P Q C cùng thuộc một đường tròn. </b></i>, , ,


Cách chứng minh hoàn toàn tương tự như lời giải bài toán gốc.


</div>
<span class='text_page_counter'>(77)</span><div class='page_container' data-page=77>

33
<b>Bài 6. </b>



<i><b>Cho một khối lập phương </b></i>10 10 10  <i><b> gồm 1000 ô vuông đơn vị màu trắng. An và Bình </b></i>
<i><b>chơi một trị chơi. An chọn một số dải </b></i>1 1 10  <i><b> sao cho hai dải bất kì khơng có chung </b></i>
<i><b>đỉnh hoặc cạnh rồi đổi tất cả các ơ sang màu đen. Bình thì được chọn một số ơ bất kì </b></i>
<i><b>của hình lập phương rồi hỏi An các ơ này màu gì. Hỏi Bình phải chọn ít nhất bao nhiêu </b></i>
<i><b>ơ để với mọi câu trả lời của An thì Bình ln xác định được những ô nào là màu đen? </b></i>


<b>Lời giải. </b>


Trước hết, ta sẽ chứng minh nhận xét tổng quát:
<i>Cho một khối lập phương </i>2<i>n</i>2<i>n</i>2<i>n gồm </i> 3


<i>8n ô vuông đơn vị màu trắng. An và Bình chơi </i>
<i>một trị chơi. An chọn một số dải 1 1 n</i>  <i> sao cho với hai dải bất kì thì chúng khơng có chung </i>
<i>đỉnh hoặc cạnh rồi đổi tất cả các ô sang màu đen. Bình thì được chọn một số ơ bất kì của hình lập </i>
<i>phương rồi hỏi An các ơ này có màu gì. Khi đó, Bình cần chọn ít nhất 6n ơ mới có thể xác định </i>2
<i>được ơ nào có màu đen. </i>


</div>
<span class='text_page_counter'>(78)</span><div class='page_container' data-page=78>

34
Do điều kiện hai dãy được chọn bất kì khơng chung cạnh và đỉnh nên với mỗi câu trả
lời về màu cho mỗi ơ <i>u mà Bình chọn. Ta thấy rằng nếu gọi R<sub>u</sub></i> là hợp của các ô được
chọn thì khi ơ đó màu đen, sẽ có đúng một trong ba dải ngang, dọc, chéo đi qua ơ đó
được tô màu đen. Trong trường hợp này, ta cần chọn thêm một số ô thuộc <i>R<sub>u</sub></i> để biết
chính xác dải đó. Nếu chỉ chọn thêm một ơ thơi thì khi An trả lời ơ đó màu trắng, Bình
sẽ khơng xác định được dải nào trong hai dải cịn lại được tơ đen. Do đó, Bình phải
chọn thêm ít nhất hai ơ nữa trong <i>R<sub>u</sub></i> mới có khả năng trả lời được. Thêm vào đó, hai ơ
đó phải thuộc hai dải khác nhau vì nếu chúng cùng thuộc một dải thì cũng tương tự
như trường hợp chọn một ô nêu trên.


Với nhận xét này, ta gán cho mỗi ơ <i>u của hình lập phương một bộ số ( , , )a b c với định </i>


nghĩa như sau:


 <i>a</i>2 nếu dải hình hộp theo chiều ngang đi qua <i>u khơng có thêm điểm nào </i>
thuộc <i>S<sub>n</sub></i> và <i>a</i>1 nếu ngược lại.


 <i>b</i>2 nếu dải hình hộp theo chiều dọc đi qua <i>u khơng có thêm điểm nào thuộc </i>


<i>n</i>


</div>
<span class='text_page_counter'>(79)</span><div class='page_container' data-page=79>

35
 <i>c</i>2 nếu dải hình hộp theo chiều chéo đi qua <i>u khơng có thêm điểm nào thuộc </i>


<i>n</i>


<i>S</i> và <i>c</i>1 nếu ngược lại.


Khi đó, có hai trong ba số <i>a b c có giá trị bằng 1 và số cịn lại khơng vượt quá 2 nên </i>, ,


4


<i>a b c</i>   .


Đặt <i>T</i> là tổng các số dùng để gán cho các ơ của hình lập phương. Khi đó, ta có


( ) 4


<i>n</i>


<i>n</i>
<i>u S</i>



<i>T</i> <i>a b c</i> <i>S</i>




<sub></sub>

   .


Mặt khác, với mỗi dãy <i>1 1 n</i>  của khối lập phương (theo cả ba chiều) đều có ít nhất
một khối thuộc tập hợp <i>S<sub>n</sub></i> vì nếu khơng, Bình sẽ khơng có thơng tin gì về dải đó và
trong trường hợp An trả lời rằng tất cả các ơ được Bình chọn đều được tơ màu trắng thì
Bình sẽ khơng biết được dải cịn lại đó có được tơ màu đen hay khơng.


Dễ thấy rằng có tất cả <sub>(2 )</sub><i><sub>n</sub></i> 2<sub> dải </sub>


<i>1 1 2n</i>  nằm ngang và tất cả các dải này sẽ đóng góp ít
nhất <sub>2(2 )</sub><i><sub>n</sub></i> 2<sub> đơn vị vào </sub><i><sub>T</sub></i><sub> (đóng góp vào các số </sub><i><sub>a theo định nghĩa như trên). </sub></i>


Tương tự với <sub>(2 )</sub><i><sub>n</sub></i> 2<sub> dải </sub>


<i>1 1 2n</i>  dọc và <sub>(2 )</sub><i><sub>n</sub></i> 2<sub> dải </sub>


<i>1 1 2n</i>  chéo nên ta suy ra


2 2


3 2(2 ) 24


<i>T</i>  <i>n</i>  <i>n</i> .


Từ đó, ta được 4<i>S<sub>n</sub></i> 24<i>n</i>2 hay <i>S<sub>n</sub></i> 6 ,<i>n n</i>2 


Đến đây, ta suy ra hai điều sau:


- Trong hình lập phương 2 2 2  , Bình cần chọn ít nhất 6 ơ.
- Trong hình lập phương 10 10 10  , Bình cần chọn ít nhất 150 ơ.


Ta sẽ chỉ ra cách tô màu 6 ô thỏa mãn đề bài và từ đó chỉ ra cách xây dựng cho hình lập
phương 10 10 10  đã cho. (Trên thực tế, ta hồn tồn có thể xây dựng cho trường hợp
tổng quát cho hình lập phương 2<i>n</i>2<i>n</i>2<i>n</i>).


Thật vậy, trong hình lập phương 2 2 2  , trừ hai ơ nào đó đối xứng nhau qua tâm, Bình
chọn 6 ơ cịn lại như hình bên dưới.


</div>
<span class='text_page_counter'>(80)</span><div class='page_container' data-page=80>

36
Dễ thấy rằng trong hình lập phương 2 2 2  , chỉ có khơng q 1 dải được tơ màu nên
hoặc khơng có ơ nào hoặc có 2 ơ của hình lập phương được tơ màu và 2 ơ đó phải thuộc
cùng 1 dải. Do đó, trong 6 ơ được chọn, ta có 3 trường hợp:


- Nếu khơng có ơ nào được tơ đen thì cả hình lập phương khơng có.


- Nếu có đúng 1 ơ được tơ đen thì ơ đen cịn lại sẽ thuộc trong 2 ơ khơng được chọn có
cùng 1 dải với ơ đen đã biết.


- Nếu có 2 ơ được tơ đen thì đó chính là tất cả các ơ đen của hình lập phương.
Do đó, cách chọn này với hình lập phương 2 2 2  thỏa mãn điều kiện đề bài.
Tiếp theo, ta xây dựng cho hình lập phương 10 10 10  như sau:


Ta chia hình lập phương thành 5 lớp 10 10 2  và ta chia nó thành 25 phần, mỗi phần là
một hình lập phương 2 2 2  rồi đánh số như hình bên dưới:


<i><b>5</b></i>


<i><b>5</b></i>


<i><b>5</b></i>


<i><b>5</b></i>
<i><b>4</b></i>
<i><b>4</b></i>


<i><b>4</b></i>


<i><b>4</b></i>


<i><b>3</b></i>
<i><b>3</b></i>


<i><b>3</b></i>
<i><b>3</b></i>


<i><b>2</b></i>


<i><b>2</b></i>
<i><b>2</b></i>


<i><b>2</b></i>


<i><b>1</b></i>


<i><b>1</b></i>
<i><b>1</b></i>



<i><b>1</b></i>


<i><b>5</b></i>
<i><b>4</b></i>
<i><b>3</b></i>


</div>
<span class='text_page_counter'>(81)</span><div class='page_container' data-page=81>

37
Ở lớp thứ <i>i</i> với <i>i</i>1, 2,3, 4, 5, ta chọn các khối được đánh số <i>i</i> và với mỗi khối đó, ta bỏ
đi 2 ơ bất kì đối xứng nhau qua tâm như cách đã nêu ở trên rồi chọn 6 còn lại.


Dễ thấy rằng với cách tô như vậy, ta chiếu các hình lập phương đã được chọn xuống
một mặt nào đó thì các hình chiếu sẽ phủ kín mặt đó. Điều này có nghĩa là với một dải
bất kì mà An đã chọn thì nó đều đi qua một trong các hình lập phương 2 2 2  mà Bình
chọn như trên. Khi đó, như đã chứng minh ở trên, ta sẽ xác định được rằng dải đó có
được tơ màu hay không, tức là xác định được màu của tất cả các ơ được tơ màu đen của
hình lập phương ban đầu. Do đó, cách chọn các ơ này thỏa mãn đề bài.


Vậy số ơ ít nhất mà Bình cần chọn là 150.


<b>Nhận xét. </b>


Có thể nói đây là bài tốn khó nhất của kỳ thi, tuy nhiên, cái khó của nó khơng nặng
tính kỹ thuật như bài 3 mà là cái khó về mặt phương pháp. Thực tế nhiều thí sinh đã
nói rằng “Em thực sự không biết phải xoay sở thế nào!”.


Để làm được bài toán này, trước hết phải thật tập trung để hiểu rõ yêu cầu của bài toán
và phải dùng trường hợp <i>n</i>2 hoặc xét bài toán 2 chiều để hình dung bài tốn một
cách cụ thể nhất. Trong các nhận xét ở trên thì nhật xét rằng mỗi dải đều có một ô được
chọn là khá hiển nhiên (và nhiều thí sinh đã phát biểu được nhận xét này), cịn nhận xét
rằng ngồi khối <i>u thì trong hợp R<sub>u</sub></i> cịn có ít nhất 2 khối nữa được chọn thì tinh tế hơn.


Khi đã có hai nhận xét này thì việc áp dụng kỹ thuật đếm bằng hai cách để đánh giá là
khá tự nhiên.


Việc xây dựng cấu hình cách chọn cũng là một thách thức.


Ở đây, trường hợp <i>n</i>2 đóng một vai trò quan trọng, như những viên gạch để ta xây
dựng. Rõ ràng các cấu trúc cơ bản như hình vuông Latin ở trong bài giải đã giúp chúng
ta giải quyết bài tốn. Các mơ hình, cấu trúc, cách sắp xếp cơ bản vì vậy ln đóng một
vai trò quan trọng trong các bài tốn xây dựng ví dụ, phản ví dụ. Nếu có làm quen
nhiều với các bài tốn về cực trị rời rạc thì có thể xử lí bài này, ít nhất là việc xây dựng
một cách chủ động hơn.


</div>
<span class='text_page_counter'>(82)</span><div class='page_container' data-page=82>

38
<i><b>(Việt Nam TST 2000) Trong mặt phẳng cho 2000 đường tròn bán kính 1 sao cho khơng có </b></i>
<i><b>hai đường trịn nào tiếp xúc nhau và hợp của các đường tròn này tạo thành một tập hợp </b></i>
<i><b>liên thông. Chứng minh rằng số các điểm thuộc ít nhất hai đường trịn trong các đường </b></i>
<i><b>trịn đã cho khơng nhỏ hơn 2000. </b></i>


<i><b>(Việt Nam TST 2010) Gọi một hình chữ nhật có kích thước </b></i>1 2 <i><b> là hình chữ nhật đơn và </b></i>
<i><b>một hình chữ nhật có kích thước</b></i>2 3 <i><b>, bỏ đi 2 ơ ở góc chéo nhau (tức có có 4 ơ vng </b></i>
<i><b>con) là hình chữ nhật kép. Người ta ghép khít các hình chữ nhật đơn và hình chữ nhật </b></i>
<i><b>kép này lại với nhau được một bảng hình chữ nhật có kích thước là</b></i>2008 2010 <i><b>. Tìm số </b></i>
<i><b>nhỏ nhất các hình chữ nhật đơn có thể dùng để ghép. </b></i>


Một lần nữa thì trong bài tốn cực trị ở kì thi chọn đội tuyển, dấu bằng xảy ra có thể xây
dựng khơng q khó nhưng để chỉ ra đó là giá trị tốt nhất thì là điều hồn tồn khơng
dễ dàng, thậm chí rất khó!


Một vấn đề thú vị đặt ra là nếu kích thước của hình lập phương là lẻ thì ta sẽ có kết quả
thế nào? Áp dụng cách đánh giá chặn dưới như trường hợp <i>n chẵn, ta được nếu </i>



2 1


<i>n</i> <i>k</i> thì số hình vng cần được đánh dấu phải thỏa mãn bất đẳng thức
2


3(2 1)
2
<i>k</i>


<i>T</i>   . Từ đây suy ra <i>T</i> 6<i>k</i>2 6<i>k</i>2<i>. Tuy nhiên, việc xây dựng cách chọn </i>
2


6<i>k</i> 6<i>k</i>2 hình lập phương đơn vị thỏa mãn yêu cầu bài toán vẫn là một câu hỏi mở.
Với <i>n</i>3, tức là <i>k</i>1, hiện nay chưa tìm được cách chọn 14 hình lập phương thỏa mãn
yêu cầu bài toán và cũng chưa chứng minh được là cần nhiều hơn. (Ta có thể chỉ được
15 hình lập phương thỏa mãn yêu cầu).


<i><b>Ta thử thay giả thiết đã nêu trong đề bài một chút để có bài tốn mới: “Bình chọn một ơ </b></i>
<i><b>bất kì của hình lập phương rồi hỏi An ơ này có màu gì rồi sau đó tiếp tục như thế” và </b></i>
các dữ kiện khác vẫn giữ tương tự.


<i><b>Hoặc bài toán trong trường hợp hai chiều: Thay khối lập phương bởi bảng ô vuông và </b></i>
<i><b>thay các dải ô vuông được chọn bởi các hàng, cột của bảng. </b></i>


</div>
<span class='text_page_counter'>(83)</span><div class='page_container' data-page=83>

Trần Nam Dũng (chủ biên)



Võ Quốc Bá Cẩn – Trần Quang Hùng – Lê Phúc Lữ


Hoàng Đỗ Kiên – Nguyễn Huy Tùng




<b>LỜI GIẢI VÀ BÌNH LUẬN</b>



<b>ĐỀ CHỌN ĐỘI TUYỂN QUỐC GIA</b>


<b>DỰ THI IMO 2014</b>



</div>
<span class='text_page_counter'>(84)</span><div class='page_container' data-page=84></div>
<span class='text_page_counter'>(85)</span><div class='page_container' data-page=85>

Việt Nam Team Selection Test 2014 2


<b>Đề thi chính thức</b>



<i><b>Ngày thi thứ nhất - 25/03/2014</b></i>



Bài 1.Tìm tất cả các hàm số <i><b>f</b></i> <b>:Z→Z</b>thỏa mãn


<i><b>f</b></i> <b>2m+</b> <i><b>f</b></i><b>(</b><i><b>m</b></i><b>) +</b> <i><b>f</b></i><b>(</b><i><b>m</b></i><b>)</b><i><b>f</b></i><b>(</b><i><b>n</b></i><b>)</b>


<b>=</b><i><b>n</b></i><b>·</b> <i><b>f</b></i><b>(</b><i><b>m</b></i><b>) +</b><i><b>m,</b></i> <b>∀</b><i><b>m,</b><b>n</b></i><b>∈Z.</b>


Bài 2.Trong mặt phẳng tọa độ vuông góc <i><b>O x y,</b></i>xét các điểm nguyên có tọa độ thuộc tập
hợp sau


<i><b>T</b></i> <b>=</b>


<b>(</b><i><b>x</b></i><b>;</b> <i><b>y</b></i><b>):−20¶</b><i><b>x</b></i><b>,</b> <i><b>y</b></i><b>¶20,(</b><i><b>x</b></i><b>;</b> <i><b>y</b></i><b>)6= (0; 0)</b> <b>.</b>


Tơ màu các điểm thuộc<i><b>T</b></i> sao cho với mọi điểm có tọa độ<b>(</b><i><b>x</b></i><b>,</b><i><b>y</b></i><b>)∈</b><i><b>T</b></i> thì có đúng một trong
hai điểm <b>(</b><i><b>x</b></i><b>;</b><i><b>y</b></i><b>)</b> và <b>(<sub>−</sub></b><i><b>x</b></i><b>;−</b><i><b>y</b></i><b>)</b> được tô màu. Với mỗi cách tô như thế, gọi <i><b>N</b></i> là số các bộ


<b>(</b><i><b>x</b></i><b>1;</b><i><b>y</b></i><b>1),(</b><i><b>x</b></i><b>2;</b><i><b>y</b></i><b>2)</b>mà cả hai điểm này cùng được tơ màu và


<i><b>x</b></i><b>1≡2x2,</b><i><b>y</b></i><b>1≡2</b><i><b>y</b></i><b>2(mod 41).</b>



Tìm tất cả các giá trị có thể có của<i><b>N</b></i><b>.</b>


Bài 3.Cho tam giác <i><b>ABC</b></i> nội tiếp trong đường trịn<b>(</b><i><b>O</b></i><b>).</b>Trên cung <i><b>BC</b></i> khơng chứa <i><b>A</b></i>của


<b>(</b><i><b>O</b></i><b>)</b> lấy điểm <i><b>D</b></i>. Giả sử<i><b>C D</b></i> cắt <i><b>AB</b></i> ở <i><b>E</b></i> và <i><b>BD</b></i> cắt <i><b>AC</b></i> ở <i><b>F</b></i>. Gọi<b>(</b><i><b>K</b></i><b>)</b>đường tròn nằm trong
tam giác<i><b>E BD</b></i>, tiếp xúc với<i><b>E B,</b><b>E D</b></i> và tiếp xúc với đường tròn <b>(</b><i><b>O</b></i><b>).</b> Gọi<b>(</b><i><b>L</b></i><b>)</b>là tâm đường
tròn nằm trong tam giác<i><b>F C D</b></i>, tiếp xúc với <i><b>F C,</b><b>F D</b></i>và tiếp xúc với đường tròn<b>(</b><i><b>O</b></i><b>).</b>


a) Gọi <i><b>M</b></i> là tiếp điểm của <b>(</b><i><b>K</b></i><b>)</b> với<i><b>BE</b></i> và <i><b>N</b></i> là tiếp điểm của <b>(</b><i><b>L</b></i><b>)</b> với<i><b>C F</b></i>. Chứng minh
rằng đường trịn đường kính <i><b>M N</b></i> ln đi qua một điểm cố định khi<i><b>D</b></i> di chuyển.
b) Đường thẳng qua <i><b>M</b></i> và song song với <i><b>C E</b></i> cắt <i><b>AC</b></i> ở <i><b>P</b></i>, đường thẳng qua <i><b>N</b></i> và song


song với<i><b>BF</b></i> cắt<i><b>AB</b></i>ở<i><b>Q</b></i>. Chứng minh rằng đường tròn ngoại tiếp tam giác<i><b>AM P</b></i><b>,</b><i><b>ANQ</b></i>
cùng tiếp xúc với một đường tròn cố định khi <i><b>D</b></i>di chuyển.


<i><b>Ngày thi thứ hai - 26/03/2014</b></i>



Bài 4.Cho tam giác <i><b>ABC</b></i> nhọn, khơng cân có đường cao <i><b>AD</b></i> và <i><b>P</b></i> thuộc <i><b>AD</b></i>. Các đường
thẳng<i><b>P B,</b><b>PC</b></i> lần lượt cắt<i><b>C A,</b><b>AB</b></i>tại <i><b>E,</b><b>F.</b></i>


a) Giả sử tứ giác<i><b>AE D F</b></i> nội tiếp. Chứng minh rằng <i><b>PA</b></i>


<i><b>P D</b></i> <b>= (tan</b><i><b>B</b></i><b>+tan</b><i><b>C</b></i><b>)cot</b>
<i><b>A</b></i>
<b>2.</b>


b) Gọi<i><b>C P</b></i>cắt đường thẳng qua<i><b>B</b></i>vng góc<i><b>AB</b></i>tại<i><b>M</b></i>.<i><b>BP</b></i>cắt đường thẳng qua<i><b>C</b></i> vng
góc <i><b>AC</b></i> tại <i><b>N</b></i>. <i><b>K</b></i> là hình chiếu của <i><b>A</b></i> lên <i><b>M N</b></i>. Chứng minh rằng <b><sub>∠</sub></b><i><b>B KC</b></i> <b>+<sub>∠</sub></b><i><b>M AN</b></i>
khơng đổi khi <i><b>P</b></i> di chuyển trên<i><b>AD.</b></i>



Bài 5.Tìm tất cả đa thức <i><b>P</b></i><b>(</b><i><b>x</b></i><b>),</b><i><b>Q</b></i><b>(</b><i><b>x</b></i><b>)</b> có hệ số nguyên và thỏa mãn điều kiện: Với dãy số


<b>(</b><i><b>x</b><b>n</b></i><b>)</b>xác định bởi


<i><b>x</b></i><b>0=2014,</b><i><b>x</b></i><b>2</b><i><b>n</b></i><b>+1=</b><i><b>P</b></i><b>(</b><i><b>x</b></i><b>2</b><i><b>n</b></i><b>),</b><i><b>x</b></i><b>2</b><i><b>n</b></i><b>=</b><i><b>Q</b></i><b>(</b><i><b>x</b></i><b>2</b><i><b>n</b></i><b>−1)</b>với<i><b>n</b></i><b>¾1.</b>


</div>
<span class='text_page_counter'>(86)</span><div class='page_container' data-page=86>

Việt Nam Team Selection Test 2014 3


Bài 6.Cho<i><b>m,</b><b>n</b></i><b>,</b><i><b>p</b></i>là các số tự nhiên không đồng thời bằng<b>0.</b>Không gian tọa độ được chia
thành các mặt phẳng song song cách đều nhau. Một cách điền vào mỗi khối lập phương
đơn vị một trong các số từ<b>1</b>đến<b>60</b>được gọi là cách điền<i><b>Điện Biên</b></i>nếu thỏa mãn: trong
mỗi hình hộp chữ nhật với các mặt trên hệ mặt đã cho và tập hợp kích thước ba cạnh (số
hình lập phương trên trên cạnh) xuất phát từ một đỉnh là<b>2m+1, 2n+1, 2</b><i><b>p</b></i><b>+1</b>, khối lập
phương đơn vị có tâm trùng với tâm của hình hộp chữ nhật được điền số bằng trung bình
cộng của các số điền ở tâm của<b>8</b>hình lập phương ở các góc của hình hộp đó. Hỏi có tất cả
bao nhiêu cách điền<i><b>Điện Biên</b></i>?


</div>
<span class='text_page_counter'>(87)</span><div class='page_container' data-page=87>

Việt Nam Team Selection Test 2014 4


<b>Nhận xét chung</b>



Xét về cấu trúc, đề thi chọn đội tuyển thi<b>IMO</b> năm nay gồm<b>6</b>bài tốn, trong đó có <b>2</b>bài
hình học phẳng,<b>2</b>bài đại số và<b>2</b>bài tổ hợp. Cụ thể như sau:


• Bài 1: Đại số (phương trình hàm trên tập số ngun).
• Bài 2: Tổ hợp (bài tốn đếm).


• Bài 3: Hình học phẳng (đường và điểm cố định).
• Bài 4: Hình học phẳng (hình học tính tốn).



• Bài 5: Đại số (phương trình hàm đa thức trên tập số ngun).
• Bài 6: Tổ hợp (bài tốn đếm)


Ngồi hai bài hình ra thì tính chất số học xuất hiện ở cả bốn bài cịn lại nhưng chỉ đóng
các vai trị làm nền để xây dựng các vấn đề chứ không thực sự là các bài số học. Trong mỗi
ngày, các bài toán được sắp xếp một cách tương đối theo độ khó tăng dần. Bài<b>1</b>và<b>4</b>có thể
coi là hai bài dễ. Tiếp theo là bài<b>2,</b>bài<b>5</b> ở mức độ trung bình và khó nhất là bài<b>3</b> và bài
<b>6.</b>Theo ý kiến của chúng tôi, để lọt vào đội tuyển, các thí sinh chắc chắn sẽ phải giải quyết
được bài<b>1, 4</b>và ít nhất làm được hồn chỉnh<b>1</b>bài trong các bài<b>2, 5</b>(cũng là các bài quyết
định), còn lại khó có thí sinh nào giải quyết được trọn vẹn bài<b>3</b>và<b>6.</b>


Hai bài hình học năm nay, xét về tính phân loại thì ổn, có một bài dễ và một bài khó, trong
đó câu<b>3b</b>sẽ là thách thức đáng kể. Bài<b>4</b>khá dễ khi xét lại các bài cùng vị trí ở kỳ thi các
năm trước. Tuy nhiên, nếu đặt vào chung với các bài<b>5</b>và <b>6</b>trong một ngày thi thì lại khá
hợp lý vì sẽ tạo một thử thách nhỏ cho thí sinh trước khi bước vào hai thử thách lớn hơn. Về
bốn bài cịn lại, kiến thức được sử dụng nói chung rất quen thuộc, địi hỏi tư duy thuần túy
và trình bày cẩn thận chứ khơng dùng các định lý khó. Bài<b>1</b>là phương trình hàm trên tập
số tự nhiên được tính vào đại số, nếu nắm vững cách xử lý phương trình hàm thì sẽ vượt
qua bài này khơng mấy khó khăn. Bài<b>5</b>về đa thức, cũng là đại số nhưng có liên quan đến
các tính chất số học, giải tích. Bài<b>2</b>và<b>6</b>đều là các bài tổ hợp nhưng được đặt trên nền số
học, xu hướng thường thấy trong các đề<b>VMO, TST</b>những năm gần đây.


Ta có thể thấy một điểm khá trùng hợp ở ba bài<b>2, 5</b> và<b>6</b> là mỗi bài đều có khai thác yếu
tố<b>chiều</b>và vì vậy mà bài tốn có thể hình dung dễ hơn khi đưa về trường hợp<b>1</b>chiều. Ba
bài này cũng có nhiều điểm khó, cả về cách tiếp cận, lập luận chặt chẽ trong trình bày lời
giải lẫn đáp số cuối cùng của bài toán. So với đề thi các năm trước thì có thể thấy đề<b>TST</b>
<b>2014</b>có phần nhẹ hơn, khơng có bài q khó và hai bài dễ của kỳ thi cũng dễ hơn. Tuy vậy,
đề thi vẫn đảm bảo tính phân loại cao, các bài hình đòi hỏi nắm các kỹ thuật nhất định để
xử lý cịn các bài khác thì lại địi hỏi tư duy là chính, hạn chế sử dụng các định lý, cơng cụ


mạnh là một đặc điểm nổi bật và cũng là ưu điểm của đề thi này.


</div>
<span class='text_page_counter'>(88)</span><div class='page_container' data-page=88>

Việt Nam Team Selection Test 2014 5


<b>Lời giải chi tiết và bình luận</b>



<b>Bài 1.</b>


Tìm tất cả các hàm <i><b>f</b></i> <b>:Z→Z</b>thỏa mãn


<i><b>f</b></i> <b>2m+</b> <i><b>f</b></i><b>(</b><i><b>m</b></i><b>) +</b> <i><b>f</b></i><b>(</b><i><b>m</b></i><b>)</b><i><b>f</b></i><b>(</b><i><b>n</b></i><b>)</b>


<b>=</b><i><b>n f</b></i><b>(</b><i><b>m</b></i><b>) +</b><i><b>m,</b></i> <b>∀</b><i><b>m</b></i><b>,</b><i><b>n</b></i><b>∈Z.(∗)</b>


Lời giải. Đặt <i><b>a</b></i><b>=</b> <i><b>f</b></i><b>(0).</b>Rõ ràng <i><b>f</b></i> <b>≡0</b>khơng phải là nghiệm của phương trình, do đó tồn
tại <i><b>m</b></i><b>0</b> <b>∈</b> <b>Z</b>sao cho <i><b>f</b></i><b>(</b><i><b>m</b></i><b>0)</b> <b>=6</b> <b>0.</b> Từ đó, bằng cách thay <i><b>m</b></i> <b>=</b> <i><b>m</b></i><b>0</b> vào đẳng thức<b>(∗)</b> ta dễ


dàng suy ra <i><b>f</b></i> đơn ánh. Mặt khác, thay<i><b>n</b></i><b>=0</b>vào<b>(∗),</b>ta được
<i><b>f</b></i> <b>2m+ (</b><i><b>a</b></i><b>+1)·</b> <i><b>f</b></i><b>(</b><i><b>m</b></i><b>)</b>


<b>=</b><i><b>m,</b></i> <b>∀</b><i><b>m</b></i><b>∈Z.(∗∗)</b>


Kết quả này chứng tỏ <i><b>f</b></i> là tồn ánh. Do đó,<b>∃</b><i><b>b</b></i><b>∈Z</b>sao cho <i><b>f</b></i><b>(</b><i><b>b</b></i><b>) =−1.</b>Thay <i><b>m</b></i> <b>=</b><i><b>n</b></i> <b>=</b> <i><b>b</b></i>


vào<b>(1),</b>ta được <i><b>f</b></i><b>(2b) =0.</b>Mặt khác, thay<i><b>m</b></i><b>=</b><i><b>n</b></i><b>=0</b>vào<b>(<sub>∗</sub>),</b>ta cũng có <i><b>f</b></i><b>(</b><i><b>a</b></i><b>2<sub>+</sub></b><i><b><sub>a</sub></b></i><b><sub>) =</sub><sub>0.</sub></b>


Từ đây, kết hợp với tính đơn ánh của <i><b>f,</b></i>ta suy ra
<i><b>b</b></i><b>=</b> <i><b>a</b></i>


<b>2<sub>+</sub></b><i><b><sub>a</sub></b></i>



<b>2</b> <b>.</b>
Bây giờ, thay<i><b>n</b></i> <b>=</b><i><b>b</b></i>vào<b>(<sub>∗</sub>),</b>ta được


<i><b>f</b></i><b>(2m) =</b> <i><b>a</b></i>


<b>2<sub>+</sub></b><i><b><sub>a</sub></b></i>


<b>2</b> <b>·</b><i><b>f</b></i><b>(</b><i><b>m</b></i><b>) +</b><i><b>m,</b></i> <b>∀</b><i><b>m</b></i> <b>∈Z.(∗ ∗ ∗)</b>
Tiếp tục, thay<i><b>m</b></i><b>=0</b>vào<b>(∗),</b>ta được


<i><b>f a</b></i><b>·</b><i><b>f</b></i><b>(</b><i><b>n</b></i><b>) +</b><i><b>a</b></i><b></b>


<b>=</b><i><b>an,</b></i> <b>∀</b><i><b>n</b></i><b>∈Z.</b>


Đến đây, bằng cách thay<i><b>m</b></i><b>=</b><i><b>an</b></i> vào<b>(∗∗)</b>rồi so sánh với đẳng thức trên, ta suy ra


<b>(</b><i><b>a</b></i><b>+1)·</b> <i><b>f</b></i><b>(</b><i><b>an</b></i><b>) +2an=</b><i><b>a</b></i><b>·</b> <i><b>f</b></i><b>(</b><i><b>n</b></i><b>) +</b><i><b>a,</b></i> <b>∀</b><i><b>n</b></i><b>∈Z.</b>


Cịn nếu thay<i><b>n</b></i><b>=</b><i><b>b,</b></i>ta được <i><b>a</b></i><b>(</b><i><b>a</b></i>


<b>2<sub>+</sub></b><i><b><sub>a</sub></b></i><b><sub>)</sub></b>


<b>2</b> <b>=</b> <i><b>f</b></i><b>(0) =</b><i><b>a,</b></i>từ đó suy ra<i><b>a</b></i><b>∈ {0, 1,−2}.</b>Ta xét các
trường hợp sau:


• Nếu <i><b>a</b></i><b>=1</b> thì dễ dàng suy ra <i><b>f</b></i><b>(</b><i><b>n</b></i><b>) =1−2n,∀</b><i><b>n</b></i> <b>∈Z.</b> Tuy nhiên, khi thử lại ta thấy


hàm này không thỏa mãn yêu cầu đề bài.



• Nếu<i><b>a</b></i><b>=0</b>thì từ<b>(∗ ∗ ∗),</b>ta suy ra <i><b>f</b></i><b>(2m) =</b><i><b>m,</b></i><b>∀</b><i><b>m</b></i> <b>∈Z.</b>So sánh kết quả này với<b>(∗∗),</b>


ta được <i><b>f</b></i><b>(</b><i><b>m</b></i><b>) =0,∀</b><i><b>m</b></i> <b>∈Z,</b>vô lý vì <i><b>f</b></i> phải là một hàm khác hằng.


• Nếu<i><b>a</b></i><b>=−2</b>, ta có


</div>
<span class='text_page_counter'>(89)</span><div class='page_container' data-page=89>

Việt Nam Team Selection Test 2014 6


Mặt khác, theo<b>(∗ ∗ ∗),</b>ta lại có <i><b>f</b></i><b>(−2n) =</b> <i><b>f</b></i><b>(−</b><i><b>n</b></i><b>)−</b><i><b>n.</b></i>Do đó, ta được
<i><b>f</b></i><b>(−</b><i><b>n</b></i><b>) +3n=2·</b><i><b>f</b></i><b>(</b><i><b>n</b></i><b>) +2,</b> <b>∀</b><i><b>n</b></i> <b>∈Z.</b>


Thay <i><b>n</b></i> bởi<b>−</b><i><b>n</b></i> vào đẳng thức trên, ta cũng có <i><b>f</b></i><b>(</b><i><b>n</b></i><b>)−3n</b> <b>=2·</b> <i><b>f</b></i><b>(−</b><i><b>n</b></i><b>) +2,</b> <b>∀</b><i><b>n</b></i><b>∈Z.</b>


Kết hợp hai điều này lại, ta suy ra <i><b>f</b></i><b>(</b><i><b>n</b></i><b>) =</b><i><b>n</b></i> <b>−2,∀</b><i><b>n</b></i> <b>∈</b> <b>Z.</b> Thử lại, ta thấy hàm này


thỏa mãn yêu cầu đề bài.


Vậy hàm số cần tìm là <i><b>f</b></i><b>(</b><i><b>n</b></i><b>) =</b><i><b>n</b></i><b>−2,∀</b><i><b>n</b></i><b>∈Z.</b>


<b>Nhận xét.</b>


Ở bài tốn trên, tính đơn ánh – tồn ánh được nhận thấy khá dễ dàng và nó cịn đúng với
mọi số thực chứ khơng chỉ trên tập số nguyên. Đây cũng là điều tiên quyết cần có để việc
thay thế các giá trị có thể thực hiện dễ dàng.


Ngồi cách giải trên, ta cũng có thể dự đoán ra biểu thức của hàm số và bằng cách chọn
các số đặc biệt, ta có thể xử lý nhanh chóng được “nhu cầu” này. Việc phát hiện ra hàm số
<i><b>f</b></i><b>(</b><i><b>n</b></i><b>) =</b><i><b>n</b></i><b>−2</b>nói chung khơng khó vì nếu hàm số cần tìm là đa thức thì dễ thấy bậc của nó
là 1 và từ đó thay vào đồng nhất hệ số. Bằng cách thay các giá trị thích hợp, ta tính được



<i><b>f</b></i><b>(4) =2,</b><i><b>f</b></i><b>(3) =1,</b><i><b>f</b></i><b>(2) =0,</b> <i><b>f</b></i><b>(1) =−1,</b><i><b>f</b></i><b>(0) =−2.</b>
Thay<i><b>m</b></i><b>=3</b>vào công thức đã cho, ta có ngay


<i><b>f</b></i> <b>7+</b><i><b>f</b></i><b>(</b><i><b>n</b></i><b>)</b>


<b>=</b><i><b>n</b></i><b>+3</b>với mọi<i><b>n.</b></i>


Từ đây có thể quy nạp để có kết quả trên. Một cách khác cũng khá nhanh như sau:


Đặt <i><b>a</b></i> <b>=</b> <i><b>f</b></i><b>(0)</b> và <i><b>u</b></i><b>,</b><i><b>v</b></i><b>,</b><i><b>w</b></i> là các số nguyên thỏa <i><b>f</b></i><b>(</b><i><b>u</b></i><b>) =</b> <b>0,</b><i><b>f</b></i><b>(</b><i><b>v</b></i><b>) =</b> <b>−1,</b> <i><b>f</b></i><b>(</b><i><b>w</b></i><b>) =</b> <b>1</b>. Ta thực
hiện thay các giá trị thích vào biểu thức của hàm số đã cho như sau: Nếu<i><b>m</b></i><b>=</b><i><b>w,</b><b>n</b></i><b>=</b><i><b>u</b></i><b>−</b><i><b>w</b></i>
thì <i><b>f</b></i> <b>2w</b> <b>+1+</b><i><b>f</b></i><b>(</b><i><b>u</b></i><b>−</b><i><b>w</b></i><b>)</b>


<b>=</b><i><b>u</b></i>.


Nếu<i><b>m</b></i> <b>=</b><i><b>u,</b><b>n</b></i><b>=0</b>thì <i><b>f</b></i><b>(2u) =</b><i><b>u</b></i>. Suy ra


<b>2w</b> <b>+1+</b> <i><b>f</b></i><b>(</b><i><b>u</b></i><b>−</b><i><b>w</b></i><b>) =2u⇒</b> <i><b>f</b></i><b>(</b><i><b>u</b></i><b>−</b><i><b>w</b></i><b>) =2(</b><i><b>u</b></i><b>−</b><i><b>w</b></i><b>)−1.</b>


Tiếp tục thay<i><b>m</b></i><b>=</b><i><b>x</b></i> <b>∈Z,</b><i><b>n</b></i><b>=</b><i><b>u</b></i><b>−</b><i><b>w</b></i> thì <i><b>f</b></i> <b>2(</b><i><b>x</b></i><b>+ (</b><i><b>u</b></i><b>−</b><i><b>w</b></i><b>)</b><i><b>f</b></i><b>(</b><i><b>x</b></i><b>))=</b><i><b>x</b></i><b>+ (</b><i><b>u</b></i><b>−</b><i><b>w</b></i><b>)</b><i><b>f</b></i><b>(</b><i><b>x</b></i><b>)</b>.


Cuối cùng, thay<i><b>m</b></i><b>=</b><i><b>x</b></i><b>+ (</b><i><b>u</b></i><b>−</b><i><b>w</b></i><b>)</b><i><b>f</b></i><b>(</b><i><b>x</b></i><b>),</b><i><b>n</b></i> <b>=</b><i><b>v</b></i> thì


<i><b>x</b></i><b>+ (</b><i><b>u</b></i><b>−</b><i><b>w</b></i><b>)</b><i><b>f</b></i><b>(</b><i><b>x</b></i><b>) =</b><i><b>u</b></i> hay <i><b>f</b></i><b>(</b><i><b>x</b></i><b>) =</b> <i><b>u</b></i><b>−</b><i><b>x</b></i>
<i><b>u</b></i><b>−</b><i><b>w</b></i><b>.</b>


Đây là biểu thức tuyến tính theo<i><b>x</b></i>, ta dễ dàng chọn được các giá trị thích hợp.


Ngồi ra, ta cịn có thể tiếp cận theo cách sau: Ta chứng minh được rằng <i><b>f</b></i><b>(2m−</b><i><b>f</b></i><b>(</b><i><b>m</b></i><b>)) =</b><i><b>m</b></i>
và do <i><b>f</b></i> song ánh nên <i><b>f</b></i><b>(</b><i><b>m</b></i><b>)+</b><i><b>f</b></i><b>−1<sub>(</sub></b><i><b><sub>m</sub></b></i><b><sub>) =</sub><sub>2m</sub></b><sub>. Đây là bài xuất hiện trong kỳ thi</sub><b><sub>APMO 1989</sub></b><sub>:</sub>



Xác định tất cả các hàm số <i><b>f</b></i> <b>:<sub>R</sub>→R</b>thỏa mãn


• <i><b>f</b></i> đơn điệu tăng.


</div>
<span class='text_page_counter'>(90)</span><div class='page_container' data-page=90>

Việt Nam Team Selection Test 2014 7


<b>Bài 2.</b>


Trong mặt phẳng tọa độ vng góc <i><b>O x y,</b></i> xét các điểm nguyên có tọa độ thuộc tập
hợp sau


<i><b>T</b></i> <b>=</b>


<b>(</b><i><b>x</b></i><b>;</b> <i><b>y</b></i><b>):−20¶</b><i><b>x</b></i><b>,</b> <i><b>y</b></i><b>¶20,(</b><i><b>x</b></i><b>;</b> <i><b>y</b></i><b>)6= (0; 0)</b> <b>.</b>


Tơ màu các điểm thuộc <i><b>T</b></i> sao cho với mọi điểm có tọa độ<b>(</b><i><b>x</b></i><b>,</b><i><b>y</b></i><b>)∈</b><i><b>T</b></i> thì có đúng một
trong hai điểm<b>(</b><i><b>x</b></i><b>;</b><i><b>y</b></i><b>)</b>và<b>(−</b><i><b>x</b></i><b>;−</b><i><b>y</b></i><b>)</b>được tơ màu. Với mỗi cách tô như thế, gọi<i><b>N</b></i> là số
các bộ<b>(</b><i><b>x</b></i><b>1;</b><i><b>y</b></i><b>1),(</b><i><b>x</b></i><b>2;</b><i><b>y</b></i><b>2)</b>mà cả hai điểm này cùng được tô màu và


<i><b>x</b></i><b>1≡2x2,</b><i><b>y</b></i><b>1≡2</b><i><b>y</b></i><b>2(mod 41).</b>


Tìm tất cả các giá trị có thể có của<i><b>N</b></i><b>.</b>


Lời giải. Trước hết, ta chuyển bài tốn từ<b>2</b>chiều thành<b>1</b>chiều.


Ta có<b>210</b> <b><sub>≡ −</sub><sub>1</sub><sub>(</sub><sub>mod 41</sub><sub>)</sub></b><sub>nên</sub><b><sub>40</sub></b><sub>số nguyên khác</sub><b><sub>0</sub></b> <sub>có giá trị tuyệt đối khơng vượt q</sub><b><sub>20</sub></b>


có thể chia thành<b>2</b>dãy, mỗi dãy có độ dài<b>20</b>sao cho nếu số hạng đầu chia<b>41</b>dư <i><b>x</b></i> thì số
hạng sau chia<b>41</b>dư<b>2x.</b>



Tô màu các số của dãy này sao cho trong<b>2</b>số đối nhau thì có đúng<b>1</b> số được tơ màu. Ta
quan tâm đến số lượng các cặp số liên tiếp cùng được tô màu trong dãy.


Dễ thấy <b>220</b> <b><sub>≡</sub></b> <b><sub>1</sub><sub>(</sub><sub>mod 41</sub><sub>)</sub></b> <sub>nên nếu thêm số hạng đầu của mỗi dãy vào cuối thì dãy mới</sub>


gồm 21 số vẫn thỏa mãn tính chất trên nên ta có thể chuyển thành vịng trịn và phát biểu
lại bài tốn như sau: Cho đa giác đều<b>20</b>đỉnh nội tiếp trong một đường tròn sao cho trong
<b>2</b>điểm đối xứng qua tâm thì có đúng <b>1</b> đỉnh được tơ màu. Tính số các cặp đỉnh liên tiếp
được tơ màu có thể có.


Với<i><b>n</b></i> là số chẵn, gọi<i><b>S</b><b>n</b></i> là tập hợp số các cặp kề nhau cùng được tô màu có thể có của đa
giác có<i><b>n</b></i> đỉnh. Ta sẽ chứng minh bằng quy nạp rằng


<i><b>S</b></i><b>4</b><i><b>n</b></i> <b>={2k+1|0¶</b><i><b>k</b></i><b>¶</b><i><b>n</b></i><b>−1}</b>và<i><b>S</b></i><b>4</b><i><b>n</b></i><b>−2={2k|0¶</b><i><b>k</b></i><b>¶</b><i><b>n</b></i><b>−1}.</b>


Điều này có thể chứng minh bằng quy nạp như sau:
Với<i><b>n</b></i> <b>=2</b>thì dễ thấy nhận xét đúng.


Giả sử nhận xét đúng đến<i><b>n</b></i> <b><sub>¾</sub>2.</b>Xét đa giác có<b>4n+2</b>đỉnh. Đa giác này có thể tạo thành
bằng cách thêm đỉnh<i><b>A</b></i>vào giữa hai đỉnh thứ<b>2n, 2n+1</b>và thêm đỉnh<i><b>B</b></i>vào giữa hai đỉnh
thứ<b>4n, 1</b>. Ta xét các trường hợp:


• Nếu đỉnh<b>2n, 2n+1</b>đều được tơ và<i><b>A</b></i>khơng được tơ thì tương ứng: đỉnh<b>4n, 1</b>không
được tô và<i><b>B</b></i>được tô. Số cặp kề nhau cùng được tơ giảm đi 1.


• Nếu đỉnh <b>2n, 2n+1</b>đều được tơ và<i><b>A</b></i>cũng được tơ thì tương ứng: đỉnh<b>4n, 1</b>khơng
được tơ và<i><b>B</b></i>khơng được tơ. Số cặp kề nhau tăng lên<b>1.</b>


• Nếu trong hai đỉnh <b>2n, 2n+1</b> có <b>1</b> đỉnh được tơ và<i><b>A</b></i> cũng được tơ thì tương ứng:


trong hai đỉnh<b>4n, 1</b>có<b>1</b>đỉnh được tơ và<i><b>B</b></i>khơng được tơ. Số cặp kề nhau tăng lên<b>1.</b>
• Nếu trong hai đỉnh <b>2n, 2n+1</b>, có<b>1</b>đỉnh được tơ và<i><b>A</b></i>khơng được tơ thì tương ứng:


</div>
<span class='text_page_counter'>(91)</span><div class='page_container' data-page=91>

Việt Nam Team Selection Test 2014 8


Do đó,<i><b>S</b></i><b>4</b><i><b>n</b></i><b>+2={</b><i><b>x</b></i><b>±1|</b><i><b>x</b></i> <b>∈</b><i><b>S</b></i><b>4</b><i><b>n</b></i><b>}</b>hay<i><b>S</b></i><b>4</b><i><b>n</b></i><b>+2={2k|0¶</b><i><b>k</b></i><b>¶</b><i><b>n</b></i><b>}</b>.


Tương tự, ta cũng có<i><b>S</b></i><b>4</b><i><b>n</b></i><b>+4={</b><i><b>x</b></i><b>±1|</b><i><b>x</b></i> <b>∈</b><i><b>S</b></i><b>4</b><i><b>n</b></i><b>+2}</b>. Tất nhiên khơng xảy ra trường hợp <i><b>S</b></i><b>4</b><i><b>n</b></i><b>+4</b>


có chứa số<b>−1</b>vì để có trường hợp<b>0</b>cặp số ở<b>4n+2</b>, các đỉnh phải được tô xen kẽ và trường
hợp giảm đi số bộ khơng xảy ra, suy ra<i><b>S</b></i><b>4</b><i><b>n</b></i><b>+4={2k+1|0¶</b><i><b>k</b></i><b>¶</b><i><b>n</b></i><b>}.</b>


Rõ ràng trong cách chứng minh trên, ta cũng đã chỉ ra được cách xây dựng các trường hợp
để có thể tơ màu thỏa mãn được tất cả các giá trị trong tập hợp tương ứng. Nhận xét được
chứng minh.


Từ đó suy ra<i><b>S</b></i><b>20</b> <b>={1, 3, 5, 7, 9}.</b>


Do đó, kết quả cho bài tốn phụ sẽ là<b>2S20</b> với định nghĩa<b>2S=</b><i><b>S</b></i><b>+</b><i><b>S</b></i><b>={</b><i><b>a</b></i><b>+</b><i><b>b</b></i><b>|</b><i><b>a,</b><b>b</b></i><b>∈</b><i><b>S</b></i><b>}</b>.


Quay trở lại bài toán ban đầu, để chuyển từ <b>1</b> thành phần <i><b>x</b></i> ở trên thành <b>2</b> thành phần


<b>(</b><i><b>x</b></i><b>,</b><i><b>y</b></i><b>)</b>, ta có thể thực hiện như sau:


Ứng với mỗi vòng tròn chứa các số thuộc dãy<i><b>A,</b></i>ta lấy một vòng tròn mới cũng gồm các số
thuộc dãy <i><b>A</b></i> đặt lên đó sao cho mỗi số thuộc đường trịn cũ khớp với đúng một số thuộc
đường tròn mới. Viết các cặp số khớp nhau thành một dãy, dãy đó chính là dãy các tọa độ
điểm mà liền sau của<b>(</b><i><b>x</b></i><b>1,</b><i><b>y</b></i><b>1)</b>là<b>(2x1, 2y1)</b>theo <b>mod 41</b>.


Dễ thấy có tất cả <b>20</b> cách ghép như thế (cố định vòng tròn cũ và xoay vòng trịn mới).


Tương tự với việc ghép các dãy<i><b>A</b></i><b>−</b><i><b>B,</b><b>B</b></i><b>−</b><i><b>A,</b><b>B</b></i><b>−</b><i><b>B</b></i>nên có tổng cộng là<b>80</b> cách ghép tạo
thành<b>80</b>dãy. Tuy nhiên, ta cũng xét thêm<b>4</b>dãy đặc biệt, tương ứng với các điểm nằm trên
trục tung và trục hoành.


Cụ thể là xét thêm dãy<i><b>C</b></i> gồm<b>20</b>số<b>0</b> và xét<b>4</b>cách ghép: <i><b>A</b></i><b>−</b><i><b>C,</b><b>C</b></i><b>−</b><i><b>A,</b><b>B</b></i><b>−</b><i><b>C</b></i><b>,</b><i><b>C</b></i> <b>−</b><i><b>B.</b></i>Do
đó, tổng cộng có<b>84</b>dãy các tọa độ.


Theo chứng minh ở trên thì ở mỗi dãy, số các cặp có thể có là<i><b>S</b></i><b>20</b> vậy nên đáp số của bài


tốn là<b>84S20</b>, cũng chính là các số chẵn từ<b>1·84=84</b>đến<b>9·84=756</b>. Bài tốn được giải


quyết hồn tồn.
<b>Nhận xét.</b>


Để giải được bài<b>2,</b> ta phải “cảm nhận” được bài toán. Một cách tự nhiên khi lấy <i><b>x</b></i><b>,</b><i><b>y</b></i> thì
nghĩ đến việc tìm<b>2x, 2</b><i><b>y</b></i> (thay vì ngược lại là xét các điểm cùng được tơ màu). Từ đó, khi
biết các số có thể xoay thành một vịng trịn thì sẽ nhận thấy được miền giá trị kia chính là
số cạnh và việc đưa thành một mơ hình như đã nêu sẽ dễ trình bày nhất, mọi thứ đều sáng
sủa hơn nhiều. Sẽ có ích khi ta phát biểu bài tốn tổng qt sau đó làm việc với các tham số
nhỏ để tìm ra quy luật. Bài này bản chất khơng khó, nhưng rất dễ sai nhất là khi kết luận
vội vàng thông qua một số nhận xét nhỏ.


Theo ý kiến chúng tôi, việc chấm bài này khá là khó khăn trong việc đánh giá các bài giải
theo hướng<b>7−</b> hay<b>0+.</b> Điểm mấu chốt ở đây là việc tách riêng hồnh độ, tung độ, mơ
hình hóa dưới dạng đa giác đều và chú ý đến đồng dư thức<b>210</b> <b><sub>=</sub></b> <b><sub>1024</sub><sub>≡ −</sub><sub>1</sub><sub>(</sub><sub>mod 41</sub><sub>)</sub><sub>.</sub></b>


Bài tốn này có phong cách của bài<b>4</b>VMO<b>2012,</b>bài<b>3</b>VMO<b>2013,</b>đều là các bài khá mới
mẻ, là dạng tổ hợp đếm có kết hợp số học, tuy phát biểu hơi có phần gượng ép trong việc
đặt vào trục tọa độ nhưng lại giúp học sinh dễ tưởng tượng hơn.



</div>
<span class='text_page_counter'>(92)</span><div class='page_container' data-page=92>

Việt Nam Team Selection Test 2014 9


Trong không gian<i><b>O x y z</b></i>, xét các điểm nguyên có tọa độ thuộc tập hợp


<i><b>T</b></i> <b>=</b>


<b>(</b><i><b>x</b></i><b>;</b><i><b>y;</b><b>z</b></i><b>):−20¶</b><i><b>x</b></i><b>,</b><i><b>y,</b><b>z</b></i><b>¶20,(</b><i><b>x</b></i><b>;</b><i><b>y;</b><b>z</b></i><b>)6= (0; 0; 0)</b> <b>.</b>


Tơ màu các điểm thuộc <i><b>T</b></i> sao cho với mọi điểm có tọa độ <b>(</b><i><b>x</b></i><b>;</b><i><b>y;</b><b>z</b></i><b>)∈</b> <i><b>T</b></i> thì có đúng một
trong hai điểm<b>(</b><i><b>x</b></i><b>;</b><i><b>y;</b><b>z</b></i><b>)</b> và<b>(<sub>−</sub></b><i><b>x</b></i><b>;−</b><i><b>y;</b></i><b>−</b><i><b>z</b></i><b>)</b>được tô màu. Với mỗi cách tô như thế, gọi <i><b>N</b></i> là
số các bộ<b>(</b><i><b>x</b></i><b>1;</b><i><b>y</b></i><b>1;</b><i><b>z</b></i><b>1),(</b><i><b>x</b></i><b>2;</b><i><b>y</b></i><b>2;</b><i><b>z</b></i><b>2)</b> mà cả hai điểm này cùng được tô màu và các tọa độ của


chúng thỏa mãn điều kiện <i><b>x</b></i><b>1≡2x2,</b><i><b>y</b></i><b>1≡2</b><i><b>y</b></i><b>2,</b><i><b>z</b></i><b>1≡2z1(mod 41).</b>Tìm tất cả các giá trị có


thể có của<i><b>N</b></i><b>.</b>


Lập luận tương tự như trên nhưng ở đây, ta lại đặt thêm một vòng tròn nữa trên hai vịng
trịn cũ. Ở trên ta có ord<b>41(2) =20</b>nên nếu hợp thay <b>20</b>và<b>41</b>bởi các số khác. Ta thấy các


số<b>41, 20</b>quyết định độ dài chu kỳ của các dãy ở trên và trong trường hợp số lượng các số
chia hết cho chu kỳ này, nghĩa là ta có thể chia đều dãy số đã cho thành<i>các vịng trịn</i> thì
bài tốn được giải quyết tương tự. Như thế, ta đi đến bài toán tổng quát:


Với<i><b>k,</b><b>n,</b><b>p,</b><b>r</b></i> là số nguyên dương mà ord<i><b><sub>p</sub></b></i><b>(</b><i><b>r</b></i><b>)(2k)</b>, xét tập hợp <i><b>T</b></i> các bộ số nguyên không
đồng thời bằng 0 là<b>(</b><i><b>x</b></i><b>1,</b><i><b>x</b></i><b>2,· · ·,</b><i><b>x</b><b>n</b></i><b>)</b>sao cho <b>|</b><i><b>x</b><b>i</b></i><b>|¶</b> <i><b>k,</b><b>i</b></i> <b>=1,</b><i><b>n</b></i>. Tơ màu các số thuộc <i><b>T</b></i> sao
cho có đúng một trong hai bộ <b>(</b><i><b>x</b></i><b>1,</b><i><b>x</b></i><b>2,· · ·,</b><i><b>x</b><b>n</b></i><b>)</b> và <b>(−</b><i><b>x</b></i><b>1,−</b><i><b>x</b></i><b>2,· · ·,−</b><i><b>x</b><b>n</b></i><b>)</b> được tô màu. Với
mỗi cách tô như thế, gọi<i><b>N</b></i> là số các bộ<b>(</b><i><b>x</b></i><b>1,</b><i><b>x</b></i><b>2,· · ·,</b><i><b>x</b><b>n</b></i><b>)</b>và<b>(</b><i><b>x</b></i><b>01,</b><i><b>x</b></i><b>02,· · ·,</b><i><b>x</b></i><b>0</b><i><b>n</b></i><b>)</b>thỏa mãn:


• Cả hai bộ này cùng được tơ màu.



• <i><b>x</b><b>i</b></i> <b>=</b><i><b>r x</b></i><b>0</b><i><b>i</b></i><b>(mod</b> <i><b>p</b></i><b>).</b>với<i><b>i</b></i> <b>=1, 2, 3,· · ·,</b><i><b>n.</b></i>


Khi đó, tất cả các giá trị có thể có của<i><b>N</b></i> là<i><b>RS</b></i>ord<i><b>p</b></i><b>(</b><i><b>r</b></i><b>)</b> với<i><b>R</b></i><b>=</b>
<b>1</b>
<i><b>k</b></i>


<b>‚‚</b>
<b>2k2</b>


ord<i><b>p</b></i><b>(</b><i><b>r</b></i><b>)</b>


<b>+1</b>
<b>Œ</b><i><b>n</b></i>


<b>−1</b>
<b>Œ</b>



số<i><b>S</b></i><b>2</b><i><b>k</b></i> được định nghĩa như trên.


Tất nhiên, có những trường hợp dãy khơng lặp lại thì phải tính tốn theo cách khác vì rõ
ràng ta khơng thể xây dựng được thành một mơ hình khép kín như đa giác đều ở trên. Bài
tốn vẫn cịn có thể phát triển theo nhiều hướng rất thú vị!


<b>Bài 3.</b>


Cho tam giác <i><b>ABC</b></i> nội tiếp trong đường tròn <b>(</b><i><b>O</b></i><b>).</b>Trên cung <i><b>BC</b></i> không chứa <i><b>A</b></i>của


<b>(</b><i><b>O</b></i><b>)</b>lấy điểm <i><b>D</b></i>. Giả sử<i><b>C D</b></i> cắt <i><b>AB</b></i> ở <i><b>E</b></i> và <i><b>BD</b></i> cắt <i><b>AC</b></i> ở <i><b>F</b></i>. Gọi <b>(</b><i><b>K</b></i><b>)</b> đường tròn nằm
trong tam giác <i><b>E BD</b></i>, tiếp xúc với <i><b>E B,</b><b>E D</b></i> và tiếp xúc với đường tròn <b>(</b><i><b>O</b></i><b>).</b>Gọi<b>(</b><i><b>L</b></i><b>)</b>là


tâm đường tròn nằm trong tam giác <i><b>F C D</b></i>, tiếp xúc với<i><b>F C</b></i><b>,</b><i><b>F D</b></i>và tiếp xúc với đường
tròn<b>(</b><i><b>O</b></i><b>).</b>


a) Gọi <i><b>M</b></i> là tiếp điểm của <b>(</b><i><b>K</b></i><b>)</b> với <i><b>BE</b></i> và <i><b>N</b></i> là tiếp điểm của<b>(</b><i><b>L</b></i><b>)</b> với<i><b>C F</b></i>. Chứng
minh rằng đường tròn đường kính <i><b>M N</b></i> ln đi qua một điểm cố định khi<i><b>D</b></i> di
chuyển trên<b>(</b><i><b>O</b></i><b>).</b>


</div>
<span class='text_page_counter'>(93)</span><div class='page_container' data-page=93>

Việt Nam Team Selection Test 2014 10


Lời giải. a) Trước hết, ta chứng minh bổ đề sau:


<b>Bổ đề (định lý Sawayama và Thébault mở rộng).</b>Cho tam giác <i><b>ABC</b></i> nội tiếp đường tròn


<b>(</b><i><b>O</b></i><b>)</b>.<i><b>D</b></i> là một điểm thuộc tia đối tia<i><b>BA</b></i>. Đường tròn<b>(</b><i><b>K</b></i><b>)</b>tiếp xúc<i><b>DA,</b><b>DC</b></i> lần lượt tại<i><b>M</b></i><b>,</b><i><b>N</b></i>
và tiếp xúc ngoài<b>(</b><i><b>O</b></i><b>)</b>. Chứng minh rằng<i><b>M N</b></i> đi qua tâm bàng tiếp ứng với đỉnh<i><b>A</b></i>của tam
giác<i><b>ABC</b></i>.


<i><b>Chứng minh.</b></i> Giả sử đường phân giác ngoài tại đỉnh <i><b>C</b></i> của tam giác <i><b>ABC</b></i> cắt đường tròn


<b>(</b><i><b>O</b></i><b>)</b>tại <i><b>E</b></i> khác<i><b>C.</b></i>Đường thẳng<i><b>C E</b></i> cắt <i><b>M N</b></i> tại <i><b>J</b></i>. Đường tròn <b>(</b><i><b>K</b></i><b>)</b>tiếp xúc với đường tròn


<b>(</b><i><b>O</b></i><b>)</b> tại <i><b>F</b></i>. Dễ dàng thấy rằng <i><b>E,</b><b>N,</b><b>F</b></i> thẳng hàng. Thật vậy, vì <i><b>E</b></i> là trung điểm cung <i><b>AB</b></i>
chứa<i><b>C</b></i> của <b>(</b><i><b>O</b></i><b>)</b>nên <i><b>OE</b></i> <b>⊥</b><i><b>AB</b></i>. Ngồi ra, ta cũng có <i><b>M N</b></i> <b>⊥</b><i><b>AB</b></i> nên <i><b>M N</b></i> <b>k</b><i><b>OE</b></i>. Mặt khác,
<i><b>K</b></i><b>,</b><i><b>O,</b><b>F</b></i> thẳng hàng nên<i><b>E,</b><b>N,</b><b>F</b></i> cũng thẳng hàng.


Từ đây, ta có tam giác<i><b>E AF</b></i> và<i><b>E N A</b></i>đồng dạng, suy ra<i><b>E A</b></i><b>2<sub>=</sub></b><i><b><sub>E B</sub></b></i><b>2<sub>=</sub></b><i><b><sub>E N</sub></b></i><b><sub>·</sub></b><i><b><sub>E F</sub></b></i><sub>.</sub>


<i><b>O</b></i>


<i><b>B</b></i>



<i><b>C</b></i>
<i><b>A</b></i>


<i><b>D</b></i>


<i><b>J</b></i>
<i><b>N</b></i>


<i><b>K</b></i>


<i><b>M</b></i>
<i><b>F</b></i>


<i><b>E</b></i>


Ta lại có


<b>∠</b><i><b>F M N</b></i> <b>=</b>


<b>1</b>


<b>2∠</b><i><b>F K N</b></i> <b>=</b>
<b>1</b>


<b>2∠</b><i><b>F OE</b></i> <b>=∠</b><i><b>F AE</b></i> <b>=∠</b><i><b>F C J</b></i>
suy ra tứ giác<i><b>C F M J</b></i> nội tiếp. Do đó,


<b>∠</b><i><b>E F J</b></i> <b>=180◦−∠</b><i><b>N F J</b></i><b>=180◦−(∠</b><i><b>N F M</b></i><b>+∠</b><i><b>M F J</b></i><b>) =180◦−(∠</b><i><b>J MC</b></i><b>+∠</b><i><b>MC J</b></i><b>) =∠</b><i><b>M JC</b></i><b>.</b>



Từ đó, ta có<b><sub>4</sub></b><i><b>E F J</b></i> <b>∼ 4</b><i><b>E J N</b></i><b>,</b>suy ra <i><b>E J</b></i><b>2<sub>=</sub></b><i><b><sub>E N</sub></b></i><b><sub>·</sub></b><i><b><sub>E F</sub></b></i> <b><sub>=</sub></b><i><b><sub>E A</sub></b></i><b>2<sub>=</sub></b><i><b><sub>E B</sub></b></i><b>2<sub>.</sub></b>


</div>
<span class='text_page_counter'>(94)</span><div class='page_container' data-page=94>

Việt Nam Team Selection Test 2014 11


Trở lại bài tốn,


Giả sử đường trịn<b>(</b><i><b>K</b></i><b>)</b>tiếp xúc<i><b>E D</b></i> tại<i><b>G</b></i> và đường tròn<b>(</b><i><b>L</b></i><b>)</b>tiếp xúc<i><b>F D</b></i> tại<i><b>H</b></i>. Theo bổ đề
ở trên thì<i><b>MG,</b><b>N H</b></i> cùng đi qua tâm bàng tiếp <i><b>J</b></i> ứng với đỉnh <i><b>A</b></i>của tam giác <i><b>ABC.</b></i>


Ta sẽ chứng minh rằng<b><sub>∠</sub></b><i><b>M J N</b></i><b>=90◦</b><sub>. Thật vậy,</sub>


<b>∠</b><i><b>K E F</b></i><b>+∠</b><i><b>LF E</b></i> <b>=</b>


<b>1</b>


<b>2∠</b><i><b>DE B</b></i><b>+</b>
<b>1</b>


<b>2∠</b><i><b>D F C</b></i><b>+∠</b><i><b>DE F</b></i><b>+∠</b><i><b>D F E</b></i> <b>=90</b>


<b>◦<sub>−∠</sub></b><i><b><sub>BAC</sub></b></i><b><sub>+</sub><sub>180</sub>◦<sub>−∠</sub></b><i><b><sub>BDC</sub></b></i> <b><sub>=</sub><sub>90</sub>◦</b>


do đó <i><b>E K</b></i> <b>⊥</b> <i><b>LF</b></i>. Ta cũng có <i><b>MG</b></i> <b>⊥</b> <i><b>E K</b></i> và <i><b>LF</b></i> <b>⊥</b> <i><b>N H</b></i> nên <i><b>MG</b></i> vng góc <i><b>N H</b></i> tại <i><b>J</b></i>. Vậy
đường trịn đường kính<i><b>M N</b></i> ln đi qua điểm <i><b>J</b></i> cố định. Ta có đpcm.


<i><b>O</b></i>


<i><b>B</b></i> <i><b>C</b></i>


<i><b>D</b></i>


<i><b>A</b></i>


<i><b>E</b></i>


<i><b>F</b></i>
<i><b>J</b></i>


<i><b>M</b></i> <i><b>K</b></i> <i><b>N</b></i>


<i><b>L</b></i>
<i><b>G</b></i>


<i><b>H</b></i>
<i><b>P</b></i>
<i><b>Q</b></i>


<i><b>Y</b></i>


<i><b>Z</b></i>


<i><b>R</b></i>


b) Dễ thấy tam giác <i><b>E MG</b></i> cân và <i><b>M P</b></i> <b>k</b><i><b>C E</b></i> nên<b><sub>∠</sub></b><i><b>E MG</b></i> <b>=∠</b><i><b>EG M</b></i> <b>=∠</b><i><b>G M P</b></i>. Do đó, <i><b>M J</b></i>


chính là phân giác ngồi tam giác<i><b>AM P</b></i> mà<i><b>AJ</b></i> là phân giác góc<i><b>BAC</b></i> nên suy ra <i><b>J</b></i> cũng là
tâm bàng tiếp góc<i><b>A</b></i>của tam giác<i><b>AM P</b></i><b>.</b>


</div>
<span class='text_page_counter'>(95)</span><div class='page_container' data-page=95>

Việt Nam Team Selection Test 2014 12


Chứng minh tương tự, ta cũng có <b>(</b><i><b>R</b></i><b>)</b> cũng tiếp xúc với đường trịn ngoại tiếp tam giác


<i><b>ANQ</b></i>. Vậy đường trịn ngoại tiếp tam giác<i><b>AM P</b></i><b>,</b><i><b>ANQ</b></i>ln tiếp xúc đường trịn<b>(</b><i><b>R</b></i><b>)</b>cố định.
Ta có đpcm.


<b>Nhận xét.</b>Với bài tốn này, nếu ai đã quen thuộc với định lý <i><b>Sawayama và Thébault</b></i> và
các mở rộng của nó thì giải quyết khơng q khó khăn. Nhưng nói chung thì với hầu hết
các thí sinh, một mặt phải xác định được mơ hình nêu ra trong đề bài chính là dạng mở
rộng; mặt khác, các bạn phải chứng minh lại thành công định lý này trong thời gian tương
đối ngắn, bài toán này xứng đáng ở vị trí số<b>3.</b> Đây là lần đầu tiên trong đề thi chọn đội
tuyển của Việt Nam có một bài hình ở vị trí này (một luật bất thành văn là bài<b>3</b>chính là
bài khó nhất đề thi).


Trước hết, ta nhắc lại định lý<i><b>Sawayama và Thébault</b></i>dạng gốc: Cho tam giác<i><b>ABC</b></i> nội tiếp
đường tròn<b>(</b><i><b>O</b></i><b>).</b> <i><b>D</b></i> là một điểm thuộc đoạn<i><b>BC</b></i><b>.</b>Đường tròn<b>(</b><i><b>K</b></i><b>)</b>tiếp xúc<i><b>DA,</b><b>DC</b></i> lần lượt
tại <i><b>M,</b><b>N</b></i> và tiếp xúc trong với<b>(</b><i><b>O</b></i><b>).</b>Chứng minh <i><b>M N</b></i>đi qua tâm nội tiếp tam giác<i><b>ABC</b></i><b>.</b>
Trước hết, ta thử tập trung vào khai thác ý a. Rõ ràng có thể thay đường trịn ngoại tiếp
thành đường tròn bất kỳ qua<i><b>B,</b><b>C,</b></i> ta đã thu được một bài tốn rất thú vị. Chú ý rằng câu
a) nói lên <i><b>MG,</b><b>N H</b></i> đi qua điểm <i><b>J</b></i>cố định. Ta có các bài toán sau:


<i><b>Bài 3.1.</b></i>Cho tam giác <i><b>ABC</b></i> và một đường tròn <b>(</b><i><b>O</b></i><b>)</b>bất kỳ cố định đi qua <i><b>B,</b><b>C</b></i><b>.</b> <i><b>D</b></i> là điểm
di chuyển trên<b>(</b><i><b>O</b></i><b>)</b> sao cho<i><b>A,</b><b>D</b></i> khác phía với <i><b>B,</b><b>C.</b></i>Giả sử<i><b>C D</b></i> cắt <i><b>AB</b></i>ở <i><b>E</b></i> và <i><b>BD</b></i>cắt <i><b>AC</b></i> ở
<i><b>F.</b></i>Gọi <b>(</b><i><b>K</b></i><b>)</b>là đường tròn tiếp xúc với <i><b>E B,</b><b>E D</b></i> lần lượt tại <i><b>M</b></i><b>,</b><i><b>N</b></i>và tiếp xúc trong với<b>(</b><i><b>O</b></i><b>).</b>
Gọi<b>(</b><i><b>L</b></i><b>)</b>là đường tròn tiếp xúc với<i><b>F C</b></i><b>,</b><i><b>F D</b></i>lần lượt tại<i><b>P</b></i><b>,</b><i><b>Q</b></i>và tiếp xúc trong với<b>(</b><i><b>O</b></i><b>).</b>Chứng
minh rằng giao điểm<i><b>M N,</b><b>PQ</b></i>ln nằm trên một đường trịn cố định khi<i><b>D</b></i> di chuyển.
<i><b>Bài 3.2.</b></i>Cho tam giác <i><b>ABC</b></i> nội tiếp đường tròn <b>(</b><i><b>O</b></i><b>).</b> Gọi <i><b>P</b></i> là điểm di chuyển trên cung
<i><b>BC</b></i> chứa<i><b>A</b></i>của<b>(</b><i><b>O</b></i><b>).</b><i><b>P B,</b><b>PC</b></i> cắt<i><b>C A,</b><b>AB</b></i>lần lượt tại <i><b>E,</b><b>F.</b></i>Đường tròn<b>(</b><i><b>K</b></i><b>)</b>tiếp xúc với đoạn
<i><b>E A,</b><b>E B</b></i>và tiếp xúc trong với<b>(</b><i><b>O</b></i><b>).</b>Đường tròn<b>(</b><i><b>L</b></i><b>)</b>tiếp xúc với đoạn<i><b>F B,</b><b>F C</b></i> và tiếp xúc với
cung<i><b>BC</b></i> không chứa<i><b>A</b></i>của<b>(</b><i><b>O</b></i><b>).</b>Chứng minh rằng đường trịn đường kính <i><b>M N</b></i>ln đi qua
một điểm cố định khi<i><b>P</b></i> di chuyển.


<i><b>Bài 3.3.</b></i>Cho tam giác<i><b>ABC</b></i> và<b>(</b><i><b>O</b></i><b>)</b>cố định đi qua<i><b>B,</b><b>C</b></i><b>.</b>Gọi<i><b>D</b></i> là điểm di chuyển trên cung


<i><b>BC</b></i> của <b>(</b><i><b>O</b></i><b>)</b> sao cho <i><b>D,</b><b>A</b></i> cùng phía với <i><b>BC</b></i><b>.</b> <i><b>DB,</b><b>DC</b></i> cắt <i><b>C A,</b><b>AB</b></i> lần lượt tại <i><b>E,</b><b>F.</b></i> Đường
tròn<b>(</b><i><b>K</b></i><b>)</b>tiếp xúc với đoạn<i><b>E A,</b><b>E B</b></i>tại <i><b>M</b></i><b>,</b><i><b>N</b></i> và tiếp xúc trong với<b>(</b><i><b>O</b></i><b>).</b>Đường tròn <b>(</b><i><b>L</b></i><b>)</b>tiếp
xúc với đoạn <i><b>F B,</b><b>F C</b></i> tại <i><b>P</b></i><b>,</b><i><b>Q</b></i>và tiếp xúc <b>(</b><i><b>O</b></i><b>)</b> tại một điểm khơng cùng phía <i><b>A</b></i> so với <i><b>BC</b></i><b>.</b>
Chứng minh rằng giao điểm của<i><b>M N</b></i> và <i><b>PQ</b></i>ln thuộc một đường trịn cố định.


Lời giải chi tiết của các bài toán trên có thể tham khảo trong<b>[2]</b>.


Ý thứ hai của bài tốn là một xây dựng thú vị khi đòi hỏi phải xây dựng thêm đường trịn
<i><b>Mixtilinear</b></i>bàng tiếp góc<i><b>A</b></i>của tam giác<i><b>ABC</b></i>. Tính chất sử dụng trong lời giải trên đã khá
quen thuộc và do đã chứng minh được định lý<i><b>Sawayama và Thébault</b></i>nên khơng cần thực
hiện lại chứng minh tính chất này. Một bài tốn có liên quan đã xuất hiện cách đây khá lâu
trong đề chọn đội tuyển năm<b>1999</b>của Việt Nam:


<i><b>Bài 5 VN TST 1999.</b></i>Cho tam giác <i><b>A</b></i><b>1</b><i><b>A</b></i><b>2</b><i><b>A</b></i><b>3</b>nội tiếp trong đường tròn <b>(</b><i><b>O</b></i><b>).</b>Một đường trịn


<b>(</b><i><b>K</b></i><b>1)</b> nằm trong góc <i><b>A</b></i><b>2</b><i><b>A</b></i><b>1</b><i><b>A</b></i><b>3</b> của tam giác <i><b>A</b></i><b>1</b><i><b>A</b></i><b>2</b><i><b>A</b></i><b>3</b>, tiếp xúc với các cạnh <i><b>A</b></i><b>1</b><i><b>A</b></i><b>2,</b><i><b>A</b></i><b>1</b><i><b>A</b></i><b>3</b> và


tiếp xúc trong với đường tròn<b>(</b><i><b>O</b></i><b>)</b>lần lượt tại các điểm <i><b>M</b></i><b>1,</b><i><b>N</b></i><b>1,</b><i><b>P</b></i><b>1</b>. Các điểm<i><b>M</b></i><b>2,</b><i><b>N</b></i><b>2,</b><i><b>P</b></i><b>2</b> và


<i><b>M</b></i><b>3,</b><i><b>N</b></i><b>3,</b><i><b>P</b></i><b>3</b>xác định một cách tương tự. Chứng minh rằng các đoạn thẳng<i><b>M</b></i><b>1</b><i><b>N</b></i><b>1,</b><i><b>M</b></i><b>2</b><i><b>N</b></i><b>2,</b><i><b>M</b></i><b>3</b><i><b>N</b></i><b>3</b>


</div>
<span class='text_page_counter'>(96)</span><div class='page_container' data-page=96>

Việt Nam Team Selection Test 2014 13


<b>Bài 4.</b>


Cho tam giác <i><b>ABC</b></i> nhọn, khơng cân có đường cao <i><b>AD</b></i> và <i><b>P</b></i> thuộc <i><b>AD</b></i>. Các đường
thẳng<i><b>P B,</b><b>PC</b></i> lần lượt cắt<i><b>C A,</b><b>AB</b></i> tại<i><b>E</b></i><b>,</b><i><b>F.</b></i>


a) Giả sử tứ giác<i><b>AE D F</b></i> nội tiếp. Chứng minh rằng <i><b>PA</b></i>



<i><b>P D</b></i> <b>= (tan</b><i><b>B</b></i><b>+tan</b><i><b>C</b></i><b>)cot</b>
<i><b>A</b></i>
<b>2.</b>
b) Gọi<i><b>C P</b></i> cắt đường thẳng qua<i><b>B</b></i> vng góc<i><b>AB</b></i> tại <i><b>M</b></i>.<i><b>BP</b></i> cắt đường thẳng qua


<i><b>C</b></i> vng góc <i><b>AC</b></i> tại <i><b>N</b></i>. <i><b>K</b></i> là hình chiếu của <i><b>A</b></i> lên <i><b>M N</b></i>. Chứng minh rằng


<b>∠</b><i><b>B KC</b></i><b>+∠</b><i><b>M AN</b></i> không đổi khi <i><b>P</b></i> di chuyển trên<i><b>AD.</b></i>


<i><b>Lời giải.</b></i> a) Gọi<i><b>Q</b></i>là giao điểm của <i><b>E F</b></i> và <i><b>BC</b></i> thì hàng điểm<b>(</b><i><b>Q,</b><b>D,</b><b>B,</b><b>C</b></i><b>)</b>điều hòa. Từ đây
suy ra chùm<b>(</b><i><b>AQ,</b><b>AD</b></i><b>,</b><i><b>AB,</b><b>AC</b></i><b>)</b>và<b>(</b><i><b>DE</b></i><b>,</b><i><b>D F,</b><b>DA,</b><b>DQ</b></i><b>)</b>cũng điều hòa.


Hơn nữa, do<i><b>DA</b></i> <b>⊥</b> <i><b>DQ</b></i> nên <i><b>DA</b></i> là phân giác trong của <b><sub>∠</sub></b><i><b>E D F</b></i>. Do tứ giác <i><b>AE D F</b></i> nội tiếp
đường tròn nên dễ thấy<i><b>AE</b></i> <b>=</b><i><b>AF</b></i>.


<i><b>A</b></i>


<i><b>B</b></i> <i><b>D</b></i> <i><b>C</b></i>


<i><b>P</b></i>
<i><b>F</b></i>


<i><b>E</b></i>


<i><b>G</b></i>
<i><b>Q</b></i>


Giả sử đường tròn ngoại tiếp tứ giác <i><b>AE D F</b></i> cắt <i><b>BC</b></i> tại <i><b>G</b></i> khác <i><b>D</b></i>. Ta có <i><b>DG</b></i> là phân giác
ngồi tại đỉnh<i><b>D</b></i> nên<i><b>GE</b></i> <b>=</b><i><b>G F</b></i> do đó <b>4</b><i><b>AGE</b></i> <b>=4</b><i><b>AG F</b></i> nên <i><b>AG</b></i> là phân giác <b><sub>∠</sub></b><i><b>BAC</b></i>. Theo
định lý<i><b>Menelaus</b></i> cho cát tuyến <i><b>BP E</b></i> của tam giác <i><b>ADC</b></i> và cát tuyến <i><b>C P F</b></i> của tam giác


<i><b>ABD</b></i> thì


<i><b>E A</b></i>


<i><b>EC</b></i> <b>=</b>


<i><b>PA</b></i>
<i><b>P D</b></i> <b>·</b>


<i><b>BD</b></i>
<i><b>BC</b></i><b>,</b>


<i><b>F A</b></i>
<i><b>F B</b></i> <b>=</b>


<i><b>PA</b></i>
<i><b>P D</b></i> <b>·</b>


<i><b>C D</b></i>
<i><b>BC</b></i>
Từ đây suy ra


<i><b>F A</b></i>
<i><b>F B</b></i> <b>+</b>


<i><b>E A</b></i>
<i><b>EC</b></i> <b>=</b>
<i><b>PA</b></i>
<i><b>P D</b></i>
<b></b><i><b><sub>BD</sub></b></i>


<i><b>BC</b></i> <b>+</b>
<i><b>C D</b></i>
<i><b>BC</b></i>
<b></b>
<b>=</b> <i><b>PA</b></i>


<i><b>P D</b></i><b>.</b>
Do đó, ta được


<i><b>PA</b></i>
<i><b>P D</b></i> <b>=</b>


<i><b>F A</b></i>
<i><b>G F</b></i><b>.</b>


<i><b>G F</b></i>
<i><b>F B</b></i> <b>+</b>


<i><b>E A</b></i>
<i><b>GE</b></i><b>.</b>


<i><b>GE</b></i>
<i><b>EC</b></i> <b>=cot</b>


<i><b>A</b></i>


<b>2tan</b><i><b>B</b></i><b>+cot</b>
<i><b>A</b></i>


</div>
<span class='text_page_counter'>(97)</span><div class='page_container' data-page=97>

Việt Nam Team Selection Test 2014 14



b) Gọi <i><b>L</b></i> là giao điểm của <i><b>BM</b></i> và <i><b>C N</b></i> thì rõ ràng điểm <i><b>L</b></i> cố định. Tứ giác <i><b>AC K N</b></i> có


<b>∠</b><i><b>AC N</b></i><b>=∠</b><i><b>AK N</b></i><b>=90◦</b>nên nội tiếp. Tương tự,<i><b>AB K M</b></i> cũng nội tiếp. Do đó, ta có
<b>∠</b><i><b>M AN</b></i><b>+∠</b><i><b>B KC</b></i> <b>=∠</b><i><b>M AK</b></i><b>+∠</b><i><b>N AK</b></i><b>+∠</b><i><b>B KC</b></i> <b>=∠</b><i><b>M B K</b></i><b>+∠</b><i><b>NC K</b></i><b>+∠</b><i><b>B KC</b></i> <b>=∠</b><i><b>BLC</b></i>


khơng đổi. Ta có đpcm.


<i><b>A</b></i>


<i><b>B</b></i>


<i><b>C</b></i>
<i><b>D</b></i>


<i><b>P</b></i>


<i><b>N</b></i>


<i><b>M</b></i>


<i><b>K</b></i>
<i><b>L</b></i>


<b>Nhận xét.</b>Đây là bài tốn ở mức độ dễ và thí sinh có thể xử lý một cách nhẹ nhàng với các
kiến thức quen thuộc về hình phẳng. Bài tốn mang màu sắc đại số khá nhiều và nói chung
địi hỏi thí sinh tính tốn, biến đổi (tương tự như bài 3 trong đề<b>IMO 201</b>3) hơn là tiếp cận
theo hướng hình học thuần túy. Hai câu a) và b) của bài tốn khơng liên quan đến nhau.
Ta sẽ tách riêng thành hai bài toán và phân tích từng bài tốn một. Câu a) phát biểu điều
kiện dưới dạng một biểu thức lượng giác như vậy chưa đẹp, ta hồn tồn có thể có một hệ


thức lượng thuần túy hình học của câu a). Ta xét bài toán sau:


<i><b>Bài toán 4.1.</b></i>Giả thiết tương tự như bài tốn. Chứng minh rằng nếu tứ giác<i><b>AE F D</b></i>nội tiếp
thì <i><b>AD</b></i>


<i><b>P D</b></i> <b>=</b>


<i><b>AB</b></i><b>·</b><i><b>AC</b></i><b>+</b><i><b>AD</b></i><b>2</b>


<i><b>DB.DC</b></i> <b>.</b>


Đây chính là dạng tổng quát của câu a của bài tốn gốc và có thể xử lý dễ dàng bằng định
lý<i><b>Van-Obel</b></i>: Cho tam giác <i><b>ABC</b></i> có <i><b>D</b></i> thuộc<i><b>BC</b></i> và <i><b>P</b></i> thuộc đoạn<i><b>AD</b></i>. Tia<i><b>BP</b></i> cắt <i><b>AC</b></i> tại <i><b>N</b></i>,
tia<i><b>C P</b></i> cắt <i><b>AB</b></i> tại <i><b>M</b></i>. Khi đó, ta có <i><b>AD</b></i>


<i><b>AP</b></i> <b>=</b>


<i><b>AM</b></i>


<i><b>AB</b></i> <b>+</b>


<i><b>AN</b></i>


</div>
<span class='text_page_counter'>(98)</span><div class='page_container' data-page=98>

Việt Nam Team Selection Test 2014 15


<i><b>Bài tốn 4.2.</b></i>Cho tam giác<i><b>ABC</b></i> có đường phân giác <i><b>AD.</b></i>Gọi<i><b>E</b></i><b>,</b><i><b>F</b></i> là hình chiếu của<i><b>D</b></i> lên
các cạnh<i><b>C A,</b><b>C B.</b></i>Giả sử<i><b>BE</b></i> giao<i><b>C F</b></i> tại <i><b>P</b></i><b>.</b>Gọi<i><b>H</b></i> là hình chiếu của<i><b>P</b></i> lên <i><b>BC</b></i><b>.</b>Chứng minh
rằng<i><b>H P</b></i> là phân giác<b><sub>∠</sub></b><i><b>E H F.</b></i>


Cuối cùng, ta các nội dung này, ta có thể phát biểu bài tốn tổng quát như sau:



<i><b>Bài toán 4.3.</b></i> Cho tam giác <i><b>ABC</b></i> với <i><b>P</b></i> là điểm bất kỳ trong tam giác <i><b>ABC.</b></i> Các đường
thẳng <i><b>PA,</b><b>P B,</b><b>PC</b></i> cắt <i><b>AB,</b><b>BC</b></i><b>,</b><i><b>C A</b></i>lần lượt tại <i><b>D,</b><b>E,</b><b>F.</b></i> Gọi <i><b>H</b></i><b>,</b><i><b>K</b></i> là hình chiếu của <i><b>F,</b><b>E</b></i> lên
<i><b>AD</b></i> và <i><b>L,</b><b>N</b></i> là hình chiếu của <i><b>F,</b><b>E</b></i> lên<i><b>BC</b></i><b>.</b>Giả sử tứ giác<i><b>AE D F</b></i> nội tiếp. Chứng minh rằng


<i><b>PA</b></i>
<i><b>P D</b></i> <b>=</b>


<i><b>F H</b></i>
<i><b>F L</b></i> <b>·</b>


<i><b>DA</b></i>


<i><b>DB</b></i> <b>+</b>


<i><b>E K</b></i>
<i><b>E N</b></i> <b>·</b>


<i><b>DA</b></i>
<i><b>DC</b></i><b>.</b>


Nói về phần b, ta thấy đây chỉ là một bài toán đẳng giác quen thuộc. Ta có thể thêm một số
ý vào bài tốn này để vấn đề phong phú hơn:


<i><b>Bài toán 4.4.</b></i>Cho tam giác<i><b>ABC</b></i> đường cao<i><b>AD.</b></i>Điểm<i><b>P</b></i> di chuyển trên<i><b>AD</b></i><b>.</b>Giả sử<i><b>P B,</b><b>PC</b></i>
lần lượt cắt các đường thẳng qua<i><b>C</b></i> vng góc với<i><b>C A</b></i>và qua<i><b>B</b></i>vng góc với<i><b>AB</b></i>tại <i><b>N</b></i><b>,</b><i><b>M</b></i><b>.</b>
Gọi <i><b>K</b></i> là hình chiếu của<i><b>A</b></i>lên <i><b>M N.</b></i>


a) Chứng minh rằng<b><sub>∠</sub></b><i><b>M AN</b></i><b>+∠</b><i><b>B KC</b></i> không đổi khi<i><b>P</b></i> di chuyển.



b) Chứng minh rằng<b><sub>∠</sub></b><i><b>M AC</b></i> <b>=∠</b><i><b>N AB.</b></i>


c) Chứng minh rằng <i><b>K A</b></i>là phân giác<b><sub>∠</sub></b><i><b>B KC</b></i><b>.</b>


Dưới đây là hai bài mở rộng theo con đường đẳng giác cho bài toán trên:


<i><b>Bài toán 4.5.</b></i>Cho tam giác <i><b>ABC</b></i> nội tiếp <b>(</b><i><b>O</b></i><b>).</b> Các điểm <i><b>E</b></i><b>,</b><i><b>F</b></i> cố định thuộc <b>(</b><i><b>O</b></i><b>)</b>sao cho
<i><b>E F</b></i> <b>k</b><i><b>BC</b></i><b>.</b>Điểm<i><b>P</b></i> di chuyển trên<i><b>AE</b></i><b>.</b>Giả sử <i><b>P B,</b><b>PC</b></i> lần lượt cắt <i><b>F C</b></i><b>,</b><i><b>F B</b></i> tại<i><b>N</b></i><b>,</b><i><b>M</b></i><b>.</b>và trung
trực<i><b>BC</b></i> cắt <i><b>M N</b></i> tại <i><b>K</b></i><b>.</b>Chứng minh rằng<b><sub>∠</sub></b><i><b>M AN</b></i><b>+∠</b><i><b>B KC</b></i> không đổi khi<i><b>P</b></i> di chuyển.


<i><b>Bài toán 4.6.</b></i>Cho tam giác<i><b>ABC</b></i> nội tiếp đường tròn<b>(</b><i><b>O</b></i><b>).</b><i><b>E,</b><b>F</b></i> cố định thuộc<b>(</b><i><b>O</b></i><b>)</b>sao cho
<i><b>E F</b></i> <b>k</b><i><b>BC</b></i><b>.</b>Các điểm<i><b>P</b></i><b>,</b><i><b>Q</b></i>lần lượt thuộc<i><b>AE</b></i><b>,</b><i><b>AF.</b></i>Giả sử<i><b>P B,</b><b>PC</b></i> lần lượt cắt<i><b>QB,</b><b>QC</b></i> tại <i><b>N</b></i><b>,</b><i><b>M</b></i><b>.</b>
Chứng minh rằng<b><sub>∠</sub></b><i><b>M AB</b></i><b>=∠</b><i><b>N AC</b></i><b>.</b>


Lời giải chi tiết của các bài tốn trên có thể tham khảo trong<b>[3]</b>.


<b>Bài 5.</b>


Tìm tất cả đa thức <i><b>P</b></i><b>(</b><i><b>x</b></i><b>),</b><i><b>Q</b></i><b>(</b><i><b>x</b></i><b>)</b> có hệ số nguyên và thỏa mãn điều kiện: Với dãy số


<b>(</b><i><b>x</b><b>n</b></i><b>)</b>xác định bởi


<i><b>x</b></i><b>0=2014,</b><i><b>x</b></i><b>2</b><i><b>n</b></i><b>+1=</b><i><b>P</b></i><b>(</b><i><b>x</b></i><b>2</b><i><b>n</b></i><b>),</b><i><b>x</b></i><b>2</b><i><b>n</b></i><b>=</b><i><b>Q</b></i><b>(</b><i><b>x</b></i><b>2</b><i><b>n</b></i><b>−1)</b>với<i><b>n</b></i><b>¾1.</b>


thì mỗi số ngun dương <i><b>m</b></i>là ước của một số hạng khác<b>0</b>nào đó của dãy<b>(</b><i><b>x</b><b>n</b></i><b>)</b>.


Lời giải. Ta sẽ chứng minh rằng nếu các đa thức <i><b>P</b></i><b>(</b><i><b>x</b></i><b>),</b><i><b>Q</b></i><b>(</b><i><b>x</b></i><b>)</b> thỏa mãn đề bài thì chúng
phải đều có bậc là<b>1.</b>Thật vậy,


Xét trường hợp một trong hai đa thức<i><b>P</b></i><b>(</b><i><b>x</b></i><b>),</b><i><b>Q</b></i><b>(</b><i><b>x</b></i><b>)</b>là đa thức hằng.<b>(1)</b>Nếu<i><b>P</b></i><b>(</b><i><b>x</b></i><b>)≡</b><i><b>a,</b></i><b>∀</b><i><b>x</b></i> <b>∈</b>


<b>Z</b>thì<b>(</b><i><b>x</b><b>n</b></i><b>)</b>có dạng:


</div>
<span class='text_page_counter'>(99)</span><div class='page_container' data-page=99>

Việt Nam Team Selection Test 2014 16


và dễ thấy mọi số hạng của dãy chỉ nhận<b>1</b>trong<b>3</b>giá trị<b>(2014,</b><i><b>a,</b><b>Q</b></i><b>(</b><i><b>a</b></i><b>))</b>.


<b>(2)</b>Nếu<i><b>Q</b></i><b>(</b><i><b>x</b></i><b>)≡</b><i><b>a,</b></i><b>∀</b><i><b>x</b></i> <b>∈Z</b>thì<b>(</b><i><b>x</b><b>n</b></i><b>)</b>có dạng


<i><b>x</b></i><b>0=2014,</b><i><b>x</b></i><b>1=</b><i><b>P</b></i><b>(2014),</b><i><b>x</b></i><b>2=</b><i><b>a,</b><b>x</b></i><b>3=</b><i><b>P</b></i><b>(</b><i><b>a</b></i><b>),· · ·</b>


và dễ thấy mọi số hạng của dãy chỉ nhận một trong ba giá trị<b>(2014,</b><i><b>P</b></i><b>(2014),</b><i><b>a,</b><b>P</b></i><b>(</b><i><b>a</b></i><b>))</b>.
Cả hai điều này đều không thỏa mãn điều kiện đề bài do mỗi số nguyên dương<i><b>m</b></i> phải là
ước của một số hạng khác<b>0</b>nào đó của dãy số <i><b>x</b><b>n</b></i>.


Tiếp theo, nếu một trong hai đa thức <i><b>P</b></i><b>(</b><i><b>x</b></i><b>),</b><i><b>Q</b></i><b>(</b><i><b>x</b></i><b>)</b> có bậc lớn hơn <b>1,</b> khơng mất tính tổng
qt, ta giả sử đó là<i><b>Q</b></i><b>(</b><i><b>x</b></i><b>)</b>. Rõ ràng khi đó<i><b>Q</b></i><b>(</b><i><b>P</b></i><b>(</b><i><b>x</b></i><b>))</b>cũng có bậc lớn hơn<b>1.</b>


Ta thấy nếu<i><b>R</b></i><b>(</b><i><b>x</b></i><b>)</b>là đa thức có bậc lớn hơn<b>1</b>thì với mọi<i><b>k</b><b>></b></i><b>0</b>lớn tùy ý, tồn tại <i><b>x</b></i> có giá trị
tuyệt đối đủ lớn sau cho<b>|</b><i><b>R</b></i><b>(</b><i><b>x</b></i><b>)|</b><i><b>></b><b>k</b></i><b>|</b><i><b>x</b></i><b>|.</b>Điều này là dễ thấy do khi<i><b>x</b></i> <b>→+∞</b> thì ta có giới
hạn


<b>lim</b>
<i><b>x</b></i><b>→+∞</b>


<b>|</b><i><b>R</b></i><b>(</b><i><b>x</b></i><b>)|</b>


<b>|</b><i><b>x</b></i><b>|</b> <b>= +∞.</b>


Ta sẽ chứng minh rằng tồn tại <i><b>N</b></i> đủ lớn sao cho <b><sub>|</sub></b><i><b>Q</b></i><b>(</b><i><b>P</b></i><b>(</b><i><b>x</b></i><b>))|</b><i><b>></b></i><b>|</b><i><b>P</b></i><b>(</b><i><b>x</b></i><b>)|+|</b><i><b>x</b></i><b>|</b>với mọi <i><b>x</b></i> <i><b>></b><b>N</b></i><b>.</b>
Ta chỉ cần xét 2 trường hợp: Nếu <i><b>P</b></i><b>(</b><i><b>x</b></i><b>)</b> là bậc 1 thì dễ thấy tồn tại <i><b>k</b></i> đủ lớn sao cho


<i><b>k</b></i><b>|</b><i><b>P</b></i><b>(</b><i><b>x</b></i><b>)|</b><i><b>></b></i><b>|</b><i><b>x</b></i><b>|</b>với mọi <i><b>x</b></i> <b>∈Z.</b>


Suy ra, tồn tại <i><b>N</b></i> đủ lớn sao cho


<b>|</b><i><b>Q</b></i><b>(</b><i><b>P</b></i><b>(</b><i><b>x</b></i><b>))|</b><i><b>></b></i><b>(</b><i><b>k</b></i><b>+1)|</b><i><b>P</b></i><b>(</b><i><b>x</b></i><b>)|=|</b><i><b>P</b></i><b>(</b><i><b>x</b></i><b>)|+</b><i><b>k</b></i><b>|</b><i><b>P</b></i><b>(</b><i><b>x</b></i><b>)|</b><i><b>></b></i><b>|</b><i><b>P</b></i><b>(</b><i><b>x</b></i><b>)|+|</b><i><b>x</b></i><b>|</b> với mọi<i><b>x</b></i> <i><b>></b><b>N</b></i><b>.</b>
Nhận xét được chứng minh.


Theo giả thiết thì trong dãy số đã cho, phải tồn tại số hạng<b>|</b><i><b>x</b><b>i</b></i><b>|</b>lớn tùy ý và rõ ràng, ta cũng
phải có <i><b>j</b></i> <i><b>></b></i><b>0</b>sao cho <b></b><i><b>x</b></i><b><sub>2</sub></b><i><b><sub>j</sub></b></i>


<b></b>


<b></b> <i><b>></b><b>N</b></i> <b>+1</b> và <i><b>x</b></i><b><sub>2</sub></b><i><b><sub>j</sub></b></i> có giá trị tuyệt đối lớn nhất trong<b>2</b><i><b>j</b></i> số hạng
đầu tiên của dãy<b>(</b><i><b>x</b><b>n</b></i><b>).</b>


Thật vậy, ta thấy rằng tồn tại vô số số hạng <i><b>x</b></i><b>2</b><i><b>j</b></i> thỏa mãn điều kiện
<b></b>
<b></b><i><b>x</b></i><b><sub>2</sub></b><i><b><sub>j</sub></b></i>


<b></b>


<b>=</b> <b>max</b>


<b>0¶</b><i><b>k</b></i><b>¶2</b><i><b>j</b></i> <b>{|</b><i><b>x</b><b>k</b></i><b>|}</b>,
gọi<i><b>T</b></i> là tập hợp các chỉ số thỏa mãn. Nếu như trong các số hạng như thế, khơng có số hạng
nào thỏa mãn<b></b><i><b>x</b></i><b><sub>2</sub></b><i><b><sub>j</sub></b></i>


<b></b>


<b></b><i><b>></b><b>N</b></i><b>+1</b>thì với mọi <i><b>t</b></i> <b>∈</b><i><b>T</b></i>, ta có<b>|</b><i><b>x</b><b><sub>i</sub></b></i><b>|<sub>¶</sub>|</b><i><b>x</b><b><sub>t</sub></b></i><b>|<sub>¶</sub></b><i><b>N</b></i><b>+1</b>với mọi<i><b>i</b></i> <b><sub>¶</sub></b><i><b>t</b></i> <b>∈</b><i><b>T</b></i>.


Tuy nhiên, do<b><sub>|</sub></b><i><b>T</b></i><b>|</b>vơ hạn nên điều giả sử ở trên là vô lý và nhận xét được chứng minh. Với
<i><b>x</b></i><b>2</b><i><b>j</b></i> là số hạng thỏa mãn điều kiện trên, chọn <i><b>m</b></i><b>=</b>


<b></b>


<b></b><i><b>x</b></i><b><sub>2</sub></b><i><b><sub>j</sub></b></i><b><sub>+</sub><sub>2</sub>−</b><i><b>x</b></i><b><sub>2</sub></b><i><b><sub>j</sub></b></i>
<b></b>


<b></b>thì ta thấy
<i><b>m</b></i><b>=</b><i><b>Q</b></i><b>(</b><i><b>P</b></i><b>(</b><i><b>x</b></i><b><sub>2</sub></b><i><b><sub>j</sub></b></i><b>))−</b><i><b>x</b></i><b><sub>2</sub></b><i><b><sub>j</sub></b></i>


<b></b>
<b>¾</b>


<b></b>


<b></b><i><b>Q</b></i><b>(</b><i><b>P</b></i><b>(</b><i><b>x</b></i><b><sub>2</sub></b><i><b><sub>j</sub></b></i><b>))</b>
<b></b>
<b>−</b>


<b></b>
<b></b><i><b>x</b></i><b><sub>2</sub></b><i><b><sub>j</sub></b></i>


<b></b>
<b></b><i><b>></b></i>


<b></b>
<b></b><i><b>P</b></i><b>(</b><i><b>x</b></i><b><sub>2</sub></b><i><b><sub>j</sub></b></i><b>)</b>


<b></b>


<b></b><i><b>></b></i>


<b></b>
<b></b><i><b>x</b></i><b><sub>2</sub></b><i><b><sub>j</sub></b></i>


<b></b>
<b></b> và
<b></b>


<b></b><i><b>x</b></i><b><sub>2</sub></b><i><b><sub>n</sub></b></i><b><sub>+</sub><sub>2</sub></b>
<b></b>
<b>=</b>


<b></b>


<b></b><i><b>Q</b></i><b>(</b><i><b>x</b></i><b><sub>2</sub></b><i><b><sub>n</sub></b></i><b><sub>+</sub><sub>1</sub>)</b>
<b></b>
<b></b><i><b>></b></i>


<b></b>
<b></b><i><b>x</b></i><b><sub>2</sub></b><i><b><sub>n</sub></b></i><b><sub>+</sub><sub>1</sub></b>


<b></b>
<b>.</b>


Do đó, trong<b>2</b><i><b>j</b></i> <b>+1</b>số hạng đầu tiên của dãy, khơng có số hạng nào chia hết cho<i><b>m</b></i>.
Mặt khác,<i><b>x</b></i><b>2</b><i><b>k</b></i><b>+2−</b><i><b>x</b></i><b>2</b><i><b>k</b></i> <b>=</b> <i><b>Q</b></i><b>(</b><i><b>P</b></i><b>(</b><i><b>x</b></i><b>2</b><i><b>k</b></i><b>))−</b><i><b>Q</b></i><b>(</b><i><b>P</b></i><b>(</b><i><b>x</b></i><b>2</b><i><b>k</b></i><b>−2))</b>


<b></b>..



. <i><b>P</b></i><b>(</b><i><b>x</b></i><b>2</b><i><b>k</b></i><b>)−</b><i><b>P</b></i><b>(</b><i><b>x</b></i><b>2</b><i><b>k</b></i><b>−2)</b>


<b></b>..


. <i><b>x</b></i><b>2</b><i><b>k</b></i><b>−</b><i><b>x</b></i><b>2</b><i><b>k</b></i><b>−2</b>


<b></b>


<b>=</b>


<i><b>m</b></i> và tương tự thì <i><b>x</b></i><b>2</b><i><b>k</b></i><b>+3−</b><i><b>x</b></i><b>2</b><i><b>k</b></i><b>+1=</b> <i><b>P</b></i><b>(</b><i><b>x</b></i><b>2</b><i><b>k</b></i><b>+2)−</b><i><b>P</b></i><b>(</b><i><b>x</b></i><b>2</b><i><b>k</b></i><b>)</b>
<b></b>..


.<b>€</b><i><b>x</b></i><b>2</b><i><b>j</b></i> <b>−</b><i><b>x</b></i><b>2</b><i><b>j</b></i><b>−2</b>


<b>Š</b>


<b>=</b><i><b>m</b></i>. Từ đây suy ra
với<i><b>k</b></i><b>¾</b> <i><b>j</b></i> thì <i><b>x</b></i><b>2</b><i><b>k</b></i><b>+2−</b><i><b>x</b></i><b>2</b><i><b>k</b></i> và <i><b>x</b></i><b>2</b><i><b>k</b></i><b>+3−</b><i><b>x</b></i><b>2</b><i><b>k</b></i><b>+1</b>đều chia hết cho<i><b>m</b></i>, tuy nhiên<i><b>x</b></i><b>2</b><i><b>j</b></i><b>+1</b>và <i><b>x</b></i><b>2</b><i><b>j</b></i><b>+2</b>đều


</div>
<span class='text_page_counter'>(100)</span><div class='page_container' data-page=100>

Việt Nam Team Selection Test 2014 17


Điều mâu thuẫn này cho ta thấy nhận xét ban đầu là đúng và<b>deg</b><i><b>P</b></i><b>(</b><i><b>x</b></i><b>) =</b> <b>deg</b><i><b>Q</b></i><b>(</b><i><b>x</b></i><b>) =1</b>.
Đặt<i><b>P</b></i><b>(</b><i><b>x</b></i><b>) =</b><i><b>ax</b></i><b>+</b><i><b>b</b></i>với<b><sub>|</sub></b><i><b>a</b></i><b>|</b><i><b>></b></i><b>1</b>và<i><b>Q</b></i><b>(</b><i><b>x</b></i><b>) =</b><i><b>c x</b></i> <b>+</b><i><b>d</b></i> vi<i><b>a,</b><b>b,</b><b>c,</b><b>d</b></i> <b>Z</b>v<i><b>ab</b></i><b>6=0</b>thỡ ta cú


<b>ă</b>


<i><b>x</b></i><b>2</b><i><b>n</b></i><b>+1=</b><i><b>ax</b></i><b>2</b><i><b>n</b></i><b>+</b><i><b>b</b></i>
<i><b>x</b></i><b>2</b><i><b>n</b></i><b>+2=</b><i><b>c x</b></i><b>2</b><i><b>n</b></i><b>+1+</b><i><b>d</b></i>



vi mi<i><b>n</b></i> <b><sub>ắ</sub>0.</b>


Suy ra<i><b>x</b></i><b>2</b><i><b>n</b></i><b>+2=</b><i><b>cax</b></i><b>2</b><i><b>n</b></i><b>+</b><i><b>bc</b></i><b>+</b><i><b>d</b></i> v <i><b>x</b></i><b>2</b><i><b>n</b></i><b>+3=</b><i><b>cax</b></i><b>2</b><i><b>n</b></i><b>+1+</b><i><b>ad</b></i><b>+</b><i><b>b</b></i>với mọi<i><b>n</b></i><b>¾0</b>.


Cả hai dãy này đều có cơng thức truy hồi dạng <i><b>y</b><b>n</b></i><b>+1</b> <b>=</b><i><b>k y</b><b>n</b></i><b>+</b><i><b>h</b></i> với<i><b>k</b></i> <b>=</b><i><b>ac,</b><b>h</b></i> <b>∈Z</b>. Giả sử
<i><b>k</b></i> <b>6=1</b> thì cơng thức tổng qt của dãy này là <i><b>y</b><b>n</b></i> <b>=</b> <i><b>k</b><b>n</b><b>y</b></i><b>0+</b><i><b>h</b></i>


<i><b>k</b><b>n</b></i><b><sub>−</sub><sub>1</sub></b>


<i><b>k</b></i><b>−1</b> với mọi <i><b>n.</b></i> Rõ ràng nếu


<i><b>k</b></i><b>=−1</b>thì dãy số tương ứng khơng thỏa mãn, ta xét<i><b>k</b></i><b>6=−1.</b>


• Nếu<i><b>h</b></i><b>=0</b>thì ta có <i><b>y</b><b>n</b></i> <b>=</b><i><b>k</b><b>n</b><b>y</b></i><b>0</b>, rõ ràng khơng thỏa mãn điều kiện.


• Nếu <i><b>h</b></i> <b>6=</b> <b>0</b>thì do <b>gcd</b>
<b>‚</b>


<i><b>k,</b><b>k</b></i>
<i><b>n</b></i><b><sub>−</sub><sub>1</sub></b>
<i><b>k</b></i><b>−1</b>


<b>Œ</b>


<b>=1</b> với mọi <i><b>n</b></i> nên giả sử giả sử <i><b>t</b></i> là số mũ lớn
nhất mà <i><b>k</b><b>t</b></i><b><sub>|</sub></b><i><b><sub>h</sub></b></i> <sub>thì các số ngun dương có dạng</sub> <i><b><sub>k</sub></b><b>s</b></i> <sub>với</sub> <i><b><sub>s</sub></b></i> <i><b><sub>></sub></b></i> <i><b><sub>t</sub></b></i> <sub>đều không là ước của</sub>
bất cứ số hạng nào của dãy, không thỏa mãn. Từ đây, suy ra <i><b>k</b></i><b>=1</b>hay <i><b>ac</b></i><b>=1</b>.
Cuối cùng, ta chỉ cần xét<b>2</b>trường hợp:


• Nếu<i><b>P</b></i><b>(</b><i><b>x</b></i><b>) =</b><i><b>x</b></i><b>+</b><i><b>a</b></i>và<i><b>Q</b></i><b>(</b><i><b>x</b></i><b>) =</b><i><b>x</b></i><b>+</b><i><b>b</b></i>với<i><b>a,</b><b>b</b></i><b>∈Z</b>thì bằng quy nạp, ta chứng minh được



<i><b>x</b></i><b>2</b><i><b>k</b></i> <b>=2014+</b><i><b>k</b></i><b>(</b><i><b>a</b></i><b>+</b><i><b>b</b></i><b>)</b>và <i><b>x</b></i><b>2</b><i><b>k</b></i><b>+1=2014+</b><i><b>a</b></i><b>+</b><i><b>k</b></i><b>(</b><i><b>a</b></i><b>+</b><i><b>b</b></i><b>)</b>. Dễ thấy rằng nếu<i><b>a</b></i><b>+</b><i><b>b</b></i><b>=0</b>


thì dãy này khơng thỏa mãn. Nếu như <i><b>a</b></i><b>+</b><i><b>b</b></i><b>6=0</b>thì gọi<i><b>S,</b><b>T,</b><b>R</b></i>lần lượt là tập hợp các
ước nguyên của <i><b>a</b></i><b>+</b><i><b>b</b></i>,<b>2014</b>và<b>2014+</b><i><b>a</b></i>. Ta xét các trường hợp sau:


<b>+</b> Với <i><b>m</b></i><b>∈</b><i><b>/</b><b>S</b></i>thì giả sử<i><b>a</b></i><b>+</b><i><b>b</b></i>chia<i><b>m</b></i> dư <i><b>t</b></i> với<i><b>t</b></i> <b>6=0</b>; do đó, khi <i><b>k</b></i>chạy qua một hệ
thặng dư đầy đủ modulo <i><b>m</b></i> thì tồn tại một số hạng của dãy chia hết cho <i><b>m</b></i> và
số lượng các số hạng như thế là vô hạn. Rõ ràng số các số hạng bằng <b>0</b>của dãy
là hữu hạn (không quá<b>2</b>) nên tồn tại số hạng khác<b>0</b>của dãy chia hết cho<i><b>m.</b></i>


<b>+</b> Với<i><b>m</b></i> <b>∈</b><i><b>S</b></i>mà <i><b>m</b></i><b>∈</b><i><b>/</b></i> <i><b>T,</b><b>R</b></i>thì dãy số tương ứng không thỏa mãn.


<b>+</b> Với <i><b>m</b></i> <b>∈</b> <i><b>S</b></i> và <i><b>m</b></i> <b>∈</b> <i><b>T</b></i> hoặc <i><b>m</b></i> <b>∈</b><i><b>R</b></i> thì tương ứng, mọi số hạng có chỉ số chẵn
hoặc mọi số hạng có chỉ số lẻ của dãy đều chia hết cho <i><b>m</b></i> và dễ thấy, tồn
tại số hạng khác <b>0</b> của dãy chia hết cho <i><b>m.</b></i> Do đó, các số <i><b>a,</b><b>b</b></i> phải thỏa mãn


<b>(</b><i><b>S</b></i><b>\</b><i><b>T</b></i><b>)∩(</b><i><b>S</b></i><b>\</b><i><b>R</b></i><b>) =∅</b>hay mỗi ước của<i><b>a</b></i><b>+</b><i><b>b</b></i>phải là ước của<b>2014</b>hoặc là ước của


<b>2014+</b><i><b>a.</b></i>


• Nếu <i><b>P</b></i><b>(</b><i><b>x</b></i><b>) =−</b><i><b>x</b></i> <b>+</b><i><b>a</b></i> và<i><b>Q</b></i><b>(</b><i><b>x</b></i><b>) =−</b><i><b>x</b></i> <b>+</b><i><b>b</b></i>thì cũng có lập luận tương tự vì các số hạng
của dãy tương ứng khi đó là<i><b>x</b></i><b>2</b><i><b>k</b></i> <b>=2014−</b><i><b>k</b></i><b>(</b><i><b>a</b></i><b>−</b><i><b>b</b></i><b>)</b>và<i><b>x</b></i><b>2</b><i><b>k</b></i><b>+1=−2014+</b><i><b>a</b></i><b>+</b><i><b>k</b></i><b>(</b><i><b>a</b></i><b>−</b><i><b>b</b></i><b>)</b>.


Điều kiện của <i><b>a,</b><b>b</b></i> là<i><b>a</b></i><b>−</b><i><b>b</b></i> <b>=6</b> <b>0</b> và mỗi ước của <i><b>a</b></i><b>−</b><i><b>b</b></i>phải là ước của <b>2014</b>hoặc là
ước của <i><b>a</b></i><b>−2014.</b>


Vậy tất cả các đa thức<i><b>P</b></i><b>(</b><i><b>x</b></i><b>),</b><i><b>Q</b></i><b>(</b><i><b>x</b></i><b>)</b>cần tìm là


• <i><b>P</b></i><b>(</b><i><b>x</b></i><b>) =</b> <i><b>x</b></i><b>+</b><i><b>a,</b><b>Q</b></i><b>(</b><i><b>x</b></i><b>) =</b><i><b>x</b></i> <b>+</b><i><b>b</b></i>trong đó <i><b>a,</b><b>b</b></i><b>∈Z</b>thỏa mãn<i><b>a</b></i><b>+</b><i><b>b</b></i><b>6=0</b> và mỗi ước của



<i><b>a</b></i><b>+</b><i><b>b</b></i>phải là ước của<b>2014</b>hoặc là ước của<b>2014+</b><i><b>a.</b></i>


• <i><b>P</b></i><b>(</b><i><b>x</b></i><b>) =−</b><i><b>x</b></i> <b>+</b><i><b>a,</b><b>Q</b></i><b>(</b><i><b>x</b></i><b>) =−</b><i><b>x</b></i><b>+</b><i><b>b</b></i>trong đó <i><b>a,</b><b>b</b></i><b>∈Z</b>thỏa mãn <i><b>a</b></i><b>−</b><i><b>b</b></i><b>6=0</b> và mỗi ước


</div>
<span class='text_page_counter'>(101)</span><div class='page_container' data-page=101>

Việt Nam Team Selection Test 2014 18


<b>Nhận xét.</b>


Đây là bài tốn liên quan đến các tính chất số học của đa thức. Trong các bài tốn này,
có tính chất cơ bản sau đây khá hữu dụng: Nếu <i><b>P</b></i><b>(</b><i><b>x</b></i><b>)</b> là đa thức có hệ số ngun thì


<b>(</b><i><b>a</b></i><b>−</b><i><b>b</b></i><b>)|(</b><i><b>P</b></i><b>(</b><i><b>a</b></i><b>)−</b><i><b>P</b></i><b>(</b><i><b>b</b></i><b>))</b> với <i><b>a</b></i> <b>6=</b> <i><b>b</b></i>. Học sinh cũng phải vận dụng tốt tính đơn điệu của đa
thức khi biến đủ lớn và một chút khéo léo trong việc xử lí các kĩ thuật ở những vấn đề nhỏ.
Bài toán yêu cầu sự cẩn thận, chi tiết; suy luận một cách logic, khoa học, có thứ tự hợp lý.
Có thể nói đây là một bài tốn hay, mới lạ và có nhiều “bẫy” trong đó.


Thoạt nhìn vào bài tốn này, ta có thể thấy khơng dễ hình dung ra mơ hình và cách tiếp
cận vì có tới 2 đa thức khơng liên quan đến nhau xuất hiện. Một suy nghĩ tự nhiên là nên
giải thử với trường hợp<i><b>P</b></i><b>,</b><i><b>Q</b></i>trùng nhau.


Khi đó ta có bài tốn chỉ với một đa thức và cách tiếp cận cũng dễ nhìn ra hơn, thậm chí
các học sinh có thể xử lý tốt hơn nếu đã có biết về bài tốn sau:


<i><b>(Arthur Angel, Problem Solving Strategies)</b></i>Cho đa thức<i><b>P</b></i><b>(</b><i><b>x</b></i><b>)</b>với hệ số nguyên thỏa mãn
điều kiện<i><b>P</b></i><b>(</b><i><b>n</b></i><b>)</b><i><b>></b><b>n</b></i> với mọi số nguyên dương<i><b>n</b></i>. Xét


<i><b>x</b></i><b>1=1,</b><i><b>x</b></i><b>2=</b><i><b>P</b></i><b>(</b><i><b>x</b></i><b>1),</b><i><b>x</b></i><b>3=</b><i><b>P</b></i><b>(</b><i><b>x</b></i><b>2),· · ·</b>


Biết rằng với mọi số nguyên dương<i><b>N,</b></i>tồn tại một số hạng của dãy số chia hết cho<i><b>N</b></i>. Chứng


minh rằng<i><b>P</b></i><b>(</b><i><b>n</b></i><b>) =</b><i><b>n</b></i><b>+1</b>.


Ta có được <i><b>x</b><b>n</b></i><b>+2−</b><i><b>x</b><b>n</b></i><b>+1</b>chia hết cho <i><b>x</b><b>n</b></i><b>+1−</b><i><b>x</b><b>n</b></i> với mọi <i><b>n</b></i>, ta suy ra các phần tử của dãy số
<i><b>x</b><b>n</b></i><b>+1,</b><i><b>x</b><b>n</b></i><b>+2,</b><i><b>x</b><b>n</b></i><b>+3...</b>khi chia cho <i><b>x</b><b>n</b></i><b>+1−</b><i><b>x</b><b>n</b></i> có cùng số dư.


Cùng với tính tăng của đa thức, ta sẽ thấy nếu <i><b>x</b><b>n</b></i><b>+1</b> “quá lớn” so với <i><b>x</b><b>n</b></i> thì đa thức tương
ứng sẽ khơng thể thỏa mãn điều kiện bài toán. Cách tiếp cận này chắc hẳn sẽ mang lại một
cái nhìn vừa bao quát, vừa đơn giản hơn cho bài số<b>5.</b>


Lời giải trên đã cố gắng làm sáng tỏ tất cả các vấn đề đặt ra trong bài toán nên tương đối
dài. Ta thấy rằng tư tưởng chính là loại trường hợp bậc<b>0</b>của các đa thức để đưa về xét 2
trường hợp quan trọng:


• Ít nhất một trong hai đa thức có bậc lớn hơn <b>1.</b>
• Bậc của cả hai đa thức đều bằng<b>1.</b>


Tuy nhiên, ta cũng cần hiểu nguyên nhân tại sao cần phải chia ra như thế, tức là tại sao lập
luận ở trường hợp (1) lại không áp dụng được cho trường hợp (2)? Câu trả lời như sau:
cách lập luận ở<b>(1)</b>đòi hỏi cần có cả hai bất đẳng thức sau


<b>|</b><i><b>Q</b></i><b>(</b><i><b>P</b></i><b>(</b><i><b>x</b></i><b>))−</b><i><b>x</b></i><b>|</b><i><b>></b></i><b>|</b><i><b>x</b></i><b>|</b> và <b><sub>|</sub></b><i><b>Q</b></i><b>(</b><i><b>P</b></i><b>(</b><i><b>x</b></i><b>))−</b><i><b>x</b></i><b>|</b><i><b>></b></i><b>|</b><i><b>P</b></i><b>(</b><i><b>x</b></i><b>)|</b>


đều phải đúng (*). Khi đó, dùng tính chất đơn điệu của đa thức để chỉ ra một số nguyên
dương<i><b>m</b></i> không là ước của số hạng khác<b>0</b>nào của dãy và dẫn đến mâu thuẫn.


</div>
<span class='text_page_counter'>(102)</span><div class='page_container' data-page=102>

Việt Nam Team Selection Test 2014 19


Áp dụng lập luận tương tự trường hợp<b>(1),</b>ta có thể giải quyết trường hợp<b>(2)</b>như sau:
Nếu hệ số cao nhất của một trong hai đa thức<i><b>P</b></i><b>(</b><i><b>x</b></i><b>),</b><i><b>Q</b></i><b>(</b><i><b>x</b></i><b>)</b>có giá trị tuyệt đối lớn hơn hoặc
bằng<b>3</b>thì tính chất (*) vẫn đúng.



Nếu hệ số cao nhất của cả hai đa thức <i><b>P</b></i><b>(</b><i><b>x</b></i><b>),</b><i><b>Q</b></i><b>(</b><i><b>x</b></i><b>)</b> đều không vượt quá 2 thì ta xét hiệu
<i><b>x</b><b>n</b></i><b>+4−</b><i><b>x</b><b>n</b></i> và lập luận tương tự.


Các bạn thử từ ý tưởng trên để tìm ra một cách lập luận ngắn gọn hơn cho bài toán trên!
Về một hướng mở rộng, chúng ta thử xem xét bài toán trong trường hợp thay<b>2</b>đa thức bởi
<b>3</b>đa thức như sau:


Tìm tất cả đa thức <i><b>P</b></i><b>(</b><i><b>x</b></i><b>),</b><i><b>Q</b></i><b>(</b><i><b>x</b></i><b>),</b><i><b>R</b></i><b>(</b><i><b>x</b></i><b>)</b> có hệ số nguyên và thỏa mãn điều kiện: Với dãy số


<b>(</b><i><b>x</b><b>n</b></i><b>)</b> xác định bởi <i><b>x</b></i><b>0</b> <b>=2014,</b><i><b>x</b></i><b>3</b><i><b>n</b></i><b>+1=</b> <i><b>P</b></i><b>(</b><i><b>x</b></i><b>3</b><i><b>n</b></i><b>),</b><i><b>x</b></i><b>3</b><i><b>n</b></i><b>+2=</b><i><b>Q</b></i><b>(</b><i><b>x</b></i><b>3</b><i><b>n</b></i><b>+1),</b><i><b>x</b></i><b>3</b><i><b>n</b></i><b>+3</b> <b>=</b><i><b>R</b></i><b>(</b><i><b>x</b></i><b>3</b><i><b>n</b></i><b>+2),</b><i><b>n</b></i> <b>¾0</b>,


thì mỗi số ngun dương<i><b>m</b></i> là ước của một số hạng khác 0 nào đó của dãy<b>(</b><i><b>x</b><b>n</b></i><b>).</b>


Các bạn thử xem xét đặc điểm chung cơ bản cũng như cái khó của bài tốn mới này hoặc
bài tốn tổng qt khi thay<b>2, 3</b>bởi <i><b>k</b></i><b>¾4</b>đa thức so với bài tốn gốc là gì nhé!


<b>Bài 6.</b>


Cho <i><b>m,</b><b>n,</b><b>p</b></i> là các số tự nhiên không đồng thời bằng <b>0.</b> Không gian tọa độ được
chia thành các mặt phẳng song song cách đều nhau. Một cách điền vào mỗi khối lập
phương đơn vị một trong các số từ <b>1</b> đến <b>60</b> được gọi là cách điền <i><b>Điện Biên</b></i> nếu
thỏa mãn: trong mỗi hình hộp chữ nhật với các mặt trên hệ mặt đã cho và tập hợp
kích thước ba cạnh (số hình lập phương trên trên cạnh) xuất phát từ một đỉnh là
<b>2m+1, 2n+1, 2p+1</b>, khối lập phương đơn vị có tâm trùng với tâm của hình hộp
chữ nhật được điền số bằng trung bình cộng của các số điền ở tâm của <b>8</b> hình lập
phương ở các góc của hình hộp đó. Hỏi có tất cả bao nhiêu cách điền<i><b>Điện Biên</b></i>?
Những cách điền là giống nhau nếu trong mỗi cách, ta chọn một hình lập phương
làm tâm thì các số được điền vào các khối lập phương đơn vị có cùng vị trí tương đối
với tâm trong hai cách điền là bằng nhau



Lời giải. Khơng mất tính tổng qt, giả sử các mặt phẳng song song trong hệ mặt cách đều
nhau một khoảng là 1 đơn vị. Ta chọn một hình lập phương nào đó rồi lấy tâm <i><b>O</b></i> của nó
làm tâm của khơng gian vng góc, các trục <i><b>O x</b></i><b>,</b><i><b>O y,</b><b>O z</b></i> song song với các đường thẳng
của hệ đã cho và chiều dương các trục tọa độ chọn tùy ý. Khi đó, ta quy ước tọa độ của
một hình lập phương trong hệ mặt đã cho là khoảng cách từ hình chiếu của nó lên các trục
<i><b>O x</b></i><b>,</b><i><b>O y,</b><b>O z</b></i> đến<i><b>O</b></i>(chọn dấu tùy vào chiều âm hay chiều dương của các trục tọa độ).
Xét một cách điền có tính chất<i><b>Điện Biên</b></i> nào đó và ta gọi hai hình lập phương phải được
điền cùng một số trong cách điền này là có<i>quan hệ</i> với nhau. Để đếm số cách điền <i>Điện</i>
<i>Biên,</i>ta đếm số lớn nhất các hình lập phương đơn vị có thể được chọn sao cho các hình đơi
một khơng có quan hệ với nhau, đặt<i><b>S</b></i>là số lượng hình này.


Trước hết, ta sẽ chứng minh rằng với mỗi hình hộp chữ nhật có kích thước các cạnh là
<b>2m+1, 2n+1, 2</b><i><b>p</b></i><b>+1</b>thì 8 số điền ở góc và số điền ở tâm là bằng nhau.


</div>
<span class='text_page_counter'>(103)</span><div class='page_container' data-page=103>

Việt Nam Team Selection Test 2014 20


kích thước<b>2m+1, 2n+1, 2p+1</b>và các hình có<i><b>A</b></i>là tâm sẽ có<b>8</b>số điền ở<b>8</b>ơ ở đỉnh đều
khơng nhỏ hơn<i><b>a</b></i>mà lại có trung bình cộng bằng <i><b>a</b></i>. Suy ra<b>8</b>số điền ở các đỉnh và số điền
ở tâm của hình hộp chữ nhật này là giống nhau. Lập luận tương tự, ta có thể mở rộng ra
trong khơng gian và được một tập hợp<i><b>T</b></i> các hình lập phương điền số <i><b>a</b></i>.


Với các hình lập phương khơng thuộc<i><b>T</b></i>, giả sử <i><b>b</b><b>></b><b>a</b></i>là số nhỏ nhất được điền cho các hình
lập phương cịn lại và tương tự trên, ta được một tập hợp<i><b>T</b></i><b>0</b> <sub>gồm các hình lập phương điền</sub>


số<i><b>b</b></i>. Rõ ràng cứ như thế, ta thấy nhận xét trên được chứng minh và đây cũng chính là điều
kiện cần để một cách điền là có tính chất Điện Biên.


Như thế, hình nằm ở tọa độ<b>(</b><i><b>x</b></i><b>,</b><i><b>y,</b><b>z</b></i><b>)</b>sẽ có quan hệ với hình nằm ở tọa độ



<b>(</b><i><b>x</b></i><b>+ (−1)</b><i><b>r</b><b>m,</b><b>y</b></i><b>+ (−1)</b><i><b>s</b><b>n,</b><b>z</b></i><b>+ (−1)</b><i><b>t</b><b>p</b></i><b>).</b>


Các số <i><b>r,</b><b>s,</b><b>t</b></i> <b>∈ {0; 1}</b> được chọn một cách độc lập với nhau (là 8 đỉnh của hình hộp chữ
nhật có tâm là <b>(</b><i><b>x</b></i><b>,</b><i><b>y,</b><b>z</b></i><b>)</b> và các cạnh là <b>2m</b> <b>+1, 2n+1, 2</b><i><b>p</b></i><b>+1</b>). Hơn nữa, ở đây ta xét
hình hộp chữ nhật có cạnh<b>2m+1, 2n+1, 2p+1</b>tương ứng song song với các mặt phẳng
<i><b>O y z,</b><b>O z x</b></i><b>,</b><i><b>O x y</b></i> và ta hồn tồn có thể hốn đổi thứ tự này.


Từ đây suy ra, hình lập phương ở tọa độ<b>(</b><i><b>x</b></i><b>,</b><i><b>y,</b><b>z</b></i><b>)</b>sẽ được điền số giống với hình lập phương
ở tọa độ <i><b>x</b></i><b>+</b><i><b>a</b></i><b>1</b><i><b>m</b></i><b>+</b><i><b>a</b></i><b>2</b><i><b>n</b></i><b>+</b><i><b>a</b></i><b>3</b><i><b>p,</b><b>y</b></i><b>+</b><i><b>b</b></i><b>1</b><i><b>m</b></i><b>+</b><i><b>b</b></i><b>2</b><i><b>n</b></i><b>+</b><i><b>b</b></i><b>3</b><i><b>p,</b><b>z</b></i><b>+</b><i><b>c</b></i><b>1</b><i><b>m</b></i><b>+</b><i><b>c</b></i><b>2</b><i><b>n</b></i><b>+</b><i><b>c</b></i><b>3</b><i><b>p</b></i>


<b></b>


. Tuy nhiên,
với hai hình có quan hệ với nhau, khi xét sự thay đổi ở các thành phần <i><b>x</b></i><b>,</b><i><b>y,</b><b>z</b></i> tương ứng
thì các đại lượng <i><b>m,</b><b>n,</b><b>p</b></i> sẽ hoặc tăng, hoặc giảm chứ không giữ nguyên và mỗi lần như
thế thì tính chẵn lẻ của<i><b>a</b></i><b>1+</b><i><b>a</b></i><b>2+</b><i><b>a</b></i><b>3,</b><i><b>b</b></i><b>1+</b><i><b>b</b></i><b>2+</b><i><b>b</b></i><b>3,</b><i><b>c</b></i><b>1+</b><i><b>c</b></i><b>2+</b><i><b>c</b></i><b>3</b>đồng thời thay đổi. Ban đầu,


xét ở vị trí<b>(</b><i><b>x</b></i><b>,</b><i><b>y,</b><b>z</b></i><b>)</b> thì ba tổng này cùng tính chẵn lẻ (cùng bằng 0) nên tính chẵn lẻ của
<i><b>a</b></i><b>1+</b><i><b>a</b></i><b>2+</b><i><b>a</b></i><b>3,</b><i><b>b</b></i><b>1+</b><i><b>b</b></i><b>2+</b><i><b>b</b></i><b>3,</b><i><b>c</b></i><b>1+</b><i><b>c</b></i><b>2+</b><i><b>c</b></i><b>3</b>là luôn giống nhau.


Một hệ quả của nhận xét trên là: hình lập phương ở <b>(</b><i><b>x</b></i><b>,</b><i><b>y,</b><b>z</b></i><b>)</b> sẽ có quan hệ với hình lập
phương ở tọa độ <b>(</b><i><b>x</b></i> <b>+</b><i><b>x</b></i><b>1,</b><i><b>y</b></i> <b>+</b> <i><b>y</b></i><b>1,</b><i><b>z</b></i> <b>+</b><i><b>z</b></i><b>1)</b> với <b>(</b><i><b>x</b></i><b>1,</b><i><b>y</b></i><b>1,</b><i><b>z</b></i><b>1)</b> là một hoán vị của bộ các số


<b>(2k m, 2kn, 2k p)</b>và<i><b>k</b></i> <b>∈Z,</b><i><b>d</b></i> <b>∈</b><i><b>m,</b><b>n,</b><b>p</b></i> .<b>(∗)</b>


Hơn nữa, theo định lý Bezout thì trong các tổ hợp tuyến tính của <i><b>m,</b><b>n</b></i><b>,</b><i><b>p</b></i>, ta có <i><b>d</b></i> <b>=</b>
<b>gcd(</b><i><b>m,</b><b>n,</b><b>p</b></i><b>)</b> <i><b>></b></i> <b>0</b> chính là tổ hợp tuyến tính có giá trị tuyệt đối nhỏ nhất. Từ đây suy
ra các hình lập phương đơn vị trong một hình lập phương lớn hơn có kích thước<i><b>d</b></i><b>×</b><i><b>d</b></i><b>×</b><i><b>d</b></i>
sẽ đơi một khơng có quan hệ với nhau (vì hai hình lập phương đơn vị bất kỳ trong hình này
sẽ có chênh lệch tọa độ nhỏ hơn<i><b>d</b></i>). Từ đó suy ra<i><b>S</b></i>là bội của<i><b>d</b></i><b>3<sub>.</sub></b>



</div>
<span class='text_page_counter'>(104)</span><div class='page_container' data-page=104>

Việt Nam Team Selection Test 2014 21


Đặt<i><b>m</b></i><b>1=</b>


<i><b>m</b></i>
<i><b>d</b></i> <b>,</b><i><b>n</b></i><b>1=</b>


<i><b>n</b></i>
<i><b>d</b></i><b>,</b><i><b>p</b></i><b>1=</b>


<i><b>p</b></i>


<i><b>d</b></i>, ta xét các trường hợp sau:


• Nếu cả ba số <i><b>m</b></i><b>1,</b><i><b>n</b></i><b>1,</b><i><b>p</b></i><b>1</b> đều lẻ, đặt<i><b>m</b></i><b>1=2m2+1,</b><i><b>n</b></i><b>1=2n2+1,</b><i><b>p</b></i><b>1=2p2+1</b>thì rõ


ràng <i><b>m</b></i><b>=2d m2+</b><i><b>d,</b><b>n</b></i> <b>=2dn2+</b><i><b>d,</b><b>p</b></i><b>=2d p2+</b><i><b>d</b></i>.


Do đó, theo<b>(<sub>∗</sub>),</b>hình lập phương ở tọa độ<b>(</b><i><b>m,</b><b>n,</b><b>p</b></i><b>)</b>sẽ có quan hệ với hình ở các tọa
độ <b>(</b><i><b>m</b></i><b>±</b><i><b>d,</b><b>n</b></i><b>±</b><i><b>d,</b><b>p</b></i><b>±</b><i><b>d</b></i><b>)</b>(các hình kề đỉnh với <i><b>A</b></i>). Trong mơ hình trên, nếu ta chọn
hình <i><b>A</b></i> và <b>3</b> hình kề mặt với nó, mỗi chiều một hình thì <b>4</b> hình này đơi một khơng
có quan hệ với nhau và không thể chọn số lượng nào lớn hơn 4 được. Từ đó, ta có
<i><b>S</b></i><b>=4d3<sub>.</sub></b>


• Nếu trong ba số <i><b>m</b></i><b>1,</b><i><b>n</b></i><b>1,</b><i><b>p</b></i><b>1</b> có <b>2</b> số lẻ và <b>1</b> số chẵn, ta có thể giả sử<i><b>m</b></i><b>1,</b><i><b>n</b></i><b>1</b> lẻ và <i><b>p</b></i><b>1</b>


chẵn. Ta lại đặt<i><b>m</b></i><b>1=2m2+1,</b><i><b>n</b></i><b>1=2n2+1,</b><i><b>p</b></i><b>1=2p2</b>thì rõ ràng<i><b>m</b></i><b>=2d m2+</b><i><b>d,</b><b>n</b></i><b>=</b>


<b>2dn2+</b><i><b>d,</b><b>p</b></i><b>=2d p2</b>.



Do đó, theo<b>(<sub>∗</sub>),</b>hình lập phương ở tọa độ<b>(</b><i><b>m,</b><b>n,</b><b>p</b></i><b>)</b>sẽ có quan hệ với hình ở các tọa
độ <b>(</b><i><b>m</b></i><b>±</b><i><b>d,</b><b>n</b></i><b>±</b><i><b>d,</b><b>p</b></i><b>)</b> (các hình kề cạnh với<i><b>A</b></i>). Ta lại hốn đổi vị trí của <i><b>m,</b><b>n,</b><b>p</b></i>, tức
là thay đổi chiều của <b>(</b><i><b>x,</b><b>y,</b><b>z</b></i><b>)</b>thì suy ra thêm<b>(</b><i><b>m,</b><b>n</b></i><b>±</b><i><b>d,</b><b>p</b></i><b>±</b><i><b>d</b></i><b>)</b>và<b>(</b><i><b>m</b></i><b>±</b><i><b>d,</b><b>n,</b><b>p</b></i><b>±</b><i><b>d</b></i><b>)</b>
có quan hệ với<i><b>A.</b></i>Trong mơ hình trên, nếu ta chọn hình <i><b>A</b></i>và<b>1</b>hình kề mặt với nó thì
hai hình này khơng có quan hệ với nhau và khơng thể chọn số lượng nào lớn hơn <b>2</b>
được. Từ đó, ta có<i><b>S</b></i><b>=2d3<sub>.</sub></b>


• Nếu trong ba số <i><b>m</b></i><b>1,</b><i><b>n</b></i><b>1,</b><i><b>p</b></i><b>1</b> có <b>1</b> số lẻ và <b>2</b> số chẵn, ta có thể giả sử<i><b>m</b></i><b>1</b> lẻ và <i><b>n</b></i><b>1,</b><i><b>p</b></i><b>1</b>


chẵn. Ta lại đặt <i><b>m</b></i><b>1</b> <b>=2m2+1,</b><i><b>n</b></i><b>1=2n2,</b><i><b>p</b></i><b>1</b> <b>=2p2</b> thì rõ ràng<i><b>m</b></i> <b>=2d m2+</b><i><b>d</b></i><b>,</b><i><b>n</b></i> <b>=</b>


<b>2dn2,</b><i><b>p</b></i><b>=2d p2</b>.


Do đó, theo<b>(∗),</b>hình lập phương ở tọa độ<b>(</b><i><b>m,</b><b>n,</b><b>p</b></i><b>)</b>sẽ có quan hệ với hình ở các tọa
độ<b>(</b><i><b>m</b></i><b>±</b><i><b>d</b></i><b>,</b><i><b>n,</b><b>p</b></i><b>)</b>(các hình kề mặt với<i><b>A</b></i>). Ta lại hốn đổi vị trí của<i><b>m,</b><b>n</b></i><b>,</b><i><b>p</b></i>, tức là thay
đổi chiều của<b>(</b><i><b>x</b></i><b>,</b><i><b>y,</b><b>z</b></i><b>)</b> thì suy ra thêm<b>(</b><i><b>m,</b><b>n</b></i><b>±</b><i><b>d</b></i><b>,</b><i><b>p</b></i><b>)</b>và <b>(</b><i><b>m,</b><b>n,</b><b>p</b></i><b>±</b><i><b>d</b></i><b>)</b>có quan hệ với
<i><b>A.</b></i>Trong mơ hình trên, ta chỉ chọn được hình<i><b>A</b></i>chứ khơng chọn được thêm hình nào
khơng có quan hệ với nó. Từ đó, ta có<i><b>S</b></i><b>=</b><i><b>d</b></i><b>3</b><sub>.</sub>


Cuối cùng, ta thấy mỗi ơ độc lập với nhau sẽ có <b>60</b> cách điền số (từ <b>1</b> đến <b>60</b>) cho nó
nên số cách điền Điện Biên tổng cộng là <b>60</b><i><b>S</b></i> với <i><b>S</b></i> được xác định như sau: Trong ba số


<i><b>m</b></i>
<b>gcd(</b><i><b>m,</b><b>n</b></i><b>,</b><i><b>p</b></i><b>),</b>


<i><b>n</b></i>
<b>gcd(</b><i><b>m,</b><b>n,</b><b>p</b></i><b>),</b>


<i><b>p</b></i>



<b>gcd(</b><i><b>m,</b><b>n,</b><b>p</b></i><b>),</b>nếu có
• <b>3</b>số lẻ thì<i><b>S</b></i><b>=4d3<sub>.</sub></b>


• <b>2</b>số lẻ thì<i><b>S</b></i><b>=2d3<sub>.</sub></b>


• <b>1</b>số lẻ thì<i><b>S</b></i><b>=</b><i><b>d</b></i><b>3<sub>.</sub></b>


<b>Nhận xét.</b>


</div>
<span class='text_page_counter'>(105)</span><div class='page_container' data-page=105>

Việt Nam Team Selection Test 2014 22


thí sinh chắc cũng nhanh chóng nhận ra rằng số<b>60</b>khơng có nhiều ảnh hưởng trong việc
đi tìm lời giải.


Bài tốn này khá lạ và khó. Có thể nói khi xử lý các bài tốn đếm tổ hợp thì cơng việc đầu
tiên chính là đốn đáp số và nếu đốn thành cơng thì nhiều khi mọi thứ sẽ dễ dàng hơn.
Điểm bất ngờ nhất ở đây lại chính là đáp số khi mà trên thực tế, có nhiều đáp số chưa
hồn tồn chính xác đã được đưa ra. Điều này có lẽ xuất phát từ việc xử lí hai điều kiện khi
chuyển về dạng số học cuối cùng khơng cẩn thận. Nếu thực sự bình tĩnh, và có thời gian, ta
có thể xét thử vài ví dụ nhỏ với<i><b>m,</b><b>n,</b><b>p</b></i> để thấy có các kết quả “tương đối khác nhau” chứ
không cùng một dạng.


Tuy nhiên, ở trong phịng thi, cùng với việc bài<b>5</b>làm mất khơng ít thời gian, đây thực sự là
một thử thách không dễ.


Đây là một bài tốn hay, có nhiều đất cho các bạn học sinh thể hiện khả năng. Bài tốn địi
hỏi những tư duy đơn giản, nhưng tinh tế ở phần suy luận tổ hợp, và sự cẩn thận, khoa học
trong phần xử lí điều kiện số học cuối cùng. Nếu chưa có một hình dung tốt về khơng gian
<b>3</b>chiều, ta hồn tồn có thể chọn cách tiếp cận từ việc, đưa bài tốn về<b>2</b>( xét mặt phẳng


và hình chữ nhật <b>2</b> cạnh lẻ) hay thậm chí <b>1</b> chiều ( đường thẳng và đoạn thẳng chứa lẻ
điểm). Cụ thể như dưới đây:


Cho<i><b>k,</b><b>a</b></i> là các số nguyên dương. Trên trục số nguyên, người ta đánh số các điểm nguyên
bằng các số từ<b>1</b> đến <i><b>k</b></i> sao cho: Mỗi đoạn thẳng có hai đầu mút ngun và có chứa đúng
<b>2a+1</b>điểm ngun thì trung điểm của nó được đánh số bằng trung bình cộng số đánh ở
hai đầu mút. Hỏi có tất cả bao nhiêu cách đánh số khác nhau?


Bằng lập luận theo nguyên lý cực hạn tương tự như lời giải bài toán gốc, ta thấy rằng:
• Với các đoạn thẳng có chứa đúng <b>2a+1</b> điểm nguyên thì số được đánh cho <b>2</b> đầu


mút bằng với số đánh cho trung điểm.


• Các điểm cách nhau ít hơn<i><b>a</b></i>đơn vị độ dài thì khơng có ràng buộc với nhau. Như thế
<i><b>a</b></i>điểm có tọa độ từ <b>0, 1, 2, ...,</b><i><b>a</b></i><b>−1</b> sẽ đôi một đánh số độc lập với nhau và kết quả
của bài toán sẽ là<i><b>k</b><b>a</b></i><sub>.</sub>


Tiếp tục phát triển lên, ta xét bài toán sau:


Cho<i><b>k,</b><b>a,</b><b>b</b></i>là các số nguyên dương. Trên trục số nguyên, người ta đánh số các điểm nguyên
bằng các số từ<b>1</b> đến <i><b>k</b></i> sao cho: Mỗi đoạn thẳng có hai đầu mút nguyên và có chứa đúng
<b>2a+1</b>hoặc<b>2b+1</b>điểm ngun thì trung điểm của nó được đánh số bằng trung bình cộng
số đánh ở hai đầu mút. Hỏi có tất cả bao nhiêu cách đánh số khác nhau?


Tiếp tục lập luận như bài trên, ta thấy các vị trí<b>0,</b><i><b>a, 2a,</b><b>b, 2</b><b>b</b></i>được đánh cùng một số hay
nói cách khác, các vị trí cách nhau<i><b>a</b></i>hoặc <i><b>b</b></i>đơn vị thì được đánh cùng một số. Từ đó suy ra
tất cả các điểm ở vị trí <i><b>x a</b></i><b>+</b><i><b>y b</b></i>với<i><b>x</b></i><b>,</b><i><b>y</b></i><b>∈Z</b>và <i><b>x</b></i><b>,</b><i><b>y</b></i> khơng đồng thời bằng<b>0</b>sẽ được đánh


cùng một số với nhau.



</div>
<span class='text_page_counter'>(106)</span><div class='page_container' data-page=106>

Việt Nam Team Selection Test 2014 23


Bằng cách tương tự, việc thay<b>2</b>số<i><b>a,</b><b>b</b></i>thành nhiều số vẫn được giải quyết tương tự và bài
tốn ln có một kết quả theo dạng lũy thừa trên. Tuy nhiên, nếu thay bởi bài toán<b>2</b>chiều,
ta lại có đến<b>2</b>kết quả: Cho<i><b>k,</b><b>a,</b><b>b</b></i>là các số nguyên dương. Trong mặt phẳng tọa độ, người
ta đánh số các điểm nguyên bằng các số từ 1 đến<i><b>k</b></i>sao cho: Mỗi hình chữ nhật có các cạnh
song song với<b>2</b>trục tọa độ và trên mỗi cạnh có<b>2a+1, 2b+1</b>điểm ngun thì trung bình
cộng<b>4</b>số điền ở đỉnh bằng số điền ở tâm. Hỏi có tất cả bao nhiêu cách đánh số khác nhau?


Kết quả của bài toán là: Đặt<i><b>c</b></i><b>=gcd(</b><i><b>a,</b><b>b</b></i><b>)</b>, nếu trong hai số <i><b>a</b></i>
<i><b>c</b></i><b>,</b>


<i><b>b</b></i>
<i><b>c</b></i> có:
• <b>2</b>số lẻ thì kết quả là<i><b>k</b></i><b>2</b><i><b>c</b></i><b>2</b>


<b>.</b>
• <b>1</b>số lẻ thì kết quả là<i><b>k</b><b>c</b></i><b>2</b>


.


</div>
<span class='text_page_counter'>(107)</span><div class='page_container' data-page=107>

Việt Nam Team Selection Test 2014 24


<b>Tài liệu tham khảo</b>



1. Topic đề thi và bình luận đề<b>VN TST 2014</b>của diễn đàn<b>mathscope.org</b>


/>


2. <b>Trần Quang Hùng</b>, Xung quanh bài hình học trong kỳ thi chọn đội tuyển VN ngày 1
/>



3. <b>Trần Quang Hùng</b>, Xung quanh bài hình học trong kỳ thi chọn đội tuyển VN ngày 2


/>


4. <b>Nguyễn Thị Hường, Lương Ánh Nguyệt, Lương Thị Thanh Mai, Đào Thị Quỳnh</b>
<b>Nga,</b> Định lý Sawayama và Thébault


/>


5. Functional equation<b>APMO 1989</b>


/>


6. Bình luận của<b>GS Nguyễn Tiến Dũng</b>trên trang<b>Sputnik Education</b>


/>


7. Bài toán <b>IMO Shortlist 1994, G4</b>


/>


8. Mẫu Latex của thầy<b>Châu Ngọc Hùng</b>


/>


</div>
<span class='text_page_counter'>(108)</span><div class='page_container' data-page=108></div>
<span class='text_page_counter'>(109)</span><div class='page_container' data-page=109></div>
<span class='text_page_counter'>(110)</span><div class='page_container' data-page=110></div>
<span class='text_page_counter'>(111)</span><div class='page_container' data-page=111></div>
<span class='text_page_counter'>(112)</span><div class='page_container' data-page=112></div>
<span class='text_page_counter'>(113)</span><div class='page_container' data-page=113></div>
<span class='text_page_counter'>(114)</span><div class='page_container' data-page=114></div>
<span class='text_page_counter'>(115)</span><div class='page_container' data-page=115></div>
<span class='text_page_counter'>(116)</span><div class='page_container' data-page=116></div>
<span class='text_page_counter'>(117)</span><div class='page_container' data-page=117></div>
<span class='text_page_counter'>(118)</span><div class='page_container' data-page=118></div>
<span class='text_page_counter'>(119)</span><div class='page_container' data-page=119></div>
<span class='text_page_counter'>(120)</span><div class='page_container' data-page=120></div>
<span class='text_page_counter'>(121)</span><div class='page_container' data-page=121></div>
<span class='text_page_counter'>(122)</span><div class='page_container' data-page=122></div>
<span class='text_page_counter'>(123)</span><div class='page_container' data-page=123></div>
<span class='text_page_counter'>(124)</span><div class='page_container' data-page=124></div>
<span class='text_page_counter'>(125)</span><div class='page_container' data-page=125></div>
<span class='text_page_counter'>(126)</span><div class='page_container' data-page=126></div>
<span class='text_page_counter'>(127)</span><div class='page_container' data-page=127></div>
<span class='text_page_counter'>(128)</span><div class='page_container' data-page=128></div>
<span class='text_page_counter'>(129)</span><div class='page_container' data-page=129></div>
<span class='text_page_counter'>(130)</span><div class='page_container' data-page=130></div>
<span class='text_page_counter'>(131)</span><div class='page_container' data-page=131></div>
<span class='text_page_counter'>(132)</span><div class='page_container' data-page=132></div>
<span class='text_page_counter'>(133)</span><div class='page_container' data-page=133></div>
<span class='text_page_counter'>(134)</span><div class='page_container' data-page=134></div>
<span class='text_page_counter'>(135)</span><div class='page_container' data-page=135></div>
<span class='text_page_counter'>(136)</span><div class='page_container' data-page=136></div>
<span class='text_page_counter'>(137)</span><div class='page_container' data-page=137></div>
<span class='text_page_counter'>(138)</span><div class='page_container' data-page=138></div>
<span class='text_page_counter'>(139)</span><div class='page_container' data-page=139></div>
<span class='text_page_counter'>(140)</span><div class='page_container' data-page=140></div>
<span class='text_page_counter'>(141)</span><div class='page_container' data-page=141>

<b>Epsilon</b>



<b>staff</b>



<b>LỜI GIẢI VÀ BÌNH LUẬN ĐỀ THI</b>


<b>CHỌN ĐỘI TUYỂN IMO 2017</b>



<b>Trần Nam Dũng – Võ Quốc Bá Cẩn – Trần Quang Hùng</b>
<b>Lê Phúc Lữ - Nguyễn Tất Thu</b>


1. Lời nói đầu




Dù Epsilon đã nói lời tạm biệt với bạn đọc từ ngày 13/2/2017 nhưng tinh thần Epsilon và đội ngũ
Epsilon thì vẫn cịn. Và có nghĩa là những sản phẩm mang tinh thần Epsilon vẫn sẽ còn được ra
đời. Tinh thần đó ngắn gọn là:<i>Chuyên nghiệp – Từ cộng đồng – Vì cộng đồng.</i>


Minh chứng cho tinh thần đó là tài liệu mà các bạn đang đọc“<i>Giải và bình luận đề thi chọn</i>
<i>đội tuyển Việt Nam dự thi Toán Quốc tế 2017</i>”, một đóng góp của đội ngũ Epsilon dành cho
cộng đồng. Khi viết đội ngũ Epsilon, chúng tôi không chỉ muốn nhắc đến các người lính ngự
lâm thuộc Ban biên tập (Epsilon staff) mà cịn là những người đã ln sát cánh cùng chúng
tôi trong suốt hơn 2 năm qua trong quá trình xây dựng Epsilon thành một niềm yêu mến và
sự chờ đợi của cộng đồng.


Giải và bình luận đề thi, chúng tôi không chỉ muốn đem lại cho độc giả lời giải, đáp án để so
khớp đúng sai mà hơn thế là những phân tích về hướng tiếp cận, về nguồn gốc, về lớp các bài
toán tương tự. Chúng tơi cũng mạn phép đưa ra những bình luận chủ quan của mình về cái hay,
cái dở, độ khó dễ, tính phù hợp, độ mới cũ của bài tốn ngõ hầu giúp cho các thầy cô trong ban
ra đề có thêm những ý kiến phản biện, để cơng tác đề thi ngày càng tốt hơn, chất lượng hơn.
Hy vọng tập tài liệu này sẽ nhận được sự đón nhận của cộng đồng. Chúng tôi luôn lắng nghe
những ý kiến đóng góp, trao đổi thẳng thắn của bạn đọc về nội dung tài liệu cũng như các vấn đề
liên quan. Chúng ta là một cộng đồng.


“<i>If you want to go far, go together.</i>”


2. Thông tin bản quyền



Bản quyền thuộc về tất cả các thành viên trong nhóm biên soạn (Trần Nam Dũng, Võ Quốc Bá
Cẩn, Trần Quang Hùng, Lê Phúc Lữ, Nguyễn Tất Thu).


Đây là thành quả của quá trình lao động miệt mài của nhóm để chia sẻ đến cộng đồng. Mọi người
đều có thể xem tài liệu MIỄN PHÍ. Tuy nhiên, vui lịng ghi rõ nguồn khi chia sẻ.



Tất cả các hoạt động mua bán, kinh doanh liên quan đến tài liệu này mà không được sự chấp
thuận của nhóm là trái pháp luật. Chúng ta hãy lên án những hành vi vi phạm bản quyền để bảo
vệ quyền lợi của các tác giả, của những sản phẩm trí tuệ. Xin cảm ơn.


</div>
<span class='text_page_counter'>(142)</span><div class='page_container' data-page=142>

<b>Epsilon</b>



<b>staff</b>



<b>2</b> <i>Lời giải và bình luận đề thi chọn đội tuyển IMO 2017</i>


3. Đề thi



3.1. Ngày thi thứ nhất (25/03/2017)



Bài 1 (7.0 điểm). Cho44cái lỗ phân biệt trên một cái rãnh là đường thẳng và2017con kiến.
Mỗi con kiến sẽ chui lên từ một cái lỗ và bò đến một cái lỗ khác với vận tốc khơng đổi rồi chui
xuống đó. GọiT là tập các thời điểm mà con kiến chui lên hoặc chui xuống các cái lỗ. Biết rằng
vận tốc của các con kiến đôi một khác nhau vàjTj 45:Chứng minh rằng tồn tại ít nhất hai
con kiến nào đó khơng gặp nhau. (Quy ước hai con kiến gặp nhau khi và chỉ khi tồn tại một thời
điểm mà cả hai ở cùng một vi trí trên rãnh kể cả lúc chui lên và xuống.)


Bài 2 (7.0 điểm). Với mỗi số nguyên dươngn, đặtxn DC2nn :


a) Chứng minh rằng nếu 2 0 1 7<sub>2</sub> k < n < 2 0 1 7k với k là số ngun dương nào đó thì xn


là bội của2 0 1 7:


b) Tìm tất cả số nguyên dương h > 1để tồn tại các số nguyên dươngN ; T sao cho với mọi



n > N thìxn là dãy số tuần hoàn theo modulohvới chu kỳT :


Bài 3 (7.0 điểm). Cho tam giácA B C ngoại tiếp đường tròn. I /và . I /tiếp xúc với các cạnh


B C ; C A ; A B lần lượt tại D ; E ; F : GọiIb; Ic lần lượt là các tâm đường trịn bàng tiếp góc


B ; C của tam giác A B C :GọiP ; Qlần lượt là trung điểm của IbE ; IcF : Giả sử . P A C /


cắtA B tại điểm thứ haiR và . QA B / cắtA C tại điểm thứ hai S :


a) Chứng minh rằngP R ; QS ; A I đồng quy.


b) Giả sử DE ; DF lần lượt cắt IbIc tại K ; J : E J cắt F K tại M và P E ; QF cắt


. P A C / ; . QA B /lần lượt tại các điểm thứ hai X ; Y :Chứng minh rằng ba đường thẳng


B Y ; C X ; A M đồng quy.


3.2. Ngày thi thứ hai (26/03/2017)



Bài 4 (7.0 điểm). Cho tam giácA B C nội tiếp đường tròn. O / : Điểm Adi động trên . O /


sao cho A B > B C và M là trung điểm của A C :Đường trịn đường kính B M cắt . O / tại


R :Giả sử R M cắt . O / tại điểm thứ hai Q ;cắt B C tại P :Đường trịn đường kínhBP cắt


A B ; B O lần lượt tại các điểm thứ hai K ; S :


a) Chứng minh rằngSRđi qua trung điểm củaKP :



b) Gọi N là trung điểm của B C : Trục đẳng phương của hai đường trịn đường kính


A N ; B M cắtSR tạiE :Chứng minh rằngM E luôn đi qua một điểm cố định.


Bài 5 (7.0 điểm). Cho 2 0 1 7 số thực dươnga1; a2; : : : ; a2 0 1 7: Với mỗin > 2 0 1 7; ta đặt


an D max˚ai1ai2ai3 ji1 C i2 C i3 D n ; 1 i1 i2 i3 n 1 :


Chứng minh rằng tồn tại số nguyên dương m không vượt quá 2 0 1 7 và số nguyên dương


</div>
<span class='text_page_counter'>(143)</span><div class='page_container' data-page=143>

<b>Epsilon</b>



<b>staff</b>



<i>Lời giải và bình luận đề thi chọn đội tuyển IMO 2017</i> <b>3</b>


Bài 6 (7.0 điểm). Với mỗi số nguyên dương n, xét a1; a2; : : : ; a2 n là hoán vị của 2 nsố


nguyên dương đầu tiên. Một hoán vị như thế được gọi là “<i>đẹp</i>” nếu với mọi1 i < j 2 n


thìai C anCi D 2 nC1 vàai aiC1không đồng dư vớiaj ajC1 theo modulo 2 nC1.


(Quy ước a2 nC1 D a1:)


a) Với n D 6, hãy chỉ ra một hoán vị đẹp.


b) Chứng minh rằng với mỗin ngun dương thì ln tồn tại một hốn vị đẹp.


4. Bình luận chung




Trong hai ngày 25, 26/3/2017 đã diễn ra kỳ thi chọn đội tuyển Việt Nam dự thi Toán Quốc tế năm
2017 với sự tham dự của 49 học sinh xuất sắc nhất đến từ các tỉnh thành và các trường chuyên
thuộc các trường đại học lớn trong cả nước. Mỗi ngày thi, các thí sinh phải giải quyết 3 bài toán
trong thời gian 240 phút.


Đề thi năm nay được đánh giá là khá khó và có phong cách gần với đề thi IMO hơn các năm
trước. Cấu trúc đề thi bao gồm 2 bài hình học, 1 bài đại số, 1 bài tổ hợp, 1 bài số học và 1 bài tổ
hợp số học. Như vậy đề chọn đội tuyển năm nay cũng theo xu hướng của IMO là thiên về tổ hợp
nhiều. Ngay cả bài 5 (bài đại số) cũng có cách phát biểu (và cả cách giải) có nhiều nét tổ hợp.
Các bài toán 2 và 4 theo đánh giá chung được coi là bài dễ nhất của mỗi ngày. Bài hình câu 4
phần a) cũng chỉ cần các kiến thức chương trình cấp 2, phần b) vận dụng các kiến thức trục
đẳng phương, ở ý này điểm cố định có đặc biệt hơn so với các bài thông thường trước đây.
Bài số 2 là một bài toán số học thuần tuý, khai thác một chủ đề khá quen thuộc đối với học
sinh Việt Nam là số dư trong phép chia hệ sốC<sub>n</sub>k cho số nguyên tốp với các định lý Lucas,
Wolstenhome, Babbage hay cơng thức Legendre, vì vậy, dù ở các mức độ khác nhau, các thí
sinh đều có thể tiếp cận được bài này.


Đáng chú ý là bài số 1, bài được ban đề thi đánh giá là dễ nhất của ngày thứ nhất lại gây nhiều
khó khăn cho các thí sinh. Các em đã không thể phát biểu lại bài tốn một cách rành mạch bằng
ngơn ngữ Tốn học nên đã khơng thể xử lý được, khơng những thế cịn để tốn nhiều thời gian
vào bài toán này. Nếu biết cách chuyển đổi bài tốn (mơ hình hố) thành đồ thị (theo nghĩa đồ
thị) hoặc đồ thị (theo nghĩa các đường biểu diễn đường đi con kiến) thì bài tốn có thể giải quyết
một cách khá đơn giản, chẳng hạn bằng quy nạp.


Các bài tốn cịn lại gồm bài số 3, 5 và 6 được đánh giá là khó. Bài hình số 3 phần a) sử dụng
kiến thức tam giác đồng dạng và tứ giác nội tiếp của chương trình cấp 2, 3 ý khó tập trung ở câu
b) địi hỏi vận dụng nhiều kiến thức tổng hợp có liên quan nhiều đến bài hình G7 trong IMO
shortlist 2002 trong đó có sử dụng cả kiến thức về hàng điều hịa. Cấu trúc là ghép nối các bài
tốn riêng lẻ thành một bài tổng hợp. Bài số 5 là một bài tốn có cách phát biểu rất tổ hợp, dạng
dãy số truy hồi với các số hạng đầu tiên bất kỳ. Để tiếp cận được bài này, học sinh phải tỉnh táo


nhận ra rằng hằng số 2017 trong đề bài có thể thay bằng một số nguyên dương bất kỳ và bắt đầu
làm thử với các trường hợp tham số nhỏ để dự đoán quy luật của dãy số. Cũng có thể thấy rằng
bằng cách logarith hố, các phép nhân trong bài toán này sẽ biến thành phép cộng và bài tốn có
thể đưa về một biến thể của bài toán số 6 trong đề thi IMO 2010. Điều này một lần nữa nhấn
mạnh độ khó của bài số 5, đồng thời cũng là một điểm chưa hay của đề thi lần này.


Bài toán số 6 là một bài toán số học-tổ hợp với yêu cầu xây dựng một hoán vị của<sub>f</sub>1 ; 2 ; : : : ; 2 ng


</div>
<span class='text_page_counter'>(144)</span><div class='page_container' data-page=144>

<b>Epsilon</b>



<b>staff</b>



<b>4</b> <i>Lời giải và bình luận đề thi chọn đội tuyển IMO 2017</i>


bằng phép thử sai, có thể xây dựng ví dụ tổng quát cho một số trường hợp. Chẳng hạn nếu2 nC1


có căn ngun thuỷ thì đặtai D ˛i .mod 2 nC1 /thì rõ ràng các hiệuaiC1 ai ˛i. ˛ 1 /


phân biệt modulo2 nC 1 vàai C anCi ˛i. 1 C ˛n/ 0 .mod 2 nC 1 / :Trong trường


hợp tổng quát, ý tưởng chính là xây dựng n số hạng đầu tiên của dãy số, còn đoạn sau lấy phần
bù. Tuy nhiên, cần có điều kiện bổ sung đối với anvà a1; vìanC1 D 2 nC 1 a1:Và đây


cũng chính là điểm mà các thí sinh có thể phạm sai lầm do ngộ nhận.


Với những nhận định trên đây cũng như tham khảo một số thông tin về bài làm của các thí sinh,
chúng tơi cho rằng bên cạnh 2 bài toán 2 và 4 thuộc dạng bài “cần phải làm được nếu muốn đậu
đội tuyển” thì chiếc vé dự IMO sẽ được quyết định ở 4 bài còn lại ở các bài 1, 3, 5, 6 với tình
trạng rất khó đốn. Khác với một số năm trước khi có những bài tốn q khó khơng ai làm
được, đề thi năm nay độ khó được chia đều, khó dễ thuỳ theo sở trường của từng em. Thực tế là


hầu như bài nào cũng có học sinh giải được tạo thành các thế “cài răng lược”, ví dụ có học sinh
làm được bài 1, 6 lại khơng làm được 3, 5, có học sinh làm được bài 5 lại không làm được bài 1.
Ngoài ra các bài 1, 5, 6 đều là các bài có thể có lỗi trong trình bày, thậm chí có những sai sót
lớn do ngộ nhận. Với đề thi như thế này, nếu làm chắc chắn 4 bài thì gần như sẽ có vé, cịn điểm
chuẩn vùng tranh chấp sẽ là 3.5++.


5. Lời giải và bình luận các bài toán



Bài 1 (7.0 điểm). Cho 4 4cái lỗ phân biệt trên một cái rãnh là đường thẳng và2 0 1 7 con
kiến. Mỗi con kiến sẽ chui lên từ một cái lỗ và bò đến một cái lỗ khác với vận tốc khơng đổi
rồi chui xuống đó. Gọi T là tập các thời điểm mà con kiến chui lên hoặc chui xuống các cái
lỗ. Biết rằng vận tốc của các con kiến đôi một khác nhau vàjTj 4 5 :Chứng minh rằng
tồn tại ít nhất hai con kiến nào đó khơng gặp nhau. (Quy ước hai con kiến gặp nhau khi và chỉ
khi tồn tại một thời điểm mà cả hai ở cùng một vi trí trên rãnh kể cả lúc chui lên và xuống.)


Lời giải. Ta xét trục toạ độO x y vớiO x là trục đánh dấu vị trí của các lỗ trên rãnh thẳng cịn


O y là trục thời gian. Các lỗ có hồnh độ là x1; x2; : : : ; x4 4 và các thời điểm mà các con


kiến chui lên hoặc chui xuống lày1; y2; : : : ; y4 5:Một con kiến sẽ đi từ điểm . xa; yb/đến


điểm. xc; yd/tương ứng với tình huống nó chui lên từ lỗ xa vào thời điểm yb và đến lỗxc vào


thời điểmyd: Do kiến bị với vận tốc khơng đổi nên đồ thị biểu diễn đường đi theo thời gian


chính là đoạn thẳng nối hai điểm này. Có tất cả 2 0 1 7đoạn thẳng như vậy. Vì vận tốc các con
kiến đơi một khác nhau nên các đoạn thẳng đều có phương khác nhau. Ta cần chứng minh có ít
nhất hai đoạn thẳng trong số này không cắt nhau.


Chú ý là số điểm là đầu mút của các đoạn thẳng là4 4 4 5 D 1 9 8 0 < 2 0 1 7nên bài tốn sẽ


được giải quyết hồn tồn nếu ta chứng minh được mệnh đề sau:<i>Nếu có</i>n <i>điểm trên mặt phẳng</i>
<i>thì khơng tạo được q</i>n<i>đoạn thẳng với các đỉnh là các điểm đó sao cho đơi một có điểm chung</i>
<i>và khơng có hai đoạn nào song song hay đè lên nhau.</i>


Ta chứng minh bằng quy nạp theo n :Với n D 2 ; 3mệnh đề là hiển nhiên. Giả sử mệnh đề đã
đúng vớin 1điểm. Xétn điểm:


</div>
<span class='text_page_counter'>(145)</span><div class='page_container' data-page=145>

<b>Epsilon</b>



<b>staff</b>



<i>Lời giải và bình luận đề thi chọn đội tuyển IMO 2017</i> <b>5</b>


Giả sử bây giờ đỉnh nào cũng có ít nhất hai đoạn xuất phát. Nếu có đỉnh Acó ba đoạn xuất
phát thì sẽ “có vấn đề”: chẳng hạn làA B ; A C ; A D:Nếu B C Dchứa Abên trong thì
đoạn thứ hai từB khơng thể cùng cắtA C vàA D: NếuB C D không chứaA bên trong
thì ta có chẳng hạn tiaA C nằm giữa hai tia A B vàA D: Khi đó đoạn thứ hai từ đỉnhC


không thể cùng cắt A B và A D: Như vậy từ mỗi đỉnh chỉ có đúng hai đoạn, suy ra số
đoạn bằng đúng số đỉnh trong trường hợp này.


Bài tốn được giải quyết hồn tồn.


Bình luận.


Đây là một bài tốn có cách phát biểu rất thú vị có lời giải khơng phức tạp nhưng thực ra
rất rối nếu khơng nhìn thấy được đúng bản chất và chuyển đổi được về mơ hình điểm, đoạn
thẳng như ở trên, với hai chú ý quan trọng: các con kiến bị với vận tốc khơng đổi (các
đường biểu diễn quỹ đạo con kiến là các đoạn thẳng) và các vận tốc đôi một khác nhau
(các đoạn thẳng không cùng phương). Trên thực tế, rất nhiều bạn đã tốn nhiều thời gian


cho bài tốn này nhưng vẫn khơng tiếp cận được lời giải. Và nếu trong lời giải, hai điều
kiện quan trọng này khơng được khai thác thì chắc chắn lời giải là khơng hồn chỉnh.


Lời giải trình bày trên đây là của GS Nguyễn Tiến Dũng, HCV IMO 1985 được chúng tơi
trình bày lại cho giống một lời giải. Còn đây là phần dẫn nhập và một cách diễn đạt lời giải
khác của GS Nguyễn Tiến Dũng:


<i>Có gì liên quan giữa các số</i>4 4 ; 4 5<i>và</i>2 0 1 7<i>? Dễ thấy</i> 4 4 4 5 D 1 9 8 0 < 2 0 1 7;<i>nên</i>
<i>có thể đoán đây là mấu chốt bài toán?</i>


<i>Vậy tại sao lại dùng tích? Bởi vì có nhiều nhất là từng đó điểm</i> .<i>vị trí, thời gian chạm</i>
<i>lỗ khi lên hoặc xuống lỗ</i>/<i>trên mặt phẳng toạ độ. Ta vẽ trên mặt phẳng toạ độ các đồ thị</i>
<i>đường đi của các con kiến, mỗi đồ thị là một đoạn thẳng nối hai trong số các điểm trên.</i>
<i>Tất cả các đoạn thẳng đó đều có hướng khác nhau</i>.<i>vì vận tốc các con kiến khác nhau</i>/ :
<i>Hướng ở đây hiểu là góc so với đường nằm ngang modulo</i> :


<i>Câu hỏi đặt ra bây giờ là nếu hai đoạn một đều có điểm chung (hai con kiến nào cũng có</i>
<i>gặp nhau) thì có nhiều nhất là bao nhiêu đoạn thẳng?</i>


<i>Ta có thể sắp xếp thứ tự các đoạn thẳng theo thứ tự vịng trịn theo góc của chúng.</i>
<i>Cố định một đoạn đầu tiên, từ điểm</i> A1 <i>đến điểm</i> A2: <i>Đoạn thứ hai có hướng quay về</i>
<i>“bên phải” so với đoạn thứ nhất, nên phải có thêm ít nhất một điểm mới</i> A3 .<i>tức là hoặc</i>
<i>là</i> A1A3 <i>với</i> A3 <i>nằm bên phải</i>A1A2; <i>hoặc</i> A3A2 <i>với</i> A3 <i>nằm bên trái</i> A1A2; <i>hoặc</i>
A3A4 .<i>hai điểm mới</i>/<i>nằm ở hai bên của</i> A1A2/ : <i>Thêm đoạn thứ ba phải thêm ít nhất</i>
<i>một điểm mới, trừ trường hợp tạo thành ba đoạn</i> A1A2; A1A3; A2A3: <i>Cứ như thế:</i>
<i>thêm một đoạn thì cần thêm ít nhất một điểm mới, trừ khi thêm đoạn cuối cùng thì dùng</i>
<i>được hai điểm cũ. Như vậy để có</i>n<i>đoạn đơi một có điểm chung thì cần ít nhất</i> n<i>điểm. Với</i>
n D 4 4<i>lần</i>4 5<i>thì có nhiều nhất là từng đó con kiến đơi một có gặp nhau. Vì</i>2 0 1 7 <i>lớn</i>
<i>hơn</i>4 4<i>lần</i> 4 5<i>nên có hai con kiến không gặp nhau.</i>



</div>
<span class='text_page_counter'>(146)</span><div class='page_container' data-page=146>

<b>Epsilon</b>



<b>staff</b>



<b>6</b> <i>Lời giải và bình luận đề thi chọn đội tuyển IMO 2017</i>


các bài toán đầu cuối, tức là các bài toán đã được phát biểu rành mạch bằng ngơn ngữ Tốn
học mà lại ít chú trọng đến các bài tốn địi hỏi khá năng đọc hiểu, phát hiện bản chất vấn
đề và phát biểu lại bằng mơ hình Tốn học.


Chúng tơi cung cấp cho bạn đọc một số bài toán mà lời giải địi hỏi bước đọc hiểu, phát
biểu lại bằng mơ hình Tốn học để tham khảo:


1. (IMO Shortlist, 2010)Trong một buổi biểu diễn ca nhạc, có2 0ca sĩ sẽ trình diễn.
Với mỗi ca sĩ có một danh sách (có thể là rỗng) các ca sĩ khác mà anh ấy hoặc cô ấy
muốn diễn sau tất cả các ca sĩ thuộc danh sách đó. Có thể xảy ra là có đúng2 0 1 0


cách sắp xếp để tất cả các mong muốn của các ca sĩ đều được thoả mãn?


2. (Tournament of the towns, 1992-1993)Trong từ điển thực vật mỗi một loài cây được
đặc trưng bởi1 0 0 dấu hiệu (mỗi một dấu hiệu có thể có, có thể khơng có). Hai lồi
cây được coi là khơng giống nhau nếu chúng khác nhau ở ít nhất5 1dấu hiệu.


a) Chứng minh rằng trong từ điển có khơng q5 0lồi cây đơi một khơng giống nhau.


b) Có thể có đúng 5 0lồi cây đôi một không giống nhau không?


3. (Moscow MO, 2010)Trên mặt phẳng ta đánh dấu4 nđiểm, sau đó ta nối tất cả các
cặp điểm có khoảng cách đúng bằng1cm. Người ta thấy răng trongn C 1điểm bất
kỳ ln có hai điểm được nối. Chứng minh rằng có ít nhất7 nđược kẻ.



4. Ban đầu có một số lẻ viên sỏi. Ở bước tiếp theo, ta chia số sỏi làm hai, một nửa để
xuống phía dưới, bên trái, một nửa để xuống phía dưới, bên phải, còn dư một viên để
ngay xuống dưới vị trí đống sỏi ban đầu. Ở mỗi lượt tiếp theo, ta lại làm như vậy với
các đống sỏi. Như vậy ở một số vị trí ở bước tiếp theo sẽ có thể được nhận sỏi từ ba
đống sỏi trên nó: ở ngay phía trên, phía trên bên trái và phía trên bên phải.


Ví dụ, nếu bắt đầu bằng đống sỏi2 1viên thì ở lượt chia đầu tiên ta sẽ có ba đống sỏi


1 0 1 1 0 :


Ở lượt chia thứ hai ta sẽ có


5 0 1 1 0 5 :


Và ở lượt chia thứ ba sẽ là


2 1 7 1 7 1 2 :


Chứng minh rằng nếu cứ tiếp tục chia như thế thì bắt đầu từ một đống sỏi có số lẻ
viên, sẽ đến một lúc ta đi đến trạng thái có các đống sỏi một viên liền nhau.


Bài 2 (7.0 điểm). Với mỗi số nguyên dương n, đặtxn D C2 nn :


a) Chứng minh rằng nếu 2 0 1 7<sub>2</sub> k < n < 2 0 1 7k với k là số nguyên dương nào đó thì


xn là bội của 2 0 1 7:


</div>
<span class='text_page_counter'>(147)</span><div class='page_container' data-page=147>

<b>Epsilon</b>




<b>staff</b>



<i>Lời giải và bình luận đề thi chọn đội tuyển IMO 2017</i> <b>7</b>


Lời giải. a)Ta chứng minh kết luận bài toán đúng cho mọi số nguyên tốp lẻ thay cho số2 0 1 7:


Giả sử tồn tại số nguyên dươngk sao cho p<sub>2</sub>k < n < pk:Ta có


vp . xn/ D vp C2 nn




D vp . 2 n / Š




2 vp . n Š / :


Do p<sub>2</sub>k < n < pk nênpk < 2 n < 2 pk < pkC1;từ đó suy ra


vp . 2 n Š /



D

2 n
p

C


2 n
p2


C C




2 n
pk




:


Mặt khác, với mọi x 2 R;ta có b2 xc 2bxc;đẳng thức xảy ra khi fxg < 12:Kết hợp với


giả thiết p<sub>2</sub>k < n < pk;ta có


vp . 2 n / Š




> 2 n
p

C

n
p2




C C




n


pk D 2 vp . n Š / ;


hay vp . xn/ > 0nên ta cóxn::: p :


b)Ở ý này, ta có hai cách tiếp cận sau.


Cách 1. (<i>Dựa theo lời giải của Hà Duy Hưng</i>)Giả sửh > 1là số thoả mãn yêu cầu bài toán.
Với mỗip nguyên tố lẻ màp j h, ta có dãy số dư xn theo modulop cũng tuần hoàn. Sử dụng


kết quả câu a) thì với p<sub>2</sub>k < n < pk thì


xn 0 .mod p / :


Chọnk đủ lớn để p<sub>2</sub>k > T C 1, ta suy ra tất cả các số dư củaxn cho p đều bằng 0với mọi


n n0, trong đón0 2 ZC đủ lớn. Tuy nhiên chọnt 2 ZC đủ lớn để pt 1 > 2 n0 và đặt


n D pt2 1 ta có ngayvp. xn/ D 0, do đóxnkhơng chia hết cho p ; vơ lý.


Vậy h chỉ có ước nguyên tố là 2 hay h D 2k với k nguyên dương. Nếu k > 1 ; ta chọn


r D k 1 và xét sốn có dạngn D 2a1 <sub>C C</sub><sub>2</sub>ar<sub>;</sub><sub>trong đó</sub><sub>a</sub>



1 > maxfT ; Ng trong đó


T ; N là các hằng số trong giả thiết củah :Khi đó


v2. xn/ D 2 S2. n / S2. 2 n / D r;


trong đóS2. x /là tổng các chữ số trong biểu diễn nhị phân. Thành thửxn 2k 1 .mod h /.


Tuy nhiên, với mọi i 2 ZC mà i < 2a1 thì số chữ số 1 trong biểu diễn nhị phân của n C i


tăng thêm ít nhất 1 đơn vị, do đó mà xnCi 0 .mod h /. Do a1 > maxfT ; Ng nên


xn xnCT 0 .mod h /, mâu thuẫn.


Vậy k D 1 do đó h D 2. Đây là đáp số bài tốn, vì dễ thấy rằng xn là số chẵn với mọi số


nguyên dươngn ; như sau: nếun D 2a1 <sub>C C</sub> <sub>2</sub>ar <sub>với</sub><sub>0</sub> <sub></sub> <sub>a</sub>


1 < < ar thì


2 n D 2a1C1 <sub>C C</sub> <sub>2</sub>arC1


nên


v2. xn/ D 2 Sn. n / S2. 2 n / D r 1 :


</div>
<span class='text_page_counter'>(148)</span><div class='page_container' data-page=148>

<b>Epsilon</b>



<b>staff</b>




<b>8</b> <i>Lời giải và bình luận đề thi chọn đội tuyển IMO 2017</i>


Cách 2.(<i>Theo lời giải của Nguyễn Song Minh</i>)Doh > 1 ; nên giả sử p là ước nguyên tố
lớn nhất củah : Giả sử T D pmT0 là chu kỳ nhỏ nhất ứng với n đủ lớn, với . T0; p / D 1 :


Theo định lý Lucas, ta có


C<sub>2 n</sub>n C<sub>2 p</sub>pmmn<sub>n</sub> C


pm<sub>n</sub><sub>C</sub><sub>T</sub>


2 pm<sub>n</sub><sub>C</sub><sub>2 T</sub> C


. nCT0<sub>/ p</sub>m


. 2 nC2 T0/ pm C


nCT0


2 nC2 T0 .mod p / :


Suy ra m D 0 :Nếup lẻ và giả sửT D . a1a2 ar/p thì với R > r đủ lớn, ta có


C<sub>2</sub>1 C<sub>2 p</sub>pRR C


pR<sub>C</sub><sub>T</sub>


2 pR<sub>C</sub><sub>2 T</sub> C



1
2: C


T


2 T .mod p / :


Suy ra CT


2 T 1 .mod p /, từ đây ta có vớiK đủ lớn thì


C<sub>2 K T</sub>K T 1 .mod p / :


Mặt khác do . T ; p / D 1nên tồn tại số nguyên dương K đủ lớn sao chop j . K T C 1 /. Kết
hợp với. K T C 1 / j C<sub>2 K T</sub>K T ;ta thu được 1 0 .modp / điều này vơ lý, do đóh D 2k:


VìT lẻ và theo lý luận trên, nếu ta chọn K sao choh j . nC K T C1 / thì


C<sub>2 n</sub>n C<sub>2 n</sub>nC<sub>C</sub>K T<sub>2 K T</sub> 0 .mod h / ;


với nđủ lớn. Ta chọnn D 2N, ta có số các số lẻ trong dãyC<sub>n</sub>k . 0 k n /bằng2 nên


C<sub>2 n</sub>n 2 .mod 4 / ;


do đó h D 2 : Hơn nữa C<sub>2 n</sub>n luôn là số chẵn với mỗi số nguyên dươngn :Vậyh D 2 :


Bình luận.


Một lần nữa chủ đề về tính chất số học của hệ số nhị thứcCnk lại được khai thác. Đó là



điều có thể dự đoán được trong các năm nguyên tố (như 2003 hay trước đó nhiều năm là
1979) nhưng khơng ngờ năm nay điều này lại được ưu ái đến hai lần (VMO bài số 6 và
bài số 2 của kỳ TST này). Vì vậy, với các cơng cụ được trang bị đầy đủ như công thức
Legendre, định lý Lucas, Lagrange, Babbage hay Wolstenhome thì bài tốn này, đặc biệt là
câu a) sẽ khơng gây khó khăn cho các bạn học sinh.


Nhắc lại ở đây định lý Babbage là đồng nhất thức C<sub>bp</sub>ap C<sub>b</sub>a .mod p2/ ; đúng cho
mọi số nguyên tố p ; còn định lý Wolstenhome ở dạng hệ số nhị thức là đồng nhất thức


C<sub>bp</sub>ap C<sub>b</sub>a .mod p3/chỉ đúng cho số nguyên tốp > 3 :


Sự kiệnC<sub>2 n</sub>n ln chẵn là một tính chất quen thuộc và có rất nhiều cách chứng minh. Hơn
thế ta có số mũ củantrong C2 nn bằng số số1trong biểu diễn nhị phân của n ;cho nên với


h D 2s thì chỉ có h D 2là thoả mãn điều kiện.


Chúng tơi dẫn một số bài toán thi Olympic những năm gần đây có sử dụng nhóm định lý này.


1. (Vietnam TST, 2010)Gọi Sn là tổng bình phương các hệ số sau khai triển của


</div>
<span class='text_page_counter'>(149)</span><div class='page_container' data-page=149>

<b>Epsilon</b>



<b>staff</b>



<i>Lời giải và bình luận đề thi chọn đội tuyển IMO 2017</i> <b>9</b>


2. (Saudi Arabia TST, 2015)Với mỗi số nguyên dươngn ; đặt


S . n / D



n


X


kD0


rk;


trong đórk là số dư trong phép chiaCnk cho 3 :Tìm tất cả các số nguyên dương


nsao cho S . n / n :


3. (Trường Xuân Toán học miền Nam, 2017)Cho dãy số. xn/xác định bởi công thức:


x0 D 1 ; xn D 2 0 1 7




x0 C x1 C Cxn 1


n




với 1 n 2 0 1 7:


Hỏi có bao nhiêu chỉ sối mà0 i 2 0 1 7 sao cho<sub>j</sub>xijchia hết cho 3?


Bài 3 (7.0 điểm). Cho tam giácA B C ngoại tiếp đường tròn . I /và. I / tiếp xúc với các
cạnhB C ; C A ; A B lần lượt tạiD ; E ; F :GọiIb; Ic lần lượt là các tâm đường trịn bàng



tiếp gócB ; C của tam giácA B C :Gọi P ; Qlần lượt là trung điểm củaIbE ; IcF : Giả


sử . P A C /cắt A B tại điểm thứ hai R và . QA B /cắt A C tại điểm thứ haiS :
a) Chứng minh rằngP R ; QS ; A I đồng quy.


b) Giả sử DE ; DF lần lượt cắt IbIc tại K ; J : E J cắt F K tại M vàP E ; QF cắt
. P A C / ; . QA B / lần lượt tại các điểm thứ hai X ; Y : Chứng minh rằng ba đường
thẳngB Y ; C X ; A M đồng quy.


Lời giải. Ta sẽ dùng các bổ đề sau để giải bài toán này.


Bổ đề 1. <i>Cho tam giác</i>A B C <i>có đường trịn nội tiếp</i>. I /<i>tiếp xúc với các cạnh</i>B C ; C A ; A B
<i>lần lượt tại</i> D ; E ; F :<i>Gọi</i> M ; N <i>lần lượt là trung điểm của</i>DE ; DF :<i>Gọi</i> P <i>là giao điểm</i>
<i>của các đường trung trực của</i>M C <i>và</i> N B :


a) <i>Chứng minh rằng đường tròn</i>. P B C / <i>tiếp xúc với đường tròn</i>. I /<i>.</i>


b) <i>Gọi</i>J <i>là điểm đối xứng với</i>D <i>qua</i> P :<i>Chứng minh rằng</i> J <i>là tâm đường trịn bàng tiếp</i>
<i>góc</i>A <i>của tam giác</i>A B C<i>.</i>


c) <i>Chứng minh rằng giao điểm của</i> P M <i>và</i> A C <i>nằm trên đường tròn</i> . P B C /<i>.</i>


Chứng minh. a)Gọi G là giao điểm thứ hai củaP D và . I / : Dễ thấy trung trực C M là trục
đẳng phương của đường tròn điểmC và đường tròn. I / ;và trung trựcB N là trục đẳng phương
của đường tròn điểm B và đường trịn . I / : Do đó


P C2 D P D P G D P B2:


Từ đây suy ra <sub>∠</sub>P G C D ∠P C B và <sub>∠</sub>P GB D ∠P B C nên tứ giácP B G C nội tiếp đường


trịn . K /. Lại có I D k KP và <sub>KP</sub>I D <sub>D</sub> <sub>K G</sub>I G nên ba điểm G ; I ; K thẳng hàng. Do đó, . K /


</div>
<span class='text_page_counter'>(150)</span><div class='page_container' data-page=150>

<i><b>Epsilon</b></i>



<i><b>staff</b></i>



<b>10</b> <i>Lời giải và bình luận đề thi chọn đội tuyển IMO 2017</i>


<i>A</i>


<i>B</i> <i><sub>C</sub></i>


<i>I</i>


<i>K</i>
<i>F</i>


<i>E</i>


<i>D</i>
<i>R</i>


<i>P</i>
<i>N</i>


<i>Q</i>


<i>M</i>
<i>G</i>



<i>J</i>


b)Do GB là đường đối trung của tam giácGF D nên


∠F G N D ∠B GD D ∠P B C




∠GF N D ∠GD C D ∠P DB :


Từ đó suy ra hai tam giác GF N và BDP đồng dạng g-g. Lại có N ; P là trung điểm của


DF ; D J nên hai tam giácGF D và BD J đồng dạng c-g-c. Từ đó suy ra


∠DB J D ∠D GF D ∠BDF


nênB J k DF : VậyB J là phân giác ngồi gócB :


Tương tự, ta cũng có C J là phân giác ngồi gócC nênJ là tâm đường trịn bàng tiếp gócA.


c)Dễ thấy tứ giácB C M N nội tiếp và tâm nội tiếp chính là giao điểmP của trung trực C M ;
B N :Từ đó suy ra


P M2 D P C2 D P D P G :


Suy ra hai tam giác P DM và P M G đồng dạng. Từ đó


∠DM P D ∠D GM D ∠E G C :


Lại có



∠GE C D 1 8 0ı ∠GE A D 1 8 0ı ∠GDE D ∠P DM D ∠P M G :


</div>
<span class='text_page_counter'>(151)</span><div class='page_container' data-page=151>

<b>Epsilon</b>



<b>staff</b>



<i>Lời giải và bình luận đề thi chọn đội tuyển IMO 2017</i> <b>11</b>


Bổ đề 2. <i>Cho tam giác</i>A B C :<i>Đường tròn nội tiếp</i>. I ; r /<i>tiếp xúc với các cạnh</i>B C ; C A ; A B
<i>lần lượt tại</i>D ; E ; F : <i>Một đường tròn qua</i> B ; C <i>và tiếp xúc</i> . I /<i>tại</i>X :<i>Khi đó, ta có</i>


a) XE XF
XD2 D


r2
I B I C :
b) XE


XF D


I B DE2
I C DF2:


Chứng minh. Gọi giao điểm của tiếp tuyến chung của hai đường tròn tiếp xúc tạiX vàB C là


M. GọiG là điểm đối xứng của D quaM thì M G2 <sub>D</sub> <sub>M D</sub>2 <sub>D</sub> <sub>M X</sub>2 <sub>D</sub> <sub>M B</sub> <sub></sub> <sub>M C</sub> <sub>nên</sub>


. B C ; D G / D 1 ;do đóEF đi quaG :Gọi A Llà đường cao của tam giácA B C thì theo
bài tốn quen thuộc thì I G ? A D nên hai tam giácI GD và DA Lđồng dạng.



<i>A</i>
<i>B</i> <i>C</i>
<i>I</i>
<i>F</i>
<i>E</i>
<i>D</i>
<i>K</i>
<i>G</i>
<i>X</i>
<i>M</i> <i>L</i>
<i>Z</i>
<i>H</i>


Lại có DX ? I M và M là trung điểm của GDnên DX đi qua trung điểmA L : GọiDK là
đường kính của . I / thì chùm D . A L ; X K / D 1, chiếu lên . I /dẫn tới tứ giác DK Z X


điều hòa vớiZ là giao điểm củaA D và . I /. Từ đó suy ra tiếp tuyến tạiZ ; D vàX K đồng
quy tạiG. Nói cách khác, X K đi qua G. Gọi H là giao điểm của A D và EF :Ta có


XE
XF


K E
K F D


GE
GF D


H E


H F D


Œ A ED 
Œ A F D  D


DE
DF


DB
I C


I B
D C :


Lại có K E I C D 2 r2 D K F I B nên


XE
XF D


I B DE2
I C DF2:


Mặt khác, ta cũng có


F X
F K


E X
E K D



GX
GK D


GD2
GK2 D


</div>
<span class='text_page_counter'>(152)</span><div class='page_container' data-page=152>

<b>Epsilon</b>



<b>staff</b>



<b>12</b> <i>Lời giải và bình luận đề thi chọn đội tuyển IMO 2017</i>


và K E I C D 2 r2 D K F I B nên


XE XF
XD2 D


r2
I B I C :


Bổ đề được chứng minh.


Từ bổ đề trên, ta suy ra hai hệ quả là


XE2
XD2 D


r2 DE2
I B2 <sub></sub> <sub>DF</sub>2





XF2
XD2 D


r2 DF2
I C2 <sub></sub> <sub>DE</sub>2:


Bổ đề 3. <i>Cho tam giác</i> A B C ; <i>đường tròn nội tiếp</i> . I / <i>tiếp xúc với</i>B C <i>tại</i> D<i>. Dựng một</i>
<i>đường tròn qua</i> A ; B <i>tiếp xúc với</i>. I /<i>tại</i>M <i>và một đường tròn qua</i>A ; C <i>tiếp xúc với</i> . I /<i>tại</i>
N : B M <i>cắt</i>C N <i>tại</i> P :<i>Chứng minh rằng</i>∠P A B D ∠DA C :


Chứng minh. Gọi các tiếp điểm của . I /vớiC A ; A B là E ; F vàX Y Z là tam giác Ceva
củaP. GọiK là giao điểm củaB Y vàDF : Sử dụng bổ đề và hệ quả trên với chú ý rằngM K


là đường đối trung của tam giácM DF ;ta có


Y C
Y A D


Œ Y B C 
Œ Y BA  D


Œ Y B C 
Œ K BD 


Œ K BD 
Œ K BF 


Œ K BF 


Œ Y BA 
D B C B Y


BD BK
K D
K F


BF BK
B Y BA
D B C


BA


M D2
M F2 D


B C
BA


r2 DF2
EF2 <sub></sub> <sub>I A</sub>2:


Tương tự, ta cũng có


ZB
Z A D


B C
C A



r2 DE2
EF2 <sub></sub> <sub>I A</sub>2:


</div>
<span class='text_page_counter'>(153)</span><div class='page_container' data-page=153>

<b>Epsilon</b>



<b>staff</b>



<i>Lời giải và bình luận đề thi chọn đội tuyển IMO 2017</i> <b>13</b>


Sử dụng định lý Ceva, ta có


XB
X C D


Y A
Y C


ZB
Z A D


A B : DE2
A C : DF2:


Do đó


DB
D C


XB
X C D



p b


p c


A B DE2
A C DF2 D


DF I B
DE I C


A B DE2
A C DF2 D


A B2
A C2:


Từ đó suy raA D và A X đẳng giác trong<sub>∠</sub>BA C :


Trở lại bài toán:


a) Theo bổ đề 1 phần a), b), ta thấy các đường tròn . P A C /và . QBA / tiếp xúc với . I / :


Từ đó, tiếp tục theo bổ đề 1 phần c) thìP R ; QS cùng đi qua trung điểmN của EF và hiển
nhiên M nằm trên A I :


<i>A</i>
<i>B</i> <i>C</i>
<i>Ib</i>
<i>I<sub>c</sub></i>


<i>I</i>
<i>F</i>
<i>E</i>
<i>P</i>
<i>Q</i>
<i>S</i>
<i>R</i>
<i>D</i>
<i>K</i>
<i>J</i>
<i>Y</i>
<i>X</i> <i>Z</i>
<i>M</i>
<i>N</i>


b)Cũng theo bổ đề 1 phần a), ta thấyX ; Y lần lượt là tiếp điểm của các đường tròn. P C A / ;
. QA B /với . I /. GọiZ là giao điểm củaB Y và C X :Theo bổ đề 3, ta thấy


∠DA B D ∠Z A C :


Ta sẽ chứng minh<sub>∠</sub>DA B D ∠M A C và từ đó suy ra A ; M ; Z thẳng hàng. Thật vậy, ta dễ
thấy <sub>∠</sub>E A J D 9 0ı 1


2∠BA C D ∠EDF nên tứ giácK A ED nội tiếp.


Chứng minh tương tự, tứ giácJA F D nội tiếp. Từ đó dễ thấy <sub>∠</sub>E K A D ∠J DA D ∠J F A


nên A J E M nội tiếp. Tương tự, tứ giácA K F M nội tiếp nên theo định lý Miquel,M nằm trên


</div>
<span class='text_page_counter'>(154)</span><div class='page_container' data-page=154>

<b>Epsilon</b>




<b>staff</b>



<b>14</b> <i>Lời giải và bình luận đề thi chọn đội tuyển IMO 2017</i>


Bình luận.


Hai ý a), b) của bài tốn này cũng không liên quan tới nhau. Ý a) của bài tốn giúp ta tìm
được trung điểmN củaEF. Ý a) là hệ quả trực tiếp của bổ đề 1. Nội dung chính của bổ
đề 1 đã có trong [1] và thực chất bổ đề này có liên quan chặt chẽ tới bài tốn IMO ShortList
2002, G7 xem [2], [3]. Phần khó nhất của ý b) là cần chứng minh<sub>∠</sub>Z A C D ∠DA B,
thực chất ý chứng minh này đã có trong [4] từ khá lâu và được chứng minh chặt chẽ như
trong trong bổ đề 3. Để chứng minh ý này có thể dùng phép nghịch đảo như trong [4]. Bạn
đọc có thể tham khảo một mở rộng của bài toán ý b) như sau, tham khảo [4].


Bài toán mở rộng.Cho tam giác A B C nội tiếp trong đường tròn. O /. K là một điểm


nằm trên phân giác <sub>∠</sub>BA C và . K / là một đường tròn tiếp xúc trong với . O / tại L.


E ; F là các điểm thuộc . K / sao cho C E ; DF là các tiếp tuyến của . K / ; trong đó


E ; F vàAkhác phía vớiB C. Đường tròn ngoại tiếp các tam giác A C E vàA BF cắt
nhau tại P khácA. Chứng minh rằng<sub>∠</sub>P A C D ∠L A B :


Tài liệu tham khảo:


[1] <i>Two circles are tangent</i>,


/>[2] <i>IMO Shortlist 2002, G7</i>,



/>


[3]
/>


[4] <i>Passes through tangent point</i>,


/>


Bài 4 (7.0 điểm). Cho tam giácABC nội tiếp đường tròn.O/:ĐiểmAdi động trên.O/


sao choAB > BC vàM là trung điểm củaAC:Đường trịn đường kínhBM cắt.O/tại


R:Giả sửRM cắt.O/tại điểm thứ haiQ;cắtBC tạiP:Đường trịn đường kínhBP cắt


AB; BO lần lượt tại các điểm thứ haiK; S:


a) Chứng minh rằngSRđi qua trung điểm củaKP:


b) Gọi N là trung điểm của BC: Trục đẳng phương của hai đường tròn đường kính


AN; BM cắtSRtạiE:Chứng minh rằngME ln đi qua một điểm cố định.


Lời giải. a)Dễ thấyBQlà đường kính của.O/. GọiI là giao điểm củaSRvàPK:Ta thấy


∠SP I D∠SBK D∠QCA




∠PSI D∠PBR D∠CQR:


</div>
<span class='text_page_counter'>(155)</span><div class='page_container' data-page=155>

<b>Epsilon</b>




<b>staff</b>



<i>Lời giải và bình luận đề thi chọn đội tuyển IMO 2017</i> <b>15</b>


<i>O</i>


<i>A</i>


<i>B</i> <i>C</i>


<i>M</i>


<i>N</i>


<i>R</i>
<i>P</i>


<i>Q</i>


<i>S</i>
<i>K</i>


<i>I</i>
<i>E</i>


<i>X</i>
<i>T</i>


<i>D</i>



<i>L</i>
<i>H</i>


b) Định nghĩa lại điểm E là chân đường cao hạ từ C của tam giác ABC: Gọi H là giao
điểm của các đường caoAD; BL của tam giác ABC: Dễ thấy tứ giácLMND nội tiếp nên


CM CL DCN CDvàHAHD DHB HL;suy raCH chính là trục đẳng phương của
đường trịn đường kínhAN vàBM.


Ta cóE nằm trênCH nên thuộc trục đẳng phương của đường trịn đường kínhAN vàBM. Do


∠BEH D∠BRP D90ınên tứ giácBHERnội tiếp, ta suy ra


∠HRE D∠EBH D∠OBC D∠PBS D∠PRS


nên ba điểmS; R; E thẳng hàng. Từ đó suy raE là giao điểm củaSRvà trục đẳng phương của
các đường trịn đường kínhAN; BM:


GọiX là giao điểm củaEM vàBQ:GọiT là trung điểm củaBC thì dễ thấy tứ giácETML


nội tiếp. Từ đó suy ra


∠MEC D90ı ∠MET D90ı ∠ALT D90ı ∠BAC D∠BAL D∠BCQ:


Kết quả này chứng tỏ tứ giácBCXE nội tiếp, suy ra<sub>∠</sub>BXC D BEC D 90ı:VậyX là hình
chiếu củaC lênBQcố định. Tóm lại, ta cóEM đi quaX cố định.


Bình luận.


</div>
<span class='text_page_counter'>(156)</span><div class='page_container' data-page=156>

<b>Epsilon</b>




<b>staff</b>



<b>16</b> <i>Lời giải và bình luận đề thi chọn đội tuyển IMO 2017</i>


Bạn đọc có thể tham khảo một phát triển của bài toán ý b) như sau:<i>Cho tam giác</i>ABC
<i>cố định và một đường tròn</i>.K/<i>thay đổi đi qua</i>B; C <i>cắt</i>CA; AB <i>lần lượt tại</i>E; F<i>. Gọi</i>
J; L<i>lần lượt là tâm ngoại tiếp các tam giác</i>ABE <i>và</i>ACF: EJ <i>cắt</i>FL<i>tại</i>P: Q<i>là tâm</i>
<i>ngoại tiếp tam giác</i>KEF:<i>Chứng minh rằng đường thẳng</i>PQ<i>luôn đi qua một điểm cố</i>
<i>định khi</i>.K/<i>thay đổi.</i>


Bài 5 (7.0 điểm). Cho 2 0 1 7số dương a1; a2; : : : ; a2 0 1 7:Với mỗin > 2 0 1 7; ta đặt
an D max


˚


ai1ai2ai3ji1 C i2 C i3 D n ; 1 i1 i2 i3 n 1 :


Chứng minh rằng tồn tại số nguyên dươngm không vượt quá 2 0 1 7 và số nguyên dương


N > 4 m sao choanan 4 m D a2n 2 m với mọin > N :


Lời giải. Với mỗin nguyên dương, ta đặtbn D lnan:Khi đó bài tốn có thể được phát biểu


lại như sau:<i>Cho</i> 2 0 1 7<i>số thực</i>b1; b2; : : : ; b2 0 1 7: <i>Với mỗi</i>n > 2 0 1 7; <i>ta đặt</i>


bn D max˚bi1 C bi2 C bi3ji1 Ci2 Ci3 D n ; 1 i1 i2 i3 n 1 :


<i>Chứng minh rằng tồn tại số nguyên dương</i> m <i>không vượt quá</i> 2 0 1 7 <i>và số nguyên dương</i>
N > 4 m <i>sao cho</i>bn C bn 4 m D 2 bn 2 m <i>với mọi</i>n > N :



Gọi` . 1 ` 2 0 1 7 /là số sao cho b`


` D max


n<sub>b</sub>


i


i


ˇ
ˇ


ˇ1 i 2 0 1 7
o


:Ta có nhận xét sau:


Nhận xét 1. <i>Với mọi</i> n<i>nguyên dương thì</i>
bn


n
b`


` :


Chứng minh. Ta chứng minh bằng quy nạp theon :Rõ ràng khẳng định đúng với n 2 0 1 7


do cách định nghĩa ` : Ta chỉ cần xét trường hợpn > 2 0 1 7: Giả sử khẳng định đúng với mọi



k < n :Từ cách xác địnhbn;ta thấy tồn tạij1; j2; j3 2 Nthỏa j1Cj2Cj3 D n sao cho


bn D bj1 C bj2 C bj3:


Sử dụng giả thiết quy nạp, ta có


bn j1


b`


` C j2
b`


` C j3
b`


` D n
b`


` ;


từ đó suy ra bn


n
b`


` :Do đó khẳng định cũng đúng với n :Nhận xét được chứng minh.


Bây giờ, với mỗi nnguyên dương, đặtcn D n b` ` bn thì từ nhận xét trên ta cócn 0 với



mọin : Đồng thời, với mọin 2 0 1 7 thì


cnC2 ` D . n C 2 ` / b` ` bnC2 `


. nC 2 ` / b` ` . bn C b` Cb`/


D n b` ` bn D cn:


Từ đó suy ra


</div>
<span class='text_page_counter'>(157)</span><div class='page_container' data-page=157>

<b>Epsilon</b>



<b>staff</b>



<i>Lời giải và bình luận đề thi chọn đội tuyển IMO 2017</i> <b>17</b>


Gọix số nguyên dương nhỏ nhất sao cho 2 x ` > 2 0 1 7và đặt


M D maxfci j1 i 4 x ` 1g:


Khi đó, với mọin > 2 x ` ; đặtn D 2 k x `C r (với0 r < 2 x `), ta có


cn crC2 . k 1 / x ` crC2 . k 2 / x ` crC2 x ` M :


Ta có các nhận xét sau:


Nhận xét 2. <i>Với mọi</i> n<i>nguyên dương, tồn tại các số tự nhiên</i> s1; s2; : : : ; s2 0 1 7 <i>sao cho</i>
cn D s1c1 Cs2c2 C C s2 0 1 7c2 0 1 7:



Chứng minh. Ta chứng minh bằng quy nạp theo n : Rõ ràng khẳng định đúng với mọi


n 2 0 1 7 (chỉ cần chọn sn D 1v si D 0 vii Ô n). Ta ch cần xét trường hợp n > 2 0 1 7


là đủ. Giả sử khẳng định đúng với mọi k < n :Tư công thức xác định củabn; ta suy ra


cn D minfci1Cci2Cci3 j1 i1 i2 i3 n 1 ; i1Ci2Ci3 D ng; 8n > 2 0 1 7:


Do đó tồn tại j1; j2; j3 2 N thỏaj1 C j2 C j3 D n sao cho


cn D cj1 C cj2 C cj3:


Theo giả thiết quy nạp, tồn tại các số tự nhiênu1; u2; : : : ; u2 0 1 7; v1; v2; : : : ; v2 0 1 7; w1;


w2; : : : ; w2 0 1 7 sao cho


cj1 D u1c1 C u2c2 C C u2 0 1 7c2 0 1 7;


cj2 D v1c1 C v2c2 C C v2 0 1 7c2 0 1 7;


cj2 D w1c1 C w2c2 C Cw2 0 1 7c2 0 1 7:


Do đó, ta có


cn D s1c1 Cs2c2 C C s2 0 1 7c2 0 1 7;


trong đósi D ui C vi C wi với1 i 2 0 1 7:Như vậy khẳng định cũng đúng với n :Nhận


xét được chứng minh.



Nhận xét 3. <i>Dãy</i>. cn/<i>chỉ nhận hữu hạn giá trị.</i>


Chứng minh. Nhận xét này được suy ra trực tiếp từ tính bị chặn củacn và kết quả của nhận


xét ở trên.


Bây giờ, docnchỉ nhận hữu hạn giá trị và


cnC2 k x ` cnC2 . k 1 / x ` cn


với mọin 2 0 1 7 nên tồn tạiN1 đủ lớn đểcn D cn 2 x ` với mọin > N1:Khi đó, ta có


n b` ` bn D . n 2 x ` / b` ` bn 2 x `; 8n > N1;


hay


</div>
<span class='text_page_counter'>(158)</span><div class='page_container' data-page=158>

<b>Epsilon</b>



<b>staff</b>



<b>18</b> <i>Lời giải và bình luận đề thi chọn đội tuyển IMO 2017</i>


Từ đây, ta cũng suy ra


bn 2 x ` D 2 b` C bn 4 x `; 8n > N1 C 2 x ` :


Cộng chéo vế hai kết quả trên, ta được


bnC bn 4 x ` D 2 bn 2 x `; 8n > N1 C 2 x ` :



ChọnN D N1 C2 x `và m D x ` ; ta có điều phải chứng minh.


Bình luận.


Bước logarith hố để đưa về dãybnđịnh nghĩa bởi bn D maxfbi Cbj Cbk j i Cj C


k D ng là khá hiển nhiên. Nó khơng tạo ra bước ngoặt đáng kể nhưng ít nhất sẽ tạo sự
tiện lợi trong trình bày và tính toán. Trong các phần sau ta coi như đang làm việc với bài
toán đã chuyển đổi sang phép cộng.


Đây là một bài toán dãy số được định nghĩa truy hồi thơng qua max;tạo một tình huống tổ
hợp khá thú vị. Để hình dung “hành vi” của dãy số này, ta cần xây dựng thử dãy số với tham
số nhỏ. Chẳng hạn, nếu thay2 0 1 7 bằng5 và với5số hạng đầu tiên là 1 ; 2 ; 4 ; 5 ; 1 0


thì các số hạng tiếp theo sẽ là 7; 1 2 ; 1 3 ; 1 5 ; 1 6 ; 2 1 ; : : :và chúng ta sẽ nhìn thầy vai
trị của số hạng thứ5 (số 1 0) trong dãy số này, từ đó dẫn đến ý tưởng xét` sao cho b`


` lớn


nhất và các bước tiếp theo.


Bài tốn này có thể coi là anh em song sinh với bài số 6 trong đề thi Toán Quốc tế năm
2010, cụ thể như sau:


<i>Cho</i>a1; a2; a3; : : :<i>là dãy các số thực dương và</i> s <i>là số nguyên dương sao cho</i>
an D maxfak Can k j 1 k n 1g


<i>với mọi</i> n > s : <i>Chứng minh rằng tồn tại các số nguyên dương</i> l s <i>và</i> N <i>sao cho</i>
an D al C an l <i>với mọi</i>n N :



Sự khác biệt chỉ nằm ở phép cộng và phép nhân, vốn có thể chuyển sang dễ dàng nhờ
logarith, cũng như độ sâu của phép truy hồi. Tuy nhiên bước chuyển từ 2 sang 3 này
gần như không ảnh hưởng đến các bước lý luận của lời giải. Ý tưởng chính vẫn là xét


t D max˚ai


i ; i D 1 ; : : : ; s và xét dãy sốbn D n t an:


Bài 6 (7.0điểm). Với mỗi số nguyên dươngn, xét a1; a2; : : : ; a2 n là hoán vị của2 nsố


nguyên dương đầu tiên. Một hoán vị như thế được gọi là “<i>đẹp</i>” nếu với mọi1 i < j 2 n


thìai CanCi D 2 nC1vàai aiC1khơng đồng dư vớiaj ajC1theo modulo2 nC1.


(Quy ước a2 nC1 D a1:)


a) Với n D 6, hãy chỉ ra một hoán vị đẹp.


</div>
<span class='text_page_counter'>(159)</span><div class='page_container' data-page=159>

<b>Epsilon</b>



<b>staff</b>



<i>Lời giải và bình luận đề thi chọn đội tuyển IMO 2017</i> <b>19</b>


Lời giải. a)Ta xét hoán vị sau:


1 ; 2 ; 4 ; 8 ; 3 ; 6 ; 1 2 ; 1 1 ; 9 ; 5 ; 1 0 ; 7:


Dễ thấy rằng1 C1 2 D 2 C1 1 D 4 C9 D 8 C5 D 3C1 0 D 6 C7. Đồng thời, hiệu của
các số liên tiếp lấy theo modulo 1 3 lần lượt là 1 ; 2 ; 4 ; 8 ; 3 ; 6 ; 1 2 ; 1 1 ; 9 ; 5 ; 1 0 ; 7 đều


phân biệt. Do đó, hốn vị này thỏa mãn điều kiện đề bài.


b)Gọi A là tập hợp các hoán vị đẹp và B là tập hợp các cách điền các số 1 ; 2 ; : : : ; 2 n


lên các đỉnh của đa giác đều X1X2X3: : : X2 n, sao cho hai số thuộc các đầu mút của một


đường kính có tổng là 2 nC 1I đồng thời khơng có bất cứ một dãy liên tiếp các số nào có
tổng là bội của 2 nC 1 :


Ta sẽ chứng minh rằng tồn tại song ánh từ A ! B và khi đó, để chứng minh tồn tại hốn vị, ta
chỉ cần chỉ ra rằng<sub>j</sub>Aj D jBj > 0 :


(1)<i>Tồn tại song ánh</i>A ! B :


Xét một hoán vị thuộcAlà. a1; a2; : : : ; a2 n/thì vớii D 1 ; 2 n, ta chọnbi 2 f1 ; : : : ; 2 ng


sao cho bi aiC1 ai .mod 2 nC 1 /. Theo giả thiết thì tất cả các sốbi này phải đôi một


phân biệt. Ta quy ước rằnga2 nC1 D a1 và b2 nC1 D b1.


Ta điền các số này lên vòng tròn theo thứ tự,bi điền cho đỉnh Xi. Dễ thấy


bi C biCn aiC1 ai C aiC1Cn aiCn D 2 . 2 n C 1 / 0 .mod 2 nC 1 / :


Do0 < bi C biCn < 2 . 2 n C 1 / nênbi C biCn D 2 n C1 :Ngồi ra, ta cũng có


ai D . ai ai 1/C . ai 1 ai 2/C C . a2 a1/C a1 D a1 C
i 1


X



kD1


bk


và các số ai đôi một phân biệt theo modulo2 nC 1nên không tồn tại i ; j sao cho


aj ai D
j 1


X


kDi


bk 0 .mod 2 nC 1 / :


Do đó, cách điền các số bi như trên thỏa mãn điều kiện cách điền trongB :Ngược lại, xét một


cách điền trong B và bi được điền cho đỉnhXi; i D 1 ; 2 n thì xét dãy số


a1 D b1; a2 D b1 C b2; a3 D b1 Cb2 C b3; : : : ; ai D
i


X


kD1


bk


và tổng lấy theo modulo2 n C1 :Khi đó, ta thấy rằng các số ai đều dương và đồng thời cũng



phân biệt, vì nếu khơng, giả sử cóai D aj thì


0 D aj ai D
j 1


X


kDi


bk .mod 2 nC 1 / ;


mâu thuẫn. Hơn nữabi C biCn ai ai 1 C aiCn aiCn 1 0 .mod 2 n C1 / nên


</div>
<span class='text_page_counter'>(160)</span><div class='page_container' data-page=160>

<b>Epsilon</b>



<b>staff</b>



<b>20</b> <i>Lời giải và bình luận đề thi chọn đội tuyển IMO 2017</i>


Suy ra, tổngai CaiCnvới mọi i D 1 ; 2 nđều có cùng số dư làa 2 f0 ; 1 ; 2 ; : : : ; 2 ngkhi


chia cho2 nC 1 :Chú ý rằng có 2 ntổng như thế và chúng đôi một rời nhau nên


2 n


X


kD1



k D


2 n


X


iD1


. ai C aiCn/ 2 n a .mod 2 n C 1 / ;


mà . 2 n ; 2 n C 1 / D 1 nên cần có a D 0hay ai C aiCn D 2 nC 1 với mọi i D 1 ; 2 n.


Do đó, dãy số ai là hốn vị đẹp nên nó thuộcA : Đến đây ta có song ánh đi từA vàoB và


. 1 / được chứng minh.


(2)<i>Chứng minh</i>jBj > 0 :


Trước hết, ta thấy rằng số cách điền sao chobi CbiCn D 2 nC1với mọii D 1 ; 2 nlàn Š2n


vì các số 1 ! 2 nthuộc các cặp rời nhau có tổng là 2 nC 1và có thể đổi chỗ số điền choXi


vàXiCn:Giả sử trong một cách điền số lên đa giác, có một dãy liên tiếp có k số mà tổng chia


hết cho 2 nC 1 thì k 3 và nếu gọiSk là số tập con có k phần tử như thế thì ở đây, ta có


n 2n k <sub></sub> <sub>. n</sub> <sub>k / Š</sub> <sub></sub> <sub>S</sub>


k cách điền. Cụ thể là cón cách chọn vị trí bắt đầu của dãy (vì tính đối



xứng qua tâm nên nếu dãyk số này thỏa thì k số đối xứng của nó cũng thỏa), còn lại2 n 2 k


số xếp tùy ý thỏa mãn tính đối xứng là 2n k . n k / Š.


Từ đó suy ra số cách điền thỏa điều kiện tổngbi C biCn D 2 n C 1nhưng có một dãy các số


liên tiếp có tổng chia hết cho2 nC 1sẽ không vượt quá


S D n


n


X


kD3


2n k. n k / Š Sk:


Để đánh giá đại lượng này, ta tiếp tục chứng minh các nhận xét sau:


a) Sk


1
kC


k 1
2 n .


b) C2 nk 1 4k 1Cnk 11 với mọi n 5 ; k 3 ; n k :Khi đó vớin D 1 ; 2 ; 3 ; 4 ta có



thể chỉ ra hốn vị cụ thể thỏa mãn đề bài.


c) x


0


0 Š C
x1


1 Š C
x2


2 Š C C
xn


n Š < e


x


với mọin.


Thật vậy:


a) Ta thấy rằng có đúng C2 nk 1 tập con tùy ý cók 1phần tử của 2 n. Với mỗi tập con như


thế thì có khơng q một cách để thêm vào một số để đượck số có tổng là bội của2 nC1.
Ngược lại, mỗi tập conk phần tử có tổng chia hết cho2 nC 1 thì tương ứng với đúngk


tập con cók 1phần tử của2 n :Dùng đếm bằng hai cách, ta có k Sk 1 C2 nk 1 hay



(a) được chứng minh.


b) Có thể chứng minh dễ dàng bằng quy nạp theon với chú ý rằng


C2 nk C
k
n 1 D


2 n k C 1


k C
k 1
2 n
n k
k C
k 1
n 1


2 n k C 1


n k 4


k 1


< 4k:


c) Có thể chứng minhf . x / D ex Pn


iD0
xi



</div>
<span class='text_page_counter'>(161)</span><div class='page_container' data-page=161>

<b>Epsilon</b>



<b>staff</b>



<i>Lời giải và bình luận đề thi chọn đội tuyển IMO 2017</i> <b>21</b>


Cuối cùng, ta có


S n


n


X


kD3


2n k. n k / Š1
kC


k


n 1 n
n


X


kD3


2n k. n k / Š1


kC


k 1
n 14


k 1


D 2nn Š1
4


n


X


k<sub>D</sub>3


2k
k Š < 2


n


n Še


2 <sub>. 1</sub>


C2 C 2 /
4 < 2


n



n Š :


Do đó,jBj 2nn Š S > 0. Bài toán được giải quyết hồn tồn.


Bình luận.


Ở bài 6 này, chúng tơi có tham khảo ý tưởng của Nguyễn Huy Tùng, IMO 2014 (ý
tưởng cho lời giải chi tiết ở trên) và Lê Hồng Quý, IMO 2006 (ý tưởng lời giải trong
phần bình luận bên dưới).


Lời giải trên tiếp cận theo hướng xác suất, cụ thể là để chứng minh tồn tại một mơ hình
thỏa mãn ràng buộc của đề bài, thay vì xây dựng trực tiếp, chúng ta có thể đếm hoặc ước
lượng tương đối số mơ hình thỏa mãn và chứng minh nó là số dương.


Một số bài tốn tương tự:


1. (Bulgaria, 2016)Hỏi có tồn tại hay khơng số nguyên dương n < 1 09sao cho ncó thể
biểu diễn dưới dạng tổng của ba số chính phương dương phân biệt bởi hơn1 0 0 0cách?
Tương tự ý tưởng đã nêu trong bài 6, ta có thể đếm số lượng tổnga2 Cb2 C c2 phân
biệt với1 a < b < c N và N là số nguyên dương nào đó. Có tất cảC<sub>N</sub>3 bộ


. a ; b ; c /nhưng nhận không quá3 N2 giá trị nên phải tồn tại một giá trị tương ứng
với ít nhất C


3
N


3 N2 bộ. ChọnN D 1 8 0 0 3thì câu trả lời cho bài toán trên là khẳng định.


2. Chứng minh rằng trong 21 0 0 người, không nhất thiết phải có 2 0 0 người đơi một


quen nhau hoặc đôi một không quen nhau.


3. Trong một bảng1 0 01 0 0 ; người ta viết vào mỗi ô một trong các số nguyên dương
không vượt quá5 0 0 0 và mỗi số xuất hiện đúng hai lần. Chứng minh rằng có thể
chọn ra1 0 0 số phân biệt thuộc các hàng và cột khác nhau từ trong bảng trên.


Để giải quyết bài 6, ta cũng có thể tiến hành trực tiếp với ý tưởng là xây dựng sao cho với
mọii thìjaiC1 aij 2 i C 1 .mod 2 nC 1 /, nếu có được điều này thì yêu cầu của


bài toán được thỏa mãn. Cụ thể là:


Xếp các số lên các đỉnh của đa giác X1X2: : : X2 n sao choXi D i vớii D 1 ; nvà


Xi D i vớii D n C 1 ; 2 n. Ta tiến hành chọn các số của hoán vị theo quy tắc sau:


Xuất phát từ đỉnhX1, ta chọna1 D 1.


Đi đến đỉnh liền sau củaXnC1là XnC2, và chọna2 D 2.


Đi đến đỉnh liền sau củaX2 làX3 và chọna3 D 3 :


</div>
<span class='text_page_counter'>(162)</span><div class='page_container' data-page=162>

<b>Epsilon</b>



<b>staff</b>



<b>22</b> <i>Lời giải và bình luận đề thi chọn đội tuyển IMO 2017</i>


Tuy nhiên, vấn đề ở đây làanC1 D 1 nên hiệujanC1 anj 2 fn 1 ; nC 1g,


do đó ta cần tránh bước thứi nào đó mà <sub>j</sub>aiC1 aij 2 fn 1 ; n C1g (nếu hai



số này chẵn thì giá trị tương ứng làn ; n C 2).


Để làm được điều này, ở bước thứi, thay vì ta đi vào điểm bên cạnh của điểm đối xứng
qua đường kính thì ta sẽ đi đến điểm bên cạnh của số đó để chojaiC1 aij D 1,


đoạn cịn lại vẫn thực hiện như cũ. Do việc chọn các giá trị theo cùng một chiều nên
không xảy ra trường hợp lặp lại số cũ.


Cuối cùng, ta chỉ cần xác định bước thứi như thế. Để ý rằng nếu thực hiện như ban đầu,
ta sẽ có các sốa1; a2; : : : ; anchỉ chứa số nguyên dương lẻ và số nguyên âm chẵn.


Khi đó, nếunchẵn thìan D nvà jan anC1j D n 1, cần tránh giá trị nC2


ra vì. n 1 / . nC 2 / .mod 2 n C1 /. Thay vì đi đến điểm nC<sub>2</sub>2, ta đi sang
điểm bên cạnh của nó.


Nếun lẻ thì tương tự, thay vì đi đến nC<sub>2</sub>1, ta cũng đi sang điểm bên cạnh. Theo quy
tắc này thì các giá trịjai aiC1j sẽ nhận tất cả các giá trị lẻ từ1 ! 2 n 1và


</div>
<span class='text_page_counter'>(163)</span><div class='page_container' data-page=163>

<b>Epsilon</b>



<b>staff</b>



<b>LỜI GIẢI VÀ BÌNH LUẬN ĐỀ THI</b>


<b>CHỌN ĐỘI TUYỂN IMO 2018</b>



<b>Trần Nam Dũng – Võ Quốc Bá Cẩn – Trần Quang Hùng</b>
<b>Lê Phúc Lữ – Nguyễn Lê Phước</b>



1. Lời nói đầu



Kỳ thi chọn đội tuyển Việt Nam dự thi toán quốc tế năm 2018 đã diễn ra trong hai ngày 30 và
31/3 vừa qua. Trong mỗi ngày, thí sinh phải làm ba bài tốn trong vịng 270 phút.


Tiếp nối truyền thống nhiều năm nay, Ban biên tập chúng tơi vẫn muốn theo sát các thí sinh
và giáo viên chun Tốn, cùng nhau giải và phân tích, khai thác các bài có trong đề thi.
Thơng qua đó, mọi người có thể thấy được bản chất, vẻ đẹp của các vấn đề, quan trọng hơn là
có nguồn tư liệu, định hướng rèn luyện cho các mùa thi sau. Ban biên tập hiểu rằng các đề thi
TST ln mang tính thử thách, tạo cảm hứng và bên cạnh VMO, IMO, đây cũng là một trong
nội dung ln được chờ đón.


Năm 2018, cùng với sự ra đời và đồng hành của chương trình BM2E – Bring Math to Everyone,
chúng tơi đã có nhiều động lực và sự hỗ trợ hơn. Với mong muốn đẩy mạnh phong trào chuyên
Toán nhiều nơi, hy vọng rằng B2ME sẽ ln nhận được sự đón nhận của mọi người và chung tay
phát triển. Ngoài ra với tinh thần hội nhập quốc tế, năm nay chúng tôi cũng sẽ xuất bản Epsilon
14, phiên bản tiếng Anh.


Cuối cùng, để hồn thành tài liệu này, chúng tơi xin gửi lời cám ơn đến thầy Nguyễn Chu Gia
Vượng (viện Toán học) và các bạn Hoàng Đỗ Kiên (HCB IMO 2013), Nguyễn Nguyễn (PTNK
TPHCM) vì những tài liệu cũng như những góp ý đáng giá. Mọi thắc mắc, góp ý, xin hãy gửi tin
nhắn cho trang chủ của tạp chí Epsilon.


</div>
<span class='text_page_counter'>(164)</span><div class='page_container' data-page=164>

<b>Epsilon</b>



<b>staff</b>



<b>2</b> <i>Lời giải và bình luận đề thi chọn đội tuyển IMO 2018</i>


2. Đề thi




2.1. Ngày thi thứ nhất (30/03/2018)



Bài 1 (7.0 điểm). Cho tam giácABC nhọn không cân cóD; E; F lần lượt là trung điểm của


các cạnhBC; CAvàAB:Gọi.O/; .O0/lần lượt là tâm ngoại tiếp và tâm Euler của tam giác.


Xét điểmP bên trong tam giácDEF vàDP; EP; FP cắt lại.O0/lần lượt tạiD0; E0; F0:Gọi


A0là điểm đối xứng vớiAquaD0:Xác định tương tự vớiB0vàC0:


a) Chứng minh rằng nếuPO DPO0thì đường tròn.A0B0C0/đi quaO:


b) LấyX đối xứng vớiA0 qua đường thẳngOD:Xác định tương tự vớiY vàZ:Gọi H là


trực tâm tam giácABC vàXH; Y H; ZH cắt BC; CA; AB theo thứ tự tạiM; N; K:


Chứng minh rằngM; N; K thẳng hàng.


Bài 2 (7.0 điểm). Vớimlà số nguyên dương, xét bảng ô vngm2018gồmmhàng,2018


cột mà trong đó có một vài ơ trống, cịn một vài ô được đánh số0hoặc1:Bảng được gọi là “đầy


đủ” nếu với bất kỳ chuỗi nhị phânS có2018ký tự nào, ta đều có thể chọn ra một hàng nào đó


của bảng rồi điền thêm0; 1vào để2018ký tự của hàng tạo thành chuỗiS (nếu chuỗiS đã có


sẵn trên hàng nào đó rồi thì coi như thỏa mãn). Bảng được gọi là “tối giản” nếu nó đầy đủ và nếu
ta bỏ đi bất kỳ hàng nào thì nó khơng cịn đầy đủ nữa.



a) Với 0 k 2 0 1 8 ; chứng minh rằng tồn tại bảng tối giản 2k 2 0 1 8 sao cho có


đúngk cột có đủ cả 0lẫn1 :


b) Cho bảng tối giảnm 2 0 1 8 có đúngk cột chứa cả0lẫn1. Chứng minh rằngm 2k<sub>:</sub>


(Một dãy nhị phân độ dài 2 0 1 8 là dãy có dạng x1x2: : : x2 0 1 8 trong đó xi 2 f0 ; 1g với


mọii 2 f1 ; 2 ; : : : ; 2 0 1 8g:)


Bài 3 (7.0 điểm). Cho số nguyênn 3và Anlà tập hợp tất cả các số nguyên dương nhỏ hơn


n ; nguyên tố cùng nhau vớin :Xét đa thức


Pn. x / D


X


k2An


xk 1:


a) Chứng minh rằngP . x /chia hết cho đa thức xr C 1với r là số ngun dương nào đó.


b) Tìm tất cả các số nguyên dương nđểPn. x /bất khả quy trênZŒ x  :


2.2. Ngày thi thứ hai (31/03/2018)



Bài 4 (7.0 điểm). Cho a là số thực thuộc đoạn 1<sub>2</sub>; 2<sub>3</sub>



: Xét các dãy số . un/ và . vn/


. n D 0 ; 1 ; : : : /được xác định như sau:


un D


3


2nC1 . 1 /


b2nC1<sub>a</sub><sub>c</sub>


; vn D


3


2nC1 . 1 /


nCb2nC1<sub>a</sub><sub>c</sub>


:


a) Chứng minh rằng


2 0 1 8


X


iD0



ui


!2
C


2 0 1 8


X


iD0


vi


!2


7 2 a2 4 8 a C 1 0 C 2


42 0 1 9:


</div>
<span class='text_page_counter'>(165)</span><div class='page_container' data-page=165>

<b>Epsilon</b>



<b>staff</b>



<i>Lời giải và bình luận đề thi chọn đội tuyển IMO 2018</i> <b>3</b>


Bài 5 (7.0 điểm). Một bảng ô vuông m n A B C D có các đỉnh là các giao lộ (có tất cả


. mC1 / . nC1 / giao lộ). Người ta muốn thiết lập một tuyến đường bắt đầu từ A ;đi theo


các cạnh song song với các cạnh của hình chữ nhật và đi qua tất cả các giao lộ đúng một lần,



sau đó quay về A :


a) Chứng minh rằng có thể xây dựng được đường đi khi và chỉ khi mlẻ hoặc nlẻ.


b) Vớim ; n thỏa mãn điều kiện câua), hỏi có ít nhất bao nhiêu giao lộ mà tại đó có ngã rẽ?


Bài 6 (7.0 điểm). Cho tam giácA B C nhọn nội tiếp . O / và. J /là đường trịn bàng tiếp góc


Acủa tam giác. Gọi D ; E ; F lần lượt là tiếp điểm của . J /vớiB C ; C A và A B :


a) Gọi L là trung điểm của B C : Đường tròn đường kính LJ cắt các đường thẳng


DE ; DF lần lượt tạiK vàH :Chứng minh rằng các đường tròn. BDK / và. C DH /


cắt nhau trên đường tròn. J / :


b) Giả sử đường thẳng EF cắt đường thẳng B C tại G và GJ cắt các đường thẳng


A B ; A C lần lượt tại M ; N : Gọi P vàQ là các điểm trên các đường thẳng J B ; J C


sao cho <sub>∠</sub>P A B D ∠QA C D 9 0ı<sub>:</sub> <sub>Gọi</sub> <sub>T</sub> <sub>là giao điểm của hai đường thẳng</sub>


P M ; QN và S là điểm chính giữa cung lớn B C của đường tròn . O / : Gọi I là


tâm đường tròn nội tiếp tam giác A B C :Chứng minh rằng đường thẳng S I cắt đường


</div>
<span class='text_page_counter'>(166)</span><div class='page_container' data-page=166>

<b>Epsilon</b>



<b>staff</b>




<b>4</b> <i>Lời giải và bình luận đề thi chọn đội tuyển IMO 2018</i>


3. Bình luận chung



Các bài tốn năm nay được phân theo phân mơn như sau:


Bài 1 và bài 6:Hình học.


Bài 2 và bài 5:Tổ hợp.


Bài 3:Số học.


Bài 4:Đại số.


Hai bài hình có cấu hình khá đẹp, đặc biệt là ở bài 1, nơi ta có thể gặp lại những ý tưởng


kinh điển của hình học qua đường tròn Euler và đường tròn Hagge: chứng minh đồng viên


thơng qua phép vị tự. Tuy nhiên có vẻ như hai bài 1và 6quá giống nhau về dạng bài, công


cụ: định lý Menelaus, đẳng giác, phương tích, phép biến hình. Nếu sử dụng hai bài hình, nên
là hai bài khác hướng.


Hai bài tổ hợp khác nhẹ nhàng dù đều liên quan đến những lĩnh vực của toán hiện đại: lý thuyết


mã hóa thơng tin và lý thuyết đồ thị.Bài 5có vẻ là bài quen thuộc nhưng nếu chưa gặp thì cũng


khơng khó xử lý.



Bài 3là một bài phát biểu dưới dạng đa thức (thậm chí có cả từ bất khả quy) nhưng bản chất là


một bài số học. Ý tưởng cơ bản là xây dựng các song ánh (dạng k ! n k ; k ! k C n<sub>2</sub> hoặc


tương tự) giữa các đơn thức, cịn trường hợp nphi chính phương (square-free) thì dùng quy nạp


theo số ước nguyên tố. Bài này có ít nhiều liên quan đến tổng Ramanujan trong số học.


Bài 4là một bài bất đẳng thức dãy số có hình thức khá lạ và khá xấu. Tuy nhiên, nếu nhìn kỹ có


thể thấy được tư tưởng dùng hệ đếm nhị phân. Nếu “bắt” được ý này thì phần sau chỉ là phần xử
lý kỹ thuật. Có lẽ vì vậy mà ban đề thi đưa bài này vào vị trí bài dễ nhất trong ngày thứ hai, dù


theo chúng tơi, bài này khó hơnbài 5nhiều và sẽ là bài “sát thủ”.


Như vậy, chúng ta có thể thấy rằng, hai bài 3, 4là hai bài Số học và Đại số song kiếm hợp bích.


Nhìn tổng thể, đề thi năm nay khá hay và tốt, có nhiều đất diễn cho thí sinh, khơng có bài siêu
sát thủ. Năm nay ban chấm thi sẽ vất vả. Dự kiến là bài nào cũng có thể có thí sinh làm trọn vẹn.


Có một điểm chưa được hay là tất cả các bài TST năm nay đều có hai ýa)vàb). Trong một số


trường hợp có hai ý là hay nhưng trong nhiều trường hợp nó làm bài tốn trở nên lắt nhắt, hay


như ýb)củabài 3trở nên tầm thường khi đã có ýa). Việc có hai bài hình “đồng dạng” cũng là


điều cần khắc phục và chúng ta nên khai thác thêm mảng Hình tổ hợp và bất đẳng thức hình học


để đa dạng hóa. Hơn nữa hai bài hình đều có hai ýa),b)gần như độc lập với nhau thành ra gần



</div>
<span class='text_page_counter'>(167)</span><div class='page_container' data-page=167>

<b>Epsilon</b>



<b>staff</b>



<i>Lời giải và bình luận đề thi chọn đội tuyển IMO 2018</i> <b>5</b>


4. Lời giải và bình luận các bài tốn



Bài 1 (7.0điểm). Cho tam giácA B C nhọn không cân có D ; E ; F lần lượt là trung điểm
của các cạnhB C ; C A vàA B : Gọi . O / ; . O0/lần lượt là tâm ngoại tiếp và tâm Euler của
tam giác. Xét điểmP bên trong tam giácDEF và DP ; EP ; F P cắt lại . O0<sub>/</sub><sub>lần lượt tại</sub>
D0; E0; F0:GọiA0là điểm đối xứng với AquaD0:Xác định tương tự với B0và C0:


a) Chứng minh rằng nếuP O D P O0 thì đường trịn. A0B0C0/ đi quaO :


b) LấyX đối xứng vớiA0qua đường thẳng OD:Xác định tương tự vớiY và Z :Gọi H
là trực tâm tam giác A B C và XH ; Y H ; ZH cắt B C ; C A ; A B theo thứ tự tại


M ; N ; K :Chứng minh rằng M ; N ; K thẳng hàng.


Lời giải. a) Gọi Ilà điểm đối xứng với O qua P : Ta có O0 <sub>là trung điểm của</sub> <sub>OH</sub> <sub>nên</sub>


O0P k I H : Lại có P O D P O0nênI O D I H :


A


B C


H O0 O



D
P
D0


A0


G


I
S


GọiS vàG lần lượt là trung điểm của các đoạn thẳngAI; AH:Ta có


SP D 1


2AO D
1


2R DO


0<sub>D</sub>


vàSP kAO kO0D nên tứ giácO0SPDlà hình bình hành. Từ đó suy raDP kO0S:


Lại cóSP D 1<sub>2</sub>R DO0<sub>D</sub>0<sub>nên tứ giác</sub><sub>SD</sub>0<sub>PO</sub>0<sub>là hình thang cân. Suy ra</sub><sub>O</sub>0<sub>P</sub> <sub>D</sub><sub>SD</sub>0<sub>và</sub>


IH D2O0P D2SD0DIA0:


Từ đó, ta cóIA0 DIH DIO nênA0nằm trên đường tròn.I; IO/:Chứng minh tương tự, ta



</div>
<span class='text_page_counter'>(168)</span><div class='page_container' data-page=168>

<b>Epsilon</b>



<b>staff</b>



<b>6</b> <i>Lời giải và bình luận đề thi chọn đội tuyển IMO 2018</i>


b)GọiRlà bán kính của đường trịn.O/:Dễ thấyGD DR:Xét phép vị tự tâmAtỉ số 1<sub>2</sub> biến


B; C; A0; X; H; M và trung trựcBC lần lượt thành thànhF; E; D0; U; G; M0và trung trực


EF:Khi đó, ta có <sub>M C</sub>MB D MM00FE vàU đối xứng vớiD0qua trung trựcEF:Suy ra


MB
M C D


M0F
M0<sub>E</sub> D


GF
GE


UF
UE D


p


R2 <sub>DF</sub>2


p



R2 <sub>DE</sub>2


D0E
D0<sub>F</sub>:


A


B C


H O0 O


D
G


E


F P


D0


A0


X


M


M0 U


Tương tự, ta cũng tính được NC<sub>NA</sub> và KA<sub>KB</sub>:DoDD0; EE0; FF0đồng quy nên



D0F
D0<sub>E</sub>


F0E
F0<sub>D</sub>


E0D
E0<sub>F</sub> D1:


Từ đó <sub>M C</sub>MB NC<sub>NA</sub> <sub>KB</sub>KA D1;hayM; N; K thẳng hàng.


Bình luận. Đây là một bài tốn hình học hay và có nhiều ý mới. Nếu như câub)chỉ là ứng


dụng của định lý Menelaus khá cơ bản thì câua)là một bài tốn rất thú vị. Điểm đáng chú ý của


câua)là ta có thể viết lại thành bài toán thú vị như sau:


Bài toán 1.<i>Cho tam giác</i>ABC <i>với</i>D; E; F <i>lần lượt là trung điểm của các cạnh</i>BC; CA<i>và</i>


AB: P <i>là một điểm bất kỳ trên mặt phẳng. Các đường thẳng</i>PD; PE; PF <i>cắt lại đường tròn</i>


.DEF /<i>lần lượt tại</i>X; Y <i>và</i>Z:<i>Gọi</i>U; V <i>và</i>W <i>lần lượt là đối xứng của</i>A; B; C <i>qua</i>X; Y; Z:


a) <i>Chứng minh rằng đường tròn</i>.U V W /<i>đi qua trực tâm tam giác</i>ABC:


b) <i>Gọi</i>K <i>là tâm của đường trịn</i>.U V W /:<i>Chứng minh rằng đường thẳng</i>PK <i>ln đi qua</i>
<i>một điểm cố định khi</i>P <i>thay đổi.</i>


</div>
<span class='text_page_counter'>(169)</span><div class='page_container' data-page=169>

<b>Epsilon</b>




<b>staff</b>



<i>Lời giải và bình luận đề thi chọn đội tuyển IMO 2018</i> <b>7</b>


Sau đây là một tổng quát hơn nữa cho bài toán đồng viên trên:


Bài toán 2.<i>Cho tam giác</i>ABC <i>nội tiếp trong đường tròn</i>.O/<i>và hai điểm</i>P; Q<i>bất kỳ trên</i>
<i>mặt phẳng. Phép vị tự tâm</i>P <i>tỉ số</i>k<i>biến các điểm</i>A; B; C <i>thành các điểm</i>A0; B0<i>và</i>C0:<i>Gọi</i>


A0A1; B0B1 <i>và</i>C0C1<i>lần lượt là các đường kính của đường trịn</i>.A0B0C0/:<i>Các đường thẳng</i>


QA1; QB1<i>và</i>QC1 <i>cắt lại đường tròn</i>.A1B1C1/<i>lần lượt tại</i>A2; B2; C2:


a) <i>Phép vị tự tâm</i>A; B; C <i>tỉ số</i> <sub>1 k</sub>1 <i>lần lượt biến các điểm</i>A2; B2; C2<i>thành</i>A3; B3 <i>và</i>C3:
<i>Chứng minh rằng đường tròn</i>.A3B3C3/<i>đi qua</i>P:


b) <i>Gọi</i>K <i>là tâm của đường tròn</i>.A3B3C3/:<i>Chứng minh rằng phép vị tự tâm</i>K <i>tỉ số</i> <sub>k</sub>1 <i>biến</i>


P <i>thành tâm ngoại tiếp tam giác</i>ABC:


Mặt khác cũng từ cấu hình bài tốn TST chúng ta cũng có thể tìm và khai thác được nhiều điểm
thú vị khác, sau đây là một khai thác thú vị từ cấu hình này:


Bài tốn 3.<i>Cho tam giác</i> A B C <i>có trực tâm</i> H : <i>Gọi</i> A1; B1; C1 <i>lần lượt là trung điểm</i>
<i>của các cạnh</i> B C ; C A <i>và</i> A B : P <i>là điểm bất kỳ trên đường thẳng Euler của tam giác</i>


A B C : . N / <i>là đường tròn Euler của tam giác</i> A B C :<i>Gọi</i>A1A0; B1B0 <i>và</i>C1C0 <i>lần lượt</i>


<i>là các đường kính của đường trịn</i> . N / :



a) <i>Chứng minh rằng các đường thẳng</i>A A0; BB0 <i>và</i> C C0<i>đồng quy tại</i> X :


b) <i>Các đường thẳng</i> A A0; BB0 <i>và</i>C C0 <i>lần lượt cắt lại đường tròn</i> . N / <i>tại</i>A2; B2; C2:


Q<i>là điểm bất kỳ trên đường thẳng</i> P X :<i>Các đường thẳng</i>QA2; QB2 <i>và</i>Q C2<i>lần lượt</i>
<i>cắt lại đường tròn</i>. N / <i>tại</i>A3; B3; C3:<i>Gọi</i>A4; B4 <i>và</i> C4 <i>lần lượt là đối xứng của</i> A ;


B ; C <i>lần lượt qua</i>A3; B3; C3:<i>Chứng minh rằng đường tròn</i> . K /<i>ngoại tiếp tam giác</i>


A4B4C4 <i>đi qua</i>H :


c) <i>Chứng minh rằng</i> K <i>luôn nằm trên một đường thẳng cố định song song với đường</i>
<i>thẳng</i> P X <i>khi</i>Q <i>thay đổi.</i>


Ngoài ra ta cũng có thể sử dụng kết quả bài toán sau làm bổ đề để chứng minh ýb)của bài TST


bằng phương pháp phương tích và trụ đẳng phương:


Bài tốn 4.<i>Cho tam giác</i>A B C <i>và điểm</i>P <i>bất kỳ trên mặt phẳng. Các điểm</i> X ; Y <i>và</i>Z <i>nằm</i>
<i>trên các đường tròn</i>. P B C / ; . P C A / ; . P A B / <i>sao cho đường tròn</i> . X Y Z /<i>đi qua</i>P :<i>Gọi</i>


U ; V <i>và</i> W <i>lần lượt là đối xứng của</i>X ; Y ; Z <i>qua trung trực của</i>B C ; C A ; A B : <i>Chứng minh</i>


<i>rằng đường tròn</i>. U V W / <i>cũng đi qua</i> P :


Có thể tham khảo thêm các mở rộng tại liên kết:gspot.


</div>
<span class='text_page_counter'>(170)</span><div class='page_container' data-page=170>

<b>Epsilon</b>



<b>staff</b>




<b>8</b> <i>Lời giải và bình luận đề thi chọn đội tuyển IMO 2018</i>


Bài 2 (7.0 điểm). Vớim là số nguyên dương, xét bảng ô vuôngm 2 0 1 8 gồm mhàng,


2 0 1 8 cột mà trong đó có một vài ơ trống, cịn một vài ơ được đánh số0 hoặc1 :Bảng được


gọi là “đầy đủ” nếu với bất kỳ chuỗi nhị phân S có 2 0 1 8ký tự nào, ta đều có thể chọn ra
một hàng nào đó của bảng rồi điền thêm0 ; 1vào để 2 0 1 8ký tự của hàng tạo thành chuỗi
S (nếu chuỗiS đã có sẵn trên hàng nào đó rồi thì coi như thỏa mãn). Bảng được gọi là “tối
giản” nếu nó đầy đủ và nếu ta bỏ đi bất kỳ hàng nào thì nó khơng cịn đầy đủ nữa.


a) Với0 k 2 0 1 8 ; chứng minh rằng tồn tại bảng tối giản2k 2 0 1 8 sao cho có


đúngk cột có đủ cả 0lẫn1 :


b) Cho bảng tối giảnm2 0 1 8có đúngkcột chứa cả0lẫn1. Chứng minh rằngm 2k:
(Một dãy nhị phân độ dài 2 0 1 8 là dãy có dạng x1x2: : : x2 0 1 8 trong đó xi 2 f0 ; 1g


với mọii 2 f1 ; 2 ; : : : ; 2 0 1 8g:)


Lời giải. a)Đầu tiên, xét một bảng ơ vng trống kích thước2k 2 0 1 8 và2k xâu nhị phân


có độ dàik ;ta sẽ tiến hành điền các số 0 ; 1của các xâu này lần lượt vào phần bên trái của từng


hàng, mỗi xâu dùng cho một hàng. Khi đó, phần cịn lại phía bên phải là2 0 1 8 k cột được


để trống. Dễ thấy rằng ở mỗi cột trongk cột bên trái của bảng đều có chứa đúng2k 1 số0 và


2k 1 số1 (tức là có đủ cả0lẫn 1như ràng buộc). Ta sẽ chứng minh bảng này tối giản.



0 0 : : : 0 : : : : : : : : :


0 0 : : : 1 : : : : : : : : :


: : : : : : : : : : : : 0 : : : : : :


1 1 : : : 0 : : : : : : : : :


1 1 : : : 1 : : : : : : : : :


Với một xâu nhị phân s D a1a2 : : : a2 0 1 8 tùy ý, ta xét xâu con s0 D a1a2: : : ak của


nó. Rõ ràng s0 đã xuất hiện ở phần đầu của hàng nào đó trên bảng nên nếu ta điền tiếp


akC1; akC2; : : : ; a2 0 1 8 vào phần trống thì sẽ thu đượcs : Ngồi ra, ta thấy rằng cũng chỉ có


đúng một hàng chứa các số củas0nên nếu bỏ hàng đó đi thì khơng có hàng nào có thể dùng để


“khơi phục” s được. Do đó, bảng đã cho thỏa mãn.


b)Giả sử rằng k cột đầu tiên bên trái của bảng có chứa đủ 0và1 :Dưới đây, ta sẽ chứng minh


nhận xét quan trọng mấu chốt để giải bài tốn này.


Nhận xét.<i>Tất cả các ơ trong</i>2 0 1 8 k <i>cột bên phải đều trống.</i>


Chứng minh.Xét xâu nhị phâns có độ dài k bất kỳ và giả sửAs là tập hợp các hàng màk ô


đầu tiên bên trái của nó có thể sinh ra s :Ta sẽ chứng minh rằng trong As sẽ có một hàng mà



tồn bộ các ơ từ vị trík C1 đến vị trí2 0 1 8 đều trống.


Xét các ơ ở vị trí thứkC1ở mỗi hàng trongAs:Dễ thấy tất cả các ơ đó thuộc về cột thứkC1


của bảng, là cột không chứa đồng thời0 và 1 :


Nếu tồn bộ cột này có chứa số thì giả sử đều là số0 (tương tự nếu là1). Khi đó, chuỗi độ dài


k C 1có dạng 0 0 : : : 0 1gồm k số0bên trái sẽ không thể biểu diễn được bởi bất cứ hàng nào,


mâu thuẫn với tính đầy đủ của bảng. Do đó, phải có một tập con của As có các hàng mà tại vị trí


</div>
<span class='text_page_counter'>(171)</span><div class='page_container' data-page=171>

<b>Epsilon</b>



<b>staff</b>



<i>Lời giải và bình luận đề thi chọn đội tuyển IMO 2018</i> <b>9</b>


Tiếp tục, lại xét vị trí thứ k C 2thì tương tự trên, nếu tồn bộ cộtk C 2có chứa số thì khơng


thỏa nên phải có cột trống, như vậy ta lại tiếp tục chọn được tập con A00s của A0s mà vị trí thứ


k C 1 ; k C 2đều trống. Cứ làm như thế cho đến khi gặp cột cuối cùng và ta được một hàng có


tất cả các vị trí từk C 1đến 2 0 1 8đều trống.


Như thế, với mỗi xâu nhị phâns có độ dàik, ta ln tìm được một hàng mà tất cả các ô từ vị trí


k C1đến2 0 1 8 đều trống. Chú ý rằng các hàng này khơng nhất thiết phân biệt vì có thể có một



hàng được sử dụng cho nhiều xâu. Gọi tập hợp các hàng này làA :


Ta thấy Acó thể biểu diễn được cho tất cả các xâu độ dài 2 0 1 8 :Rõ ràng mỗi hàng trên bảng


đều phải thuộc vàoAvì nếu khơng, ta có thể loại bỏ hàng đó đi mà bảng vẫn cịn đầy đủ, mâu


thuẫn với tính chất tối giản. Như vậy,A cũng chính là tập hợp tất cả các hàng của bảng. Do đó,


tồn bộ các cột từk C 1đến 2 0 1 8của bảng đều trống. Nhận xét được chứng minh. <sub></sub>


Tiếp theo, vì tồn bộ bảng con bên phải trống nên có thể biểu diễn được mọi xâu nhị phân độ dài


2 0 1 8 k. Vì bảng ban đầu là tối giản nên buộc bảng con bên trái cũng tối giản.


Xóa đi2 0 1 8 k cột trống để còn bảng con kích thước m k :Đánh số các hàng từ1đến m


và đặt Ai . i D 1 ; 2 ; : : : ; m /là tập hợp các xâu nhị phân độ dàik có thể sinh ra từ hàng thứ


i (chú ý rằng trên các hàng đó vẫn có thể cịn các ơ trống xen kẽ, khơng nhất thiết phải được điền


tồn bộ bởi các số 0và 1).


Vì bảng ban đầu tối giản đối với các xâu nhị phân độ dài 2 0 1 8nên bảng mới sinh ra cũng tối


giản với các xâu nhị phân độ dài k :Suy ra nếu đặtB D A1 [ A2 [ [Amthì jBj D 2k


(nghĩa là bảng này có thể sinh ra được tất cả 2k xâu nhị phân độ dài k).


Rõ ràng với mỗi i . i D 1 ; 2 ; : : : ; m / ; phải có một xâu nhị phân độ dài k nào đó chỉ được



sinh ra bởi hàng thứi bởi nếu ngược lại, tất cả các xâu nhị phân sinh ra từ hàngi cũng được sinh


ra từ hàng khác thì ta bỏ hàng đó đi dẫn đến bảng khơng cịn tối giản nữa. Điều này có nghĩa là


Ai phải đóng góp ít nhất một phần tử vàoAvà phần tử này khơng thuộc các tập khác. Do đó m


tập hợp sẽ đóng góp ít nhấtm phần tử vào B và kéo theo jBj m :


So sánh các đánh giá trên, ta có ngaym 2k. Đây chính là điều phải chứng minh.


Bình luận. Ở câua), với số hàng của bảng là2k <sub>và với điều kiện đặt ra là đúng</sub><sub>k</sub> <sub>cột có chứa</sub>


cả0lẫn 1 ;ta có thể dễ dàng nghĩ đến việc xét tất cả các xâu nhị phân có độ dàik (hiển nhiên có


đúng2k xâu như thế).


Cái khó nhất của câub)chính là tính tùy ý của các cột trong2 0 1 8 kcột cịn lại, ngồi trường


hợp đặc biệt vừa xét, vẫn có thể cịn các cột chỉ được điền0 ;chỉ được điền 1 :Ta phải dùng tính


tối giản của bảng để loại các trường hợp đó đi. Chú ý rằng câub)trong đề cho sốk tùy ý nên có


thể xuất phát từ trường hợpk D 0để thấy rằng tất cả các ơ đều phải trống. Đây cũng chính là


định hướng của lời giải trên.


Với quan hệ giữa các hàng của bảng và các chuỗi nhị phân, bài toán này hứa hẹn sẽ có cách tiếp
cận sử dụng lý thuyết graph, đặc biệt là bipartie graph. Mong được trao đổi thêm với bạn đọc.
Dạng tốn về tập hợp mơ hình hóa trên bảng này đã từng xuất hiện một lần trong đề VMO 2015.


Dưới đây, ta xét một số bài toán tương tự:


Bài toán 1 (IMO Shortlist, 1998).<i>Cho số nguyên</i> n 2<i>và tập hợp</i> A D f1 ; 2 ; : : : ; ng:


<i>Một họ</i>F <i>gồm</i>t <i>tập hợp con của</i> A ; <i>đặt là</i>A1; A2; : : : ; At <i>được gọi là “rời nhau” nếu</i>
<i>như với mỗi cặp số</i>fx ; yg <i>lấy từ</i> A<i>thì tồn tại</i>Ai 2 F <i>mà</i>


ˇ


ˇAi \ fx ; yg


ˇ


ˇ D 1 :<i>Họ</i> F <i>được</i>


<i>gọi là “bao phủ” nếu như mỗi phần tử của</i>A <i>đều thuộc vào ít nhất một tập nào đó trong</i> F :


</div>
<span class='text_page_counter'>(172)</span><div class='page_container' data-page=172>

<b>Epsilon</b>



<b>staff</b>



<b>10</b> <i>Lời giải và bình luận đề thi chọn đội tuyển IMO 2018</i>


Bài toán 2 (Bổ đề Kleitman).<i>Một họ</i>F <i>gồm các tập con của tập hợp</i> n<i>số nguyên dương đầu</i>
<i>tiên được gọi là “down closed” nếu mỗi</i>X 2 F <i>thì tất cả tập con của</i>X <i>cũng thuộc</i>F : <i>Một họ</i>


F <i>gồm các tập hợp được gọi là “up closed” nếu mỗi</i> X 2 F <i>thì tất cả các tập hợp nhận</i> X <i>là</i>
<i>tập con cũng thuộc</i>F : <i>Chứng minh rằng</i>


a) <i>Nếu</i> F1; F2 <i>là hai họ “down closed” thì</i>


jF1 \F2j


jF1j jF2j


2n :


b) <i>Nếu</i> F1; F2 <i>lần lượt là họ “down closed” và “up closed” thì</i>
jF1 \F2j j


F1j jF2j


2n :


Bài toán 3 (Iran, 2001).<i>Với số nguyên dương</i> n 2 ; <i>xét bảng ô vuông</i> n n <i>được điền</i>
<i>các số</i> 0 <i>và</i>1 ; <i>một “đường chéo suy rộng” là tập hợp</i>n <i>phần tử không cùng hàng và cùng</i>
<i>cột. Giả sử trên bảng này có đúng một đường chéo suy rộng chứa tồn số</i> 1 : <i>Chứng minh</i>
<i>rằng có thể sắp xếp lại các hàng và cột của bảng để được bảng mới mà mọi số tại vị trí</i>


. i ; j / <i>với</i> 1 j < i n <i>đều là</i>0 :


Bài tốn 4.<i>Xét bảng ơ vng</i>m n <i>có chứa các số</i>0 <i>hoặc</i>1 :<i>Giả sử tồn bộ bảng có ít nhất</i>


˛ m n<i>số</i>1<i>với</i>0 < ˛ < 1 :<i>Chứng minh rằng có ít nhất một trong hai ràng buộc sau phải đúng</i>:


i) <i>Có ít nhất một hàng có chứa ít nhất</i>np˛ <i>số</i>1 :


</div>
<span class='text_page_counter'>(173)</span><div class='page_container' data-page=173>

<b>Epsilon</b>



<b>staff</b>




<i>Lời giải và bình luận đề thi chọn đội tuyển IMO 2018</i> <b>11</b>


Bài 3 (7.0 điểm). Cho số nguyênn 3và An là tập hợp tất cả các số nguyên dương nhỏ


hơnn ; nguyên tố cùng nhau vớin : Xét đa thức
Pn. x / D


X


k2An


xk 1:


a) Chứng minh rằngP . x /chia hết cho đa thức xr C1 vớir là số ngun dương nào đó.


b) Tìm tất cả các số nguyên dương nđểPn. x /bất khả quy trênZŒ x  :


Lời giải. a)Trước hết, ta thấy rằng vớim ; k 2 ZCvà klẻ thìxk m C1chia hết cho xm C1 :


Để thuận lợi trong lập luận, xét đa thứcQn. x / D x Pn. x / D Pk2Anx


k <sub>và dễ thấy chỉ cần</sub>


chứng minhQn. x /chia hết choxrC1với sốr 2 ZCnào đó là được. Xét các trường hợp sau:


Nếu n là số lẻ:Vì gcd. n ; k / D gcd. n ; n k / với mọi k D 1 ; 2 ; : : : ; n 1


nên với k 2 An thì n k 2 An: Suy ra, ta có thể nhóm các đơn thức trongPn. x /


thành các cặp rời nhau có dạng . xk<sub>; x</sub>n k<sub>/ :</sub> <sub>Vì</sub> <sub>k ; n</sub> <sub>k</sub> <sub>khác tính chẵn lẻ nên</sub>



. n k / k D n 2 k lẻ và xk C xn k D xk. xn 2 k C1 / chia hết chox C 1 :Từ


đó suy ra Qn. x /chia hết chox C1 ; thỏa mãn.


Nếu n chia hết cho 4:Tương tự như trường hợp trên, ta cũng ghép cặp.xk; xn k/với


chú ý hiệun 2k là số chia4 dư2:Suy raxk Cxn k D xk.xn 2k C1/chia hết cho


x2<sub>C</sub><sub>1:</sub><sub>Do đó,</sub><sub>Q</sub>


n.x/chia hết chox2C1;cũng thỏa mãn.


Nếu n chia 4 dư 2:Ta sẽ chứng minh các nhận xét sau:


Nhận xét 1.<i>Nếu</i>Qn.x/<i>chia hết cho</i>xrC1 .r 2ZC/<i>thì với số nguyên tố lẻ</i>p<i>mà</i>pjn;
<i>đa thức</i>Qpn.x/<i>cũng thế.</i>


Chứng minh.Giả sử a 2 An thì các sốkp C a 2 Ap n vớik D 0 ; 1 ; : : : ; p 1 :


Ứng với mỗi số k ;tổng các đơn thức với mũ là kp C avới a 2 Antương ứng sẽ có


cùng nhân tử là kp và sẽ chia hết cho P


a2Ax
a


, tức là cũng chia hết cho xr C 1 :Do


đó, Qp n. x / chia hết choxr C 1 :



Nhận xét 2.<i>Với</i> n D 2 p1p2 pm; <i>trong đó</i>m 2 ZC <i>và</i> p1; p2; : : : ; pm <i>là các</i>
<i>số nguyên tố lẻ phân biệt tùy ý thì</i>Qn. x /<i>sẽ phân tích được.</i>


Chứng minh.Ta sẽ chứng minh bằng quy nạp theo mlà số ước nguyên tố lẻ của n :


Nếum D 1thì khin D 2 pvớip nguyên tố, dễ thấyAn D f1 ; 2 ; : : : ; 2 pg nfp ; 2 pg


nênQn. x /sẽ chia hết choxp C 1 :


Giả sử khẳng định đúng với m 1 : Xétn D 2 p1p2 pm và p là số nguyên tố lẻ,


gcd. p ; n / D 1 ; trong đó Qn. x / chia hết cho xr C 1 : Ta sẽ chứng minh rằng với


N D p n thìQN. x /cũng chia hết choxr C1 :


Trước hết, giả sử trongAN có tính ln cả các số ngun tố cùng nhau vớinnhưng lại


chia hết chop thì thay vì có QN. x / ;ta sẽ có một đa thức mới là RN. x / :Đa thức này


sẽ là sự lặp lại p lần của của đa thức Qn. x / (nghĩa là RN. x / có thể chia thành các


nhóm mà mỗi nhóm so với Qn. x /chỉ sai khác lũy thừaxkp với1 k p 1). Do


</div>
<span class='text_page_counter'>(174)</span><div class='page_container' data-page=174>

<b>Epsilon</b>



<b>staff</b>



<b>12</b> <i>Lời giải và bình luận đề thi chọn đội tuyển IMO 2018</i>



Tiếp theo, ta cần bỏ đi các lũy thừa chia hết chop, đó là các đơn thức có dạng xap với


a 2 An:Khi đó, dễ thấy tổng của chúng sẽ là đa thứcQn. xp/mà Qn. xp/chia hết cho


xp r <sub>C</sub><sub>1</sub><sub>, tức là cũng chia hết cho</sub> <sub>x</sub>r <sub>C</sub><sub>1 :</sub><sub>Tóm lại</sub> <sub>Q</sub>


N. x / D RN. x / Qn. xp/chia


hết choxr C 1 :Do đó, khẳng định cũng đúng với sốncóm C 1ước nguyên tố lẻ. Theo


nguyên lý quy nạp thìnhận xét 2được chứng minh. <sub></sub>


Kết hợp hai nhận xét lại, ta thấy trường hợp này vẫn thỏa mãn điều kiện. Bài tốn được
giải quyết hồn tồn.


b)Theo câua), ta đưa về tìm tất cả các số nguyên dươngn 3 sao chojAnj D 2 ; điều này


tương đương với việc tìmn 3để' . n / D 2 : Ta xét các trường hợp sau:


Nếun D p nguyên tố thì2 D ' . n / D n 1 nênn D 3thỏa mãn.


Nếu n D p˛ với ˛ > 1 vàp nguyên tố thì 2 D ' . n / D p˛ 1. p 1 / nên dễ thấy
rằngp D 2 ; ˛ D 2và n D 4thỏa mãn.


Nếu n có ít nhất hai ước nguyên tố khác nhau là p ; q thì theo tính chất nhân tính của


hàm Euler,2 D ' . n /sẽ chia hết cho' . p / D p 1 và' . q / D q 1nên chỉ có thể


p D 2 và q D 3 ;tương ứng với n D 6thỏa mãn.



Vậy tất cả các số ncần tìm làn 2 f3 ; 4 ; 6g:


Bình luận. Câub)của bài tốn thực sự rất nhẹ nhàng và được hướng dẫn trực tiếp từ câua).


Nếu đề bài chỉ cho ýa)hoặc ýb)thì sẽ trọn vẹn hơn nhiều. Câu a)thực sự là một kết quả đẹp


và thú vị, nhất là trong trường hợp n là số square-free chẵn (n không chia hết cho số chính


phương lớn hơn1 nào). Có một ý tưởng để giải quyết trường hợp này là chứng minh số lượng


các số có dạng4 k C 1và 4 k C 3 trong An là bằng nhau; tuy nhiên, điều này chỉ đúng khi


n có ước nguyên tố dạng4 k C 1. Điều này có thể chứng minh nhờ hai nhận xét:


TrongAnln chứa số có dạng b D 4 k C 3(nếun D 4 mC 2thì đó là một trong hai


sốb D 2 mC 3và b D 2 mC 5).


Tập hợpA0


n D fb a ja 2 Angcó cùng số lượng số chia 4dư1 và dư3 nhưAn. Chú ý


rằng nếu a 1 .mod4 / thìb a 3 .mod4 /và ngược lại.


Một kết quả thú vị khác là đa thứcPn. x /chia hết cho x2


k


C 1nếu như có một số nguyên tốp



thỏa mãn pjn và v2. p C1 / D k :Bạn đọc hãy tự chứng minh kết quả này.


Bài toán cũng có thể giải bằng tổng Ramanujan với chú ý rằng nếu đặtCn. r / D Qn. r/mà


D e2 i n thìC<sub>m n</sub> D C<sub>m</sub>C<sub>n</sub> với mọim ; n nguyên dương thỏa gcd. m ; n / D 1 :


Dưới đây là một số bài toán tương tự:


Bài toán 1.<i>Với mỗi số nguyên dương</i> n ;<i>xét đa thức</i> Pn. x / D Pa2Anx


a 1


<i>với</i>An<i>là tập</i>
<i>hợp các ước dương của</i> n :<i>Tìm tất cả các số nguyên dương</i>n <i>sao cho</i>Pn. x /<i>bất khả quy.</i>


Bài toán 2(China MO, 2013).<i>Với mỗi số nguyên dương</i>n ;<i>xét đa thức</i>Pn. x / D Pn<sub>i</sub><sub>D</sub><sub>0</sub>cixi
<i>với</i> ci 2 f0 ; 1g <i>và</i> ci Cni .mod2 / :


a) <i>Với</i> m ; n ; x0 <i>là các số nguyên dương và</i> x0 C1 <i>không phải là lũy thừa của</i> 2 ; <i>chứng</i>


<i>minh rằng nếu</i>Pm. x0/<i>chia hết cho</i> Pn. x0/<i>thì</i>Pm. x /<i>chia hết cho</i> Pn. x /<i>với mọi</i>
<i>số nguyên dương</i> x :


</div>
<span class='text_page_counter'>(175)</span><div class='page_container' data-page=175>

<b>Epsilon</b>



<b>staff</b>



<i>Lời giải và bình luận đề thi chọn đội tuyển IMO 2018</i> <b>13</b>


Bài 4 (7.0 điểm). Cho a là số thực thuộc đoạn 1


2;


2
3




: Xét các dãy số . un/ và . vn/


. n D 0 ; 1 ; : : : /được xác định như sau:


un D


3


2nC1 . 1 /


b2nC1<sub>a</sub><sub>c</sub>


; vn D


3


2nC1 . 1 /


nCb2nC1<sub>a</sub><sub>c</sub>
:


a) Chứng minh rằng



2 0 1 8


X


iD0


ui


!2


C


2 0 1 8


X


iD0


vi


!2


7 2 a2 4 8 a C 1 0C 2


42 0 1 9:
b) Tìm tất cả các giá trị của ađể đẳng thức xảy ra.


Lời giải. a)Từ giả thiết, ta cóvi D ui vớii chẵn vàvi D ui vớii lẻ nên bất đẳng thức cần
chứng minh có thể được viết lại thành



1 0 0 9


X


iD0


u2 i C
1 0 0 8


X


iD0


u2 iC1


!2
C


1 0 0 9


X


iD0


u2 i


1 0 0 8


X



iD0


u2 iC1


!2


7 2 a2 4 8 aC1 0C 2


42 0 1 9;
hay


1 0 0 9


X


iD0


u2 i


!2
C


1 0 0 8


X


iD0


u2 iC1



!2


3 6 a2 2 4 a C5 C 1


42 0 1 9: . 1 /


Bây giờ, gọi biểu diễn nhị phân của alà


a D
C1
X


iD1


xi


2i


với xi 2 f0 ; 1g: Do 1<sub>2</sub> a 2<sub>3</sub> nên dễ thấy x1 D 1 : Với mỗi số tự nhiên i ; ta thấy tính


chẵn lẻ của b2iC1ac phụ thuộc vào xiC1; cụ thể: NếuxiC1 D 0thì b2iC1ac chẵn và nếu


xiC1 D 1thì b2iC1ac lẻ. Suy ra. 1 /b2


iC1<sub>a</sub><sub>c</sub>


D 1nếu xiC1 D 0 và. 1 /b2


iC1<sub>a</sub><sub>c</sub>



D 1


nếuxiC1 D 1 :Trong mọi trường hợp, ta đều có


. 1 /b2iC1ac D 1 2 xiC1:
ĐặtA D P1 0 0 9


iD0


x2 iC1


22 iC1 và B D


P1 0 0 8


iD0
x2 iC2


22 iC2;ta có


1 0 0 9


X


iD0


u2 i D
1 0 0 9


X



iD0


3 . 1 2 x2 iC1/


22 iC1 D 2


1


2 41 0 0 9 6 A ;
1 0 0 8


X


iD0


u2 iC1 D
1 0 0 8


X


iD0


3 . 1 2 x2 iC2/


22 iC2 D 1


1


41 0 0 9 6 B :



Ngồi ra, ta cũng cóa A C B 12 nên


3 6 a2 2 4 a C 5C 1


42 0 1 9 D 4 . 3 a 1 /
2


C 1C 1


42 0 1 9


4 . 3 A C 3B 1 /2 C 1 C 1


</div>
<span class='text_page_counter'>(176)</span><div class='page_container' data-page=176>

<b>Epsilon</b>



<b>staff</b>



<b>14</b> <i>Lời giải và bình luận đề thi chọn đội tuyển IMO 2018</i>


Ta sẽ chứng minh




2 1


2 41 0 0 9 6 A


2
C





1 1


41 0 0 9 6 B


2


4 . 3 AC3B 1 /2C1C 1


42 0 1 9: . 3 /
Bằng biến đổi tương đương, ta viết được bất đẳng thức trên dưới dạng


6


41 0 0 9A C 1 2 B




1 C 1


41 0 0 9 6 A




1


41 0 0 8



1
42 0 1 8:
DoA 12 nên6 A > 1 C


1


41 0 0 9:Suy ra


6


41 0 0 9A C 1 2 B




1 C 1


41 0 0 9 6 A




6


41 0 0 9A


6


41 0 0 9
1 0 0 9


X



iD0


1
22 iC1


D 6


41 0 0 9


2
3




1 1


41 0 1 0




D 1


41 0 0 8


1
42 0 1 8:


Bất đẳng thức (3) được chứng minh. Bằng cách sử dụng các bất đẳng thức (2) và (3), ta thu được
bất đẳng thức (1). Ta có điều phải chứng minh.



b)Từ các đánh giá ở trên, ta thấy dấu đẳng thức xảy ra khi và chỉ khi a D A C B ; B D 0 và


A D 23 1
1
41 0 1 0




; hay a D 23 1
1
41 0 1 0




:


Bình luận. Biểu thức <sub>b</sub>2nC1acchính là gợi ý quan trọng trong việc xét biểu diễn nhị phân của


a :Thực tế thì khơng có cách nào xác định được tính chẵn lẻ của đại lượng này nếu không dùng


đến biểu diễn nhị phân của a : Ngồi ra, trong bài tốn trên, chúng tơi đã sử dụng đẳng thức


. 1 /a D 1 2 a với a 2 f0 ; 1gđể lời giải được gọn gàng hơn.


Biểu diễn nhị phân (và biểu diễn p-phân nói chung) là một cơng cụ quan trọng trong tốn học,


có nhiều ứng dụng trong tổ hợp, số học, đại số và giải tích. Theo dõi các Shortlist của IMO, ta
thấy ý tưởng này xuất hiện khá nhiều. Dưới đây chúng tơi xin đưa ra một số bài tốn như vậy:



Bài toán 1 (Putnam, 1981).<i>Gọi</i> f . n /<i>là số chữ số</i> 1<i>trong biểu diễn nhị phân của số nguyên</i>
<i>dương</i> n :<i>Tính giá trị của tổng</i>:


1
X


nD1


f . n /
n . n C 1 /:


Bài toán 2 (IMO Shortlist, 1983).<i>Cho</i> f W Œ 0 ; 1  ! R <i>là hàm số liên tục thỏa mãn đồng</i>
<i>thời các điều kiện</i>:


8
ˆ
<


ˆ
:


f . x / D bf . 2 x / ; 0 x 1


2;


f . x / D b C . 1 b / f . 2 x 1 / ; 1


2 x 1 ;


</div>
<span class='text_page_counter'>(177)</span><div class='page_container' data-page=177>

<b>Epsilon</b>




<b>staff</b>



<i>Lời giải và bình luận đề thi chọn đội tuyển IMO 2018</i> <b>15</b>


Bài toán 3 (IMO, 1988).<i>Hàm số</i> f <i>xác định trên tập hợp các số nguyên dương được cho</i>


<i>bởi</i>: f . 1 / D 1 ; f . 3 / D 3 ; f . 2 n / D f . n / ; f . 4 n C 1 / D 2f . 2 n C 1 / f . n / <i>và</i>


f . 4 nC3 / D 3f . 2 nC1 / 2f . n /<i>với mọi số nguyên dương</i>n :<i>Tìm số tất cả các số nguyên</i>


<i>dương</i> n ;<i>nhỏ hơn hay bằng</i> 1 9 8 8<i>thỏa mãn điều kiện</i>f . n / D n :


Bài toán 4 (IMO Shortlist, 1996).<i>Cho dãy số</i> a . n / . n D 1 ; 2 ; 3 ; : : : / <i>được xác định bởi</i>


a . 1 / D 0 <i>và với mọi</i>n > 1<i>thì</i>


a . n / D a jn


2
k


C . 1 /n . n2C1 /<sub>:</sub>


a) <i>Tìm giá trị lớn nhất của</i> a . n /<i>với</i>n <i>là các số nguyên dương</i> 1 9 9 6 <i>và tìm tất cả các</i>
<i>giá trị</i> n 1 9 9 6<i>để giá trị lớn nhất đó đạt được.</i>


</div>
<span class='text_page_counter'>(178)</span><div class='page_container' data-page=178>

<b>Epsilon</b>



<b>staff</b>




<b>16</b> <i>Lời giải và bình luận đề thi chọn đội tuyển IMO 2018</i>


Bài 5 (7.0 điểm). Một bảng ô vuông m n A B C D có các đỉnh là các giao lộ (có tất
cả . mC 1 / . n C 1 / giao lộ). Người ta muốn thiết lập một tuyến đường bắt đầu từA ;
đi theo các cạnh song song với các cạnh của hình chữ nhật và đi qua tất cả các giao lộ
đúng một lần, sau đó quay về A :


a) Chứng minh rằng có thể xây dựng được đường đi khi và chỉ khi mlẻ hoặc nlẻ.


b) Vớim ; nthỏa mãn điều kiện câu a), hỏi có ít nhất bao nhiêu giao lộ mà tại đó có ngã rẽ?


Lời giải. Đánh số các hàng và cột theo thứ tự từ trái sang phải1 ! m C1 ; trên xuống dưới


1 ! n C 1 :Khi đó, điểmA sẽ ở góc trên bên trái với vị trí. 1 ; 1 / :


a) Điều kiện cần:Ta thấy rằng đường đi có thể được biểu diễn bởi dãy các ký tự L ; R ; U


và D ; trong đó các ký tự này lần lượt chỉ hướng đi sang trái, sang phải, lên trên và xuống


dưới tại mỗi giao lộ.


Do có tổng cộng. m C 1 / . nC 1 / giao lộ nên cũng có bấy nhiêu đó số ký tựL ; R ; U ; D:


Vì đường đi xuất phát từ A và quay trở về A nên số lần rẻ trái bằng số lần rẻ phải, số lần đi


xuống bằng số lần đi lên. Điều này chứng tỏ số ký tự L ; R bằng nhau, số ký tựU ; D bằng


nhau. Suy ra tổng số ký tự là . mC 1 / . n C 1 /phải chẵn. Vì thế nên phải có m lẻ hoặcn lẻ.



Điều kiện đủ:Giả sửm là số lẻ, trường hợpn lẻ hoàn toàn tương tự. Ta sẽ đi theo quy tắc sau
(có mơ tả như hình trên):


Lần đi ngang đầu tiên: xuất phát tạiA ; đi từ cột1 ! n.


Mỗi lần đi dọc thì chỉ xuống một ơ; đi ngang thì di chuyển giữa cột 2 $ n :


Lần đi ngang cuối cùng: đi từ cộtn ! 1sau đó đi ngược từ hàng n ! 1 đến điểmA :


Điều này thực hiện được dom lẻ nên số tuyến đường nằm ngang là chẵn. Vậy điều kiện cần và


đủ để đi được như đề bài là mhoặc nphải lẻ.


b)Xét hai giao lộ có ngã rẽ gần nhau nhất (điểmA tạm tính là một trong các giao lộ đó) thì giữa


hai giao lộ sẽ có một đoạn đường, nằm ngang hoặc nằm dọc. Ta thấy đường đi sẽ là một dãy các
đoạn dọc và ngang luân phiên (mỗi đoạn đi qua hai hoặc nhiều giao lộ), nếu xuất phát theo chiều
ngang thì về theo chiều dọc và ngược lại.


Gọi số đoạn ngang là r, đoạn dọc làc và k là số ngã rẽ (khơng tính A). Ta sẽ chứng minh


các nhận xét sau:


</div>
<span class='text_page_counter'>(179)</span><div class='page_container' data-page=179>

<b>Epsilon</b>



<b>staff</b>



<i>Lời giải và bình luận đề thi chọn đội tuyển IMO 2018</i> <b>17</b>


Chứng minh.Tại mỗi giao lộ chuyển có ngã rẽ sẽ đúng một đoạn ngang và dọc. Số ngã rẽ,



tính thêm cả A ;chính bằng số cặp khơng thứ tự có dạng fđoạn ngang, đoạn dọcgmà đoạn


ngang và dọc có chung đầu mút tại một giao lộ nào đó. Hơn nữa, mỗi đoạn ngang có chung


đầu mút với đúng 2đoạn dọc, và mỗi đoạn dọc có chung đầu mút với đúng 2đoạn ngang nên


k C 1 D 2 r D 2 c : <sub></sub>


Để tìm giá trị nhỏ nhất củak, ta đưa về tìm giá trị nhỏ nhất của r và c :


Nhận xét 2.r m C 1<i>hoặc</i>c nC 1 :


Chứng minh.Giả sử rằng r m thì số đoạn ngang không xuất hiện đủ trên m C 1hàng,


do đó có một hàng mà cả n C 1 giao lộ trên đó đều ứng với đoạn dọc, tức là c n C 1 :


Tương tự nếuc n thìr m C 1 :


Từ đó, ta đưa về xét các trường hợp sau đây:


Nếu m lẻ, n chẵn:Giả sửr mthì theo nhận xét trên, có một hàng mà cảnC1giao


lộ đều ứng với đoạn dọc, nhưng vì có lẻ điểm trên hàng này nên với các đoạn dọc đó, ta


khơng thể quay về đượcA, khơng thỏa. Do đó r mC1vàk D 2 r 1 2 mC1.


Ta dễ dàng xây dựng được đường đi với đúngm C 1 đoạn dọc tương tự câu a). Vì thế


nên mink D 2 m C1 :



Nếu m chẵn, n lẻ:Lập luận tương tự, ta có minkD2nC1:


Nếu m; n đều lẻ:Dễ thấy rằng cả hai đẳng thức trong các đánh giár mC1; c nC1


đều có thể xảy ra nên mink D2min.m; n/C1:


Nói tóm lại:


Nếum; nđều lẻ thì mink D2min.m; n/C1:


Nếumchẵn,nlẻ thì minkD2nC1:


Nếumlẻ,nchẵn thì minkD2mC1:


Bình luận. Đây là một bài toán khá kinh điển về đường đi Hamilton qua tất cả các đỉnh trên


grid graph, có thể tham khảo thêm tại liên kết: />


GridGraph.html.


Kết quả câua)đã cũ và thậm chí, ta cịn có thể đếm được số đường đi như thế theo m ; nnhờ


công thức truy hồi. Ngồi cách giải như trên, ta cũng có thể tiếp cận theo hướng tô màu như sau:


</div>
<span class='text_page_counter'>(180)</span><div class='page_container' data-page=180>

<b>Epsilon</b>



<b>staff</b>



<b>18</b> <i>Lời giải và bình luận đề thi chọn đội tuyển IMO 2018</i>



Khi đó, mỗi lần đi từ giao lộ này sang giao lộ kia, màu sẽ được thay đổi. Để đi qua hết các giao


lộ, mỗi giao lộ một lần rồi quay về đúng A thì rõ ràng, màu phải được đổi chẵn lần. Suy ra


. m C1 / . n C 1 / chẵn hay ít nhất một trong hai sốm ; nlẻ.


Ở câub), ta cần phải đánh giá cẩn thận hơn về quan hệ giữa số lần đi ngang và đi dọc cũng như


vai trị tính chẵn lẻ của sốm ; n :


Một số bài toán tương tự:


Bài tốn 1 (Tạp chí Crux). <i>Một con chuột ăn một miếng phơ mai hình lập phương kích</i>
<i>thước</i> 3 3 3<i>gồm</i> 2 7 <i>miếng nhỏ</i>1 1 1 .<i>nó chỉ ăn phần ruột bên trong mỗi miếng</i>/ :


<i>Hỏi nếu nó xuất phát từ một miếng ở góc thì có thể đi qua được</i> 2 7 <i>miếng và kết thúc tại</i>
<i>miếng ở giữa khơng, biết rằng nó chỉ được di chuyển giữa hai miếng có chung mặt với nhau</i>
<i>và không quay về miếng đã đi qua</i>‹


</div>
<span class='text_page_counter'>(181)</span><div class='page_container' data-page=181>

<b>Epsilon</b>



<b>staff</b>



<i>Lời giải và bình luận đề thi chọn đội tuyển IMO 2018</i> <b>19</b>


Bài 6 (7.0 điểm). Cho tam giác A B C nhọn nội tiếp . O / và . J /là đường trịn bàng tiếp
góc Acủa tam giác. Gọi D ; E ; F lần lượt là tiếp điểm của. J /với B C ; C A và A B :


a) Gọi L là trung điểm của B C : Đường trịn đường kính LJ cắt các đường thẳng



DE ; DF lần lượt tại K và H : Chứng minh rằng các đường tròn . BDK / và


. C DH / cắt nhau trên đường tròn. J / :


b) Giả sử đường thẳng EF cắt đường thẳng B C tại G và GJ cắt các đường thẳng


A B ; A C lần lượt tại M ; N :GọiP vàQlà các điểm trên các đường thẳngJ B ; J C


sao cho <sub>∠</sub>P A B D ∠QA C D 9 0ı: Gọi T là giao điểm của hai đường thẳng


P M ; QN và S là điểm chính giữa cung lớnB C của đường tròn. O / : GọiI là tâm


đường tròn nội tiếp tam giác A B C : Chứng minh rằng đường thẳng S I cắt đường
thẳngA T tại một điểm thuộc đường tròn. O / :


Lời giải. a)Gọi D0và F0 lần lượt là tiếp điểm của đường tròn . I /nội tiếp tam giác A B C


với B C ; BA :Dựng đường kínhD W của đường trịn. J / : Ta có


A I
A J D


I F0


J F D
I D0


J W


nên ba điểmA ; D0<sub>và</sub> <sub>W</sub> <sub>thẳng hàng.</sub>



A


B C


J
F


E
O


D
L
I


H
D0


F0


D1


W


X U


U0


Z
V



</div>
<span class='text_page_counter'>(182)</span><div class='page_container' data-page=182>

<b>Epsilon</b>



<b>staff</b>



<b>20</b> <i>Lời giải và bình luận đề thi chọn đội tuyển IMO 2018</i>


GọiD1là giao điểm của các đường thẳngID0vàAD:Ta cóJD DJ W nênID0DID1:Mà


BD0 DCDnênLD DLD0;từ đó suy raILkDD1:


GọiXlà trung điểm củaD0W thì tứ giácDLXJ là hình chữ nhật. Suy ra<sub>∠</sub>XHD D90ı;hay


XH kJB:Từ đóXH ?BI:


Dựng hình chữ nhật C D J U : Ta có J U k C D và J U D C D nên J U k BD0 và


J U D BD0; suy ra tứ giác BD0U J là hình bình hành. Từ đây, ta thu được D0U k XH :


Gọi Y ; V vàZ lần lượt là giao điểm của đường thẳngXH với các đường thẳng D J ; U W ;
U C :Khi đó, ta cóV là trung điểm củaU W ; màU Z k Y W nênV là trung điểm của Y Z :


Xét tứ giác C E U J nội tiếp có C E D C D D U J nên U E k C J hay U E ? DE : Mà


E W ? ED nên ba điểmW ; U và E thẳng hàng.


Xét các tam giácB I C và Y V W ;ta có


∠Y W V D ∠C ED D ∠I C B





∠W Y V D ∠X Y J D ∠B J D D ∠I B C


nên4B I C 4Y V W (g-g). MàV là trung điểm củaY ZvàLlà trung điểm củaB C nên ta
có4B I L 4Y Z W (c-g-c). Do đó <sub>∠</sub>Y W Z D ∠B LI D ∠BDA D 9 0ı ∠W D U0:


Từ đó suy ra <sub>∠</sub>D U0<sub>W</sub> <sub>D</sub> <sub>9 0</sub>ı <sub>với</sub> <sub>U</sub>0 <sub>là giao điểm của các đường thẳng</sub> <sub>W Z</sub> <sub>và</sub> <sub>A D:</sub> <sub>Mặt</sub>


khác, ta lại có <sub>∠</sub>D U0W D ∠D U0Z D 9 0ınên U0 thuộc các đường tròn. C DH /và . J / :


Chứng minh tương tự, đường trịn. BDK / cũng đi quaU0:Từ đó, ta có điều phải chứng minh.


b)(Hình vẽ xem trang sau) Gọi S0 và R lần lượt là giao điểm thứ hai của các đường thẳng


A I ; S I và đường trịn. O / : Ta cóS0I2 D S0L S0S nên <sub>∠</sub>S0I L D ∠S0S I D ∠SAR :


MàI L k A D nên<sub>∠</sub>S0I L D ∠S0A D ;từ đó suy ra <sub>∠</sub>S0AR D ∠S0A D hay


∠BAR D ∠C A D: . 1 /


KẻDH0 ? J G . H0 2 J G / : Ta cóGH0 GJ D GD2 D GE GF nên tứ giác EF J H0


nội tiếp. Lại có tứ giácA E J F nội tiếp nên các điểmA ; E ; F ; J và H0đồng viên, từ đó suy


ra<sub>∠</sub>A H0<sub>J</sub> <sub>D</sub> <sub>9 0</sub>ı <sub>hay ba điểm</sub><sub>A ; D</sub> <sub>và</sub> <sub>H</sub>0 <sub>thẳng hàng.</sub>


Mặt khác, ta có <sub>∠</sub>P A J D ∠QA J D 9 0ı C 21∠BA C D ∠B I C và ∠I B C D ∠I J C ;


∠I C B D ∠I J B nên<sub>4</sub>I B C 4AP J 4A J Q(g-g).



Gọi P0; Q0 lần lượt là trung điểm của J P và J Q thì <sub>4</sub>I B L 4AP P0 4A J Q0 và


4I C L 4AQ Q0:Suy ra tứ giác AP0J Q0nội tiếp.


Từ <sub>∠</sub>B I L D ∠JAQ0 <sub>D</sub> <sub>∠</sub><sub>J P</sub>0<sub>Q</sub>0 <sub>D</sub> <sub>∠</sub><sub>BP Q ;</sub> <sub>ta suy ra tứ giác</sub> <sub>P B I L</sub>0 <sub>nội tiếp (</sub><sub>L</sub>0 <sub>là</sub>


giao điểm của các đường thẳng LI và P Q). Mà <sub>∠</sub>P B I D 9 0ı nên I L ? P Q : Lại có


I L k A D và A D ? M N nên M N k P Q :


QuaM kẻ đường thẳng song song với đường thẳngAP cắt đường thẳngA T tạiA0:Ta có


T A0
T A D


T M
T P D


T N
T Q;


suy ra NA0 k AQ và <sub>∠</sub>A M A0 D ∠A NA0 D 9 0ı: Từ đó


∠BA T D 9 0ı <sub>∠</sub>A A0M D 9 0ı <sub>∠</sub>A N M D ∠C A D: . 2 /


</div>
<span class='text_page_counter'>(183)</span><div class='page_container' data-page=183>

<b>Epsilon</b>



<b>staff</b>




<i>Lời giải và bình luận đề thi chọn đội tuyển IMO 2018</i> <b>21</b>


A


B


C


J
F


E
O


D
L
I


G


M


N
P


Q


T
S



S0


R
L0


H0


P0


Q0


A0


Bình luận. Đây là bài tốn khá khó. Hai ý của bài tốn khơng liên quan mấy tới nhau. Ýa)có
thể tổng quát hơn (và đơn giản hơn) thành bài toán sau:


</div>
<span class='text_page_counter'>(184)</span><div class='page_container' data-page=184>

<b>Epsilon</b>



<b>staff</b>



<b>22</b> <i>Lời giải và bình luận đề thi chọn đội tuyển IMO 2018</i>


Tính chất này cịn có thể mở rộng hơn nữa bằng cách sử dụng điểm Miquel:


Bài toán 2.<i>Cho tam giác</i>A B C <i>có các điểm</i>D ; E ; F <i>bất kỳ nằm trên cạnh</i>B C ; C A ; A B :


<i>Các đường tròn</i>. A EF / ; . C F D / ; . C DE / <i>có một điểm chung là</i>M : <i>Một đường tròn</i> . K /


<i>đi qua</i> P <i>và</i>D <i>cắt lại các đường thẳng</i>DE ; DF <i>lần lượt tại</i> Q <i>và</i>R :<i>Chứng minh rằng các</i>
<i>đường tròn</i>. DB Q / ; . D C R / <i>và</i> . DEF / <i>có một điểm chung khác</i> D:



Lời giải hai bài tốn tổng qt trên ngồi phương pháp cộng góc thơng thường thì ta cũng có


thể sử dụng phép nghịch đảo cực D:


Ýb)của bài TST cũng là bài toán khá thách thức địi hỏi học sinh phải có nhiều kỹ năng và biết


một số bổ đề mới có thể giải trọn vẹn. Ý này có thể tổng quát cho tam giác pedal bất kỳ như sau:


Bài toán 3.<i>Cho tam giác</i>A B C <i>và điểm</i> P <i>bất kỳ trên mặt phẳng. Gọi</i> DEF <i>là tam giác</i>
<i>pedal của điểm</i>P : <i>Đường thẳng</i>EF <i>cắt đường thẳng</i> B C <i>tại</i> G : <i>Đường thẳng</i> GP <i>cắt các</i>
<i>đường thẳng</i>C A <i>và</i>A B <i>lần lượt tại</i>Q ; R :<i>Các đường thẳng qua</i>A<i>vng góc</i>A B <i>và</i>A C <i>cắt</i>
<i>các đường thẳng</i>P B ; P C <i>tại</i>M ; N : <i>Đường thẳng</i> N Q<i>cắt đường thẳng</i>M R<i>tại</i> L :<i>Đường</i>


<i>tròn</i> . DEF /<i>cắt lại đường thẳng</i> B C <i>tại</i> K :<i>Chứng minh rằng</i> ∠K A B D ∠L A C :


Khi P trùng với tâm đường trịn bàng tiếp góc Acủa tam giác A B C thì ta thu được ýb)của


bài TST. Đây là một mở rộng theo kiểu đẳng giác có ý nghĩa và khá phức tạp. Sơ lược các bước
chứng minh cho bài toán tổng quát này như sau:


Chứng minh M N song song vớiGP bằng cách sử dụng tỉ số kép, tức là chứng minh


P . B C ; D G / D P . M N ; D1/ :


Chứng minh hai tam giác K QRvà A M N thấu xạ tâm T bằng định lý Desargues, cùng


với các định lý dùng để biến đổi tỉ số là Thales, Menelaus.


Cuối cùng là chỉ ra hai đường thẳngA LvàA T đẳng giác với góc<sub>∠</sub>M A N (cũng là đẳng



giác trong góc<sub>∠</sub>BA C) thơng qua bổ đề:<i>Cho hình thang</i>A B C D <i>có hai đáy là</i>A B <i>và</i>


C D: <i>Đường thẳng</i>A D <i>cắt đường thẳng</i>B C <i>tại</i> E : <i>Các đường chéo</i> A C <i>và</i> BD <i>cắt</i>
<i>nhau tại</i> F : <i>Nếu một điểm</i>M <i>thỏa mãn</i> M A ; M B <i>đẳng giác với góc</i> <sub>∠</sub>C M D <i>thì</i> M A ;
M B <i>cũng đẳng giác với góc</i> ∠E M F :


Có thể tham khảo thêm các mở rộng tại liên kết:gspot.


</div>

<!--links-->
<a href=' /><a href=' /><a href=' /><a href=' /><a href=' /><a href=' /><a href=' /><a href=' /><a href=' /><a href=' /><a href=' /><a href=' /><a href=' /><a href=' /> Tổng hợp đề thi các môn HK II năm 2008 - 2009
  • 26
  • 502
  • 0
  • ×